Sie sind auf Seite 1von 538

Deeper Understanding, Faster Calc: SOA MFE

and CAS Exam 3F


Yufeng Guo
October 26, 2009

ii

Contents
Introduction

ix

9 Parity and other option relationships


9.1 Put-call parity . . . . . . . . . . . . . . . . . . .
9.1.1 Option on stocks . . . . . . . . . . . . . .
9.1.2 Options on currencies . . . . . . . . . . .
9.1.3 Options on bonds . . . . . . . . . . . . . .
9.1.4 Generalized parity and exchange options .
9.1.5 Comparing options with respect to style,
strike . . . . . . . . . . . . . . . . . . . .

. . . . . . . . .
. . . . . . . . .
. . . . . . . . .
. . . . . . . . .
. . . . . . . . .
maturity, and
. . . . . . . . .

10 Binomial option pricing: I


10.1 One-period binomial model: simple examples
10.2 General one-period binomial model . . . . . .
10.2.1 Two or more binomial trees . . . . . .
10.2.2 Options on stock index . . . . . . . .
10.2.3 Options on currency . . . . . . . . . .
10.2.4 Options on futures contracts . . . . .

.
.
.
.
.
.

.
.
.
.
.
.

.
.
.
.
.
.

.
.
.
.
.
.

.
.
.
.
.
.

.
.
.
.
.
.

.
.
.
.
.
.

.
.
.
.
.
.

.
.
.
.
.
.

.
.
.
.
.
.

35
35
36
49
64
67
71

11 Binomial option pricing: II


11.1 Understanding early exercise . . . . . . . . . . .
11.2 Understanding risk-neutral probability . . . . . .
11.2.1 Pricing an option using real probabilities
11.2.2 Binomial tree and lognormality . . . . . .
11.2.3 Estimate stock volatility . . . . . . . . . .
11.3 Stocks paying discrete dividends . . . . . . . . .
11.3.1 Problems with discrete dividend tree . . .
11.3.2 Binomial tree using prepaid forward . . .

.
.
.
.
.
.
.
.

.
.
.
.
.
.
.
.

.
.
.
.
.
.
.
.

.
.
.
.
.
.
.
.

.
.
.
.
.
.
.
.

.
.
.
.
.
.
.
.

.
.
.
.
.
.
.
.

.
.
.
.
.
.
.
.

.
.
.
.
.
.
.
.

79
79
80
81
88
91
95
96
98

12 Black-Scholes
12.1 Introduction to the Black-Scholes formula . . . .
12.1.1 Call and put option price . . . . . . . . .
12.1.2 When is the Black-Scholes formula valid?
12.2 Applying the formula to other assets . . . . . . .

.
.
.
.

.
.
.
.

.
.
.
.

.
.
.
.

.
.
.
.

.
.
.
.

.
.
.
.

.
.
.
.

.
.
.
.

105
105
105
107
107

iii

.
.
.
.
.
.

1
1
1
11
12
13
19

iv

CONTENTS

12.3

12.4

12.5

12.6

12.2.1 Black-Scholes formula in terms of prepaid forward price


12.2.2 Options on stocks with discrete dividends . . . . . . . .
12.2.3 Options on currencies . . . . . . . . . . . . . . . . . . .
12.2.4 Options on futures . . . . . . . . . . . . . . . . . . . . .
Option the Greeks . . . . . . . . . . . . . . . . . . . . . . . . .
12.3.1 Delta . . . . . . . . . . . . . . . . . . . . . . . . . . . .
12.3.2 Gamma . . . . . . . . . . . . . . . . . . . . . . . . . . .
12.3.3 Vega . . . . . . . . . . . . . . . . . . . . . . . . . . . . .
12.3.4 Theta . . . . . . . . . . . . . . . . . . . . . . . . . . . .
12.3.5 Rho . . . . . . . . . . . . . . . . . . . . . . . . . . . . .
12.3.6 Psi . . . . . . . . . . . . . . . . . . . . . . . . . . . . . .
12.3.7 Greek measures for a portfolio . . . . . . . . . . . . . .
12.3.8 Option elasticity and volatility . . . . . . . . . . . . . .
12.3.9 Option risk premium and Sharp ratio . . . . . . . . . .
12.3.10 Elasticity and risk premium of a portfolio . . . . . . . .
Profit diagrams before maturity . . . . . . . . . . . . . . . . . .
12.4.1 Holding period profit . . . . . . . . . . . . . . . . . . . .
12.4.2 Calendar spread . . . . . . . . . . . . . . . . . . . . . .
Implied volatility . . . . . . . . . . . . . . . . . . . . . . . . . .
12.5.1 Calculate the implied volatility . . . . . . . . . . . . . .
12.5.2 Volatility skew . . . . . . . . . . . . . . . . . . . . . . .
12.5.3 Using implied volatility . . . . . . . . . . . . . . . . . .
Perpetual American options . . . . . . . . . . . . . . . . . . . .
12.6.1 Perpetual calls and puts . . . . . . . . . . . . . . . . . .
12.6.2 Barrier present values . . . . . . . . . . . . . . . . . . .

.
.
.
.
.
.
.
.
.
.
.
.
.
.
.
.
.
.
.
.
.
.
.
.
.

107
108
108
110
110
111
111
112
112
112
112
112
113
114
115
115
115
118
119
119
120
121
121
121
125

13 Market-making and delta-hedging


13.1 Delta hedging . . . . . . . . . . . . . . . . . . . .
13.2 Examples of Delta hedging . . . . . . . . . . . .
13.3 Textbook Table 13.2 . . . . . . . . . . . . . . . .
13.4 Textbook Table 13.3 . . . . . . . . . . . . . . . .
13.5 Mathematics of Delta hedging . . . . . . . . . . .
13.5.1 Delta-Gamma-Theta approximation . . .
13.5.2 Understanding the market makers profit

.
.
.
.
.
.
.

.
.
.
.
.
.
.

.
.
.
.
.
.
.

.
.
.
.
.
.
.

.
.
.
.
.
.
.

.
.
.
.
.
.
.

.
.
.
.
.
.
.

.
.
.
.
.
.
.

.
.
.
.
.
.
.

129
129
129
138
140
141
141
142

14 Exotic options: I
14.1 Asian option (i.e. average options)
14.1.1 Characteristics . . . . . . .
14.1.2 Examples . . . . . . . . . .
14.1.3 Geometric average . . . . .
14.1.4 Payo at maturity T . . . .
14.2 Barrier option . . . . . . . . . . . .
14.2.1 Knock-in option . . . . . .
14.2.2 Knock-out option . . . . . .
14.2.3 Rebate option . . . . . . . .

.
.
.
.
.
.
.
.
.

.
.
.
.
.
.
.
.
.

.
.
.
.
.
.
.
.
.

.
.
.
.
.
.
.
.
.

.
.
.
.
.
.
.
.
.

.
.
.
.
.
.
.
.
.

.
.
.
.
.
.
.
.
.

.
.
.
.
.
.
.
.
.

.
.
.
.
.
.
.
.
.

145
145
145
146
146
147
147
147
147
148

.
.
.
.
.
.
.
.
.

.
.
.
.
.
.
.
.
.

.
.
.
.
.
.
.
.
.

.
.
.
.
.
.
.
.
.

.
.
.
.
.
.
.
.
.

.
.
.
.
.
.
.
.
.

.
.
.
.
.
.
.
.
.

.
.
.
.
.
.
.
.
.

CONTENTS
14.2.4 Barrier parity . .
14.2.5 Examples . . . .
14.3 Compound option . . .
14.4 Gap option . . . . . . .
14.4.1 Definition . . . .
14.4.2 Pricing formula .
14.4.3 How to memorize
14.5 Exchange option . . . .

v
. . . . . . . . . . . .
. . . . . . . . . . . .
. . . . . . . . . . . .
. . . . . . . . . . . .
. . . . . . . . . . . .
. . . . . . . . . . . .
the pricing formula
. . . . . . . . . . . .

.
.
.
.
.
.
.
.

.
.
.
.
.
.
.
.

.
.
.
.
.
.
.
.

.
.
.
.
.
.
.
.

.
.
.
.
.
.
.
.

.
.
.
.
.
.
.
.

.
.
.
.
.
.
.
.

148
148
149
151
151
151
151
152

18 Lognormal distribution
18.1 Normal distribution . . . . . . . . . . . . . . . . . . .
18.2 Lognormal distribution . . . . . . . . . . . . . . . . . .
18.3 Lognormal model of stock prices . . . . . . . . . . . .
18.4 Lognormal probability calculation . . . . . . . . . . . .
18.4.1 Lognormal confidence interval . . . . . . . . . .
18.4.2 Conditional expected prices . . . . . . . . . . .
18.4.3 Black-Scholes formula . . . . . . . . . . . . . .
18.5 Estimating the parameters of a lognormal distribution

.
.
.
.
.
.
.
.

.
.
.
.
.
.
.
.

.
.
.
.
.
.
.
.

.
.
.
.
.
.
.
.

.
.
.
.
.
.
.
.

.
.
.
.
.
.
.
.

155
155
156
156
157
158
162
162
163

.
.
.
.

.
.
.
.

.
.
.
.

.
.
.
.

.
.
.
.

.
.
.
.

165
165
166
167

19 Monte Carlo valuation


19.1 Example 1 Estimate E (ez ) . . . . . . . . . . . . . . . . . . .
19.2 Example 2 Estimate . . . . . . . . . . . . . . . . . . . . . .
19.3 Example 3 Estimate the price of European call or put options
19.4 Example 4 Arithmetic and geometric options . . . . . . . . .
19.5 Ecient Monte Carlo valuation . . . . . . . . . . . . . . . . .
19.5.1 Control variance method . . . . . . . . . . . . . . . . .
19.6 Antithetic variate method . . . . . . . . . . . . . . . . . . . .
19.7 Stratified sampling . . . . . . . . . . . . . . . . . . . . . . . .
19.7.1 Importance sampling . . . . . . . . . . . . . . . . . . .
19.8 Sample problems . . . . . . . . . . . . . . . . . . . . . . . . .

.
.
.
.
.
.
.
.
.
.

.
.
.
.
.
.
.
.
.
.

173
173
177
180
184
193
193
197
199
199
199

18.6 How are asset prices distributed


18.6.1 Histogram . . . . . . . .
18.6.2 Normal probability plots
18.7 Sample problems . . . . . . . .

.
.
.
.

.
.
.
.

.
.
.
.

.
.
.
.

.
.
.
.

.
.
.
.

20 Brownian motion and Itos Lemma


20.1 Introduction . . . . . . . . . . . . . . . . .
20.1.1 Big picture . . . . . . . . . . . . .
20.2 Brownian motion . . . . . . . . . . . . . .
20.2.1 Stochastic process . . . . . . . . .
20.2.2 Definition of Brownian motion . .
20.2.3 Martingale . . . . . . . . . . . . .
20.2.4 Properties of Brownian motion . .
20.2.5 Arithmetic Brownian motion and
motion . . . . . . . . . . . . . . . .

.
.
.
.

.
.
.
.

.
.
.
.
.
.
.
.

.
.
.
.

.
.
.
.
.
.
.
.

.
.
.
.

.
.
.
.
.
.
.
.

.
.
.
.

.
.
.
.
.
.
.
.

.
.
.
.

.
.
.
.

. . . . . . . . . . . . .
. . . . . . . . . . . . .
. . . . . . . . . . . . .
. . . . . . . . . . . . .
. . . . . . . . . . . . .
. . . . . . . . . . . . .
. . . . . . . . . . . . .
Geometric Brownian
. . . . . . . . . . . . .

205
205
206
207
207
207
209
210
214

vi

CONTENTS
20.2.6 Ornstein-Uhlenbeck process . . . . . . . . .
20.3 Definition of the stochastic calculus . . . . . . . . .
20.4 Properties of the stochastic calculus . . . . . . . .
20.5 Itos lemma . . . . . . . . . . . . . . . . . . . . . .
20.5.1 Multiplication rules . . . . . . . . . . . . .
20.5.2 Itos lemma . . . . . . . . . . . . . . . . . .
20.6 Geometric Brownian motion revisited . . . . . . .
20.6.1 Relative importance of drift and noise term
20.6.2 Correlated Ito processes . . . . . . . . . . .
20.7 Sharpe ratio . . . . . . . . . . . . . . . . . . . . . .
20.8 Risk neutral process . . . . . . . . . . . . . . . . .
20.9 Valuing a claim on S a . . . . . . . . . . . . . . . .
20.9.1 Process followed by S a . . . . . . . . . . . .
20.9.2 Formula for S A (t) and E S A (t) . . . . . .
20.9.3 Expected return of a claim on S A (t) . . . .
20.9.4 Specific examples . . . . . . . . . . . . . . .

.
.
.
.
.
.
.
.
.
.
.
.
.
.
.
.

.
.
.
.
.
.
.
.
.
.
.
.
.
.
.
.

.
.
.
.
.
.
.
.
.
.
.
.
.
.
.
.

.
.
.
.
.
.
.
.
.
.
.
.
.
.
.
.

.
.
.
.
.
.
.
.
.
.
.
.
.
.
.
.

.
.
.
.
.
.
.
.
.
.
.
.
.
.
.
.

.
.
.
.
.
.
.
.
.
.
.
.
.
.
.
.

.
.
.
.
.
.
.
.
.
.
.
.
.
.
.
.

215
216
222
223
223
223
225
225
226
230
233
233
233
234
235
235

21 Black-Scholes equation
21.1 Dierential equations and valuation under certainty .
21.1.1 Valuation equation . . . . . . . . . . . . . . . .
21.1.2 Bonds . . . . . . . . . . . . . . . . . . . . . . .
21.1.3 Dividend paying stock . . . . . . . . . . . . . .
21.2 Black-Scholes equation . . . . . . . . . . . . . . . . . .
21.2.1 How to derive Black-Scholes equation . . . . .
21.2.2 Verifying the formula for a derivative . . . . . .
21.2.3 Black-Scholes equation and equilibrium returns
21.3 Risk-neutral pricing . . . . . . . . . . . . . . . . . . .

.
.
.
.
.
.
.
.
.

.
.
.
.
.
.
.
.
.

.
.
.
.
.
.
.
.
.

.
.
.
.
.
.
.
.
.

.
.
.
.
.
.
.
.
.

.
.
.
.
.
.
.
.
.

245
245
245
246
246
246
246
247
250
252

22 Exotic options: II
253
22.1 All-or-nothing options . . . . . . . . . . . . . . . . . . . . . . . . 253
23 Volatility
24 Interest rate models
24.1 Market-making and bond pricing . . . . . . . . . . . . .
24.1.1 Review of duration and convexity . . . . . . . . .
24.1.2 Interest rate is not so simple . . . . . . . . . . .
24.1.3 Impossible bond pricing model . . . . . . . . . .
24.1.4 Equilibrium equation for bonds . . . . . . . . . .
24.1.5 Delta-Gamma approximation for bonds . . . . .
24.2 Equilibrium short-rate bond price models . . . . . . . .
24.2.1 Arithmetic Brownian motion (i.e. Merton model)
24.2.2 Rendleman-Bartter model . . . . . . . . . . . . .
24.2.3 Vasicek model . . . . . . . . . . . . . . . . . . .
24.2.4 CIR model . . . . . . . . . . . . . . . . . . . . .
24.3 Bond options, caps, and the Black model . . . . . . . .

255

.
.
.
.
.
.
.
.
.
.
.
.

.
.
.
.
.
.
.
.
.
.
.
.

.
.
.
.
.
.
.
.
.
.
.
.

.
.
.
.
.
.
.
.
.
.
.
.

.
.
.
.
.
.
.
.
.
.
.
.

257
257
257
264
265
268
270
271
271
272
272
274
275

CONTENTS
24.3.1 Black formula . . . . .
24.3.2 Interest rate caplet . .
24.4 Binomial interest rate model
24.5 Black-Derman-Toy model . .

vii
.
.
.
.

.
.
.
.

.
.
.
.

.
.
.
.

.
.
.
.

.
.
.
.

.
.
.
.

.
.
.
.

.
.
.
.

.
.
.
.

.
.
.
.

.
.
.
.

.
.
.
.

.
.
.
.

.
.
.
.

.
.
.
.

.
.
.
.

.
.
.
.

.
.
.
.

.
.
.
.

275
279
279
283

viii

CONTENTS

Introduction
This study guide is for SOA MFE and CAS Exam 3F. Before you start, make
sure you have the following items:
1. Derivatives Markets, the 2nd edition.
2. Errata of Derivatives Markets. You can download the errata at http://
www.kellogg.northwestern.edu/faculty/mcdonald/htm/typos2e_01.
html.
3. Download the syllabus from the SOA or CAS website.
4. Download the sample MFE problems and solutions from the SOA website.
5. Download the recent SOA MFE and CAS Exam 3 problems.

ix

INTRODUCTION

Chapter 9

Parity and other option


relationships
9.1

Put-call parity

9.1.1

Option on stocks

Notation
t=0
T
S0
ST
K
CEur (K, T )
CEur (K, 0)
PEur (K, T )
PEur (K, 0)
r

F0,T

Current date (date when an option is sold or bought)


Option expiration date (maturity date)
Current price of the underlying asset
The price of the underlying asset at the option expiration date
Strike price or exercise price
Premium of a European call option with strike price K and T years to expiration
Premium of a European call option on the expiration date
Premium of a European put option with strike price K and T years to expiration
Premium of a European put option on the expiration date
The continuously compounded annual risk-free interest rate
The continuously compounded annual dividend rate
Delivery price in a forward contract expiring in T

Put-call parity
The textbook gives the following formula
CEur (K, T ) PEur (K, T ) = P V0,T (F0,T K) = erT (F0,T K)

(9.1)

The textbook explains the intuition behind Equation 9.1. If we set the
forward price F0,T as the common strike price for both the call and the put
1

CHAPTER 9. PARITY AND OTHER OPTION RELATIONSHIPS

(i.e. K = F0,T ) , then CEur (K, T ) PEur (K, T ) = P V0,T (F0,T K) = 0.


Buying a call and selling a put with K = F0,T synthetically creates a forward
contract and the premium for a forward contract is zero.
However, often I find that Equation 9.1 is not intuitive at all. In fact, its
annoyingly complex and hard to memorize. So I like to rewrite 9.1 as follows:
CEur (K, T ) + P V (K) = PEur (K, T ) + S0

(9.2)

Later I will explain the intuition behind 9.2. First, lets prove 9.2. The proof
is extremely important.
Suppose at time zero we have two portfolios:
Portfolio #1 consists of a European call option on a stock and P V (K) =
KerT . P V (K) is the present value of the strike price K and r is the
continuously compounded risk free interest rate per year.
Portfolio #2 consists of a European put option on the stock and one share
of the stock with current price S0 .
Both the call and put have the same underlying stock, the same strike price
K, and the same expiration date T . Notice that at time zero Portfolio #1
is worthy CEur (K, T ) + P V (K); Portfolio #2 is worth PEur (K, T ) + S0 .
Since its dicult to compare the value of Portfolio #1 and the value of
Portfolio #2 at time zero, lets compare them at the expiration date T . Well
soon see that the two portfolios have the same value at T .
Payo of Portfolio #1 at the expiration date T
If ST < K If ST K
Call payo is max (0, ST K) 0
ST K
Payo of P V (K)
K
K
Total
K
ST
If you have P V (K) at t = 0, youll have K at T .
Payo of Portfolio #2 at the expiration date T
If ST < K If ST K
Put payo max (0, K ST ) K ST
0
Payo of S0
ST
ST
Total
K
ST
If you have one stock worth S0 at t = 0, youll have one stock at T worth
ST .
You see that Portfolio #1 and Portfolio #2 have identical payos of max (K, ST )
at T . If ST < K, both portfolios are worth the strike price K; if ST K, both

9.1. PUT-CALL PARITY

portfolios are worth the stock price ST . Since Portfolio #1 and #2 are worth
the same at T , to avoid arbitrage, they must be worth the same at any time
prior to T . Otherwise, anyone can make free money by buying the lower priced
portfolio and selling the higher priced one. So Portfolio #1 and #2 are worth
the same at time zero. Equation 9.2 holds.
I recommend that from this point now you throw Equation 9.1 away and use
Equation 9.2 instead.
How to memorize Equation 9.2:
Tip 9.1.1. Many candidates have trouble memorizing Equation 9.2. For example, its very easy to write a wrong formula CEur (K, T ) + S0 = PEur (K, T ) +
P V (K). To memorize Equation 9.2, notice that for a call to work, a call must
go hand in hand with the strike price K. When exercising a call option, you
give the call seller both the call certificate and the strike price K. In return, the
call seller gives you one stock. Similarly, for a put to work, a put must go hand
in hand with one stock. When exercising a put, you must give the put seller both
the put certificate and one stock. In return, the put seller gives you the strike
price K.
Tip 9.1.2. Another technique that helps me memorize Equation 9.2 is the
phrase Check (CK) Please (PS). At expiration T , CEur (K, 0)+K = PEur (K, 0)+
ST . Discounting this equation to time zero, we get: CEur (K, T ) + P V (K) =
PEur (K, T ) + S0 .

Example 9.1.1. The price of a 6-month 30-strike European call option is 12.22.
The stock price is 35. The continuously compounded risk-free interest rate is 8%
per year. Whats the price of a 6-month 30-strike European put option on the
same stock?
Solution.
K = 30
T = 6/12 = 0.5
S0 = 35
r = 0.08
CEur = 12.22
CEur + P V (K) = PEur + S0
12.22 + 30e0.08(0.5) = PEur + 35,
PEur = 6. 043
Parity if the stock pays discrete dividends
If the stock pays discrete dividend, the parity formula is
CEur (K, T ) + P V (K) = PEur (K, T ) + S0 P V (Div)

(9.3)

This is why we need to subtract the term P V (Div). At expiration, CEur (K, T )+
K = PEur (K, T ) + ST . If we discount ST from T to time zero, well get

CHAPTER 9. PARITY AND OTHER OPTION RELATIONSHIPS

S0 P V (Div). If you have one stock worth S0 at time zero, then during [0, T ],
youll receive dividend payments. Then at T , you not only have one share of
stock, you also have the accumulated value of the dividend. To get exactly one
stock at T , you need to have S0 P V (Div) at time zero.
Discounting this equation back to time zero, we get Equation 9.3
Please note that Equation 9.3 assumes that both the timing and the amount
of each dividend are 100% known in advance.

Example 9.1.2. The price of a 9-month 95-strike European call option is 19.24.
The stock price is 100. The stock pays dividend of $1 in 3 months and $2 in 6
months. The continuously compounded risk-free interest rate is 10% per year.
Whats the price of a 9-month 95-strike European put option on the same stock?
Solution.

T = 9/12 = 0.75
K = 95
S0 = 100
CEur = 19.24
P V (Div) = e0.1(3/12) + 2e0.1(6/12) = 2. 877 8

r = 0.1

CEur (K, T ) + P V (K) = PEur (K, T ) + S0 P V (Div)


19.24 + 95e0.1(0.75) = PEur (K, T ) + 100 2. 877 8, PEur (K, T ) = 10. 25

Example 9.1.3. The price of a 9-month 83-strike European put option is 13.78.
The stock price is 75. The stock pays dividend of $1 in 3 months, $2 in 6 months,
$3 in 9 months, and $4 in 12 months. The continuously compounded risk-free
interest rate is 6% per year. Whats the price of a 9-month 95-strike European
call option on the same stock?
Solution.

T = 9/12 = 0.75
K = 83
S0 = 75
r = 0.06
PEur = 13.78
P V (Div) = e0.06(3/12) + 2e0.06(6/12) + 3e0.06(9/12) = 5. 794
CEur (K, T ) + P V (K) = PEur (K, T ) + S0 P V (Div)
CEur (K, T ) + 83e0.06(0.75) = 13.78 + 75 5. 794
CEur (K, T ) = 3. 64
Tip 9.1.3. When calculating P V (Div) in Equation 9.3, discard any dividend
paid after the option expiration date. In this example, the $4 is paid in 12
months, which is after the expiration date of the option. This dividend is ignored
when we use Equation 9.3.

9.1. PUT-CALL PARITY

Parity if the stock pays continuous dividends


If the stock pays dividends at a continuously compounded rate of per year,
the parity formula is:
CEur (K, T ) + P V (K) = PEur (K, T ) + S0 eT

(9.4)

At expiration, CEur (K, T ) + K = PEur (K, T ) + ST . If we discount ST from


T to time zero, well get S0 eT . If you have eT share of a stock, by investing
dividends and buying additional stock, youll have exactly one stock at T . This
concept is called tailing. Refer to Derivatives Markets Section 5.2 about tailing.
Example 9.1.4. The price of a 6-month 90-strike European put option is 5.54.
The stock price is 110. The stock pays dividend at a continuously compounded
rate of 2% per year. The continuously compounded risk-free interest rate is 6%
per year. Whats the price of a 6-month 90-strike European call option on the
same stock?
Solution.
T = 6/12 = 0.5
K = 90
S0 = 110
r = 0.06
= 0.02
PEur = 5.54
CEur (K, T ) + P V (K) = PEur (K, T ) + S0 eT
CEur (K, T ) + 90e0.06(0.5) = 5.54 + 110e0.02(0.5)
CEur (K, T ) = 27. 11
Example 9.1.5. The price of a 3-month 40-strike European call option is 6.57.
The stock price is 44. The stock pays dividend at a continuously compounded
rate of 5% per year. The continuously compounded risk-free interest rate is 8%
per year. Whats the price of a 3-month 40-strike European put option on the
same stock?
Solution.
T = 3/12 = 0.25
K = 40
S0 = 44
r = 0.08
= 0.08
CEur = 6.57
CEur (K, T ) + P V (K) = PEur (K, T ) + S0 eT
6.57 + 40e0.08(0.25) = PEur (K, T ) + 44e0.05(0.25)
PEur (K, T ) = 2. 32

Synthetic stock
Rearranging Equation 9.3, we get:
S0 = CEur (K, T ) PEur (K, T ) + P V (K) + P V (Div)
To understand the meaning of Equation 9.5, notice

(9.5)

CHAPTER 9. PARITY AND OTHER OPTION RELATIONSHIPS


Symbol
+CEur (K, T )
PEur (K, T )
+P V (K)
+P V (Div)

Meaning at t = 0
buy a K-strike call expiring in T
sell a K-strike put expiring in T
buy a zero-coupon bond that pays K at T
buy a zero-coupon bond that pays Div at T

Next, notice CEur (K, T ) PEur (K, T ) + P V (K) is worth ST . At T , you


always receive K from the zero-coupon bond seller. However, the call and put
values depend on whether ST K.
If ST K, the sold put expires worthless; you exercise the call, paying K
and receiving one stock.
If on the other hand, ST K, the purchased call expires worthless and the
sold put is exercised against you. You pay the put holder K and receive
one stock from him.
Either way, if at time zero you buy a call, sell a put, and invest P V (K)
in a zero-coupon bond, then at T you are guaranteed to have one stock.
Once you understand CEur (K, T )PEur (K, T )+P V (K) is worth one stock
at T , the meaning of Equation 9.5 is obvious: if you buy one stock at time zero,
then at time T , youll have one stock worth ST . In addition, youll receive the
future value of the dividends due to owning a stock.
Example 9.1.6. The price of a 9-month 52-strike European call option on a
non dividend-paying stock is 33.4420. The price of a 9-month 52-strike European
put option on the same stock is 15.1538. The continuously compounded risk-free
interest rate is 6% per year. How can you synthetically create one stock at time
zero? Whats the price of this synthetically created stock at time zero?
Solution.
The parity for a non-dividend paying stock is
CEur (K, T ) + P V (K) = PEur (K, T ) + S0
Rearranging this equation, we get:
S0 = CEur (K, T ) + P V (K) PEur (K, T )
To synthetically create the ownership of one stock, you need to do the following at time zero:
Buy a 9-month 52-strike European call option
Sell a 9-month 52-strike European put option
Invest 52e0.06(0.75) = 49. 711 9 in an account earning risk-free interest
rate (i.e. buying a zero coupon bond that pays 52 in 9 months)
The current price of this synthetically created stock is:
S0 = 33.4420 + 49. 711 9 15.1538 = 68

9.1. PUT-CALL PARITY

Synthetic T-bill
Rearranging Equation 9.3, we get:
P V (Div) + P V (K) = S0 + PEur (K, T ) CEur (K, T )

(9.6)

According to Equation 9.6, buying one stock, buying a K-strike put, and
selling a K-strike call synthetically creates a zero coupon bond with a present
value equal to P V (Div) + P V (K).
Creating synthetic T-bill by buying the stock, buying a put, and selling a
call is called a conversion. If we short the stock, buy a call, and sell a put, we
create a short position on T-bill. This is called a reverse conversion.
Example 9.1.7. Your mother-in-law desperately wants to borrow $1000 from
you for one year. Shes willing to pay you 50% interest rate for using your
money for one year. You really want to take her oer and earn 50% interest.
However, state anti-usury laws prohibits any lender from charging an interest
rate equal to or greater than 50%. Since you happen to know the put-call parity,
you decide to synthetically create a loan and circumvent the state anti-usury
law. Explain how you can synthetically create a loan and earn 50% interest.
You want to lend $1000 at time zero and receive $1000 (1.5) = 1500 at T = 1.
To achieve this, at time zero you can
have your mother-in-law sell you an asset thats worth $1000
have your mother-in-law sell you a 1500-strike, 1-year to expiration put
option on the asset
sell your mother-in-law a 1500-strike, 1-year to expiration call option on
the asset.
Lets see whats happens at T = 1.
If ST 1500, you exercise the put and sell the asset to your mother-in-law
for 1500
if ST 1500, your mother-in-law exercises the call and buys the asset
from you for 1500.
The net eect is that you give your mother-in-law $1000 at time zero and
receive $1500 from her at T = 1.
In this example, you have a long position on the synthetically created Tbill. This is an example of conversion. In contrast, your mother-in-law has a
short position in the synthetically created T-bill. This is an example of reverse
conversion.

CHAPTER 9. PARITY AND OTHER OPTION RELATIONSHIPS

Synthetic call option


Rearranging Equation 9.3, we get:
CEur (K, T ) = PEur (K, T ) + S0 P V (Div) P V (K)
Example 9.1.8. The price of a 6-month 75-strike European put option on a
dividend-paying stock is 8.06. The stock price is 80. The continuously compounded risk-free interest rate is 5% per year. The continuously compounded
dividend rate is 3% per year. Explain how you can create a synthetic 6-month
75-strike European call option on the stock. Calculate the premium for such a
synthetic call option.
Solution.

The put-call parity for a stock paying continuous dividend is


CEur (K, T ) + P V (K) = PEur (K, T ) + S0 eT
Rearranging this equation, we get:
CEur (K, T ) = PEur (K, T ) + S0 eT P V (K)
= 8.06 + 80e0.03(0.5) 75e0.05(0.5) = 13. 72
This is how to synthetically create a 6-month 75-strike European call
symbol
at t = 0
+PEur (K, T ) buy a 6-month 75-strike European put
+S0 eT
buy e0.03(0.5) = 0.985 share of stock
P V (K)
sell a bond that pays 75 in 6 months
Lets see why a call is synthetically created. 0.985 share of stock at t = 0
will grow into one stock if you reinvest the dividend and buy additional share of
stock. If at T = 0.5 the stock price is greater than 75 (i.e. ST > 75), then two
things happen: your purchased put expires worthless; the bond matures and
you need to pay the bond holder K = 75. So the net eect is that if ST > 75
then at T you pay K = 75 and own one stock. This is the same as if ST > 75
you exercise the call, paying K = 75 and receiving one stock.
If ST 75, then two things will happen. You exercise the put, selling your
stock for K = 75; the bond matures and you pay the bond holder K = 75. So
the net eect is that if ST 75 you net cash flow is zero and you dont own a
stock. This is the same as if ST 75 you do nothing and let your call expire
worthless.
Example 9.1.9. The price of a 9-month 110-strike European put option on
a dividend-paying stock is 42.81. The stock price is 100. The continuously
compounded risk-free interest rate is 8% per year. The stock will pay $1 dividend
in 3 months and $1 dividend in 6 months. Explain how you can create a synthetic
9-month 110-strike European call option on the stock. Calculate the premium
for such a synthetic call option.

9.1. PUT-CALL PARITY

CEur (K, T ) = PEur (K, T ) + S0 P V (Div) P V (K)


= 42.81 + 100 e0.08(0.25) e0.08(0.5) 110e0.08(0.75) = 37. 27
How to synthetically create a 9-month 110-strike European call
symbol
at t = 0
+PEur (K, T ) buy a 9-month 110-strike European put
+S0
buy one share of stock paying 100
P V (Div)
sell Bond #1 that pays $1 in 3 months and $1 in 6 months
P V (K)
sell Bond #2 that pays $110 in 6 months
Lets see why a synthetic call is created.
If at expiration date ST > 110
Youll receive $1 dividend at the end of Month 3 and $1 dividend at the
end of Month 6. You use the dividends to pay o Bond #1
Your purchased put expires worthless
You have one stock at T = 0.75
Your pay K = 110 at T = 0.75
The net eect is that youll pay K = 110 at T = 0.75 and own one stock.
This is the same as owning a call option and ST > 110.
If at expiration date ST 110
Youll receive $1 dividend at the end of Month 3 and $1 dividend at the
end of Month 6. You use the dividends to pay o Bond #1
You have one stock at T = 0.75
You exercise your put, surrendering one stock and receiving K = 110
Your pay K = 110 at T = 0.75
The net eect is that you have zero cash left and dont own one stock. This
is the same as owning a call option and ST 110.
Synthetic put option
Rearranging Equation 9.3, we get:
PEur (K, T ) = CEur (K, T ) S0 + P V (Div) + P V (K)
If the stock pays continuous dividend, then
PEur (K, T ) = CEur (K, T ) + P V (K) S0 eT

10

CHAPTER 9. PARITY AND OTHER OPTION RELATIONSHIPS

Example 9.1.10. The price of a 6-month 45-strike European call option on a


dividend-paying stock is 18.62. The stock price is 50. The continuously compounded risk-free interest rate is 6% per year. The stock will pay $1 dividend in
3 months. Explain how you can create a 6-month 45-strike European put option
on the stock. Calculate the premium for such a synthetic put option.
Solution.
PEur (K, T ) = CEur (K, T ) S0 + P V (Div) + P V (K)
= 18.62 50 + e0.06(0.25) + 45e0.06(0.5) = 13. 28
How to create a synthetic 6-month 45-strike European put option
symbol
+CEur (K, T )
S0
+P V (Div)
+P V (K)

at t = 0
buy a 6-month 45-strike European call
short sell one share receiving 50 and invest in a savings account
buy Bond #1 that pays $1 in 3 months
sell Bond #2 that pays $45 in 6 months

This is why CEur (K, T ) S0 + P V (Div) + P V (K) behaves like a put.


If ST 45
(1) You will receive K = 45 at T = 0.5 from Bond #2
(2) You exercise the call at T = 0.5, paying K = 45 which you get from
(1) and receiving one stock
(3) at T = 0.5 you give the stock you get from (2) to the broker from
whom you borrowed the stock for short sale
(4) Bond #1 pays you $1 dividend at the end of Month 3. After receiving
this dividend, you Immediately pay this dividend to the original owner of
the stock you sold short
(2)+(3) will close out your short position on the stock
The net eect is that if ST 45 you keep the proceeds from the short sale,
which you can use to buy a stock. This is "keeping your asset (i.e. proceeds from
the short sale)." This is the same as if you own a 6-month 45-strike European
put and ST 45. If you own a 6-month 45-strike European put and ST 45,
you let the put option expire worthless and you still own a stock.
If ST < 45
(1) You will receive K = 45 at T = 0.5 from Bond #2
(2) You let the call expire worthless
(3) At T = 0.5 you buy a stock from the open market using the proceeds
from the short sale; you give the stock to the broker from whom you
borrowed the stock for short sale

9.1. PUT-CALL PARITY

11

(4) Bond #1 pays you $1 dividend at the end of Month 3. After receiving
this dividend, you Immediately pay this dividend to the original owner of
the stock you sold short
(3)+(4) will close out your short position on the stock.
The net eect is that if ST 45 you receive K = 45 and you spent the
proceeds from the short sale. This is "giving up an asset (proceeds from the
short sale) and getting the strike price K."

9.1.2

Options on currencies

The put-call parity when currencies are underlying is


CEur (K, T ) + P V (K) = x0 er + PEur (K, T )

(9.7)

In Equation 9.7, the underlying asset is 1 euro. The call holder has the
right, at T , to buy the underlying (i.e. 1 euro) by paying a fixed dollar amount
K. The premium of this call option is CEur (K, T ) dollars. Similarly, the put
holder has the right, at T , to sell the underlying (i.e. 1 euro) for a fixed dollar
amount K. The premium of this put option is PEur (K, T ) dollars. x0 is the
price, in dollars, of buying the underlying (i.e. 1 euro) at time zero. r is the
continuously compounded euro interest rate per year earned by the underlying
(i.e. 1 euro).
To understand the term x0 er in 9.7, notice that to have 1 at T , you need
to have er at time zero. Since the cost of buying 1 at time zero is x0
dollars, the cost of buying er at time zero is x0 er dollars.
Equation 9.7 is very similar to Equation 9.4. If you set S0 = x0 and = r ,
Equation 9.4 becomes 9.7. This shouldnt surprise us. Both S0 and x0 refer to
the price of an underlying asset at time zero. S0 is the dollar price of a stock;
x0 is the dollar price of 1 euro. Both and r measure the continuous rate of
reward earned by an underlying asset.
Tip 9.1.4. How to memorize Equation 9.7. Just memorize Equation 9.4. Next,
set S0 = x0 and = r .
Tip 9.1.5. When applying Equation 9.7, remember that K, CEur (K, T ), PEur (K, T ),
and x0 are in U.S. dollars. To remember this, assume that you are living in the
U.S. (i.e. US dollar is your home currency); that your goal is to either buy or
sell the underlying asset (i.e.1) with a fixed dollar amount. Also remember
that r is the euro interest rate on euro money.
Example 9.1.11. The current exchange rate is 1= 1.33 US dollars. The
dollar-denominated 6-month to expiration $1.2-strike European call option on
one euro has a premium $0.1736. The continuously compounded risk-free interest rate on dollars is 6% per year. The continuously compounded risk-free

12

CHAPTER 9. PARITY AND OTHER OPTION RELATIONSHIPS

interest rate on euros is 4% per year. Calculate the premium for the dollardenominated 6-month to expiration $1.2-strike European put option on one euro.
Solution.
CEur (K, T ) + P V (K) = PEur (K, T ) + S0 eT
K = 1.2
T = 0.5
CEur (K, T ) = $0.1736
r = 0.06
= 0.04
S0 = 1.33
0.1736 + 1.2e0.06(0.5) = PEur (K, T ) + 1.33e0.04(0.5)
PEur (K, T ) = 0.03 447
Example 9.1.12. The current exchange rate is $1 = 0.78. The dollar-denominated
9-month to expiration $1-strike European put option on one euro has a premium
$0.0733. The continuously compounded risk-free interest rate on dollars is 7%
per year. The continuously compounded risk-free interest rate on euros is 3% per
year. Calculate the premium for the dollar-denominated 9-month to expiration
$1-strike European call option on one euro.
CEur (K, T ) + P V (K) = PEur (K, T ) + S0 eT
K=1
T = 0.5
PEur (K, T ) = 0.0733
1
r = 0.07
= 0.03
S0 =
= 1. 282 05
0.78
1 0.03(0.75)
e
CEur (K, T )+e0.07(0.75) = 0.0733+
0.78

9.1.3

CEur (K, T ) = $0.3780

Options on bonds

This is an option contract where the underlying asset is a bond. Other than
having dierent underlying assets, a stock option and a bond option have no
major dierences. You buy a call option on stock if you think the stock price
will go up; you buy a put option if you think the stock price will drop. Similarly,
you buy a call option on a bond if you think that the market interest rate will
go down (hence the price of a bond will go up); you buy a put option on a bond
if you think that the market interest rate will go up (hence the price of the bond
will go down).
The coupon payments in a bond are like discrete dividends in a stock. Using
Equation 9.3, we get the
CEur (K, T ) + P V (K) = PEur (K, T ) + B0 P V (Coupon)

(9.8)

Please note that in Equation 9.8, in the term P V (Coupon), "coupon" refers
to the coupon payments during [0, T ]. In other words, only the coupons made
during the life of the options are used in Equation 9.8. Coupons made after T
are ignored.

9.1. PUT-CALL PARITY

13

Example 9.1.13. A 10-year $1,000 par bond pays 8% annual coupons. The
yield of the bond is equal to the continuously compounded risk-free rate of 6%
per year. A 15-month $1,000-strike European call option on the bond has a
premium $180. Calculate the premium for a 15-month $1,000-strike European
put option on the bond.
Solution.
The annual eective interest rate is i = e0.06 1 = 6. 184%

1 1.0618410
B0 = 1000 (0.08) a10| +1000v 10 = 1000 (0.08)
+1000 1.0618410 =
0.06184
1132. 50
We need to be careful about calculating P V (Coupon). The bond matures
in 10 years. There are 10 annual coupons made at t = 1, 2, ..., 10. However,
since the option expires at T = 15/12 = 1. 25, only the coupon paid at t = 1 is
used in Equation 9.8.
P V (Coupon) = e0.06 (1000) (0.08) = 75. 34
CEur (K, T ) + P V (K) = PEur (K, T ) + B0 P V (Coupon)
180 + 1000e0.06(1.25) = PEur (K, T ) + 1132. 50 75. 34
PEur (K, T ) = 50. 59

9.1.4

Generalized parity and exchange options

General definition of call and put


Most times, the strike price is a constant and we can easily tell whether an
option is a call or a put. For example, an option that gives you the privilege of
buying one Google stock in 1 year for $35 is a call option; an option that gives
you the privilege of selling one Google stock in 1 year for $35 is a put option.
However, occasionally, the strike price is not a constant and its hard to
determine whether an option is a call or put. For example, you purchase an
option that gives you the privilege of receiving one Google stock by surrendering
one Microsoft stock in 1 year. If at T = 1, the price of a Google stock is higher
than that of a Micros0ft stock (i.e. STGoogle >.STMicrosof t ), its advantageous for
you to exercise the option. To exercise the call, you buy one Microsoft stock
from the open market for STM icrosof t , give it to the option writer. In return,
the option writer gives you one Google stock worth STGoogle . Your payo is
STGoogle .STMicrosof t . If, on the other hand, STGoogle .STMicrosof t , you let your
option expires worthless and your payo is zero. Is this option a call or a put?
It turns out that this option can be labeled as either a call or a put. If
you view the Google stock as the underlying asset and the Microsoft stock as
the strike asset, then its a call option. This option gives you the privilege of
buying, at T = 1, one Google stock by paying one Microsoft stock. If you view

14

CHAPTER 9. PARITY AND OTHER OPTION RELATIONSHIPS

the Microsoft stock as the underlying asset and the Google stock as the strike
asset, then its a put option. This option gives you the privilege of selling, at
T = 1, one Microsoft stock for the price of one Google stock.
Definition 9.1.1. An option gives the option holder the privilege, at T , of
surrendering an asset AT and receiving an asset BT (we denote the option as
AT BT ). This is a call option if we view B as the underlying asset and A as
the strike asset (the option holder has the privilege of buying BT by paying AT ).
This is a put option if we view A as the underlying asset and B as the strike
asset (the option holder has the privilege of selling AT and receiving BT ).
Please note that the payo of Option AT BT is max (0, BT AT ).
Example 9.1.14. An option gives the option holder the privilege, at T = 0.25
(i.e. 3 months later), of buying 1 with $1.25. Explain why this option can be
viewed (perhaps annoyingly) as either a call or a put.

Solution.
This option is $1.25 1. The option holder has the privilege, at T = 0.25,
of surrendering $1.25 and receiving 1 (i.e. give $1.25 and get 1).
This is a call option if we view 1 as the underlying asset. The option holder
has the privilege of buying 1 by paying $1.25.
This is also a put option if we view $1.25 as the underlying asset. The option
1
holder has the privilege of selling $1.25 for 1 (i.e. selling $1 for
=
1.25
0.8).
Example 9.1.15. An option gives the option holder the privilege, at T = 0.25,
of buying one Microsoft stock for $35. Explain why this option can be viewed
(perhaps annoyingly) as either a call or a put.
This option is $35 1 M icrosof t stock (give $35 and get 1 Microsoft stock).
If we view the Microsoft stock as the underlying asset, this is a call option. The
option holder has the privilege of buying one Microsoft stock by paying $35.
This is also a put option if we view $35 as the underlying. The option holder
has the privilege of selling $35 for the price of one Microsoft stock.
Example 9.1.16. An option gives the option holder the privilege, at T = 0.25,
of selling one Microsoft stock for $35. Explain why this option can be viewed as
either a call or a put.
This option is 1 M icrosof t stock $35 (give 1 Microsoft stock and get $35).
If we view the Microsoft stock as the underlying asset, this is a put option. The
option holder has the privilege of selling one Microsoft stock for $35. This is
also a call option if we view $35 as the underlying. The option holder has the
privilege of buying $35 by paying one Microsoft stock.

9.1. PUT-CALL PARITY

15

Generalized put and call parity


(AT BT )0 + P V (AT ) = (BT AT )0 + P V (BT )

(9.9)

In Equation 9.9, (AT BT )0 is the premium paid at t = 0 for the privilege


of giving AT and getting BT at T . Similarly, (BT AT )0 is the premium paid
at t = 0 for the privilege of giving BT and getting AT at T . P V (AT ) is the
present value of AT . If you have P V (AT ) at time zero and invest it from time
zero to T , youll have exactly AT . Similarly, if you have P V (BT ) at time zero
and invest it from time zero to T , youll have exactly BT .
This is the proof of Equation 9.9. Suppose we have two portfolios. Portfolio
#1 consists a European option AT BT and the present value of the asset AT .
Portfolio #2 consists a European option BT AT and the present value of the
asset BT .
Payo of Portfolio #1 at T
If AT BT If AT < BT
Option AT BT 0
BT AT
P V (AT )
AT
AT
Total
AT
BT

Payo of Portfolio #2 at T
If AT BT
Option BT AT AT BT
P V (BT )
BT
Total
AT

If AT < BT
0
BT
BT

Portfolio #1 and Portfolio #2 have the identical payo at T . To avoid


arbitrage, these two portfolios must cost us the same to set up at any time prior
to T . The cost of setting up Portfolio #1 is (AT BT )0 + P V (AT ). The cost
of setting up Portfolio #2 is (BT AT )0 +P V (BT ). Hence Equation 9.9 holds.
Example 9.1.17. Stock A currently sells for $30 per share. It doesnt pay
any dividend. Stock B currently sells for $50 per share. It pays dividend at a
continuously compounded rate of 5% per year. The continuously compounded
risk-free interest rate is 6% per year. A European option gives the option holder
the right to surrender one share of Stock B and receive one share of Stock A at
the end of Year 1. This option currently sells for $8.54. Calculate the premium
for another European option that gives the option holder the right to surrender
one Stock A and receive one Stock B at the end of Year 1.
(AT BT )0 + P V (AT ) = (BT AT )0 + P V (BT )
(AT BT )0 + 30 = 8.54 + 50e0.05
(AT BT )0 = 26. 1
Please note that the risk free interest rate 6% is not needed for solving the
problem. In addition, you dont need to decide whether to call the option "give

16

CHAPTER 9. PARITY AND OTHER OPTION RELATIONSHIPS

Stock B and receive Stock A" or "give Stock A and receive Stock B" as a call or
put. Equation 9.9 holds no matter you call the option AT BT or BT AT
a call or put.
Example 9.1.18. Stock A currently sells for $55 per share. It pays dividend of
$1.2 at the end of each quarter. Stock B currently sells for $72 per share. It pays
dividend at a continuously compounded rate of 8% per year. The continuously
compounded risk-free interest rate is 6% per year. A European option gives the
option holder the right to surrender one share of Stock A and receive one share
of Stock B at the end of Year 1. This option currently sells for $27.64. Calculate
the premium for another European option that gives the option holder the right
to surrender one Stock B and receive one Stock A at the end of Year 1.
(AT BT )0 + P V (AT ) = (BT AT )0 + P V (BT )
P V (AT ) = A0 1.2a4|i = 55 1.2a4|i
i is the eective interest rate per quarter.
i = e0.25r 1 = e0.25(0.06) 1 = 1. 511
3%

1 1.01 511 34
P V (AT ) = 55 1.2a4|i = 55 1.2
= 50. 38
0.01 511 3
P V (BT ) = B0 eB T = 72e0.08 = 66. 46
27.64 + 50. 38 = (BT AT )0 + 66. 46
(BT AT )0 = 11. 56
Currency options
Example 9.1.19. Lets go through the textbook example. Suppose that a 1-year
dollar-denominated call option on 1 with the strike price $0.92 is $0.00337.
The current exchange rate is 1 = $0.9. Whats the premium for a 1-year
1
euro-denominated put option on $1 with strike price
= 1. 087?
0.92
First, lets walk through the vocabulary. The phrase "dollar-denominated
option" means that both the strike price and the option premium are expressed
in U.S. dollars. Similarly, the phrase "euro-denominated option" means that
both the strike price and the option premium are expressed in euros.
Next, lets summarize the information using symbols. "dollar-denominated
call option on 1 with the strike price $0.92" is $0.92 1. The premium for
this option is $0.00337. This is represented by ($0.92 1)0 =$0.0337.
1
= 1. 087" can
"Euro-denominated put option on $1 with strike price
0.92

1
1
be represented by $1
. The premium for this option is $1
0.92
0.92 0
Now the solution
be simple.

should

1
1
1
$1
=
$1 0.92
0.92
0.92 0 0.92
0.92
0
1
1
=
($0.92 1)0 =
$0.0337
0.92
0.92

9.1. PUT-CALL PARITY


1
Since the exchange rate is 1 = $0.9 or $1 =
, we have:
0.9

1
1
1
1
=
$0.0337 =
0.0337
= 0.04 07
$1
0.92 0 0.92
0.92
0.9

17

18

CHAPTER 9. PARITY AND OTHER OPTION RELATIONSHIPS


General formula:
1
The current exchange rate is 1 = $x0 or $1=
x0
A dollar-denominated $K strike call on 1 has a premium of $a
m
( $K 1)0 = $a or ( $K 1)$0 = a
1
strike put on $1 has a premium of b
K
m

1
1
=b or $1
=b
$1
K 0
K 0

A euro-denominated

It then follows:

$1

1
K

x0 =

$1

1
K

$
0

1
( $K 1)$0
K

(9.10)

1
is the euro-cost of "give $1, get ." Since
K
0

1
x0 is the dollar cost of "give
the exchange rate is 1 = $x0 , $1
K 0
1
$1, get
." Similarly, ( $K 1)$0 is the dollar-cost of "give $K, get 1."
K
1
1
Equation 9.10 essentially says that the dollar cost of "give $1, get " is
of
K
K
the dollar cost "give $K, get 1." This should make intuitive sense.

1 1
Tip 9.1.6. The textbook gives you the complex formula C$ (x0 , K, T ) = x0 KPf
, ,T .
x0 K
Do not memorize this formula or Equation 9.10. Memorizing complex formulas
is often prone to errors. Just translate options into symbols. Then a simple
solution should emerge. See the next example.
In Equation 9.10,

1
$1
K

9.1. PUT-CALL PARITY

19

Example 9.1.20. The current exchange rate is 0.9 per dollar. A European
euro-denominated call on 1 dollar with a strike price 0.8 and 6 months to
expiration has a premium 0.0892. Calculate the price of a European dollardenominated put option on 1 euro with a strike price $1.25.
Just translate the options into symbols. Then youll see a solution.
The current exchange rate is 0.9 per dollar. $1 =0.9 or 1 = $

1
0.9

Euro-denominated call on 1 dollar with strike price 0.8 has a premium 0.0892
(0.8 $1)0 =0.0892
Calculate the price of a dollar-denominated put on 1 euro with strike price $1.25
(1 $1.25)0 = $?

1
$1 = 1.25 (0.8 $1)0
(1 $1.25)0 = 1.25
1.25
0
1
= 1.25 0.0892 = 1.25 0.08928 $
= $0.124
0.9

9.1.5

Comparing options with respect to style, maturity,


and strike

European vs American options


American options can be exercised at any time up to (and including) the maturity. In contrast, European options can be exercised only at the maturity. Since
we can always convert an American option into a European option by exercising
the American option only at the maturity date, American options are at least
as valuable as an otherwise identical European option.
CAmer (K, T ) CEur (K, T )

(9.11)

PAmer (K, T ) PEur (K, T )

(9.12)

Equation 9.11 and Equation 9.12 are not earth-shaking observations. You
shouldnt have trouble memorizing them.
Maximum and minimum price of a call
1. The price of a call option is always non-negative. CAmer (K, T )
CEur (K, T ) 0. Any option (American or European, call or put) is a
privilege with non-negative payo. The price of a privilege can never be
negative. The worst thing you can do is to throw away the privilege.
2. The price of a call option cant exceed the current stock price.
S0 CAmer (K, T ) CEur (K, T ). The best you can do with a call option
is to own a stock. So a call cant be worth more than the current stock.

20

CHAPTER 9. PARITY AND OTHER OPTION RELATIONSHIPS


3. The price of a European call option must obey the put call
parity. For a non-dividend paying stock, the parity is CEur (K, T ) =
PEur (K, T ) + S0 P V (K) S0 P V (K)
Combining 1, 2, and 3, we have:
For a non-dividend paying stock
S0 CAmer (K, T ) CEur (K, T ) max [0, S0 P V (K)]

(9.13)

For a discrete-dividend paying stock


S0 CAmer (K, T ) CEur (K, T ) max [0, S0 P V (Div) P V (K)] (9.14)
For a continuous-dividend paying stock

S0 CAmer (K, T ) CEur (K, T ) max 0, S0 eT P V (K)

(9.15)

Tip 9.1.7. You dont need to memorize Equation 9.13, 9.14, or 9.15. Just memorize basic ideas behind these formulas and derive the formulas from scratch.
Maximum and minimum price of a put
1. The price of a put option is always non-negative. PAmer (K, T )
PEur (K, T ) 0.
2. The price of a European put option cant exceed the present
value of the strike price. PEur (K, T ) P V (K). The best you can
do with a European put option is to get the strike price K at T . So a
European put cant be worth more than the present value of the strike
price.
3. The price of an American put option cant exceed the strike
price. K CAmer (K, T ). The best you can do with an American put
option is to exercise it immediately after time zero and receive the strike
price K. So an American put cant be worth more than the strike price.
4. The price of a European put option must obey the put call
parity. For a non-dividend paying stock, the parity is PEur (K, T ) =
CEur (K, T ) + P V (K) S0 P V (K) S0
Combining 1, 2, and 3, we have:
K PAmer (K, T ) PEur (K, T ) 0
For a non-dividend paying stock

(9.16)

9.1. PUT-CALL PARITY

P V (K) PEur (K, T ) max [0, P V (K) S0 ]

21

(9.17)

For a discrete-dividend paying stock


P V (K) PEur (K, T ) max [0, P V (K) + P V (Div) S0 ]

(9.18)

For a continuous-dividend paying stock

P V (K) CEur (K, T ) max 0, P V (K) S0 eT

(9.19)

Tip 9.1.8. You dont need to memorize Equation 9.16, 9.17, 9.18, or 9.19. Just
memorize the basic ideas behind these formulas and derive the formulas from
scratch.
Early exercise of American options
Suppose an American option is written at time zero. The option expires in date
T . Todays date is t where 0 t < T . The stock price at the option expiration
date is ST . Todays stock price is St . The continuously compounded risk-free
interest rate is r per year.
Proposition 9.1.1. Its never optimal to exercise an American call early on a
non-dividend paying stock.
If at t you exercise an American call option , your payo is St K.
If at t you sell the remaining
call option, youll
get at least CEur (St , K, T )
max [0, St P V (K)] = max 0, St Ker(T t) , the premium for a European
call option written in t and expiring in T .

Clearly, CEur (St , K, T ) max [0, St P V (K)] = max 0, St Ker(T t) >


St K for r > 0
Its not optimal to exercise an American call option early if you can sell the
remaining call option.
What if you cant sell the remaining call option? If you cant sell the call
option written in t and expiring in T ., you can short sell one stock at t, receiving
St and accumulating to St er(T t) . Then at T , if ST > K, you exercise your call,
paying K and receiving one stock. Next, you return the stock to the broker.
Your total profit is St er(T t) K > St K for a positive r; if at T , ST < K,
you let your call option expire worthless, purchase a stock in the market, and
return it to the broker. Your profit is St er(T t) ST > St er(T t) K > St K
for a positive r.
Intuition behind not exercising American call option early. If you exercise the
call option early at t, you pay the strike price K at t and gain physical possession
of the stock at t. You lose the interest you could have earned during [t, T ] had
you put K in a savings account, yet you gain nothing by physically owning a

22

CHAPTER 9. PARITY AND OTHER OPTION RELATIONSHIPS

stock during [t, T ] since the stock doesnt pay any dividend. In addition, by
exercising the call option t, you throw away the remaining call option during
[t, T ].
Example 9.1.21. You purchase a 30-strike American call option expiring in 6
months on a non-dividend paying stock. 2 months later the stock reaches a high
price of $90. You are 100% sure that the stock will drop to $10 in 4 months.
You are attempted to exercise the call option right now (i.e. at t = 2/12 = 1/6
year) and receive 90 30 = 60 profit. The continuously compounded risk-free
interest rate is 6% per year. Explain why its not optimal to exercise the call
early.
Solution. The problem illustrates the pitfall in common thinking "If you I know
for sure that the stock price is going to fall, shouldnt I exercise the call now and
receive profit right away, rather than wait and let my option expire worthless?"
Suppose indeed the stock price will be $10 at the call expiration date T =
6/12 = 0.5. If you exercise the call early at t = 2/12 = 1/6, youll gain
St K = 90 30 = 60, which will accumulate to 60e0.06(4/12) = 61. 212 at
T = 0.5.
Instead of exercising the call early, you can short-sell the stock at t = 2/12 =
1/6. Then youll receive 90, which will accumulate to 90e0.06(4/12) = 91. 818 at
T = 0.5. Then at T = 0.5, you purchase a stock from the market for 10 and
return it to the brokerage firm where you borrow the stock for short sale. Your
profit is 91. 818 10 = 81. 818, which is the greater than 61. 212 by 81. 818 61.
212 = 20. 606.
Proposition 9.1.2. It might be optimal to exercise an American call option
early for a dividend paying stock.
Suppose the stock pays dividend at tD .
Time 0 ... ... tD ... ... T
Pro and con for exercising the call early at tD .
+. If you exercise the call immediately before tD , youll receive dividend
and earn interest during [tD , T ]
. Youll pay the strike price K at tD , losing interest you could have
earned during [tD , T ]
. You throw away the remaining call option during [tD , T ]. Had you
waited, you would have the call option during [tD , T ]
However, if the accumulated value of the dividend is big enough, then it can
optimal to exercise the stock at tD .
Proposition 9.1.3. If its optimal to exercise an American call early, then the
best time to exercise the call is immediately before the dividend payment.

9.1. PUT-CALL PARITY

23

Here is the proof. Suppose the dividend is paid at tD


Time 0 t1 ... ... tD t2 ... ... T
Its never optimal to exercise an American call at t1 . If you exercise the call
at t1 instead of tD , youll
lose interest that can be earned on K during [t1 , tD ]
lose a call option during [t1 , tD ]
gain nothing (theres no dividend during [t1 , tD ])
Its never optimal to exercise an American call at t2 . If you exercise the call
at t2 instead of tD , youll
gain a tiny interest that can be earned during [tD , t2 ] but lose the dividend
that can be earned if the call is exercised at tD
So for a dividend paying stock, if its ever worthwhile to exercise an American
call early, you should exercise the call immediately before the dividend payment,
no sooner or later.
Combining these two proposition, we have:
Proposition 9.1.4. Its only optimal to exercise an American call option either
at maturity or immediately before a dividend payment date. Any other time is
not optimal.
Proposition 9.1.5. It might be optimal to exercise an American put early.
Time 0 t1 ... ... t ... ... T
The pros and cons of exercising an American put at t instead of T
+. You receive K at t and earn interest during [t, T ]
. You lose the remaining put option during [t, T ]. If you wait and
delay exercising the option, youll have a put option during [t, T ] and can
decide whether to exercise it or discarding it. This is especially painful if
ST > K. If ST > K and you exercise the put at t, youll get K at t, which
accumulates to Ker(T t) at T . If you wait and ST > K, youll let the put
option expire worthless and have ST at T . If ST > Ker(T t) , then you
lose money by exercising the put at t. So one danger of exercising the put
at t is that the stock might be worth more than K after t.
However, if the interest earned on K during [t, T ] is big enough, it can be
optimal to exercise the put at t instead of T .

24

CHAPTER 9. PARITY AND OTHER OPTION RELATIONSHIPS

Appendix 9.A: Parity bound for American options


Appendix 9.5A (and 9.5B) is not on the syllabus. However, I still recommend
that you study it.
Appendix 9.5A has two important ideas
1. Why the put-call parity doesnt hold for American options
2. How to calculate the non-arbitrage boundary price for American options
Why the put-call parity doesnt hold for American options The putcall parity such as Equation 9.2 holds only for European options. It doesnt hold
for American options. To understand why, lets start from European options.
For Equation 9.2 to hold, among other things, the call and the put must be
exercised at the same time. Recall our proof of the put-call parity. At time t
we have two portfolios. Portfolio #1 consists a European call option on a stock
and P V (K), the present value of the strike price K. Portfolio #2 consists a
European put option on the stock and one share of the stock with current price
St . Both the call and put have the same underlying stock, have the same strike
price K, and the same expiration date T . We have found that Portfolio #1 and
Portfolio #2 have an identical payo of max (K, ST ) at the common exercise
date T . To avoid arbitrage, the two portfolios must cost us the same at time
zero. Hence we have Equation 9.2.
Now suppose the call is exercised at T1 and the put is exercised at T2 where
T1 6= T2 . Portfolio #1 consists of a European call option and P V (K) = KerT1 ;
Portfolio #2 consists an American put and one stock worth S0 . Then at
T1 , Portfolio #1 has a payo of max (K, ST1 ); Portfolio #2 has a payo of
max (K, ST2 ). Now the two payos dier in timing and the amount. As a
result, we dont know whether the two portfolios have the same set-up cost.
Now you should understand why Equation 9.2 doesnt hold for American
options. American options can be exercised at any time up to (and including)
the maturity. Even when an American call and an American put have the same
maturity T , the American call can be exercised at T1 where 0 T1 T ; the
American put can be exercised at T1 where 0 T2 T . Hence an American
call and an American put can be exercised at dierent times, they dont follow
the put-call parity.
Non-arbitrage boundary price for American options For a stock that
pays discrete dividend, the key formula is
S0 P V (K) CAme (K, T ) PAme (K, T ) S0 P V (Div) K

(9.20)

Many people find it hard to understand or memorize Equation 9.20. Here is


an intuitive proof without using complex math.

9.1. PUT-CALL PARITY

25

First, we establish the boundary of American options:


CEur (K, T ) + P V (Div) CAme (K, T ) CEur (K, T )
PEur (K, T ) + K P V (K) PAme (K, T ) PEur (K, T )

(9.21)

(9.22)

This is why Equation 9.21 holds. Clearly, CAme (K, T ) CEur (K, T ). This
is because an American call option can always be converted to a European call
option.
To understand why CEur (K, T ) + P V (Div) CAme (K, T ), consider an
American call option and an otherwise identical European call option. Both
call options have the same underlying stock, the same strike price K, the same
expiration date T . The European call option is currently selling for CEur (K, T ).
How much more can the American call option sell for?
The only advantage of an American option over an otherwise identical European call option is that the American call option can be exercised early. The
only good reason for exercising an American call early is to get the dividend.
Consequently, the value of an American call option can exceed the value of an
otherwise identical European call option by no more than the present value of
the dividend. So CEur (K, T ) + P V (Div) CAme (K, T ). A rational person
will pay no more than CEur (K, T ) + P V (Div) to buy the American call option.
So Equation 9.21 holds.
Similarly, an American put is worth at least as much as an otherwise identical
European put.
In addition, the value of an American put exceeds the value of an otherwise
identical European put by no more than K P V (K). The only advantage of
an American put over an otherwise identical European put is that the American
put can be exercised early. The only good reason for exercising an American
put early is to receive the strike price K immediately at time zero (as opposed
to receiving K at T in a European put) and earn the interest on K from time
zero to T . The maximum interest that can be earned on K during [0, T ] is
K P V (K) = K KerT . Consequently, PEur (K, T ) + K P V (K)
PAme (K, T ). Equation 9.22 holds.
Next, we are ready to prove Equation 9.20. CAme (K, T ) PAme (K, T )
reaches its minimum value when CEur (K, T ) reaches it minimum value CEur (K, T )
and PAme (K, T ) reaches its maximum value PEur (K, T ) + K P V (K):
CAme (K, T ) PAme (K, T ) CEur (K, T ) [PEur (K, T ) + K P V (K)]
From the put-call parity, we have:
CEur (K, T ) + P V (K) = PEur (K, T ) + S0 P V (Div)
CEur (K, T ) = PEur (K, T ) + S0 P V (Div) P V (K)
CEur (K, T ) [PEur (K, T ) + K P V (K)]
= PEur (K, T ) + S0 P V (Div) P V (K)

26

CHAPTER 9. PARITY AND OTHER OPTION RELATIONSHIPS


[PEur (K, T ) + K P V (K)]
= S0 P V (Div) K
CAme (K, T ) PAme (K, T ) S0 P V (Div) K

Similarly, CAme (K, T )PAme (K, T ) reaches it maximum value when CAme (K, T )
reaches its maximum value and PAme (K, T ) reaches it minimum value.
CAme (K, T ) PAme (K, T ) CEur (K, T ) + P V (Div) PEur (K, T )
CEur (K, T )+P V (Div)PEur (K, T ) = [PEur (K, T ) + S0 P V (Div) P V (K)]
+P V (Div) PEur (K, T )
= S0 P V (Div)
Hence S0 P V (K) CAme (K, T ) PAme (K, T ) S0 P V (Div) K.
Equation 9.20 holds.
Tip 9.1.9. If you cant memorize Equation 9.20, just memorize Equation 9.21,
Equation 9.22, and Equation 9.3. Then can derive Equation 9.20 on the spot.
Example 9.1.22. If the interest rate is zero. Is it ever optimal to exercise an
American put on a stock?
Solution. According to Equation 9.22, if the risk-free interest rate is zero, then
K = P V (K) and PEur (K, T ) PAme (K, T ) PEur (K, T ). This gives us
PAme (K, T ) = PEur (K, T ). So its never optimal to exercise an American put
early if the interest rate is zero.
Example 9.1.23. Is it ever optimal to exercise an American call on a nondividend paying stock?
Solution. According to Equation 9.21, if Div = 0, then CEur (K, T ) CAme (K, T )
CEur (K, T ) or CAme (K, T ) = CEur (K, T ). Its never optimal to exercise the
American call option.
Example 9.1.24.
An American call on a non-dividend paying stock with
exercise price 20 and maturity in 5 months is worth 1.5. Suppose that the current
stock price is 1.9 and the risk-free continuously compounded interest rate is 10%
per year. Calculate the non-arbitrage boundary price of the American put option
on the stock with strike price 20 and 5 months to maturity.
Solution.
If you can memorize Equation 9.20, then
S0 P V (K) CAme (K, T ) PAme (K, T ) S0 P V (Div) K
19 20e0.1(5/12) 1.5 PAme (K, T ) 19 0 20
0.184 1.5 PAme (K, T ) 1
1 PAme (K, T ) 1.5 0.184

9.1. PUT-CALL PARITY

27

1 + 1.5 PAme (K, T ) 0.184 + 1.5


2. 5 PAme (K, T ) 1. 684
If you cant memorize Equation 9.20, this is how to solve the problem using
basic reasoning.
The American put is worth at least the otherwise identical European put.
PAme (K, T ) PEur (K, T )
Using the put-call parity: PEur (K, T ) = CEur (K, T ) + P V (K) S0
Since the stock doesnt pay any dividend, CAme (K, T ) = CEur (K, T ) = 1.5
PEur (K, T ) = CEur (K, T ) + P V (K) S0
= 1.5 + 20e0.1(5/12) 19 = 1. 684
So PAme (K, T ) PEur (K, T ) = 1.684
The value of an American put can exceed the value of an otherwise identical
European put by no more the early-exercise value. Since the only possible reason
to exercise an American put early is to receive K and earn interest K P V (K)
during [0, T ], so the maximum early-exercise value is
K P V (K) = 20 20e0.1(5/12) = 0.816
PAme (K, T ) PEur (K, T ) + K P V (K) = 1.684 + 0.816 = 2. 5
Time to expiration
American option An American option (call or put) has more time to expiration is at least as valuable as an otherwise identical American option with less
time to expiration. If options are on the same stock and T1 > T2 , we have:
CAmer (K, T1 ) > CEur (K, T2 )
PAmer (K, T1 ) > PEur (K, T2 )

(9.23)
(9.24)

European option A European call on a non-dividend paying stock will be at


least as valuable as an otherwise identical European call option with a shorter
time to expiration. This is because for a non-dividend paying stock, an European
call option is worth the same as an otherwise identical American call option.
And an American call option with a longer time to expiration is more valuable
than an otherwise identical American option with a shorter time to expiration.
If both options are on the same non-dividend paying stock and T1 > T2 , we
have:
CEur (K, T1 ) > CEur (K, T2 )
(9.25)
If the stock pays dividend, then a longer-lived European option may be less
valuable than an otherwise identical but shorter-lived European option. The
textbook gives two good examples.
In Example #1, a stock is valuable only because of its dividend. The stock
pays a dividend at the end of Week 2. Once the dividend is paid, the stock dies

28

CHAPTER 9. PARITY AND OTHER OPTION RELATIONSHIPS

and is worth nothing. In this case, if T > 2 weeks, CEur (K, T ) = 0. If if T 2


weeks, CEur (K, T ) might be worth something depending on how high the strike
price K is.
In Example #2, a put is written on the asset of a bankrupt company. The
asset of the bankrupt company is a constant c. The put is worth P V (c) = cerT .
If the risk-free interest rate r > 0, cerT will decrease if time to expiration T
increases.

European options when the strike price grows over time Typically,
a call or put has a fixed strike price K. However, theres nothing to prevent
someone from inventing a European option whose strike price changes over time.
Consider a European option whose strike price grows with the risk-free interest rate. That is, KT = KerT . What can we say about the price of such a
European option?
For a European option with strike price KT = KerT , a longer-lived European
option is at least as valuable as an otherwise identical but shorter-lived European
option.

CEur KerT1 , T1 CEur KerT2 , T2 if T1 > T2

PEur KerT1 , T1 PEur KerT2 , T2 if T1 > T2

(9.26)
(9.27)

This is why Equation 9.26 holds. Suppose at time zero we buy two European
calls on the same stock. The first call expires at T1 and has a strike price KerT1 .
The second call expires at T2 and has a strike price KerT2 , where T1 > T2 .
Lets choose a common time T1 and compare the payos of these two calls
at T1 . The payo of the longer-lived call is max 0, ST1 KerT1 .
The payo of the shorter-lived call is calculated as follows. First, we calculate
its payo at T2 . Next, we accumulate this payo from
T2 to T1 . rT
The payo of the shorter-lived
call
at
T
is
max
0, ST2 Ke 2 . Next, we
2

rT2
accumulate this payo max 0, ST2 Ke
from T2 to T1 .
As well soon see, its much harder for a short-lived call to have a positive
payo at T1 .
The longer-lived call will have a positive payo at ST1 KerT1 at T1 if
ST1 > KerT1 ; all else, the payo is zero.
On the other hand, the shorter-lived call will have a positive payo ST1
KerT1 at T1 only if the following two conditions are met
ST2 > KerT2
ST1 > KerT1

9.1. PUT-CALL PARITY

29

If ST2 KerT2 , the shorter-lived call will expire worthless, leading to zero
payo at T2 , which accumulates to zero payo at T1 .
If ST2 > KerT2 , well receive a positive payo ST2 KerT2 at T2 . If we
accumulate ST2 KerT2 from T2 to T1 , ST2 will accumulate to ST1 and KerT2
to KerT2 er(T1 T2 ) = KerT1 , leading to a total amount ST1 KerT1 at T1 . The
total payo amount ST1 KerT1 is positive if ST1 > KerT1 .
In summary, both calls can reach the common positive payo ST1 KerT1 at
T1 . The longer-lived call will reach this payo if ST1 > KerT1 . The shorter-lived
call will reach this payo if both ST2 > KerT2 and ST1 > KerT1 . Consequently,
the long-lived call has a better payo and should be at least as valuable as the
shorter-lived call. Hence Equation 9.26 holds.
If you still have trouble understanding why the longer-lived call has a richer
payo, you can draw the following payo table:
The accumulated payo of the shorter-lived call at T1
If ST1 KerT1
If ST1 > KerT1
rT2
If ST2 Ke
P ayof f = 0
P ayof f = 0
If ST2 > KerT2 P ayof f = ST1 KerT1 0 P ayof f = ST1 KerT1 > 0
The payo of the longer-lived call at T1
If ST1 KerT1 If ST1 > KerT1
rT2
If ST2 Ke
P ayof f = 0
P ayof f = ST1 KerT1 > 0
rT2
If ST2 > Ke
P ayof f = 0
P ayof f = ST1 KerT1 > 0
You can see that the longer-lived call has a slightly better payo than the
shorter-lived payo. To avoid arbitrage, the longer-lived call cant sell for less
than the shorter-lived call. Hence Equation 9.26 holds. Similarly, you can prove
Equation 9.27.
Dierent strike price
Proposition 9.1.6. A call (European or American) with a low strike price
is at least as valuable as an otherwise identical call with a higher strike price.
However, the excess premium shouldnt exceed the excess strike price.
0 C (K1 , T ) C (K2 , T ) K2 K1 if K1 < K2

(9.28)

Equation 9.28 should make intuitive sense. The lower-strike call allows the
call holder to buy the underlying asset by the guaranteed lower strike price.
Clearly, the payo of a lower-strike call can never be less than the payo of
an otherwise identical but higher-strike call. Consequently, 0 C (K1 , T )
C (K2 , T ). And this is why C (K1 , T ) C (K2 , T ) K2 K1 . The only advantage of a K1 -strike call over the K2 -strike call is that the guaranteed purchase
price of the underlying asset is K2 K1 lower in the K1 -strike call; to buy the
asset, the K1 -strike call holder can pay K1 yet the K2 -strike call holder will pay

30

CHAPTER 9. PARITY AND OTHER OPTION RELATIONSHIPS

K2 . Consequently, no rational person will pay more than C (K2 , T ) + K2 K1


to buy the K1 -strike call.
Proposition 9.1.7. A European call with a low strike price is at least as valuable as an otherwise identical call with a higher strike price. However, the excess
premium shouldnt exceed the present value of the excess strike price.

0 CEur (K1 , T ) CEur (K2 , T ) P V (K2 K1 ) if K1 < K2

(9.29)

The only advantage of a K1 -strike European call over the K2 -strike European
call is that the guaranteed purchase price of the underlying asset is K2 K1
lower in the K1 -strike call at T . Consequently, no rational person will pay more
than CEur (K2 , T ) + P V (K2 K1 ) to buy the K1 -strike call.
Please note that CEur (K1 , T )CEur (K2 , T ) P V (K2 K1 ) doesnt apply
to American call options because two American options can be exercised at
dierent dates.
Proposition 9.1.8. A put (European or American) with a higher strike price
is at least as valuable as an otherwise identical put with a lower strike price.
However, the excess premium shouldnt exceed the excess strike price.

0 P (K2 , T ) P (K1 , T ) K2 K1 if K1 < K2

(9.30)

Clearly, a higher strike put is at least as valuable as an otherwise identical put


with a lower strike price. Since the only advantage of a K2 -strike put over the
K1 -strike put is that the guaranteed sales price of the underlying asset is K2 K1
high in the K2 -strike put, no rational person will pay C (K1 , T ) + K2 K1 to
buy the K2 -strike put.
Proposition 9.1.9. A European put with a higher strike price is at least as
valuable as an otherwise identical put with a lower strike price. However, the
excess premium shouldnt exceed the present value of the excess strike price.

0 PEur (K2 , T ) PEur (K1 , T ) P V (K2 K1 ) if K1 < K2

(9.31)

Please note that PEur (K2 , T ) PEur (K1 , T ) P V (K2 K1 ) wont apply
to two American put options because they can be exercised at two dierent
dates.
Proposition 9.1.10. A diversified option portfolio is at least as valuable as one
undiversified option. For K1 < K2 and 0 < < 1

9.1. PUT-CALL PARITY

31

C [K1 + (1 ) K2 ] C (K1 ) + (1 ) C (K2 )

(9.32)

P [K1 + (1 ) K2 ] P (K1 ) + (1 ) P (K2 )

(9.33)

Please note that Equation 9.32 and Equation 9.33 apply to both European
options and American options.
A call portfolio consists of portion of K1 -strike call and (1 ) portion
of K2 -strike call. The premium of this portfolio, C (K1 ) + (1 ) C (K2 ),
can be no less than the premium of a single call with a strike price K1 +
(1 ) K2 . Similarly, a call portfolio consists of portion of K1 -strike put and
(1 ) portion of K2 -strike put. The premium of this portfolio, P (K1 ) +
(1 ) P (K2 ), can be no less than the premium of a single call with a strike
price K1 + (1 ) K2 .
Before proving Equation 9.32, lets look at an example.
Example 9.1.25. (Textbook example 9.8 K2 and K3 switched) K1 = 50, K2 =
65, K3 = 0.4 (50) + 0.6 (65) = 59. C (K1 , T ) = 14, C (K2 , T ) = 5. Explain why
C (59) 0.4C (50) + 0.6C (65).
Payo
59-strike call (a)

ST < 50
0

50 ST < 59
0

59 ST < 65
ST 59

ST 65
ST 59

50-strike call (b)


65-strike call (c)
0.4b + 0.6c

0
0
0

ST 50
0
0.4 (ST 50)

ST 50
0
0.4 (ST 50)

ST 50
ST 65
ST 591

(0.4b + 0.6c) a

0.4 (ST 50) 0

0.6 (65 ST ) > 02

The above table says the following:


If we buy 0.4 unit of 50-strike call, buy 0.6 unit of 65-strike call, and sell
1 unit of 59-strike call, our initial cost is (0.4b + 0.6c) a and our payo at
T is always non-negative. To avoid arbitrage, the position of always having a
non-negative payo at expiration T surely has a non-negative cost at t = 0.
Imagine what happens otherwise. For example, you always have a non-negative
payo at T and it costs you $10 (i.e. you receive $10) to set up this position
at t = 0. Then youll make at least $10 free money.
Hence we have (0.4b + 0.6c) a 0 no matter what ST is.
Clearly, the call portfolio consisting of 40% 50-strike call and 60% 65-strike
call is at least as good as the 59-strike call. Consequently, the portfolio is at
least as valuable as the 59-strike call. C (59) 0.4C (50) + 0.6C (65) .
1 0.4 (S
T
2 0.4 (S
T

50) + 0.6 (ST 65) = ST 59


50) (ST 59) = 0.6 (65 ST )

32

CHAPTER 9. PARITY AND OTHER OPTION RELATIONSHIPS

Example 9.1.26. Prove Equation 9.32.


Let K3 = K1 + (1 ) K2 . Clearly, K1 < K3 < K2
Payo
ST < K1 K1 ST < K3
K3 ST < K2
K3 -strike call (a)
0
0
ST K3

ST K2
ST K3

K1 -strike call (b)


K2 -strike call (c)
b + (1 ) c

0
0
0

ST K1
0
(ST K1 )

ST K1
0
(ST K1 )

ST K1
ST K2
ST K3

b + (1 ) c a

(ST K1 ) 0

(1 ) (K2 ST ) > 0

Please note
(ST K1 ) + (1 ) (ST K2 ) = ST [K1 + (1 ) K2 ] = ST K3
In addition,
(ST K1 ) (ST K3 ) = K3 K1 (1 ) ST
= K1 + (1 ) K2 K1 (1 ) ST = (1 ) K2 (1 ) ST
= (1 ) (K2 ST )
If we buy unit of K1 -strike call, buy (1 ) unit of K2 call, and sell 1 unit
of K3 = K1 + (1 ) K2 strike call, well have a non-negative payo at T . To
avoid arbitrage, the initial cost must be non-negative. Hence b+(1 ) ca >
0.
Anyway, the call portfolio consisting of portion of K1 -strike call and 1
portion of K2 -strike call is at least as good as the call with the strike price
K3 = K1 + (1 ) K2 . To avoid arbitrage, C [K1 + (1 ) K2 ] C (K1 ) +
(1 ) C (K2 ).
Example 9.1.27. (Textbook example 9.9 K2 and K3 switched) K1 = 50, K2 =
70, K3 = 0.75 (50) + 0.25 (70) = 55. P (K1 , T ) = 4, P (K2 , T ) = 16. Explain
why P (55) 0.75P (50) + 0.25P (70).
Payo
55-strike put (a)

ST < 50
55 ST

50-strike put (b)


70-strike put (c)
0.75b + 0.25c

50 ST
70 ST
55 ST

0
70 ST
0.25 (70 ST )

0
70 ST
0.25 (70 ST )

0
0
0

0.75 (ST 50) > 0

0.25 (70 ST ) > 03

(0.75b + 0.25c) a

50 ST < 55
55 ST

Please note
0.75 (50 ST ) + 0.25 (70 ST ) = 55 ST
0.25 (70 ST ) (55 ST ) = 0.75 (ST 50)
3 0.4 (S
T

50) (ST 59) = 0.6 (65 ST )

55 ST < 70
0

ST 70
0

9.1. PUT-CALL PARITY

33

(0.75b + 0.25c) a 0 no matter what ST is. Clearly, the put portfolio


consisting of 75% 50-strike put and 25% 70-strike put is at least as good as
the 55-strike put. Consequently, to avoid arbitrage, the portfolio is at least as
valuable as the 55-strike put. P (55) 0.75P (50) + 0.25P (70).
Example 9.1.28. Prove Equation 9.33.
Let K3 = K1 + (1 ) K2 . Clearly, K1 < K3 < K2
Payo
ST < K1 K1 ST < K3
K3 ST < K2
K3 -strike put (a) K3 ST
K3 ST
0

ST K2
0

K1 -strike put (b)


K2 -strike put (c)
b + (1 ) c

K1 ST
K2 ST
K3 ST

0
K2 ST
(1 ) (K2 ST )

0
K2 ST
(K2 ST )

0
0
ST K3

b + (1 ) c a

(ST K1 ) 0

(K2 ST ) > 0

Please note
(K1 ST ) + (1 ) (K2 ST ) = K3 ST
In addition,
(1 ) (K2 ST ) (K3 ST )
= (1 ) (K2 ST ) K1 (1 ) K2 + ST = (ST K1 )
(ST K1 ) (ST K3 ) = K3 K1 (1 ) ST
= K1 + (1 ) K2 K1 (1 ) ST = (1 ) K2 (1 ) ST
= (1 ) (K2 ST )
The payo is always non-negative. Consequently, the call portfolio consisting
of portion of K1 -strike put and 1 portion of K2 -strike put is at least as
good as the put with the strike price K3 = K1 +(1 ) K2 . To avoid arbitrage,
P [K1 + (1 ) K2 ] P (K1 ) + (1 ) P (K2 ).
Exercise and moneyness
Proposition 9.1.11. If its optimal to exercise an option, its also optimal to
exercise an otherwise identical option thats more in-the-money.
This is just common sense. The textbook gives an example. Suppose its
optimal to exercise a 50-strike American call on a dividend paying stock. The
current stock price is 70. If its optimal to exercise the American a 50-strike
American call, then it must also be optimal to exercise an otherwise identical
call but with a strike price 40.

34

CHAPTER 9. PARITY AND OTHER OPTION RELATIONSHIPS

Chapter 10

Binomial option pricing: I


This chapter is one of the easiest chapters in Derivatives Markets. The textbook
did a good job explaining the mechanics of how to calculate the option price in
a one-period binomial model. Besides learning the mechanics of option pricing,
you should focus on understanding two basic ideas: the non-arbitrage pricing
and the risk-neutral probabilities.

10.1

One-period binomial model: simple examples

Example 10.1.1. Suppose we want to find the price of a 12-month European


call option on a stock with strike price $15. The stock currently sells for $20. In
12 months, the stock can either go up to $30 or go down to $10. The continuously
compounded risk-free interest rate per year is 10%.
30

15

20
Time 0

?
10
T =1

Time 0

Stock price binomial tree

0
T =1

Call payo binomial tree

Its hard to directly calculate the price of the call option. So lets build
something that behaves like a call, something that has the same payo pattern
as the call. Suppose at time zero we create a portfolio by buying X stocks and
putting Y dollars in a savings account. We want this portfolio to have the exact
payo as the call.
Y e0.1

30X
20X
Time 0

+
10X
T =1

Y
Time 0

15
=

Y e0.1
T =1
35

?
Time 0

0
T =1

36

CHAPTER 10. BINOMIAL OPTION PRICING: I

In the above diagram, 2X stocks at time zero are worth either 30X or 10X
at T = 1. Putting Y dollars in a savings account at time zero will produce Y e0.1
at T = 1.
We want our portfolio to behave like a call. So the payo of our portfolio
should
as the payo of the call. We set up the following equation:
the same 0.1
30X + Y e = 15
X = 0.75
Y = 10 (0.75) e0.1 = 6. 786
10X + Y e0.1 = 0
Y = 6. 786 means that we borrow 6. 786 at t = 0 and pays 6. 786e0.1 = 7.
5 at T = 1
If at t = 0 we buy 0.75 share of a stock and borrow $6. 786, then at T = 1,
this portfolio will have the same payo as the call. To avoid arbitrage, the
portfolio and the call should have the same cost at t = 0.
C = 20X + Y = 20 (0.75) 6. 786 = 8. 214
Example 10.1.2. Find the price of a 12-month European put option on a stock
with strike price $15. The stock currently sells for $20. In 12 months, the
stock can either go up to $30 or go down to $10. The continuously compounded
risk-free interest rate per year is 10%.
Suppose at time zero we create a replicating portfolio by buying X stocks
and investing Y dollars in a savings account. We want this portfolio to have the
exact payo as the put.
Y e0.1

30X
20X

Time 0

10X
T =1

30X + Y e0.1 = 0
10X + Y e0.1 = 5

0
=

Time 0

Y e0.1
T =1

X = 0.25

?
Time 0

5
T =1

Y = 6. 786

So the replicating portfolio is at t = 0 by short-selling 0.25 stock and investing 6. 786 in a savings account.
The price of the put at t = 0 is:
P = 20X + Y = 20 (0.25) + 6. 786 = 1. 786
Lets check whether the put-call parity holds.
C + P V (K) = 8. 214 + 15e0.1 = 21. 787
P + S0 = 1. 786 + 20 = 21. 786
Ignoring the rounding dierence, we get: C + P V (K) = P + S0

10.2

General one-period binomial model

Suppose we have two points in time, t = 0 (today) and t = h (some point in


the future). The continuously compounded risk-free interest rate per year is r
( a positive constant). We have two assets: a stock that pays zero dividend and

10.2. GENERAL ONE-PERIOD BINOMIAL MODEL

37

a savings account. The savings account is the same as a zero-coupon bond. At


time h, the stock price is Sh ; the bond price is Bh .
The bond price is deterministic:
B0 = 1
Bh = erh
The stock price isstochastic:
Su
S0 = S
Sh =
Sd
So at h the stock price either goes up to Su ("up state") or goes down to Sd
("down state").
Su
S
Time 0

Sd
Time h

We assume theres no tax, no transaction cost, or margin requirements; one


is allowed to short sell any security and receive full proceeds. In addition, we
assume that anyone can buy or sell any number of securities without aecting
the market price.
We assume that the market is an invisible cop automatically enforcing Sd <
Serh < Su . For example, if Su > Sd > Serh , then the stocks return is guaranteed to be higher than the risk-free interest rate. If this is the case, then the
risk-free interest rate was set too low and stocks return is too good. Everyone
will jump on this opportunity, withdraw all his money from his savings account,
and invest it in the stock. This will instantaneously bid up the price of the stock
and the risk-free interest rate, forcing Sd < Serh < Su to hold.
Whats the non-arbitrage condition is the stock pays dividend continuously
at the yield ?
Suppose you buy eh share of a stock at t = 0 (thus you pay Seh ) , buy
reinvesting dividend in the stock, youll have exactly one eh eh = 1 stock at
time h, which is worth either Su or Sd .
Su
Seh
Time 0

Sd
Time h

Suppose
you
invest Seh in a savings account, then your wealth at h is
h
rh
simply Se
e = Se(r)h .
Se(r)h
Seh
Se(r)h
Time 0 Time h
To avoid arbitrage, the following condition needs to be met:

38

CHAPTER 10. BINOMIAL OPTION PRICING: I


At good times (i.e. when the stock goes up), the return you earn from the
stock should exceed the risk free interest rate. So Su > Se(r)h
At bad times (i.e. when the stock goes down), the return you earn from
the stock should be less than the risk free interest rate. So Sd < Se(r)h

If the condition is not met, well end up in a weird situation where the stock
is always better o than the savings account or the savings account is always
better o than the stock. Then everyone will invest his money in the better
performing asset, instantly bidding up the price of the lower-performing asset
and forcing the above condition to be met.
To avoid arbitrage, Su , Sd , and r need to satisfy the following condition:
Su > Se(r)h > Sd

(10.1)

If Su = uS and Sd = dS, then Equation 10.1 becomes:


u > e(r)h > d

(10.2)

Equation 10.2 is the textbook Equation 10.4.


Lets continue.
Let C represent the option price at time zero. Let Cu and Cd represent the
payo at time h of an option in the up state and down state respectively:
Cu
C
Time 0

Cd
Time h

Our task is to determine C by setting a portfolio that replicates the option


payo. We build the replicating portfolio by buying 4 shares of the stock and
investing $B in a zero-coupon bond. So we set up the following equation:
4Su
Berh
Cu
4S
+ B
= C
4Sd
Berh
Cd
t=0 t=h
t=0 t=h
t=0 t=h

4Su + Berh = Cu
4Sd + Berh = Cd

Solving these equations, we get:


4=

Cu Cd
Su Sd

(10.3)

10.2. GENERAL ONE-PERIOD BINOMIAL MODEL

B = erh

Su Cd Sd Cu
Su Sd

39

(10.4)

Rearranging Equation 10.3, we get 4Su Cu = 4Sd Cd . This equation


is critical as well soon see.
The value of the portfolio at the up state is
4Su + Berh = 4Su + erh (Cu 4Su ) erh = Cu
The value of the portfolio at the down state is
4Sd + Berh = 4Sd + erh (Cu 4Sd ) erh = Cd
Using Equation 10.3 and Equation 10.4, we get (verify this for yourself):
rh

Su Serh
Se Sd
rh
Cu +
Cd
(10.5)
C = 4S + B = e
Su Sd
Su Sd

Define

Serh Sd
Su Sd
rh
S
u Se
d = q =
Su Sd

u = p =

(10.6)
(10.7)

Because market automatically enforces Sd < Serh < Su , we have


0 < u < 1
0 < p < 1
u + d = 1

(10.8)
(10.9)
(10.10)

According to Equation 10.8 , we can pretend that u and d are probabilities.


Then Equation 10.5 becomes
C = erh ( u Cu + d Cd )

(10.11)

Doing some algebra, we also get (verify this for yourself):

S=e

rh

Su Serh
Serh Sd
Su +
Sd
Su Sd
Su Sd

= erh ( u Su + d Sd )

(10.12)

According to Equation 10.11 and 10.12, once we set up faked probabilities


u and d , the call price at t = 0 is simply the expected present value of the

40

CHAPTER 10. BINOMIAL OPTION PRICING: I

call payos discounted at the risk-free interest rate; the stock price at t = 0 is
simply the expected present value of the future stock prices discounted at the
risk-free interest rate. Since the discounting rate is risk-free interest rate, u
and d are called risk neutral probabilities.
Please note that u and d are not real probabilities. They are artificially
created probabilities so that Equation 10.11 and 10.12 have simple and intuitive
explanations.
Example 10.2.1. Using the risk-neutral probabilities, find the price of a 12month European call option on a stock with strike price $15. The stock currently
sells for $20. In 12 months, the stock can either go up to $30 or go down to
$10. The continuously compounded risk-free interest rate per year is 10%.
Solution.
Stock price tree
Time 0

T
Su = 30

Option terminal payo


Time 0

S = 20

T
Cu = max (0, 30 15) = 15

C =?
Sd = 10
Cd = max (0, 10 15) = 0
Serh Sd
20e0.1(1) 10
u =
=
= 0.605
Su Sd
30 10
d = 1 0.605 = 0.395
Cu = 30 15 = 15
Cd = 0
C = erh ( u Cu + d Cd ) = e0.1(1) (0.605 15 + 0.395 0) = 8. 211
Example 10.2.2. Using the risk-neutral probabilities, find the price of a 12month European put option on a stock with strike price $15. The stock currently
sells for $20. In 12 months, the stock can either go up to $30 or go down to
$10. The continuously compounded risk-free interest rate per year is 10%.
Stock price tree
Time 0

T
Su = 30

S = 20

Option terminal payo


Time 0
C =?

Sd = 10

T
Cu = max (0, 15 30) = 0
Cd = max (0, 15 10) = 5

Serh Sd
20e0.1(1) 10
=
= 0.605
Su Sd
30 10
d = 1 0.605 = 0.395

u =

Cu = 0
Cd = 5
C = erh ( u Cu + d Cd ) = e0.1(1) (0.605 0 + 0.395 5) = 1. 787

10.2. GENERAL ONE-PERIOD BINOMIAL MODEL

41

Suppose the stock pays dividends at a continuously compounded rate per


year. At time zero, our replicating portfolio consists of 4 shares of stocks and
$B in a bond (or a savings account). If we continuously reinvest dividends and
buy additional stocks during [0, h], our 4 shares at time zero will grow into
4eh shares at time h. Our 4eh shares will be worth either 4eh Su in the up
state or 4eh Sd in the down state. We want our replicating portfolio to have
the same payo as the option at time h.

4eh Su + Berh = Cu
4eh Sd + Berh = Cd
Solving the equations, we get:
Cu Cd
Su Sd

(10.13)

Su Cd Sd Cu
Su Sd

(10.14)

4 = eh
B = erh

Notice whether the stock pays dividend or not, at time zero, we always need
Su Cd Sd Cu
Su Cd Sd Cu
in a savings account, which grows into
to have erh
Su Sd
Su Sd
Cu Cd
dollars at t = h. If the stock doesnt pay dividend, at t = 0 we hold
Su Sd
Cu Cd
shares of stock, which is
shares of stock at t = h ; if the stock pays
Su Sd
Cu Cd
dividend at a continuously compounded rate , at t = 0 we hold eh
,
Su Sd
Cu Cd
which grows
shares of stock at t = h.
Su Sd
Cu Cd
Su Cd Sd Cu
So at time h we need to have
units of stocks and
Su Sd
Su Sd
dollars in a savings account (or a bond), regardless of whether the stock pays
dividend or not.
To see why, suppose our replicating portfolio at t = h (not t = 0) consists
of U shares of stocks and V dollars in a savings account. Then regardless of
whether the stock pays dividend or not, we need to have:

U Su + V = Cu
U Sd + V = Cd

This gives us:


Cu Cd
U=
Su Sd

V =

Su Cd Sd Cu
Su Sd

To have V dollars in a savings account at t = h, we need to have V erh =


Cu Cd
Su Cd Sd Cu
erh
at t = 0. To have U =
shares of stocks at t = h,
Su Sd
Su Sd

42

CHAPTER 10. BINOMIAL OPTION PRICING: I

if the stock pays dividend at a continuously compounded rate of , we need to


Cu Cd
have U eh = eh
shares of stocks at t = 0. If we use the dividends
Su Sd
received to buy additional stocks, then well have U shares of stock at t = h.
Now lets find the cost of the option on a stock that pays dividends at a
continuously compounded rate per year. The option cost at time zero is
Cu Cd
Su Cd Sd Cu
4S + B = eh
S + erh
S

S
Su Sd
d
u (r)h

Se
S

S
Se(r)h
d
u
rh
=e
Cu +
Cd
Su Sd
Su Sd
C = 4S + B = erh (u Cu + d Cd )

(10.15)

where

u =

Se(r)h Sd
Su Sd

(10.16)

d =

Su Se(r)h
Su Sd

(10.17)

If Su = Su and Sd = Sd, then Equation 10.16 and 10.17 becomes:


u =

e(r)h d
ud

(10.18)

d =

u e(r)h
ud

(10.19)

Tip 10.2.1. If you dont want to memorize Equation 10.12, 10.15, 10.16, 10.17,
just set up the replication portfolio and calculate 4 and B from scratch.
Example 10.2.3. Find the price of a 12-month European call option on a stock
with strike price $15. The stock pays dividends at a continuously compounded
rate 6% per year. The stock currently sells for $20. In 12 months, the stock can
either go up to $30 or go down to $10. The continuously compounded risk-free
interest rate per year is 10%.
Stock price tree
Time 0

T
Su = 30

S = 20

Option terminal payo


Time 0
C =?

Sd = 10

T
Cu = max (0, 30 15) = 15
Cd = max (0, 10 15) = 0

10.2. GENERAL ONE-PERIOD BINOMIAL MODEL


Replicating portfolio
Time 0
(4, B) = (0.706 32, 6. 786 28)

43

T
(4u , Bu ) = (0.75, 7. 50)

(4d , Bd ) = (0.75, 7. 50)


Our replicating portfolio at t = 0 consists of 4 shares of stocks and $B in a
savings account.
Cu Cd
15 0
4 = eh
= e0.06(1)
= 0.706 32
Su Sd
30 10
Su Cd Sd Cu
30 (0) 10 (15)
= e0.1(1)
B = erh
= 6. 786 28
Su Sd
30 10
C = 4S + B = 0.706 3 (20) 6. 786 3 = 7. 34
The replicating portfolio at T is:
4u = 4d = 4eh = 0.706 32e0.06(1) = 0.75
Bu = Bd = Berh = 6. 786 28e0.1(1) = 7. 5
Verify that the replicating portfolio and the option have the same value:
The value of the replicating portfolio at the up state:
4u Su + Bu = 0.75 (30) 7. 50 = 15 = Cu
The value of the replicating portfolio at the down state:
4d Sd + Bd = 0.75 (10) 7. 50 = 0 = Cd
The value of the portfolio at t = 0 :
C = 4S + B = 0.706 32 (20) 6. 78628 = 7. 34
Alternatively,
Se(r)h Sd
20e(0.10.06)1 10
u =
=
= 0.540 81
Su Sd
30 10
d = 1 u = 1 0.540 81 = 0.459 19
C = erh (u Cu + d Cd ) = e0.1(1) (0.540 81 15 + 0.459 19 0) = 7. 34
Example 10.2.4. Find the price of a 12-month European put option on a stock
with strike price $15. The stock pays dividends at a continuously compounded
rate 6% per year. The stock currently sells for $20. In 12 months, the stock can
either go up to $30 or go down to $10. The continuously compounded risk-free
interest rate per year is 10%.
Solution.
Stock price tree
Time 0

T
Su = 30

S = 20

Option terminal payo


Time 0
C =?

Sd = 10

T
Cu = max (0, 15 30) = 0
Cd = max (0, 15 10) = 5

44

CHAPTER 10. BINOMIAL OPTION PRICING: I


Replicating portfolio
Time 0

T
(4u , Bu ) = (0.25, 7. 50)

(4, B) = (0.235 4, 6. 786 3)


(4d , Bd ) = (0.25, 7. 50)
Our replicating portfolio at t = 0 consists of 4 shares of stocks and $B in a
savings account.
Cu Cd
05
4 = eh
= e0.06(1)
= 0.25e0.06(1) = 0.235 4
Su Sd
30 10
Su Cd Sd Cu
30 (5) 10 (0)
= e0.1(1)
= 6. 786 3
B = erh
Su Sd
30 10
C = 4S + B = 0.235 4 (20) + 6. 786 3 = 2. 078
The replicating portfolio at T is:
4u = 4d = 4eh = 0.235 4e0.06(1) = 0.25
Bu = Bd = Berh = 6. 786 3e0.1(1) = 7. 5
Verify that the replicating portfolio and the option have the same value:
The value of the replicating portfolio at the up state:
4u Su + Bu = 0.25 (30) + 7. 50 = 0 = Cu
The value of the replicating portfolio at the down state:
4d Sd + Bd = 0.25 (10) + 7. 50 = 5 = Cd
The value of the portfolio at t = 0 :
C = 4S + B = 0.235 4 (20) + 6. 786 3 = 2. 078
Alternatively,
Se(r)h Sd
20e(0.10.06)1 10
=
= 0.540 8
u =
Su Sd
30 10
d = 1 u = 1 0.540 8 = 0.459 2
C = erh ( u Cu + d Cd ) = e0.1(1) (0.540 8 0 + 0.459 2 5) = 2. 078
Arbitrage a mispriced option
If an option sells for more or less than the price indicated by Equation 10.15,
we can make money by "buy low, sell high."
Example 10.2.5. A 12-month European call option on a stock has strike price
$15. The stock pays dividends at a continuously compounded rate 6% per year.
The stock currently sells for $20. In 12 months, the stock can either go up to
$30 or go down to $10. The continuously compounded risk-free interest rate per
year is 10%. This call currently sells for $8. Design an arbitrage strategy.

Solution.

Stock price tree

Option terminal payo

10.2. GENERAL ONE-PERIOD BINOMIAL MODEL


Time 0

T
Su = 30

S = 20

Time 0
C =?

Sd = 10
Replicating portfolio
Time 0
(4, B) = (0.706 32, 6. 786 28)

45

T
Cu = max (0, 30 15) = 15
Cd = max (0, 10 15) = 0
T
(4u , Bu ) = (0.75, 7. 50)
(4d , Bd ) = (0.75, 7. 50)

There are two calls. One is in the market selling for $8 at t = 0. The other is
a synthetic call, which consists, at t = 0, of holding 0.706 3 stock and borrowing
$6. 786 3 at risk-free interest rate. The synthetic call sells for $7. 34 at t = 0.
These two calls have identical payos at t = 1.
To make a riskless profit, we buy low and sell high. At t = 0, we sell a
call for $8 (sell high). Then at t = 1, if the stock price is $30, the call holder
exercises the call and our payo is 15 30 = 15; if the stock is $10, the call
expires worthless and our payo is zero.

15

0
Time 0 Time t = 1
Payo of a written call
Simultaneously, at t = 0 we buy 0.706 3 stock and borrow 6. 786 3 dollars
at risk-free interest rate (buy low). This costs us 0.706 3 (20) 6. 786 3 = 7.
34 at t = 0. At t = 1, our initial 0.706 3 stock becomes 0.706 3e0.06(1) stock
and our initial debt 6. 786 3 grows into 6. 786 3e0.1(1) . Our portfolio is worth
0.706 3e0.06(1) S1 6. 786 3e0.1(1) .
If S1 = 30, our portfolio is worth
0.706 3e0.06(1) (30) 6. 786 3e0.1(1) = 15
If S1 = 10, our portfolio is worth
0.706 3e0.06(1) (10) 6. 786 3e0.1(1) = 0
15
7.34
Time 0

0
Time t = 1

Payo of a replicating portfolio


So at t = 0, we gain 8 7. 34 = 0.66. At t = 1, the portfolio exactly osets
our payo in the call. We earn 0.66 sure profit at t = 0.

46

CHAPTER 10. BINOMIAL OPTION PRICING: I

Example 10.2.6. A 12-month European call option on a stock has strike price
$15. The stock pays dividends at a continuously compounded rate 6% per year.
The stock currently sells for $20. In 12 months, the stock can either go up to
$30 or go down to $10. The continuously compounded risk-free interest rate per
year is 10%. This call currently sells for $7. Design an arbitrage strategy.
Stock price tree
Time 0

Option terminal payo

T
Su = 30

S = 20

Time 0
C =?

Sd = 10
Replicating portfolio
Time 0
(4, B) = (0.706 32, 6. 786 28)

T
Cu = max (0, 30 15) = 15
Cd = max (0, 10 15) = 0
T
(4u , Bu ) = (0.75, 7. 50)
(4d , Bd ) = (0.75, 7. 50)

There are two calls. One is in the market selling for $7 at t = 0. The other is
a synthetic call, which consists, at t = 0, of holding 0.706 3 stock and borrowing
6. 786 3 dollars at risk-free interest rate. The synthetic call sells for $7. 34 at
t = 0. These two calls have identical payos at t = 1.
To make a riskless profit, we buy low and sell high. At t = 0, we buy a
call for $7 (buy low). Then at t = 1, if the stock price is $30, the call holder
exercises the call and our payo is 30 15 = 15; if the stock is $10, the call
expires worthless and our payo is zero.
15
7

0
Time 0 Time t = 1
Payo of a purchased call
Simultaneously, at t = 0 we sell the replicating portfolio. We short sell
0.706 3 stock and lend 6. 786 3 dollars at risk-free interest rate (sell high). We
gain 0.706 3 (20) 6. 786 3 = 7. 34 at t = 0.
At t = 1, our initially borrowed 0.706 3 stock becomes 0.706 3e0.06(1) stock
and our lent principal 6. 786 3 grows into 6. 786 3e0.1(1) . Our portfolio is worth
6. 786 3e0.1(1) 0.706 3e0.06(1) S1 .
If S1 = 30, our portfolio is worth
6. 786 3e0.1(1) 0.706 3e0.06(1) (30) = 15
If S1 = 10, our portfolio is worth

10.2. GENERAL ONE-PERIOD BINOMIAL MODEL

47

6. 786 3e0.1(1) 0.706 3e0.06(1) (10) = 0


7.34

15

Time 0

0
Time t = 1

Payo of a replicating portfolio


So at t = 0, we gain7.34 7 = 0.34. At t = 1, the call payo exactly osets
our liabilities in the replicating portfolio We earn 0.34 sure profit at t = 0.
Tip 10.2.2. An option and its replicating portfolio are exactly the same in terms
of payo and cost. If an option in the market sells for more than the fair price
indicated in Equation 10.15, we can make a sure profit by buying the replicating
portfolio and selling the option. If an option in the market sells for less than
the fair price indicated in Equation 10.15, we can make a sure profit by selling
the replicating portfolio and buying the option.
Risk neutral probability and forward price
If we use u and d to calculate the undiscounted stock price, we get:
Se(r)h Sd
Su Se(r)h
Su +
Sd
Su Sd
Su Sd
(r)h
= Se
= F0,h

u Su + d Sd =

u Su + d Sd = Se(r)h = F0,h

(10.20)

F0,h is the delivery price at t = h of a forward contract signed at t = 0.


The textbook uses the symbol Ft,t+h to indicate that the forward contract is
signed at time t and the asset is to be delivered at t + h. If we treat the contract
initiation date t as time zero, then the asset deliver date is time h. So the
notation doesnt matter. If you want to use the textbook notation, youll have
u Su + d Sd = Se(r)h = Ft,t+h

(10.21)

If you want to use my notation, youll get Equation 10.20.


Suppose you enter into a forward contract as a seller agreeing to deliver one
stock for a guaranteed price F0,h at t = h. To ensure you indeed can deliver
one stock at time h, youll want to buy eh stock at time zero. If you reinvest
dividend and buy additional stocks, your initial eh stock will grow into exactly
one stock at time h. Your cost of buying eh stock at time zero is Seh . Since
youll tie up your money Seh during [0, h], youll want the forward price to
include the interest you could otherwise earn on Seh . So the forward price is
just the future value of Seh :
F0,h = Seh erh = Se(r)h

48

CHAPTER 10. BINOMIAL OPTION PRICING: I

According to Equation 10.20, the undiscounted stock price equals the forward price under the risk neutral probability. If a problem gives you a forward
price, you can use Equation 10.20 to calculate the risk-neutral probability.
Constructing a binomial tree
Suppose we are standing at time t. If we can be 100% certain about the stock
price at time t + h, then investing in stocks doesnt have any risk. Then stocks
must earn a risk-free interest rate. Since the stock already pays dividends at
rate , to earn a risk free interest rate, the stock price just needs to grow at
the rate of r . Hence the stock price at t + h is Se(r)h , which is just the
forward price Ft,t+h because Ft,t+h is also equal to Se(r)h .
However, the stock price is a random variable and we generally cant be
100% certain about a stocks future price. To incorporate uncertainty, we use
Formula 11.16 in Derivatives Markets:
ln

St+h
= (r ) h h
St

(Textbook 11.16)

St+h
is the continuously compounded rate of return
St
during [t, t+ h]. This return consists of a known element (r ) h and a random
element h, where is the annualized standard deviation of the continuously
compounded stock return. The variance of the stock return in one year is 2 .
The variance during the interval [t, t + h] (which is h year long) is 2 h and this
is why. [t, t + h] can be broken down into h intervals, with each interval being
one year long. Assume stock return during each year is independent identically
distributed. The total return during [t, t + h] is the sum of the returns over h
intervals. Then the total variance is just the sum of the variance over h intervals,
2 h.
In the above equation, ln

e h . In the
binomial
So St+h = St e(r)h h = Se(r)h h = Ft,t+h

(r)h+ h
h
model, the stock price
either goes up to Se
= Ft,t+h e
or goes

down to Se(r)h h = Ft,t+h e h . So we have

uSt = Ft,t+h e

dSt = Ft,t+h e

(10.22)
(10.23)

If we set volatility to zero, then Equation 10.22 and 10.23 becomes uSt =
dSt = Ft,t+h . This means that if the stock price is 100% certain, then the stock
price is just the forward price.
Apply Equation 10.21 to Equation 10.22 and 10.23, we get:

10.2. GENERAL ONE-PERIOD BINOMIAL MODEL

u = e(r)h+
d = e(r)h

10.2.1

49

(10.24)
(10.25)

Two or more binomial trees

Example 10.2.7. Lets reproduce Derivatives Markets Figure 10.4. Here is the
recap of the information on a European call option. The current stock price is 41.
The strike price K = 40. The annualized standard deviation of the continuously
compounded stock return is = 30%. The continuously compounded risk-free
rate per year is r = 8%. The continuously compounded dividend rate per year
is = 0%.The option expiration date is T = 2 years. Use a 2-period binomial
tree to calculate the option premium.
T
Each period is h =
= 1 year long.
2
Step 1
Draw a stock price tree.

u = e(r)h+ h = e(0.080)1+0.31 = 1. 462 28


d = e(r)h h = e(0.080)10.3 1 = 0.802 52
Stock price
Period 0
1
2
Period 0
1
Su2
Su
Su = 59.9537
=
S
Sud
S = 41
Sd
Sd = 32.9033
2
Sd

2
Suu = 87.6693
Sud = 48.1139
Sdd = 26.4055

Step 2
Draw the terminal payos at T = 2.
47.6693 = max (0, 87.6693 40)
8.1139 = max (0, 48.1139 40)
Option Payo
Period 0
1
Cuu

2
= 47.6693

Cu
C =?

Cud = 8.1139
Cd
Cdd = 0

Step 3
Work backward from right to left (called backwardization). Calculate the premium using the formula
C = erh (u Cu + d Cd )
e(r)h d
e(0.080)1 0.802 52
u =
=
= 0.425 56
ud
1. 462 28 0.802 52

50

CHAPTER 10. BINOMIAL OPTION PRICING: I


d = 1 u = 1 0.42556 = 0.574 44
Option premium
Period 0

2
Cuu = 47.6693

Cu = 23.0290
C = 10.7369

Cud = 8.1139
Cd = 3.1875
Cdd = 0

Cu = erh ( u Cuu + d Cud )


= e0.08(1) (0.425 56 47.6693 + 0.574 44 8.1139) = 23. 029 0
Cd = erh ( u Cud + d Cud )
= e0.08(1) (0.425 56 8.1139 + 0.574 44 0) = 3. 187 5
The call premium is
C = erh ( u Cu + d Cd )
= e0.08(1) (0.425 56 23.0290 + 0.574 44 3.1875) = 10.7369

10.2. GENERAL ONE-PERIOD BINOMIAL MODEL

51

Step 4

Calculate 4 and B using the formula


Cd
Su Cd Sd Cu
4=e
B = erh
Su Sd
Su Sd
To avoid errors, put the stock price tree and the premium tree side by side:
h Cu

Period 0

2
Suu = 87.6693

Period 0

Sud = 48.1139

C = 10.7369

Su = 59.9537
S = 41

1
Cu = 23.0290

Sd = 32.9033

Cud = 8.1139
Cd = 3.1875

Sdd = 26.4055
Period 0

(4, B) = (0.7335, 19.3367)

Cdd = 0
1

(4u , Bu ) = (1.0, 36.9247)


(4d , Bd ) = (0.3738, 9.1107)

Cuu Cud
47.6693 8.1139
= e0(1)
= 1.0
Suu Sdd
87.6693 48.1139
Cud Cdd
8.1139 0
4d = eh
= e0(1)
= 0.373 8
Sud Sdd
48.1139 26.4055
Cu Cd
23.0290 3.1875
4 = eh
= e0(1)
= 0.733 5
Su Sd
59.9537 32.9033
4u = eh

Suu Cud Sud Cuu


Suu Sud
0.08(1) 87.6693 (8.1139) 48.1139 (47.6693)
=e
= 36.9247
87.6693 48.1139

Bu = erh

36.9247 means that $36.9247 needs to be borrowed at a risk-free rate.


Sdu Cdd Sdd Cdu
Sdu Sdd
0.08(1) 48.1139 (0) 26.4055 (8.1139)
=e
= 9. 110 7
48.1139 26.4055
Su Cd Sd Cu
B = erh
Su Sd
0.08(1) 59.9537 (3.1875) 32.9033 (23.0290)
=e
= 19. 3367
59.9537 32.9033
Bd = erh

2
Cuu = 47.6693

52

CHAPTER 10. BINOMIAL OPTION PRICING: I

Step 5
Verify that the portfolio replicates the premium tree. Here is
recap of the information:
Period 0
1
2
0
1
2
Suu = 87.6693
Cuu = 47.6693
Su = 59.9537
Cu = 23.0290
S = 41
Sud = 48.1139 C = 10.7369
Cud = 8.1139
Sd = 32.9033
Cd = 3.1875
Sdd = 26.4055
Cdd = 0
Period 0

(4, B) = (0.7335, 19.3367)

1
(4u , Bu ) = (1.0, 36.9247)
(4d , Bd ) = (0.3738, 9.1107)

The value of (4, B) at time zero:


4S + B = 0.7335 (41) 19.3367 = 10. 736 8 = C
The value of(4u , Bu ) at Node u (up state):
4u Su + Bu = 1.0 (59.9537) 36.9247 = 23. 029 = Cu
The value of(4d , Bd ) at Node d (down state):
4d Sd + Bd = 0.3738 (32.9033) 9.1107 = 3. 188 = Cd
Finally, lets verify that the portfolio replicates the terminal payo. First,
we need to find the replicating portfolio at the expiration date.
Period 1
2
(4uu , Buu ) = (1.0, 40)
(4u , Bu ) = (1.0, 36.9247)
(4ud , Bud ) = (1.0, 40) = (0.3738, 9. 869 5)
(4d , Bd ) = (0.3738, 9.1107)
(4dd , Bdd ) = (0.3738, 9. 869 5)
If we reinvest dividends, 4u stocks grows into 4u eh after h years; Bu grows
into Bu erh after h years.

(4uu , Buu ) = 4u eh , Bu erh = 1.0e0(1) , 36.9247e0.08(1) = (1.0, 40)


4uu Suu + Buu = 1.0 (87.6693)
40 = 47.
669
3 = Cuu

Similarly, (4dd , Bdd ) = 4d eh , Bd erh = 0.3738e0(1) , 9.1107e0.08(1) =


(0.373 8, 9. 869 5)
4dd Sdd + Bdd = 0.3738 (26.4055) 9. 869 5 = 0.00 = Cdd
Finally, (4ud , Bud ) = (1.0, 40) = (0.3738, 9. 869 5)
Portfolios (1.0, 40) and (0.3738, 9. 869 5) have equal values at Node ud.
1.0 (48.1139) 40 = 8. 113 9 = Cud
0.3738 (48.1139) 9. 869 5 = 8. 115 = Cud (ignore rounding dierence)

10.2. GENERAL ONE-PERIOD BINOMIAL MODEL

53

Tip 10.2.3. For a multi-binomial tree, using the risk neutral probability to find
the premium is faster than using the replicating portfolio. The risk neutral probabilities u and d are constant cross nodes. However, the replicating portfolio
(4, B) varies by node.
Tip 10.2.4. For European options, you can calculate the premium using the
terminal payos. This is how to quickly find the premium for this problem.
Node at T = 2
uu
ud
dd
Total

Payo at T
Cuu = 47.6693
Cud = 8.1139
Cdd = 0

Risk neutral probability of reaching this node1


2u = 0.425 562 = 0.181 1
2 u d = 2 (0.425 56) (0.574 44) = 0.488 9 2
2d = 0.574 442 = 0.3300
2
(u + d ) = 0.181 1 + 0.488 9 + 0.3300 = 1 3

The premium is the expected present value of the terminal payo using the
risk neutral probability.

C = erT 2u Cuu + 2 u d Cud + 2d Cdd

= e0.08(2) 0.425 562 47.6693 + 0.488 9 8.1139 + 0.33 0 = 10. 736 9


Tip 10.2.5. If you purchased the textbook Derivatives Markets, you should see
a CD attached to the back cover of the book. Install the CD in your computer.
Run the spreadsheet titled "optall2" or "optbasic2." These two spreadsheets can
calculate European and American option prices. When you solve a practice problem, you can use either of these two spreadsheets to double check you answer.
Please note that these two spreadsheets dont calculate the replicating portfolio (4, B). So you cant use them to verify your calculation of the replicating
portfolio.
Example 10.2.8. Lets reproduce Derivatives Markets Figure 10.5. Here is
the recap of the information on a European call. The current stock price is 41.
The strike price K = 40. The annualized standard deviation of the continuously
compounded stock return is = 30%. The continuously compounded risk-free
rate per year is r = 8%. The continuously compounded dividend rate per year
is = 0%.The option expiration date is T = 1 year. Use a 3-period binomial
tree to calculate the option premium.
Solution.
Each period is h =
u = e(r)h+

1
year long.
3

= e(0.080)1/3+0.3 1/3 = 1. 221 246

probabilities in this column are the 3 terms of ( u + d )2 = 2d + 2 d u + 2u


are two ways of reaching Node ud: up and dow or down and up.
3 The total probability is one. Otherwise, you made an error.
1 The

2 There

54

CHAPTER 10. BINOMIAL OPTION PRICING: I

d = e(r)h h = e(0.080)1/30.3
Stock price
Period 0
1
2

1/3

= 0.863 693
3
Su3 = 74.6781

Su2 = 61.1491
Su2 d = 52.8140

Su = 50.0711
S = 41

Sud = 43.2460
Sd2 u = 37.3513

Sd = 35.4114
2

Sd = 30.5846
Sd3 = 26.4157
Calculate the premium by working backward from right to left.
Period 0

2
Cu2 = 22. 201 6

Cu = 12. 889 5
C = 7. 073 9

Cud = 5. 699 5
Cd = 2. 535 1
Cd2 = 0

rh

3
Cu3 = max (0, 74.6781 40) = 34. 678 1
Cu2 d = max (0, 52.8140 40) = 12. 814
Cd2 u = max (0, 37.3513 40) = 0
Cd3 = max (0, 26.4157 40) = 0

C=e
( u Cu + d Cd )
e(r)h d
e(0.080)1/3 0.863 693
u =
=
= 0.456 806
ud
1. 221 246 0.863 693
d = 1 u = 1 0.456 806 = 0.543 194

Cu2 = erh ( u Cu3 + d Cu2 d )


= e0.08(1/3) (0.456 806 34. 678 1 + 0.543 194 12. 814) = 22. 201 7
Cud = erh ( u Cu2 d + d Cd2 u )
= e0.08(1/3) (0.456 806 12. 814 + 0.543 194 0) = 5. 699 5
Cd2 = erh (Cd2 u + d Cd3 ) = 0
Cu = erh ( u Cu2 + d Cud )
= e0.08(1/3) (0.456 806 22. 201 6 + 0.543 194 5. 699 5) = 12. 889 5
Cd = erh ( u Cud + d Cd2 )
= e0.08(1/3) (0.456 806 5. 699 5 + 0.543 194 0) = 2. 535 1
C = erh (u Cu + d Cd )
= e0.08(1/3) (0.456 806 12. 889 5 + 0.543 194 2. 535 1) = 7. 073 9

10.2. GENERAL ONE-PERIOD BINOMIAL MODEL

55

Now Im going to tell you a calculator shortcut I used when I was preparing
for the old Course 6. The above calculations are intense and prone to errors.
To quickly and accurately calculate the premium at each node, use TI-30X IIS
calculator because TI-30X IIS allows you to modify formulas easily.
For example, to calculate Cu2 = erh ( u Cu3 + d Cu2 d ), enter
e(0.08/3)(0.456 806 34. 678 1 + 0.543 194 12. 814
Please note for TI-30 IIS, the above expression is the same as
e(0.08/3)(0.456 806 34. 678 1 + 0.543 194 12. 814)
In other words, you can omit the ending parenthesis ")". I tell you this
because occasionally people emailed me saying they discovered a typo. This is
not a typo.
Now you have entered the formula
e(0.08/3)(0.456 806 34. 678 1 + 0.543 194 12. 814.
Press "=" and you should get: 22.20164368
Next, to calculate Cud = erh (ud Cu2 d + d Cd2 u ), you dont need to enter
a brand new formula. Just reuse the formula
e(0.08/3) (0.456 806 34. 678 1 + 0.543 194 12. 814)
Change 34. 678 1 to 12. 8140 (so 0.456 806 34. 678 1 becomes 0.456 806 12.
8140).
Change 12. 814 into 00.000 (so 0.543 19412. 814 becomes 0.543 19400.000).
Now the modified formula is
e(0.08/3) (0.456 806 12. 8140 + 0.543 194 00.000)
Press "=" and you should get: 5.6994813
To calculate Cu = erh (u Cu2 + d Cud ), once again reuse a previous formula. Change the formula
e(0.08/3) (0.456 806 12. 8140 + 0.543 194 00.000)
into
e(0.08/3) (0.456 806 22. 201 6 + 0.543 194 05. 699 5)
Press "=" and you should get:12.8894166
Reusing formulas avoids the need to retype erh , u , and d (these three
terms are constant across all nodes) and increases your speed and accuracy. By
reusing formulas, you should quickly find C = 7. 073 9.

56

CHAPTER 10. BINOMIAL OPTION PRICING: I


Next, we calculate the replicating portfolio.
Cu Cd
Su Cd Sd Cu
4 = eh
B = erh
Su Sd
Su Sd
To avoid errors, put the stock price tree and the premium tree side by side:
Period 0

3
Su3 = 74.6781

Su2 = 61.1491
Su2 d = 52.8140

Su = 50.0711
S = 41

Sud = 43.2460
Sd2 u = 37.3513

Sd = 35.4114
2

Sd = 30.5846
Sd3 = 26.4157
Period 0

3
Cu3 = 34. 678 1

Cu2 = 22. 201 6


Cu = 12. 889 5
C = 7. 073 9

Cu2 d = 12. 814


Cud = 5. 699 5

Cd = 2. 535 1

Cd2 u = 0
Cd2 = 0
Cd3 = 0

Period 0

(4, B) = (0.706 3, 21. 885 2)

(4, B)u = (0.921 8, 33. 263 6)


(4, B)d = (0.450 1, 13. 405 2)

2
(4, B)u2 = (1, 38. 947 4)
(4, B)ud = (0.828 7, 30. 138 6)
(4, B)d2 = (0, 0)

Cu3 Cu2 d
34. 678 1 12. 814
= e0(1/3)
=1
Su3 Su2 d
74.6781 52.8140
Su3 Cu2 d Su2 dCu2 d
74.6781 (12. 814) 52.8140 (34. 678 1)
= e0.08(1/3)
=
Bu2 = erh
Su3 Su2 d
74.6781 52.8140
38. 947 4
4u2 = eh

Cu2 d Cd2 u
12. 814 0
= e0(1/3)
= 0.828 7
2
2
Su d Sd u
52.8140 37.3513
2
2
Su dCd2 u Sd uCu2 d
52.8140 (0) 37.3513 (12. 814)
= e0.08(1/3)
=
Bud = erh
2
2
Su d Sd u
52.8140 37.3513
30. 138 6
4ud = eh

4d2 = eh

Cd2 u Cd3
=0
Su2 d Sd3

10.2. GENERAL ONE-PERIOD BINOMIAL MODEL


Bd2 = erh

57

Su2 dCd3 Sd3 Cd2 u


=0
Su2 d Sd3

Cu2 Cud
22. 201 6 5. 699 5
= e0(1/3)
= 0.921 8
Su2 Sud
61.1491 43.2460
Su2 Cud SudCu2
61.1491 (5. 699 5) 43.2460 (22. 201 6)
Bu = erh
= e0.08(1/3)
=
2
Su Sud
61.1491 43.2460
33. 263 6
4u = eh

Cud Cd2
5. 699 5 0
= e0(1/3)
= 0.450 1
Sud Sd2
43.2460 30.5846
SudCd2 Sd2 Cud
43.2460 (0) 30.5846 (5. 699 5)
= e0.08(1/3)
=
Bd = eh
2
Sud Sd
43.2460 30.5846
13. 405 2
4d = eh

Cu Cd
12. 889 5 2. 535 1
= e0(1/3)
= 0.706 3
Su Sd
50.0711 35.4114
Cu Cd
50.0711 (2. 535 1) 35.4114 (12. 889 5)
B = erh
= e0.08(1/3)
=
Su Sd
50.0711 35.4114
21. 885 2
4 = eh

If our goal is to calculate the premium without worrying about the replicating
portfolio, then we just need to know the risk neutral probability and the terminal
payo.
Node
u3
u2 d
ud2
d3

Payo
Cu3 = 34. 678 1
Cu2 d = 12. 814
Cd2 u = 0
Cd3 = 0

Risk neutral prob of reaching this node4


3
3u= 0.456 806

2
2
3 u d = 3 0.456 806 0.543 194
3 u 2d = 3 (0.456 806) 0.543 1942
3d = 0.543 1943

The option premium is just the expected present value of the terminal payos
using the riskneutral probability.

C = erT Cu3 3u + Cu2 d 3 2u d + Cd2 u 3u 2d + Cd3 3d

= e0.08(1) 34. 678 1 0.456 8063 + 12. 814 3 0.456 8062 0.543 194
= 7. 073 9

Example 10.2.9. Lets reproduce Derivatives Markets Figure 10.6. Here is


the recap of the information on a European put. The current stock price is 41.
The strike price K = 40. The annualized standard deviation of the continuously
compounded stock return is = 30%. The continuously compounded risk-free
rate per year is r = 8%. The continuously compounded dividend rate per year
is = 0%.The option expiration date is T = 1 year. Use a 3-period binomial
tree to calculate the option premium.
4 The probabilities of this column is just the four terms in ( + )3 = 3 + 3 2 +
u
d
d u
u
3 2d u + 3d = 1.

58

CHAPTER 10. BINOMIAL OPTION PRICING: I

Solution.

Each period is h =

1
year long.
3

u = e(r)h+ h = e(0.080)1/3+0.31/3 = 1. 221 246


d = e(r)h h = e(0.080)1/30.3 1/3 = 0.863 693
Stock price
Period 0
1
2
3
Su3 = 74.6781
Su2 = 61.1491
Su = 50.0711
Su2 d = 52.8140
S = 41
Sud = 43.2460
Sd = 35.4114
Sd2 u = 37.3513
2
Sd = 30.5846
Sd3 = 26.4157
Calculate the premium by working backward from right to left.
Period 0

2
Cu2 = 0

Cu = 0.740 9
C = 2. 998 5

Cud = 1. 400 9
Cd = 5. 046 2
Cd2 = 8. 362 9

3
Cu3 = max (0, 40 74.6781) = 0
Cu2 d = max (0, 40 52.8140) = 0
Cd2 u = max (0, 40 37.3513) = 2. 648 7
Cd3 = max (0, 40 26.4157) = 13. 584 3

C = erh ( u Cu + d Cd )
e(r)h d
e(0.080)1/3 0.863 693
u =
=
= 0.456 806
ud
1. 221 246 0.863 693
d = 1 u = 1 0.456 806 = 0.543 194
0

Cu2 = erh ( u Cu3 + d Cu2 d ) = e0.08(1/3) (0.456 806 0 + 0.543 194 0) =

Cud = erh ( u Cu2 d + d Cd2 u ) = e0.08(1/3) (0.456 806 0 + 0.543 194 2. 648 7) =
1. 400 9
Cd2 = erh (u Cd2 u + d Cd3 ) = e0.08(1/3) (0.456 806 2. 648 7 + 0.543 194 13. 584 3) =
8. 362 9
Cu = erh ( u Cu2 + d Cud ) = e0.08(1/3) (0.456 806 0 + 0.543 194 1. 400 9) =
0.740 9
Cd = erh ( u Cud + d Cd2 ) = e0.08(1/3) (0.456 806 1. 400 9 + 0.543 194 8. 362 9) =
5. 046 2

10.2. GENERAL ONE-PERIOD BINOMIAL MODEL

59

C = erh (u Cu + d Cd ) = e0.08(1/3) (0.456 806 0.740 9 + 0.543 194 5. 046 2) =


2. 998 5
Next, we calculate the replicating portfolio.
Cu Cd
Su Cd Sd Cu
B = erh
4 = eh
Su Sd
Su Sd
Period 0

2
(4, B)u2 = (0, 0)

(4, B)u = (0.07 82, 4. 658 9)


(4, B) = (0.293 7, 15. 039 5)

(4, B)ud = (0.171 3, 8. 808 8)


(4, B)d = (0.549 9, 24. 517 3)
(4, B)d2 = (1, 38. 947 4)

Cu3 Cu2 d
00
= e0(1/3)
=0
Su3 Su2 d
74.6781 52.8140
Su3 Cu2 d Su2 dCu2 d
74.6781 (0) 52.8140 (0)
= erh
= e0.08(1/3)
=0
Su3 Su2 d
74.6781 52.8140

4u2 = eh
Bu2

Cu2 d Cd2 u
0 2. 648 7
= e0(1/3)
= 0.171 3
2
2
Su d Sd u
52.8140 37.3513
2
2
Su dCd2 u Sd uCu2 d
52.8140 (2. 648 7) 37.3513 (0)
= erh
= e0.08(1/3)
=
2
2
Su d Sd u
52.8140 37.3513

4ud = eh
Bud
8. 808 8

Cd2 u Cd3
2. 648 7 13. 584 3
= e0(1/3)
= 1.0
Su2 d Sd3
37.3513 26.4157
2
3
Su dCd3 Sd Cd2 u
37.3513 (13. 584 3) 26.4157 (2. 648 7)
= e0.08(1/3)
=
Bd2 = erh
Su2 d Sd3
37.3513 26.4157
38. 947 4
C 2 Cud
0 1. 400 9
= e0(1/3)
= 0.07 82
4u = eh u2
Su Sud
61.1491 43.2460
2
Su Cud SudCu2
61.1491 (1. 400 9) 43.2460 (0)
= e0.08(1/3)
=
Bu = erh
2
Su Sud
61.1491 43.2460
4. 659
4d2 = eh

Cud Cd2
1. 400 9 8. 362 9
= e0(1/3)
= 0.549 9
Sud Sd2
43.2460 30.5846
SudCd2 Sd2 Cud
43.2460 (8. 362 9) 30.5846 (1. 400 9)
= e0.08(1/3)
=
Bd = eh
Sud Sd2
43.2460 30.5846
24. 517 3
4d = eh

Cu Cd
0.740 9 5. 046 2
= e0(1/3)
= 0.293 7
Su Sd
50.0711 35.4114
Cu Cd
50.0711 (5. 046 2) 35.4114 (0.740 9)
= e0.08(1/3)
= 15.
B = erh
Su Sd
50.0711 35.4114
039 5
4 = eh

60

CHAPTER 10. BINOMIAL OPTION PRICING: I

If our goal is to calculate the premium without worrying about the replicating
portfolio, then we just need to know the risk neutral probability and the terminal
payo.
Node
u3
u2 d
ud2
d3

Payo
Cu3 = 0
Cu2 d = 0
Cd2 u = 2. 648 7
Cd3 = 13. 584 3

Risk neutral prob of reaching this node5


3
3u= 0.456 806

2
2
3u d = 3 0.456 806 0.543 194
3 u 2d = 3 (0.456 806) 0.543 1942
3d = 0.543 1943

The option premium is just the expected present value of the terminal payos
using the riskneutral probability.

C = erT Cd2 u 3 u 2d + Cd3 3d

= e0.08(1) 2. 648 7 3 0.456 806 0.543 1942 + 13. 584 3 0.543 1943 =
2. 998 5

Example 10.2.10. Lets reproduce Derivatives Markets Figure 10.7. Here is


the recap of the information on an American put. The current stock price is 41.
The strike price K = 40. The annualized standard deviation of the continuously
compounded stock return is = 30%. The continuously compounded risk-free
rate per year is r = 8%. The continuously compounded dividend rate per year
is = 0%.The option expiration date is T = 1 year. Use a 3-period binomial
tree to calculate the option premium.
Solution.
Each period is h =

1
year long.
3

u = e(r)h+ h = e(0.080)1/3+0.31/3 = 1. 221 246


d = e(r)h h = e(0.080)1/30.3 1/3 = 0.863 693
Step 1
Find the stock price tree
Period 0
1
2
3
3
Su = 74.6781
Su2 = 61.1491
Su = 50.0711
Su2 d = 52.8140
S = 41
Sud = 43.2460
Sd = 35.4114
Sd2 u = 37.3513
2
Sd = 30.5846
Sd3 = 26.4157
5 The probabilities of this column is just the four terms in ( + )3 = 3 + 3 2 +
u
d
d u
u
3 2d u + 3d = 1.

10.2. GENERAL ONE-PERIOD BINOMIAL MODEL


Step 2
Period 0

61

Calculate the payo at expiration (terminal payo)


1

3
Vu3 = max (0, 40 74.6781) = 0

Vu2
Vu
V

Vu2 d = max (0, 40 52.8140) = 0

Vud
Vd
Vd2

Vd2 u = max (0, 40 37.3513) = 2. 648 7


Vd3 = max (0, 40 26.4157) = 13. 584 3

Step 3
Calculate the value of the American put one step left to the
expiration.
An American put can be exercised immediately. The value of an American
option if exercised immediately is called the exercise value (EV ) or intrinsic
value. At Period 2, we compare the value calculated using backwardization and
the exercise value. We take the greater of the two as the value of the American
put.

The backwardized values at Period 2 are:


Cu2 = erh (u Cu3 + d Cu2 d ) = e0.08(1/3) (0.456 806 0 + 0.543 194 0) =

Cud = erh (u Cu2 d + d Cd2 u ) = e0.08(1/3) (0.456 806 0 + 0.543 194 2. 648 7) =
1. 400 9
Cd2 = erh ( u Cd2 u + d Cd3 ) = e0.08(1/3) (0.456 806 2. 648 7 + 0.543 194 13. 584 3) =
8. 362 9
The exercise values at Period 2 are:

EVu2 = max 0, K Su2 = max (0, 40 61.1491) = 0


EVud = max (0, K Sud)
= max (0, 40 43.2460) = 0
EVd2 = max 0, K Sd2 = max (0, 40 30.5846) = 9. 415 4
We take the greater of the two as the value of the American put.
Vu2 = max (Cu2 , EVu2 ) = max (0, 0) = 0
Vud = max (Cud , EVud ) = max (1. 400 9, 0) = 1. 400 9
Vd2 = max (Cd2 , EVd2 ) = max (8. 362 9, 9. 415 4) = 9. 415 4
Now we have:
Period 0
1

3
Vu3 = 0

Vu2 = 0
Vu
V

Vu2 d = 0
Vud = 1. 400 9

Vd

Vd2 u = 2. 648 7
Vd2 = 9. 415 4
Vd3 = 13. 584 3

62

CHAPTER 10. BINOMIAL OPTION PRICING: I

Step 3
Move one step to the left. Repeat Step 2. Compare the backwardized value and the exercise value. Choose the greater.
The backwardized values are:
Cu = e0.08(1/3) (0.456 806 0 + 0.543 194 1. 400 9) = 0.740 9
Cu = e0.08(1/3) (0.456 806 1. 400 9 + 0.543 194 9. 415 4) = 5. 602 9
The exercise value at Period 1 is:
EVu = max (0, K Su ) = max (0, 40 50.0711) = 0
EVd = max (0, K Sd ) = max (0, 40 35.4114) = 4. 588 6
Vu = max (Cu , EVu ) = max (0.740 9, 0) = 0.740 9
Vud = max (Cd , EVd ) = max (5. 602 9, 4. 588 6) = 5. 602 9
Hence we have:
Period 0

3
Vu3 = 0

Vu2 = 0
Vu = 0.740 9
V

Vu2 d = 0
Vud = 1. 400 9

Vd = 5. 602 9

Vd2 u = 2. 648 7
Vd2 = 9. 415 4

Vd3 = 13. 584 3


Step 4
Repeat Step 3. Move one step left. Compare the backwardized
value and the exercise value. Choose the greater value.
The backwardized value at t = 0 is:
C = e0.08(1/3) (0.456 806 0.740 9 + 0.543 194 5. 602 9) = 3. 292 9
The exercise value is:
EV = max (0, K S) = max (0, 40 41) = 0
Hence the premium for the American put is:
V = max (C, EV )= max (3. 292 9, 0) = 3. 292 9

Now we have:
Period 0

3
Vu3 = 0

Vu2 = 0
Vu = 0.740 9
V = 3. 292 9

Vu2 d = 0
Vud = 1. 400 9

Vd = 5. 602 9

Vd2 u = 2. 648 7
Vd2 = 9. 415 4
Vd3 = 13. 584 3

Step 5

Calculate the replicating portfolio at each node.

10.2. GENERAL ONE-PERIOD BINOMIAL MODEL


4 = eh

Cu Cd
Su Cd Sd Cu
B = erh
Su Sd
Su Sd
Period 0

63

2
(4, B)u2 = (0, 0)

(4, B)u = (0.07 82, 4. 658 9)


(4, B) = (0.331 66, 16. 890 9)

(4, B)ud = (0.171 3, 8. 808 8)


(4, B)d = (0.632 99, 28. 017 8)
(4, B)d2 = (1, 38. 947 4)

Vu3 Vu2 d
00
= e0(1/3)
=0
3
2
Su Su d
74.6781 52.8140
Su3 Vu2 d Su2 dVu2 d
74.6781 (0) 52.8140 (0)
= erh
= e0.08(1/3)
=0
3
2
Su Su d
74.6781 52.8140

4u2 = eh
Bu2

Vu2 d Vd2 u
0 2. 648 7
= e0(1/3)
= 0.171 3
2
2
Su d Sd u
52.8140 37.3513
2
2
Su dVd2 u Sd Vu2 d
52.8140 (2. 648 7) 37.3513 (0)
= erh
= e0.08(1/3)
=
2
2
Su d Sd u
52.8140 37.3513

4ud = eh
Bud
8. 808 8

Vd2 u Vd3
2. 648 7 13. 584 3
= e0(1/3)
= 1.0
Su2 d Sd3
37.3513 26.4157
Su2 dVd3 Sd3 Vd2 u
37.3513 (13. 584 3) 26.4157 (2. 648 7)
= e0.08(1/3)
Bd2 = erh
=
Su2 d Sd3
37.3513 26.4157
38. 947 4
V 2 Vud
0 1. 400 9
= e0(1/3)
= 0.07 82
4u = eh u2
Su Sud
61.1491 43.2460
Su2 Vud SudVu2
61.1491 (1. 400 9) 43.2460 (0)
Bu = erh
= e0.08(1/3)
=
Su2 Sud
61.1491 43.2460
4. 659
4d2 = eh

Vud Vd2
1. 400 9 9. 415 4
= e0(1/3)
= 0.632 99
2
Sud Sd
43.2460 30.5846
2
SudVd2 Sd Vud
43.2460 (9. 415 4) 30.5846 (1. 400 9)
= e0.08(1/3)
Bd = eh
=
2
Sud Sd
43.2460 30.5846
28. 017 8
Vu Vd
0.740 9 5. 602 9
4 = eh
= e0(1/3)
= 0.331 66
Su Sd
50.0711 35.4114
Vu Vd
50.0711 (5. 602 9) 35.4114 (0.740 9)
B = erh
= e0.08(1/3)
= 16.
Su Sd
50.0711 35.4114
890 9
4d = eh

Please note that for an American option, you cant use the following approach to find the option price:

64

CHAPTER 10. BINOMIAL OPTION PRICING: I

Node
Payo
Risk neutral prob of reaching this node
3
u3
Vu3 = 0
3u= 0.456 806

2
2
2
u d
Vu2 d = 0
3u d = 3 0.456 806 0.543 194
ud2
Vd2 u = 2. 648 7
3 u 2d = 3 (0.456 806) 0.543 1942
3
d
Vd3 = 13. 584 3
3d = 0.543 1943

V = erT Cd2 u 3 u 2d + Cd3 3d

= e0.08(1) 2. 648 7 3 0.456 806 0.543 1942 + 13. 584 3 0.543 1943 =
2. 998 5
This approach is wrong because it ignores the possibility that the American
option can be exercised early.

10.2.2

Options on stock index

The price of an option on stock index can be calculated the same way as the
price of an option on a stock is calculated.
Example 10.2.11. Lets reproduce Derivatives Markets Figure 10.8. Here is
the recap of the information on an American call. The current stock index
is 110. The strike price K = 100. The annualized standard deviation of the
continuously compounded stock index return is = 30%. The continuously
compounded risk-free rate per year is r = 5%. The continuously compounded
dividend rate per year is = 3.5%.The option expiration date is T = 1 year.
Use a 3-period binomial tree to calculate the option premium.
Solution.
Each period is h =

1
year long.
3

u = e(r)h+ h = e(0.050.035)1/3+0.31/3 = 1. 195 07


d = e(r)h h = e(0.050.035)1/30.3 1/3 = 0.845 18
e(r)h d
e(0.050.035)1/3 0.845 18
u =
=
= 0.456 807
ud
1. 195 07 0.845 18
d = 1 u = 1 0.456 807 = 0.543 193
Step 1
Find the stock price tree
Period 0
1
2
3
Su3 = 187.7471
Su2 = 157.1013
Su = 131.4577
Su2 d = 132.7789
S = 110
Sud = 111.1055
Sd = 92.9699
Sd2 u = 93.9042
2
Sd = 78.5763
Sd3 = 66.4112
Step 2

Calculate the payo at expiration (terminal payo)

10.2. GENERAL ONE-PERIOD BINOMIAL MODEL


Period 0

2
Vu2

Vu
V

Vud

65

3
Vu3 = max (0, 187.7471 100) = 87. 747 1
Vu2 d = max (0, 132.7789 100) = 32. 778 9

Vd

Vd2 u = max (0, 93.9042 100) = 0

Vd2

Vd3 = max (0, 66.4112 100) = 0


Step 3
Calculate the value of the American call at Period 2 by taking
the greater of the backwardized value and the exercise value at each node.
The backwardized values at Period 2 are:
Cu2 = erh (u Vu3 + d Vu2 d ) = e0.05(1/3) (0.456 807 87. 747 1 + 0.543 193 32. 778 9) =
56. 931 9
Cud = erh (u Vu2 d + d Vd2 u ) = e0.05(1/3) (0.456 807 32. 778 9 + 0.543 193 0) =
14. 726 1
Cd2 = erh ( ud Vd2 u + d Vd3 ) = e0.05(1/3) (0.456 807 0 + 0.543 193 0) =
0
The exercise values at Period 2 are:

EVu2 = max 0, Su2 K = max (0, 157.1013 100) = 57. 101 3


EVud = max (0, Sud K)
= max (0, 111.1055 100) = 11. 105 5
EVd2 = max 0, Sd2 K = max (0, 78.5763 100) = 0
We take the greater of the two as the values.
Vu2 = max (Cu2 , EVu2 ) = max (56. 931 9, 57. 101 3) = 57. 101 3
Vud = max (Cud , EVud ) = max (14. 726 1, 11. 105 5) = 14. 726 1
Vd2 = max (Cd2 , EVd2 ) = max (0, 0) = 0
Now we have:
Period 0
1

3
Vu3 = 87. 747 1

Vu2 = 57. 101 3


Vu
V

Vu2 d = 32. 778 9


Vud = 14. 726 1

Vd

Vd2 u = 0
Vd2 = 0
Vd3 = 0

Step 4
Calculate the value of the American call at Period 1 by taking
the greater of the backwardized value and the exercise value at each node.
Cu = erh (u Vu2 + d Vud ) = e0.05(1/3) (0.456 807 57. 101 3 + 0.543 193 14. 726 1) =
33. 520 02

66

CHAPTER 10. BINOMIAL OPTION PRICING: I

Cd = erh ( u Vud + d Vd2 ) = e0.05(1/3) (0.456 807 14. 726 1 + 0.543 193 0) =
6. 615 8
EVu = max (0, Su K) = max (0, 131.4577 100) = 31. 457 7
EVd = max (0, Sd K) = max (0, 92.9699 100) = 0
Vu = max (Cu , EVu ) = max (33. 520 02, 31. 457 7) = 33. 520 02
Vd = max (Cd , EVd ) = max (6. 615 8, 0) = 6. 615 8
Now we have:
Period 0

3
Vu3 = 87. 747 1

Vu2 = 57. 101 3


Vu = 33. 520 02
V

Vu2 d = 32. 778 9


Vud = 14. 726 1

Vd = 6. 615 8

Vd2 u = 0
Vd2 = 0
Vd3 = 0

Step 5
Calculate the value of the American call at Period 0 by taking
the greater of the backwardized value and the exercise value at each node.
C = erh ( u Vu + d Vd ) = e0.05(1/3) (0.456 807 33. 520 02 + 0.543 193 6. 615 8) =
18. 593 35
EV = max (0, S K) = max (0, 110 100) = 10
V = max (C, EV ) = max (18. 593 35, 10) = 18. 593 35
Step 6
Calculate the replicating portfolio
Our goal is to replicate the following values:
Period 0
1
2

3
Vu3 = 87. 747 1

Vu2 = 57. 101 3


Vu = 33. 520 02
V = 18. 593 35

Vu2 d = 32. 778 9


Vud = 14. 726 1

Vd = 6. 615 8

Vd2 u = 0
Vd2 = 0
Vd3 = 0

Cu Cd
Su Sd
Period 0

4 = eh

4 = 0.690 923

B = erh
1

Su Cd Sd Cu
Su Sd
2

4u = 0.910 598
4d = 0.447 45

4u2 = 0.988 401


4ud = 0.988 401
4d2 = 0

10.2. GENERAL ONE-PERIOD BINOMIAL MODEL


Period 0

B = 57. 408 1

Bu = 86. 185 1
Bd = 34. 983 9

67

2
Bu2 = 98. 347 1
Bud = 77. 870 8
Bd2 = 0

Vu3 Vu2 d
87. 747 1 32. 778 9
= e0.035(1/3)
= 0.988 401
3
2
Su Su d
187.7471 132.7789
3
2
Su Vu2 d Su dVu3
187.7471 (32. 778 9) 132.7789 (87. 747 1)
= e0.05(1/3)
=
Bu2 = erh
3
2
Su Su d
187.7471 132.7789
98. 347 1
4u2 = eh

Vu2 d Vud2
32. 778 9 0
= e0.035(1/3)
= 0.988 401
Su2 d Sud2
187.7471 132.7789
3
2
Su dVu2 d Su dVu3
187.7471 (0) 132.7789 (32. 778 9)
= e0.05(1/3)
=
Bud = erh
Su3 Su2 d
187.7471 132.7789
77. 870 8
4ud = eh

Vd2 u Vd3
=0
Sd2 u Sd3
Sd2 uVd3 Sd3 Vd2 u
= erh
=0
Sd2 u Sd3

4d2 = eh
Bd2

Vu2 Vud
57. 101 3 14. 726 1
= e0.035(1/3)
= 0.910 598
2
Su Sud
157.1013 111.1055
2
Su Vud SudVu2
157.1013 (14. 726 1) 111.1055 (57. 101 3)
= e0.05(1/3)
=
Bu = erh
2
Su Sud
157.1013 111.1055
86. 185 1
4u = eh

Vud Vd2
14. 726 1 0
= e0.035(1/3)
= 0.447 45
Sud Sd2
111.1055 78.5763
2
Su Vud SudVu2
111.1055 (0) 78.5763 (14. 726 1)
= e0.05(1/3)
=
Bd = erh
Su2 Sud
111.1055 78.5763
34. 983 9
4d = eh

Vu Vd
33. 520 02 6. 615 8
= e0.035(1/3)
= 0.690 923
Su Sd
131.4577 92.9699
SuVd SdVu
131.4577 (6. 615 8) 92.9699 (33. 520 02)
B = erh
= e0.05(1/3)
=
Su Sd
131.4577 92.9699
57. 408 1
4 = eh

10.2.3

Options on currency

Now lets find the price of a European call option on 1. The underlying asset is
1. The option expires in h years. The current dollar value of the underlying is

68

CHAPTER 10. BINOMIAL OPTION PRICING: I

S (so 1 = $S at t = 0). In h years the dollar value of the underlying asset 1


can go up to $Su or go down to $Sd . The strike price is $K. The continuously
compounded risk-free interest rate on dollars is r per year (so dollars earn a
return r). The continuously compounded interest rate on the underlying asset
of 1 is per year (so euros earn a return ).
Lets calculate the call price using one-period binomial tree. The asset price
tree and the option payo tree are as follows:
Asset price tree (in dollars)
Option payo (in dollars)
time 0

h
Su

time 0

h
Cu = max (0, Su K)

C
Sd
Cd = max (0, Sd K)
To replicate the payo, at t = 0 well buy units of the underlying (i.e.
buy euros) and simultaneously invest B dollars in a savings account. Since
the underlying asset earns interest at a continuous interest , it will grow
into eh at T , which is worth $eh Su in the up state and $eh Sd in the
down state.
We want our replicating portfolio to match the option payo. So we have:
eh Su
Berh
Cu
4S
+ B
= C
eh Sd
Berh
Cd
t=0 t=h
t=0 t=h
t=0 t=h

eh Su + Berh = Cu
eh Sd + Berh = Cd
Solving these equations, we get:
Cu Cd
Su Sd

(10.26)

Su Cd Sd Cu
Su Sd

(10.27)

4 = eh
B = erh

Equation 10.26 and 10.27 are exactly the same as Equation 10.13 and 10.14.
This tells us that if we treat the currency as a stock and treat the euro return
as the stocks dividend rate, we can find the currency options price and the
replicating portfolios using all the formulas available for a stock option.
Example 10.2.12. Reproduce Derivatives Markets Figure 10.9. Here is the recap of the information on an American put option on 1. The current exchange
rate is S = $1.05/. The strike price is K = $1.10. The annualized standard
deviation of the continuously compounded return on dollars is = 10%. The
continuously compounded risk-free rate on dollars is r = 5% per year. The
continuously compounded return on euros is = 3.1% per year. The option

10.2. GENERAL ONE-PERIOD BINOMIAL MODEL

69

expiration date is T = 0.5 year. Use a 3-period binomial tree to calculate the
option premium.
Solution.
The length period is h =

T
1
=
3
6

u = e(r)h+ h = e(0.0550.031)1/6+0.11/6 = 1. 045 845


d = e(r)h h = e(0.0550.031)1/60.1 1/6 = 0.963 845
e(r)h d
e(0.0550.031)1/6 0.963 845
u =
=
= 0.489 795
ud
1. 045 845 0.963 845
d = 1 u = 1 0.489 795 = 0.510 205
Step 1
Find the underlying asset price tree
Period 0
1
2
3
Su3 = 1.2011
Su2 = 1.1485
Su = 1.0981
Su2 d = 1.1070
S = 1.05
Sud = 1.0584
Sd = 1.0120
Sd2 u = 1.0202
2
Sd = 0.9754
Sd3 = 0.9402
Step 2
Period 0

Calculate the payo at expiration (terminal payo)


1

2
Vu2

Vu
V

Vud
Vd
Vd2

3
Vu3 = max (0, 1.10 1.2011) = 0
Vu2 d = max (0, 1.10 1.1070) = 0
Vd2 u = max (0, 1.10 1.0202) = 0.079 8
Vd3 = max (0, 1.10 0.9402 ) = 0.159 8

Step 3
Calculate the value of the American put at Period 2 by taking
the greater of the backwardized value and the exercise value at each node.

The backwardized values at Period 2 are:


Cu2 = erh (u Vu3 + d Vu2 d ) = e0.055(1/6) (0.489 795 0 + 0.510 205 0) =

Cud = erh (u Vu2 d + d Vd2 u ) = e0.055(1/6) (0.489 795 0 + 0.510 205 0.079 8) =
0.04 034
Cd2 = erh ( ud Vd2 u + d Vd3 ) = e0.055(1/6) (0.489 795 0.079 8 + 0.510 205 0.159 8) =
0.119 5

70

CHAPTER 10. BINOMIAL OPTION PRICING: I


The exercise values at Period 2 are:

EVu2 = max 0, K Su2 = max (0, 1.10 1.1485) = 0


EVud = max (0, K Sud)
= max (0, 1.10 1.0584) = 0.041 6
EVd2 = max 0, K Sd2 = max (0, 1.10 0.9754) = 0.124 6

We take the greater of the two as the values.


Vu2 = max (Cu2 , EVu2 ) = max (0, 0) = 0
Vud = max (Cud , EVud ) = max (0.04 034 , 0.041 6) = 0.041 6
Vd2 = max (Cd2 , EVd2 ) = max (0.119 5, 0.124 6) = 0.124 6
Now we have:
Period 0
1

3
Vu3 = 0

Vu2 = 0
Vu
V

Vu2 d = 0
Vud = 0.041 6

Vd

Vd2 u = 0.079 8
Vd2 = 0.124 6
Vd3 = 0.159 8

Step 4
Calculate the value of the American put at Period 1 by taking
the greater of the backwardized value and the exercise value at each node.
Cu = erh ( u Vu2 + d Vud ) = e0.055(1/6) (0.489 795 0 + 0.510 205 0.041 6) =
0.02 10
Cd = erh ( u Vud + d Vd2 ) = e0.055(1/6) (0.489 795 0.041 6 + 0.510 205 0.124 6) =
0.08 31
EVu = max (0, K Su ) = max (0, 1.10 1.0981) = 0.001 9
EVd = max (0, K Sd ) = max (0, 1.10 1.0120 ) = 0.088
Vu = max (Cu , EVu ) = max (0.02 10 , 0.001 9) = 0.021
Vd = max (Cd , EVd ) = max (0.088 , 0.021) = 0.088
Now we have:
Period 0

3
Vu3 = 0

Vu2 = 0
Vu = 0.021
V

Vu2 d = 0
Vud = 0.041 6

Vd = 0.088

Vd2 u = 0.079 8
Vd2 = 0.124 6
Vd3 = 0.159 8

Step 5
Calculate the value of the American put at Period 0 by taking
the greater of the backwardized value and the exercise value at each node.

10.2. GENERAL ONE-PERIOD BINOMIAL MODEL

71

C = erh (u Vu + d Vd ) = e0.055(1/6) (0.489 795 0.021 + 0.510 205 0.088) =


0.05 47
EV = max (0, K S ) = max (0, 1.10 1.05) = 0.05
V = max (C, EV ) = max (0.05 47, 0.05) = 0.054 7
Now we have:
Period 0

3
Vu3 = 0

Vu2 = 0
Vu = 0.021
V = 0.054 7

Vu2 d = 0
Vud = 0.041 6

Vd = 0.088

Vd2 u = 0.079 8
Vd2 = 0.124 6
Vd3 = 0.159 8

You can verify that the replicating portfolios are as follows:


Period 0
1
2

4 = 0.7736

Period 0

4u = 0.4592
4d = 0.9948

4u2 = 0.0000
4ud = 0.9151
4d2 = 0.9948
2
Bu2 = $0.0000

Bu = $0.5253
B = $0.8669

Bud = $1.0089
Bd = $1.0900
Bd2 = $1.0900

10.2.4

Options on futures contracts

Suppose we want to find the price of a European call option on a stock futures
contract. The underlying asset is futures. The option expires in h years. The
current price of the underlying asset is F0,h = F , where F0,h is the price of a
futures contract signed at t = 0 and expiring on date h.In h years the futures
price can go up to Fh,h = Fu = F u or go down to Fh,h = Fd = F d, where Fh,h
is the price of a futures contract signed at t = h and expiring on date h years
(i.e. expiring immediately). Fh,h is equal to the spot price Sh . The fact that
Fh,h can be either Fu or Fd is the same as fact that the stock price at t = h is

72

CHAPTER 10. BINOMIAL OPTION PRICING: I

either Fu or Fd . The continuously compounded risk-free interest rate is r per


year. Stocks pay dividend at a continuously compounded rate of per year.
The strike price is K.
The find the European call price on the futures, we draw the price tree of
the underlying asset and the payo tree.
Asset price tree
Option payo
time 0

h
Fu = F u

time 0

h
Cu = max (0, Fu K)

C
Fd = F d
Cd = max (0, Fd K)
We form a replicating portfolio at t = 0 by entering futures contracts as
a buyer and simultaneously putting $B in the savings account. Assume that
no margin account is needed before one enters a futures contract. Then the
cost of entering a futures contracts is zero. At the contract expiration date h,
the futures contracts are settled in cash. If the futures price at expiration is
Fu > F , then the seller in the futures pays (Fu F ) to us, the buyer.
If on the other hand, the futures price at h is Fd < F , then we pay the seller
(F Fd ). Paying (F Fd ) is the same as receiving (Fd F ).
We assume that Fd < F < Fu holds so theres no arbitrage.
So the cash flow of the underlying asset (futures) is:
time 0
h
$ (Fu F )
$0
$ (Fd F )
We want the replicating portfolio
(Fu F )
0
+ B
(Fd F )
t=0 t=h
t=0

and the option have


Berh
= C
Berh
t=h
t=0

the same payo.


Cu
Cd
t=h

(Fu F ) + Berh = Cu
(Fd F ) + Berh = Cd

Solving these equations, we get:


Cu Cd
Cu Cd
=
Fu Fd
F (u d)

1d
u1
rh
Cu
+ Cd
B=e
ud
ud

1d
u1
+ Cd
V = 4 0 + B = B = erh Cu
ud
ud
4=

(10.28)
(10.29)
(10.30)

10.2. GENERAL ONE-PERIOD BINOMIAL MODEL

73

Define

u =

F Fd
1d
=
Fu Fd
ud

(10.31)

d =

Fu 1
u1
=
Fu Fd
ud

(10.32)

Then
V = erh (Cu u + Cd d )

(10.33)

Equation 10.30 is the same as Equation 10.16 if we set r = 0 and S = F .


Consequently, we can find the price of a futures option by using Equation 10.16
Cu Cd
if we set = r. We just need to remember that 4 =
instead of
Fu Fd

Cu Cd
1d
u1
and that B = V = erh Cu
+ Cd
instead of
4 = er
Fu Fd
ud
ud
Su Cd Sd Cu
.
B = erh
Su Sd
How we can specify u or d?
u=

up price of Fh,h
F0,h

u=

We know that Fh,h = Sh =

down price of Fh,h


F0,h

S0 e(r)h+h
. In addition, F0,h = S0 e(r)h .
S0 e(r)h h

up price of Fh,h
S0 e(r)h+ h
h
u=
=
=
e
F0,h
S0 e(r)h

down price of Fh,h


S0 e(r)h h
h
d=
=
=
e
F0,h
S0 e(r)h

u = e

d = e

(10.34)
(10.35)

Equation 10.34 and 10.35 are the same as Equation 10.24 and 10.25 if we
set = r. We can use the stock options formula on u and d for futures options.
Tip 10.2.6. To find the price of a futures option, just use the price formula for
a stock option and set = r. However, remember that for a futures option, 4 =
Cu Cd
Cu Cd
1d
u1
instead of 4 = erh
and B = V = erh Cu
+ Cd
Fu Fd
Fu Fd
ud
ud
Su Cd Sd Cu
.
instead of B = erh
Su Sd

74

CHAPTER 10. BINOMIAL OPTION PRICING: I

Example 10.2.13. Lets reproduce Derivatives Markets Figure 10.10. Here is


the recap of the information on an American call on a futures contract. The
current futures price is S = 300. The strike price K = 300. The annualized
standard deviation of the continuously compounded stock index return is =
10%. The continuously compounded risk-free rate per year is r = 5%.The option
expiration date is T = 1 year. Use a 3-period binomial tree to calculate the
option premium.
Solution.
Well reuse the stock option formula and set = r.
1
year long.
3

h
= e0.1 1/3
u = e(r)h+ h = e
= 1. 059 434
d = e(r)h h = e h = e0.1 1/3 = 0.943 900
e(r)h d
1d
1 0.943 900
u =
=
=
= 0.485 57
ud
ud
1. 059 434 0.943 900
d = 1 u = 1 0.485 57 = 0.514 43
Each period is h =

Step 1
Find the underlying asset price tree
Period 0
1
2

3
Su3 = 356.7330

Su2 = 336.7203
Su2 d = 317.8303

Su = 317.8303
F = 300

Sud = 300.0000
Sd2 u = 283.1700

Sd = 283.1700
2

Sd = 267.2842
Sd3 = 252.2895
Step 2
Period 0

Calculate the payo at expiration (terminal payo)


1

2
Vu2

Vu
V

Vud
Vd
Vd2

3
Vu3 = max (0, 356.7330 300) = 56. 733
Vu2 d = max (0, 317.8303 300) = 17. 830 3
Vd2 u = max (0, 283.1700 300) = 0
Vd3 = max (0, 252.2895 300) = 0

Step 3
Calculate the value of the American call at Period 2 by taking
the greater of the backwardized value and the exercise value at each node.
The backwardized values at Period 2 are:

10.2. GENERAL ONE-PERIOD BINOMIAL MODEL

75

Cu2 = erh (u Vu3 + d Vu2 d ) = e0.05(1/3) (0.485 57 56. 733 + 0.514 43 17. 830 3) =
36. 113 3
Cud = erh (u Vu2 d + d Vd2 u ) = e0.05(1/3) (0.485 57 17. 830 3 + 0.514 43 0) =
8. 514 7
Cd2 = erh ( ud Vd2 u + d Vd3 ) = e0.05(1/3) (0.485 57 0 + 0.514 43 0) =
0
The exercise values at Period 2 are:

EVu2 = max 0, Su2 K = max (0, 336.7203 300) = 36. 720 3


EVud = max (0, Sud K) = max (0, 300.0000 300) = 0

EVd2 = max 0, Sd2 K = max (0, 267.2842 300) = 0


We take the greater of the two as the values.
Vu2 = max (Cu2 , EVu2 ) = max (36. 113 3, 36. 720 3) = 36. 720 3
Vud = max (Cud , EVud ) = max (8. 514 7, 0) = 8. 514 7
Vd2 = max (Cd2 , EVd2 ) = max (0, 0) = 0
Now we have:
Period 0
1

3
Vu3 = 56. 733

Vu2 = 36. 720 3


Vu
V

Vu2 d = 17. 830 3


Vud = 8. 514 7

Vd

Vd2 u = 0
Vd2 = 0
Vd3 = 0

Step 4
Calculate the value of the American call at Period 1 by taking
the greater of the backwardized value and the exercise value at each node.
Cu = erh (u Vu2 + d Vud ) = e0.05(1/3) (0.485 57 36. 720 3 + 0.514 43 8. 514 7) =
21. 843 4
Cd = erh ( u Vud + d Vd2 ) = e0.05(1/3) (0.485 57 8. 514 7 + 0.514 43 0) =
4. 066 2
EVu = max (0, Su K) = max (0, 317.8303 300) = 17. 830 3
EVd = max (0, Sd K) = max (0, 283.1700 300) = 0
Vu = max (Cu , EVu ) = max (21. 843 4, 17. 830 3) = 21. 843 4
Vd = max (Cd , EVd ) = max (4. 066 2, 0) = 4. 066 2
Now we have:

76

CHAPTER 10. BINOMIAL OPTION PRICING: I


Period 0

3
Vu3 = 56. 733

Vu2 = 36. 720 3


Vu = 21. 843 4
V

Vu2 d = 17. 830 3


Vud = 8. 514 7

Vd = 4. 066 2

Vd2 u = 0
Vd2 = 0
Vd3 = 0

Step 5
Calculate the value of the American call at Period 0 by taking
the greater of the backwardized value and the exercise value at each node.
C = erh ( u Vu + d Vd ) = e0.05(1/3) (0.485 57 21. 843 4 + 0.514 43 4. 066 2) =
12. 488 4
EV = max (0, S K) = max (0, 300 300) = 0
V = max (C, EV ) = max (12. 488 4, 0) = 12. 488 4
Now we have:
Period 0
1
2
3
Vu3 = 56. 733
Vu2 = 36. 720 3
Vu = 21. 843 4
Vu2 d = 17. 830 3
V = 12. 488 4
Vud = 8. 514 7
Vd = 4. 066 2
Vd2 u = 0
Vd2 = 0
Vd3 = 0
Next, we need to find the replicating portfolio. Our goal is to replicate the
following values:
Period 0
1
2
3
Vu3 = 56. 733
Vu2 = 36. 720 3
Vu = 21. 843 4
Vu2 d = 17. 830 3
V = 12. 488 4
Vud = 8. 514 7
Vd = 4. 066 2
Vd2 u = 0
Vd2 = 0
Vd3 = 0
Using Equation 10.29 and 10.29, you should get:
Period 0
1
2

4 = 0.5129

4u = 0.7681
4d = 0.2603

4u2 = 1
4ud = 0.5144
4d2 = 0

10.2. GENERAL ONE-PERIOD BINOMIAL MODEL


Period 0

77

2
Bu2 = $36. 720 3

Bu = $21. 843 4
B = $12. 488 4

Bud = $8. 514 7


Bd = $4. 066 2
Bd2 = $0

For example,
V 3 Vu2 d
56. 733 17. 830 3
4u2 = u
=
= 1.0
Su3 Su2 d
356.7330 317.8303
Bu2 = Vu2 = $36. 720 3
21. 843 4 4. 066 2
Vu Vd
=
= 0.512 9
Su Sd
317.8303 283.1700
B = V = $12. 488 4
I recommend that you reproduce my replicating portfolio in each node.
4=

Options on commodities
The textbook is brief on this topic. So you dont need to spend lot of time on it.
This is the main idea: we can price commodity options using the same framework
for pricing stock options if we can build a replicating portfolio using commodities
and bonds with zero transaction cost. In reality, its hard to build a replicating
portfolio using commodities. Unlike stocks, commodities such as corn may incur
storage cost or other cost. It may be impossible to short sell commodities.
As such, our ability to build a replicating portfolio is limited. However, if
we can build any replicating portfolios using commodities instantaneously and
eortlessly, commodity options and stock options are conceptually the same.
We can use the same framework to calculate the price of a commodity option
and the price of a stock option.
Example 10.2.14. Here is the information on an American call on a commodity. The current commodity price is 110. The strike price is K = 100.
The annualized standard deviation of the continuously compounded return on
the commodity is = 30%. The continuously compounded risk-free rate per
year is r = 5%. The continuously compounded lease rate of the commodity is
= 3.5% per year. The option expiration date is T = 1 year. Use a 3-period
binomial tree to calculate the option premium. Assume that you can eortlessly
and instantaneously build any replicating portfolio using the commodity and the
bond.
We just treat the commodity as a stock. The lease rate = 3.5% is the same
as the stock dividend rate. We can use the framework for pricing stock options
to price this commodity option. The solution to this problem is in the textbook
Figure 10.8.

78

CHAPTER 10. BINOMIAL OPTION PRICING: I

Options on bonds
The textbook points out two major dierences between bonds and stocks:
1. A bonds volatility decreases over time as the bond approaches its maturity. A stocks volatility doesnt have this pattern.
2. When pricing a stock option, we assume that the interest rate is constant
over time. The random variable is the stock price under a fixed interest.
However, when pricing a bond, we cant assume that the interest rate is
constant any more. If the interest rate is constant, then the bonds price
is known with 100% certainty. If the bond price doesnt change randomly,
an option on the bond has zero value. Who wants to buy a call or put on
an asset whose price is known with 100% certainty?
Because of these dierences, options on bonds should be treated dierently
from options on stocks. This is all you need to know about bond options right
now. Chapter 24 will cover more on options on bonds.

Chapter 11

Binomial option pricing: II


11.1

Understanding early exercise

Pros and cons of exercising an American call option early


Pro:
Receive the stock and future dividends
Cons
Pay the strike price early and lost interest that could have earned on the
strike price.
Lose the insurance implicit in the call. If you hold the option, the stock
price might be below the strike price at expiration, in which case you
would not exercise the option. However, if you exercise the American call
early, you lose the privilege of not exercising it. To understand this point,
suppose you go to a garage sale and find a book you like that sells for
only $1. You think "How cheap the book is. I must buy it." You pay $1
and buy the book. You think you get a good deal. However, if you resist
the temptation to buy the book immediately and wait till the end of the
garage sale, the books price may drop to $0.25. Better yet, you may even
get the book for free. That same thing may happen when you exercise
an American call early. At the moment, the stock price is high and you
might be attempted to exercise the call. However, if we wait for a while,
the stock price may drop below the strike price.
Next, the textbook gives us a rule to determine when its optimal to exercise
a perpetual 1 American call early. For a perpetual American call with zero
volatility, its optimal to exercise a perpetual American call early if the dividend
to be received exceeds the interest savings on the strike price:
1 The

textbook errata say the formulas work for an infinitely-lived American call option.

79

80

CHAPTER 11. BINOMIAL OPTION PRICING: II


ST > rK


The annual dividend you
gain if you exercise the call early is ST e 1 =

1
ST 1 + + 2 + ... 1 ST for a small . The annual interest earned on
2

1
r
K is K (e 1) = K 1 + r + r2 + ... 1 rK for a small r. If you early
2
exercise, youll pay K and receive ST ; during a year youll receive ST dividend
but you will lose rK interest. Hence early exercise is optimal if the annual
dividend exceeds the annual interest, ST > rK.
And its optimal to defer exercising a perpetual American call if the interest
savings on the strike price exceeds dividends lost:
ST < rK
Example 11.1.1. A perpetual American call option on a stock has a strike
price $50. The stock pays dividend at a continuously compounded rate of 8%
per year. The continuously compounded risk-free interest rate is 6% per year.
The volatility of the stock price is zero. When is it optimal to exercise this
American call option early? When is it optimal to defer exercise?
S > rK
rK
0.06 (50)
S>
=
= 37. 5

0.08
Once the stock price becomes greater than 37. 5, then its optimal to exercise
this perpetual American call option early.
If the stock price is less than 37. 5, then its optimal to defer exercising this
perpetual American call option early.
Please note that zero volatility doesnt mean that the stock price is a constant. It means that the stock price is known in advance with 100% certainty.
Next, the textbook says that the decision to exercise a perpetual American
call option early is complex if the volatility of the stock price is greater than
zero. In this case, the insurance in the call option is greater than zero. For
each non-zero volatility, theres a lowest stock price at which the early exercise
is optimal.

11.2

Understanding risk-neutral probability

Risk neutral probability is explained in the previous chapter. Theres not much
new information about the risk neutral probability in this chapter. The key
point to remember is that risk neutral probability is a shortcut or a math trick
that enables us to quickly find the price of an option. We can use risk neutral
probability to find the correct price of an option whether consumers are really
risk neutral or not.
The risk neutral probability is similar to the moment generating function in
Exam P. The moment generating function (M GF ) is merely a math trick that

11.2. UNDERSTANDING RISK-NEUTRAL PROBABILITY

81

allows us to quickly find the mean and variance of a random variable (hence
the name "moment generating"). If we dont use GM F , we can still find the
mean and variance, but we have to work a lot harder. With the help of GM F ,
we can quickly find the mean and variance. Similarly, if we dont use the risk
neutral probability, we can still find the option price, but we have to work a lot
harder. Once we use risk-neutral probability, we can quickly find an options
price. Risk neutral probability is merely a math risk.
By the way, one investment consultant told me that risk neutral probability
is often hard to non-technical clients to understand. If you tell a non-technical
client that an option price is calculated using risk-neutral probability and that
the risk neutral probability not real, the client often immediately ask "So the
price you calculated is wrong then?" It may take the consultant a while to
explain why the risk neutral probability is not real yet the price is still correct.

11.2.1

Pricing an option using real probabilities

Next, the textbook answers a frequently asked question: Can we calculate the
option price as the expected payo using real probabilities of the stock price?
The answer is "Yes if you know the discount rate."
Let p and q = 1p represent the real probability of stock going up and down
respectively. Let represent the real discount rate (instead of the risk-free rate,
which is used to discount payo in the risk neutral world).
uS with real probability p
S
t=0

Cu with real probability p


C =?

dS with real probability q


t=h

t=0

Cd with real probability q


t=h

Then the price of a European option expiring in h years is:


C = eh (pCu + qCd )
How can we find the real probability p and the real discount rate ? Suppose
we know that the expected return on the stock during [0, h] is . Assume that
the continuously compounded dividend rate is per year. If we have one stock
at t = 0, then at t = h well have eh stocks. The value at t = 0 is the expected
value at t = h discounted at rate .
uSeh with real probability p
S
dSeh with real probability q
Value t = 0 Value t = h

e()h d
p=
S = eh puSeh + qdSeh
ud
p=

e()h d
ud

(11.1)

82

CHAPTER 11. BINOMIAL OPTION PRICING: II

q=

u e()h
ud

(11.2)

Tip 11.2.1. If we set r = Equation 10.16 and 10.17 become Equation 11.1
and 11.2. So you just need to memorize Equation 10.16 and 10.17. To get the
formulas for the real probability, just set r = .
We can use the replicating portfolio to find . Suppose the replicating portfolio at t = 0 consists of shares of stock and putting $B in a savings account.
We already know that we can calculate and B using Equation 10.13 and
10.13.
At t = 0, the replicating portfolio is worth S + B. At t = h, the replicating
portfolio consists of eh shares of stock and $Berh in a savings account, which
is worth eh dS + Berh in the up state and eh dS Berh in the down state.
uSeh + Berh with real probability p
S + B
Value t = 0

dSeh + Berh with real probability q


Value t = h

The value at
value
at t = h discounted
at rate :

t = 0 is the expected
S + B = p uSeh + Berh + q dSeh + Berh eh

= puSeh + qdSeh + Berh (p + q) eh

= Seh + Berh eh
eh =

S + B
Seh + Berh

(11.3)

S + B
(pCu + qCd )
Seh + Berh
Since C = S + B, we just need to prove that pCu + qCd = Seh + Berh .

C = eh (pCu + qCd ) =

pCu + qCd
e()h d
u e()h
=
Cu +
C
ud
ud
(r)h

()h
e
e(r)h e()h
e
u e(r)h
d
e(r)h
=
Cu +
Cu +
Cd +
Cd
ud
ud
ud
ud

(r)h
e
d
u e(r)h
e()h e(r)h
=
Cu +
Cd +
(Cu Cd )
ud
ud
ud
According to Equation 10.15:
u e(r)h
e(r)h d
Cu +
Cd = erh (S + B)
ud
ud

11.2. UNDERSTANDING RISK-NEUTRAL PROBABILITY

83

Cu Cd
= eh S
ud
pCu + qCd

= erh (S + B) + eh S e()h e(r)h


= erh (S + B) + S eh erh = eh S + Berh
According to Equation 10.13,

S + B
(pCu + qCd ) = S + B
Seh + Berh
The above derivation tell us that
C=

Real probabilities lead to the same answer as the risk neutral probability
Any consistent pair of (, ) will produce the correct answer. The above
derivation doesnt require that has to be reasonable or precise. Any
will generate a corresponding . Together, and will produce the option
price correctly.
The simplest calculation is to set = = r. Setting = = r means
using risk neutral probabilities.

Example 11.2.1. Reproduce the textbook Figure 11.3 (which is the same as the
textbook Figure 10.5). A European call option has strike price K = 40. The
current price is S = 41. The annualized standard deviation of the continuously
compounded stock return is = 30%. The continuously compounded risk-free
rate per year is r = 8%. The continuously compounded dividend rate per year
is = 0%. The continuously compounded expected return on the stock per year
is = 15%. The option expiration date is T = 1 year. Use a 1-period binomial
tree and real probabilities to calculate the option premium.
Solution.

u = e(r)h+ h = e(0.080)1+0.31 = 1. 462 3


u = e(r)h h = e(0.080)10.3 1 = 0.802 5
1. 462 3 (41) = 59. 954 with real probability p
41
0.802 5 (41) = 32. 9023 with real probability q
t=0 t=h=1

C =?

Cu = max (0, 59. 954 40) = 19. 954 with real probability p

t=0

Cd = max (0, 32. 9023 40) = 0 with real probability q


t=h=1

Calculate the real probabilities:


41e0.15(1) = 59. 954p + 32. 9023 (1 p)

84

CHAPTER 11. BINOMIAL OPTION PRICING: II


p = 0.544 6

q = 1 p = 1 0.544 6 = 0.455 4

Calculate the replicating portfolio:


Cu Cd
19. 954 0
4 = eh
= e0(1)
= 0.737 6
Su Sd
59. 954 32. 9023
Su Cd Sd Cu
59. 954 (0) 32. 9023 (19. 954)
= e0.08(1)
B = erh
= 22.
Su Sd
59. 954 32. 9023
403 6
Calculate the

discounting rate:
S + B = Seh + Berh eh
(S + B) eh = Seh + Berh
(0.737 6 41 22. 403 6) e(1) = 0.737 6 41e0.15(1) 22. 403 6e0.08(1)
e(1) = 1. 386
= 0.326 4
Calculate the option price:
C = eh (pCu + qCd ) = e0.326 4(1) (0.544 6 19. 954 + 0.455 4 0) = 7. 84
This option price is the same as the price in the textbook Figure 10.3.

Example 11.2.2. Reproduce the textbook Figure 11.4 (the risk neutral solution
to an otherwise identical European call is in the textbook Figure 10.5). Here is
the recap of the information on an American call. The current stock price is 41.
The strike price K = 40. The annualized standard deviation of the continuously
compounded stock return is = 30%. The continuously compounded risk-free
rate per year is r = 8%. The continuously compounded expected return on the
stock per year is = 15%.The continuously compounded dividend rate per year
is = 0%.The option expiration date is T = 1 year. Use a 3-period binomial
tree and real probabilities to calculate the option premium.
Solution.
Each period is h =

1
year long.
3

u = e(r)h+ h = e(0.080)1/3+0.31/3 = 1. 221 246


d = e(r)h h = e(0.080)1/30.3 1/3 = 0.863 693
Stock price
Period 0
1
2
3
Su3 = 74.6781
Su2 = 61.1491
Su = 50.0711
Su2 d = 52.8140
S = 41
Sud = 43.2460
Sd = 35.4114
Sd2 u = 37.3513
2
Sd = 30.5846
Sd3 = 26.4157

11.2. UNDERSTANDING RISK-NEUTRAL PROBABILITY

85

The replicating portfolios are copied over from the textbook Figure 10.5.
Period 0

(4, B) = (0.706 3, 21. 885 2)

(4, B)u = (0.921 8, 33. 263 6)


(4, B)d = (0.450 1, 13. 405 2)

2
(4, B)u2 = (1, 38. 947 4)
(4, B)ud = (0.828 7, 30. 138 6)
(4, B)d2 = (0, 0)

In addition, we need to calculate the discount rate for each node. We put
the stock price table and the replicating portfolio table side by side:
Stock price
(4, B)
Period 0
1
2
3
Period 0
1
74.6781
61.1491
50.0711
52.8140
(0.921 8, 33. 263 6)
41
43.2460
(0.706 3, 21. 885 2)
35.4114
37.3513
(0.450 1, 13. 405 2)
30.5846
26.4157

0:

Calculate the common discounting factor Node u Node 0 and Node d Node
e(1/3) =

S + B
0.706 3 (41) 21. 885 2
=
=
Seh + Berh
0.706 3 (41) e0.15(1/3) 21. 885 2e0.08(1/3)

0.887 87
e(1/3) = 0.887 87

= 0.356 8

Calculate the common discounting factor Node u2 Node u and Node


ud Node u :
e(1/3) =

S + B
0.921 8 1 (50.0711) 33. 263 6
=
=
h
rh
Se + Be
0.921 8 (50.0711) e0.15(1/3) 33. 263 6 2e0.08(1/3)

0.897 84
e(1/3) = 0.897 84

= 0.323 3

Calculate the common discounting factor Node ud Node d and Node


d2 Node d :
e(1/3) =

S + B
0.450 1 (35.4114) 13. 405 2
=
=
Seh + Berh
0.450 1 (35.4114) e0.15(1/3) 13. 405 2 64e0.08(1/3)

0.847 79
e(1/3) = 0.847 79

= 0.495 4

2
(1, 38. 947 4)
(0.828 7, 30. 138 6)
(0, 0)

86

CHAPTER 11. BINOMIAL OPTION PRICING: II

Calculate the common discounting factor Node u3 Node u2 and Node


u d Node u2 :
2

S + B
1 (61.1491) 38. 947 4
=
=
Seh + Berh
1 (61.1491) e0.15(1/3) 38. 947 4e0.08(1/3)

e(1/3) =

0.914 24
e(1/3) = 0.914 24

= 0.2690

Calculate the common discounting factor Node u2 d Node ud and Node


ud Node ud :
2

e(1/3) =

S + B
0.828 7 (43.2460) 30. 138 6
=
=
h
rh
Se + Be
0.828 7 (43.2460) e0.15(1/3) 30. 138 6e0.08(1/3)

0.847 83
e(1/3) = 0.847 83

= 0.495 2

Calculate the common discounting factor Node ud2 Node d2 and Node
d Node d2 :
3

S + B
0 (30.5846) 0
=
= N/A
Seh + Berh
0 (30.5846) e0.15(1/3) 0 6e0.08(1/3)
= N/A
= N/A

e(1/3) =
e(1/3)

We calculate the premium by working backward from right to left. At each


node, we take the greater of the backwardized value and the exercise value.
Calculate the common real probability of stock going up and down at each
node.
41e0.15(1/3) = 50.0711p + 35.4114 (1 p)
p = 0.524 6
q = 1 0.524 6 = 0.475 4
Period 0

2
= 0.2690

3
Cu3 = max (0, 74.6781 40) = 34. 678 1

= 0.495 2

Cu2 d = max (0, 52.8140 40) = 12. 814

= N/A

Cd2 u = max (0, 37.3513 40) = 0

= 0.323 3
= 0.356 8
= 0.495 4

Cd3 = max (0, 26.4157 40) = 0


Period 0

3
Su3 = 74.6781

Su2 = 61.1491
Su2 d = 52.8140

Su = 50.0711
S = 41

Sud = 43.2460
Sd2 u = 37.3513

Sd = 35.4114
2

Sd = 30.5846
Sd3 = 26.4157

11.2. UNDERSTANDING RISK-NEUTRAL PROBABILITY


Cu2 = (34. 678 1 0.524 6 + 12. 814 0.475 4) e0.2690(1/3) = 22. 201 2
EVu2 = max (0, 61.1491 40) = 21. 149 1
Vu2 = max (22. 201 2, 21. 149 1) = 22. 201 2
Cud = (12. 814 0.524 6 + 0 0.475 4) e0.495 2(1/3) = 5. 699 4
EVud = max (0, 43.2460 40) = 3. 246
Vud = max (5. 699 4, 3. 246) = 5. 699 4
Cd2 = 0
EVd2 = max (0, 30.5846 40) = 0
Vd2 = max (0, 0) = 0
Now we have:
Period 0
1

3
Vu3 = 34. 678 1

Vu2 = 22. 201 2


= 0.323 3
= 0.356 8

Vu2 d = 12. 814


Vud = 5. 699 4

= 0.495 4

Vud2 = 0
Vd2 = 0
Vd3 = 0

Similarly,
Cu = (22. 201 2 0.524 6 + 5. 699 4 0.475 4) e0.323 3(1/3) = 12. 889 6
EVu = max (0, 50.0711 40) = 10. 071 1
Vu = max (12. 889 6, 10. 071 1) = 12. 889 6
Cd = (5. 699 4 0.524 6 + 0 0.475 4) e0.495 4(1/3) = 2. 534 8
EVd = max (0, 35.4114 40) = 0
Vd = max (2. 534 8, 0) = 2. 534 8
Now we have:
Period 0
1

3
Vu3 = 34. 678 1

Vu2 = 22. 201 2


Vu = 12. 889 6
= 0.356 8

Vu2 d = 12. 814


Vud = 5. 699 4

Vd = 2. 534 8

Vud2 = 0
Vd2 = 0

Vd3 = 0
Finally,
C = (12. 889 6 0.524 6 + 2. 534 8 0.475 4) e0.356 8(1/3) = 7. 0734
EV = max (0, 41 40) = 1

87

88

CHAPTER 11. BINOMIAL OPTION PRICING: II


V = max (7. 0734, 1) = 7. 073 4
So we have:
Period 0
1

3
Vu3 = 34. 678 1

Vu2 = 22. 201 2


Vu = 12. 889 6
V = 7. 073 4

Vu2 d = 12. 814


Vud = 5. 699 4

Vd = 2. 534 8

Vud2 = 0
Vd2 = 0
Vd3 = 0

Tip 11.2.2. Real probability pricing requires intensive calculation. Not only
do we need to find the real probability of up and down, we also need to find
the replicating portfolio at each node. In contrast, in risk neutral pricing, we
either use risk neutral probabilities or use replicating portfolio but not both. In
comparison, risk neutral pricing is more ecient than real probability pricing.

11.2.2

Binomial tree and lognormality

Random Walk model


Heres a brief review of the random walk model. There are 3 schools of thoughts:
the chartist approach (or technical analysis), fundamental analysis, and the
random walk model. Those who use the chartist approach draw charts to predict
stock future prices. They believe that history repeats itself and that past stock
prices help predict future stock prices. Fundamental analysis believes that at
any point the stock has an intrinsic value that depends on the earning potential
of the stock. Random Walk model, on the other hand, believes that the price of
a stock is purely random and that past price cant help predict the stock price
in the future.
Is the random walk model true? Is stock price purely random? Some scholars
challenged the random walk theory. If interested, you can look into the book
A Non-Random Walk Down Wall Street at Amazon.com http://www.amazon.
com/
Now lets look at the random walk math model. Imagine that an object zero
starts from point zero and travels along a straight line (such as Y axis). At each
step, the object either move up by 1 unit with probability p or move down by 1
unit with probability 1 p. An equivalent description of the movement is this:
at each move a coin is tossed. If we get a head, the object moves up by 1 unit;
if we get a tail, the object moves down by 1 unit.
Let Y
i represent the movement in the i-th step. Then
1
with probability p
Yi =
1 with probability 1 p
Let Zn represent the position of the article after n steps. Then
Zn = Y1 + Y2 + ... + Yn
Y1 , Y2 , ...Yn are independent identically distributed.

11.2. UNDERSTANDING RISK-NEUTRAL PROBABILITY

89

To apply the random walk model to stock prices, we can use Yi to represent
the price movement during an interval of time. We can use Zn to represent the
ending price of a stock after n equal intervals.
To get a good feel of the random variable Zn , check out the simulation of
the random walk model at
http://math.furman.edu/~dcs/java/rw.htmll
Just type in the url in your web browser.
Modeling stock as a random walk
There are 3 problems if we use Zn = Y1 + Y2 + ... + Yn to model the price of a
stock:
1. Zn can be negative yet the price of a stock cant be negative.
2. The incremental change of any stock price Yi is either 1 or 1. Though
$1 change might be OK for modeling the change of a low priced stock, it
may be inappropriate to model the change of a high priced stock.
3. The expected return on stock whose price following a random walk is zero,
that is E (Zn ) = nE (Y ) = n 0 = 0. However, stock on average should
have a positive return.
Continuously compounded returns
Let rt,t+h represent the continuously compounded return earned during the time
interval [t, t + h]. Let St and St+h represent the stock price at time t and t + h
respectively. Then
St+h
St+h
St ert,t+h = St+h
ert,t+h =
rt,t+h = ln
St
St
Continuously compounded returns are additive. Consider the time interval
[t, t + nh]. We have:
St+nh
St+h
St+2h
St+3h
St+nh
ert,t+nh =
=

...
St
St
St+h
St+2h
St+(n1)h
= ert,t+h ert+h,t+2h ert+2h,t+3h ...ert+(n1)h,t+nh
rt,t+nh = rt,t+h +rt+h,t+2h +rt+2h,t+3h + ... + rt+(n1)h,t+nh
Continuously compounded returns can be negative. Even if r < 0, we still
have er > 0. Thus if ln S follows a random walk, S cant be negative.
Standard deviation of returns
The annual return is the sum of the returns in each of the 12 months:
rannual = rJan + rF eb + ... + rDec
Assume each monthly return is independent identical distributed with common variance 2Monthly . Let 2 represent the variance of the return over 1 year
period.

90

CHAPTER 11. BINOMIAL OPTION PRICING: II

V ar (rannual ) = V ar (rJan + rF eb + ... + rDec ) = 12 2Monthly

2 = 122M onthly
Monthly =
12
Suppose we split one year into n intervals with each interval being h long (so
nh = 1). Let h represent the standard deviation of the return over an interval
of h long, then
rannual = r0,h + rh,2h + r2h,3h
+ ... + r(n1)h,nh

2
= V ar (rannual ) = V ar r0,h + rh,2h + r2h,3h + ... + r(n1)h,nh = n 2h

h =
n

However, nh = 1. So we have: h = = h
n
Binomial model

Previously, we set u = e(r)h+ h and d =


e(r)h h . Now lets
see why do

(r)h+ h
(r)h h
and d = e
is equivalent
ing so is reasonable. Setting u = e
to setting

St+h = St e(r)h h , which is equivalent to

rt,t+h = (r ) h h
(11.4)
Lets see why Equation 11.4 solves the three problems in the random walk
model:
1. Even when rt,t+h is negative, the stock price St+h is always positive.
2. The change
priceiis proportional to the stock price. S = St+h
h in stock

(r)h h
1 .
St = St e

3. The expected return during [t, t + h] is largely driven by the constant term
(r ) h. Hence the expected return is no long always zero.
Alternative binomial tree
The Cox-Ross-Rubinstein binomial tree
u = e

d = e

(11.5)

(11.6)

The lognormal tree (also called the Jarrow-Rudd binomial model)


2
u = e(r0.5 )h+

2
d = e(r0.5 )h

(11.7)
(11.8)

11.2. UNDERSTANDING RISK-NEUTRAL PROBABILITY

11.2.3

91

Estimate stock volatility

Formulas
n+1
number of stock prices observed: S0 , S1 ,...,Sn1 ,and Sn
n
number of stock returns observed. Continuously compounded
return per period is estimated as
S1
S2
S3
Sn
r1 = ln
r2 = ln
r3 = ln
...rn = ln
S0
S1
S2
Sn1
Since our focus is stock returns not stock price, the number of observations
is n (i.e. the number of stock prices observed minus one). Remember this point.
The expected return is:
r1 + r2 + ... + rn

r=
n
The v
estimated standard deviation is:
2

u
2
2
u r
r
+
r

r
+
...
+
r

r
1
2
n
t

=
n1
Example 11.2.3. Reproduce the textbook Figure 11.1 and estimate the standard
deviation of the continuously compounded return per year earned by S&P 50
index.

St
2
Week S&P 500 price rt =ln
rt r
St1
0
829.85
1
804.19
0.0314
0.001846
2
874.02
0.0833
0.005143
3
869.95
0.0047
0.000263
4
880.9
0.0125
0.000001
5
865.99
0.0171
0.000819
6
879.91
0.0159
0.000019
7
919.02
0.0435
0.001020
8
916.92
0.0023
0.000191
9
929.62
0.0138
0.000005
10
939.28
0.0103
0.000001
11
923.42
0.0170
0.000817
12
953.22
0.0318
0.000409
Total
0.1386
0.010534
The expected continuously compounded return per week is estimated as:
r1 + r2 + ... + r12
0.1386

r=
=
= 0.011 55
n
12
The standard deviation of the continuously compounded return per week is
estimated as:

92

CHAPTER 11. BINOMIAL OPTION PRICING: II


r

0.010534
= 0.03 095
12 1
1 Year = 52 Weeks

Let Y represent the continuously compounded return per year and Xi represent the continuously compounded return earned in the i-th week.
Then Y = X1 + X2 + ... + X52
Where X1 , X2 , ..., X52 are assumed to be independent identically distributed.
V ar (Y) = V ar (X
1 + X2 + ... + X52 ) = 52V ar (X)
Y = 52 Y = 52 (0.03 095) = 0.223 18
Please note my calculation was done using Excel. If you cant perfectly
reproduce my numbers, thats fine.
By the way, in Excel, the formula for the mean is AVERAGE; the formula
for the sample variance is VAR
This is how to use Excel to calculate the continuously compounded return
per year earned by S&P 500 index. Suppose the stock prices are entered in
Column C (from C3 to C15) and the weekly returns are calculated in Column
D (from D4 to D15).
1

Column B

Column C

Week

S&P 500 price

Column D
St
rt =ln
St1

3
0
829.85
4
1
804.19
0.0314
5
2
874.02
0.0833
6
3
869.95
0.0047
7
4
880.9
0.0125
8
5
865.99
0.0171
9
6
879.91
0.0159
10
7
919.02
0.0435
11
8
916.92
0.0023
12
9
929.62
0.0138
13
10
939.28
0.0103
14
11
923.42
0.0170
15
12
953.22
0.0318
The expected continuously compounded return per week is:

r =AVERAGE(D4:D15)= 0.01155006
The sample variance of the continuously compounded return per week is
estimated as:
2
=VAR(D4:D15)= 0.000957682

= 0.000957682 = 0.030946434
Make sure you dont use the population variance formula:
VARP(D4:D15)= 0.000877875

11.2. UNDERSTANDING RISK-NEUTRAL PROBABILITY

93

In Excel, VARP is:

2
2
2
r1 r + r2 r + ... + rn r
V ARP =
n
While VAR iscalculated
as

2
2
2
r1 r + r2 r + ... + rn r
V AR =
n1
So V AR = V ARP

n
n1

Finally, Ill give you a BA II Plus (or BA II Plus Professional) calculator


shortcut for quickly finding the sample mean and the sample standard deviation.
BA II Plus and BA II Plus Professional have a 1-V Statistics Worksheet. This
worksheet can calculate the sample mean, the sample standard deviation, the
population mean (which is the same as the sample mean), and the population
standard deviation.
In 1-V Statistics Worksheet, enter:
1 Column B
Column C Column D
St
2
Week S&P 500 price rt =ln
St1
3
0
829.85
4
1
804.19
0.0314
5
2
874.02
0.0833
6
3
869.95
0.0047
7
4
880.9
0.0125
8
5
865.99
0.0171
9
6
879.91
0.0159
10
7
919.02
0.0435
11
8
916.92
0.0023
12
9
929.62
0.0138
13
10
939.28
0.0103
14
11
923.42
0.0170
15
12
953.22
0.0318
804.19
X01 = ln
= 0.03 140 940
829.85
Y 01 = 1 (we observed X01 only once)
874.02
X02 = ln
= 0.08 326 770
804.19
Y 02 = 1 (we observed X02 only once)

So on and so forth. The final entry is:


953.22
X12 = ln
= 0.03 176 156
923.42
Y 12 = 1 (we observed X12 only once)

94

CHAPTER 11. BINOMIAL OPTION PRICING: II

Press 2nd STAT of the calculator key, you should get:


n = 12
X = 0.01155006
This is the sample mean (or population mean) of the continuously compounded return per week.
SX = 0.03094643
This is the estimated standard deviation (or sample standard deviation) of
the continuously compounded return per week.
X = 0.02962895
This is the estimated standard deviation (or population standard deviation)
of the continuously compounded return per week.
You should use SX and discard X when estimating the stock volatility.
Example 11.2.4. Reproduce the textbook Figure 11.1 and estimate the standard
deviation of the continuously compounded return per year earned by IBM.
Week

S&P 500 price

0
1
2
3
4
5
6
7
8
9
10
11
12

77.73
75.18
82
81.55
81.46
78.71
82.88
85.75
84.9
86.68
88.7
86.18
87.57

rt =ln

St
St1

0.0334
0.0868
0.0055
0.0011
0.0343
0.0516
0.0340
0.0100
0.0207
0.0230
0.0288
0.0160

In BA II Plus (or BA II Plus Professional) 1-V Statistics Worksheet, enter:


X01 = 0.0334
Y 01 = 1
X02 = 0.0868
Y 02 = 1
X03 = 0.0055
Y 03 = 1
......
X13 = 0.0160
Y 12 = 1
You should get: SX = 0.0365
So the estimated standard deviation (or sample standard deviation) of the
continuously compounded return per week is:
X = 0.0365
The standard deviation of the continuously compounded return per year is:

11.3. STOCKS PAYING DISCRETE DIVIDENDS

95

52 (0.0365) = 0.263 2

11.3

Stocks paying discrete dividends

Previously, we assume that dividends are paid continuously at rate . However,


in reality, dividends are paid discretely (such as quarterly or annually). Now
lets build a binomial tree to calculate the price of a stock that pays discrete
dividends.
Suppose we have a European option on a stock that pays a discrete dividend.
The option is written at t (today) and expires in t+h. The stock pays a dividend
or several dividends during [t, t + h]. The future value of the dividends at t + h
is D. The continuously compounded risk-free interest rate per year is r ( a
positive constant). The stock price today is St . At t + h, the stock price either
goes up to Stu = uSt or goes down to Std = dSt . The standard deviation of the
continuously compounded return earned by the stock per year is . We want
to calculate the option price.

Using Equation 10.22 and 10.23, we have:

Stu = uSt = Ft,t+h e h


Std = dSt = Ft,t+h e h
However, Ft,t+h = St erh D
This gives us:


Stu = St erh D e h

Std = St erh D e h

(11.9)
(11.10)

To find the price of the European option, we calculate the cost of the replicating portfolio. We have two assets: the stock and a savings account. The
savings account is the same as a zero-coupon bond. At time t + h, the stock
price is Sh ; the bond price is Bt+h . The bond price is deterministic:
Bt+h = erh
Bt = 1
The stock
( price at t + h is stochastic:

Stu = St erh D e h

St+h =
Std = St erh D e h
So at t + h the stock price either goes up to Stu ("up state") or goes down
to Std ("down state").

96

CHAPTER 11. BINOMIAL OPTION PRICING: II


Stu
St
Time t

Std
Time t + h

Our task is to determine C by setting a portfolio that replicates the option


payo of Cu in the up state and Cd in the down state. We build the replicating
portfolio by buying 4 stocks and investing $B in a zero-coupon bond.

If we own one stock at t, then at t + h our total wealth is St+h + D. We


not only can sell the stock in the market for St+h , well also have D, the future
value of the dividend earned during [t, t + h].
So we need to set up the following equation:
4St
t

4 (Stu + D)

4 Std + D
t+h

Berh
+

Cu
=

Berh
t+h

C
t

Cd
t+h

4 (Stu + D) + Berh = Cu
4 Std + D + Berh = Cd
Solving these equations, we get:

4=

B = erh

11.3.1

Cu Cd
Stu Std

Stu Cd Std Cu
4D
Stu Std

(11.11)

(11.12)

Problems with discrete dividend tree

One major problem with the stock price tree using Equation 11.9 and 11.10 is
that the tree doesnt complete recombine after the discrete dividend.
Example 11.3.1. Reproduce the textbook Figure 11.1 but just for the 2 periods.
Show that the stock price tree doesnt recombine at Period 2. This is the recap
of the information. The current stock price is 41. The stock pays a dividend
during Period 1 and Period 2. The future value of the dividend accumulated
at the risk-free interest rate r from Period 1 to Period 2 is 5. Other data are:
r = 0.08, = 0.3, t = 1,and h = 1/3.

11.3. STOCKS PAYING DISCRETE DIVIDENDS


Period 0

Stu

2
Stuu = 55. 203
= 50. 071
Stud

97

3
Stuuu = 67. 417 15
Stuud = Studu = 47. 678 91

= 39. 041
Studd = 33. 719 59

St = 41

Std

Stdu = 37. 300


= 35. 411
Stdd

Stduu = 45. 553 05


Stdud = Stddu = 32. 216 14

= 26. 380
Stddd = 22. 783 97

No dividend is paid during Period 0 and Period 1.

Stu = St erh D e h = 41e0.081/3 0 e0.3 1/3 = 50. 071 09

Std = St erh D e h = 41e0.081/3 0 e0.3 1/3 = 35. 411 39

Dividend is paid during


Period 1 to Period 2, whose value
at Period 2 is 5.

Stuu = Stu erh D e h = 50. 071 09e0.081/3 5 e0.3 1/3 = 55. 203 57

Stud = Stu erh D e h = 50. 071 09e0.081/3 5 e0.3 1/3 = 39. 041 20

Stdu = Std erh D e h = 35. 411 39e0.081/3 5 e0.3 1/3 = 37. 300 47

Stdd = Std erh D e h = 35. 411 39e0.081/3 5 e0.3 1/3 = 26. 379 73

Now you see that Stud 6= Stdu , . The tree doesnt recombine.
No dividend is paid during
Period 2 and Period 3.

Stuuu = Stuu erh D e h = 55. 203 57e0.081/3 0 e0.3 1/3 = 67. 417 15

Stuud = Stuu erh D e h = 55. 203 57e0.081/3 0 e0.3 1/3 = 47.


678 91

Studu = Stud erh D e h = 39. 041 20e0.081/3 0 e0.3 1/3 = 47. 678 91

Studd = Stud erh D e h = 39. 041 20e0.081/3 0 e0.3 1/3 = 33.


719 59

Stduu = Stdu erh D e h = 37. 300 47e0.081/3 0 e0.3 1/3 = 45. 553 05

Stdud = Stdu erh D e h = 37. 300 47e0.081/3 0 e0.3 1/3 = 32.


216 14

Stddu = Stdd erh D e h = 26. 379 73e0.081/3 0 e0.3 1/3 = 32. 216 14

Stddd = Stdd erh D e h = 26. 379 73e0.081/3 0 e0.3 1/3 = 22.


783 97
Please note that in this example Stuud = Studu and Stdud = Stddu . This is not
a coincidence.

98

CHAPTER 11. BINOMIAL OPTION PRICING: II


Lets verify that Stuud = Studu .


Stu = St erh 0 e h = Sterh e h


Stuu = Stu erh 5 e h = St erh e h 5 e h

Stuud = Stuu erh 0 e h = St erh e h 5 e h erh e h

= St erh e h 5 erh

Stud = Stu erh 5 e h = St erh e h 5 e h


Studu = Stud erh 0 e h = St erh e h 5 e h erh 0 e h

= St erh e h 5 erh

Clearly, Stuud = Studu . Similarly, you can verify for yourself that Stdud =

Stddu .

In this problem, Period 2 had 4 prices. If the stock pays continuous dividend,
Period 2 will have only 3 prices.
Similarly, in this problem, Period 3 has 5 distinct prices. In contrast, if the
stock pays continuous dividend, Period 3 will have only 4 prices.
In addition to non-combining stock prices, the above method may produce
negative stock prices.

11.3.2

Binomial tree using prepaid forward

Schroder presents a method that overcomes the two shortcomings of the above
method.
This is the idea behind Schroders method. Instead of directly building a
stock price tree (which proves to be non-combining after the discrete dividend is
paid), well build a tree of a series of prepaid forward prices on the same stock.
Hopefully, the prepaid forward
price

tree is recombining and looks like this:


P
Ft,T

u
P
Ft+h,T

d
P
Ft+h,T

In the above table,

P
Ft+2h,T

uu

ud
P
Ft+2h,T

dd
P
Ft+2h,T

P
Ft,T
is the prepaid price of a forward contract signed at t expiring on date
T

11.3. STOCKS PAYING DISCRETE DIVIDENDS

99

P
Ft+h,T
is the prepaid price of a forward contract signed at t + h expiring
on date T
P
Ft+2h,T
is the prepaid price of a forward contract signed at t + 2h expiring
on date T

Since theres a one-to-one relationship between a prepaid forward price and


the stock price, we can change the (recombining) prepaid forward price tree into
a (recombining) stock price tree. Once we have a recombining stock price tree,
well can easily find the price of the option using either risk-neutral probability
or the replicating portfolio.
First, lets find the relationship between the stock price and the prepaid
forward price on the stock. Suppose today is time t . At t we enter into a
forward contract agreeing to buy a stock on date T . The stock will pay dividend
D on date TD where TD < T .
Suppose we want to prepay the seller at t. The price of this prepaid forward
contract is the current stock price
minus the present value of the dividend:
St r(T
D t)
if TD t
De
P
= St P Vt (D) = St
Ft,T
0
if TD < t

Der(TD t) if TD t
P
St = Ft,T +
0
if TD < t
Similarly,

Der(TD th)
= St+h P Vt+h (D) = St+h
0

r(TD th)
if TD t + h
De
P
+
St+h = Ft+h,T
0
if TD < t + h
P
Ft+h,T

if TD t + h
if TD < t + h

So theres a one-to-one mapping between the prepaid forward price and the
stock price.
Next, lets find out how to build a prepaid forward price tree. We need to
know how the prepaid forward price changes over time. Suppose today is time
t . At t + h we enter into a forward contract agreeing to buy a stock at date T
where T > t + h. The stock will pay dividend D in date TD where t < TD < T .
If the stock volatility is zero (meaning that the future stock price is known today
with 100% certainty), then the price of the prepaid forward contact at t + h is
P
P rh
Ft+h,T
= Ft,T
e
P
Ft+h,T
P
Ft,T

= erh

(11.13)

(11.14)

P
at t, well receive one stock
This is why Equation 11.13 holds. If we pay Ft,T
P
P
at T . This gives us Ft,T
= ST er(T t) . Similarly, if we pay Ft+h,T
at t + h, well

100

CHAPTER 11. BINOMIAL OPTION PRICING: II

P
also receive a stock at T . This gives us Ft+h,T
= ST er(T th) . Then Equation
11.13 holds.
Now suppose that the forward price has a volatility of F per year. Then
its reasonable
( to assume that

P rh h
P
F
e
e
= Ft,T
u
in the up state
t,T
P

Ft+h,T
=
P rh h
P
Ft,T
e e
= Ft,T
d in the down state

where u = erh+F
P
Similarly, Ft+2h,T

and d = erhF h

P
Ft+h,T
u
in the up state
=
P
d in the down state
Ft+h,T

Typically, we know the volatility of the stock, S . We need to calculate F


using the following approximation:
St
P
Ft,T
Now the prepaid forward pricetree is:
uu
P
P
= Ft,T
u2
Ft+2h,T

u
P
P
= Ft,T
u
Ft+h,T

ud
du
P
P
P
P
Ft,T
= Ft+2h,T
= Ft,T
ud
Ft+2h,T

d
P
P
= Ft,T
d
Ft+h,T

dd
P
P 2
= Ft,T
d
Ft+2h,T

F = S

Once we have the prepaid forward price tree, well transform it into the stock
price tree:

P
St = Ft,T
+ P Vt (D)

u
u
P
St+h
= Ft+h,T
+ P Vt+h (D)

d
d
P
St+h
= Ft+h,T
+ P Vt+h (D)

uu
uu
P
St+2h
= Ft+2h,T
+ P Vt+2h (D)

ud
ud
P
St+2h
= Ft+2h,T
+ P Vt+2h (D)

dd
dd
P
St+2h
= Ft+2h,T
+ P Vt+2h (D)

Example 11.3.2. Let s reproduce the textbook Figure 11.11. Here is the recap
of the information on an American call option. The stock pays a dividend of $5
in 8 months. Current stock price is $41. The strike price K = $40. The stock
volatility is S = 0.3. The continuously compounded risk-free rate is r = 0.08.
The option expires in T = 1 year. Use a 3-period binomial tree to calculate the
option price.

11.3. STOCKS PAYING DISCRETE DIVIDENDS


First, well build a prepaid forward price tree.

F = S

St
P
Ft,T

P
Ft,T
= St P Vt (D) = 41 5e0.08(8/12) = 36. 26
St
41
= 0.339 2
F = S P = 0.3
36. 26
Ft,T

u = erh+F h = e0.08(1/3)+0.3392 1/3 = 1. 249 20


d = erhF h = e0.08(1/3)0.3392 1/3 = 0.844 36

Prepaid forward price tree:

P
Ft,T

u
P
Ft+h,T

d
P
Ft+h,T

uu
P
Ft+2h,T

ud
P
Ft+2h,T

dd
P
Ft+2h,T

uuu
P
Ft+3h,T

uud
P
Ft+3h,T

udd
P
Ft+3h,T

ddd
P
Ft+3h,T

u
P
P
= Ft,T
u = 36. 26 1. 249 20 = 45. 296
Ft+h,T

d
P
P
Ft+h,T
= Ft,T
d = 36. 26 0.844 36 = 30. 616

uu
u

P
P
P
= Ft,T
u2 = Ft+h,T
u = 45. 296 1. 249 20 = 56. 584
Ft+2h,T

ud
u
P
P
Ft+2h,T
= Ft+h,T
d = 45. 296 0.844 36 = 38. 246

dd
d
P
P 2
P
Ft+3h,T
= Ft,T
d = Ft+h,T
d = 30. 616 0.844 36 = 25. 851
uuu
uu

P
P
= Ft+2h,T
u = 56. 584 1. 249 20 = 70. 685
Ft+3h,T

uud
uu
P
P
Ft+3h,T
= Ft+2h,T
d = 56. 584 0.844 36 = 47. 777

udd
ud
P
P
Ft+3h,T
= Ft+2h,T
d = 38. 246 0.844 36 = 32. 293

ddd
dd
P
P
Ft+3h,T
= Ft+2h,T
d = 25. 851 0.844 36 = 21. 828
Prepaid forward price tree:

101

102

CHAPTER 11. BINOMIAL OPTION PRICING: II


70. 685
56. 584
45. 296
36. 26

47. 777
38. 246

30. 616

32. 293
25. 851

21. 828
Next, we change the prepaid forward price tree into a stock price tree. The
one-to-one mapping between
the prepaid forward price and the stock price is

r(TD tt)
if TD t + t
De
P
+
St+t = Ft+t,T
0
if TD < t + t
where 0 t T t

uuu
St+3/3

uu
St+2/3

uud
St+1/3
St+3/3

ud
St
St+2/3

udd
St+1/3
St+3/3

dd
St+2/3

ddd
St+3/3
8
+ t, D = 5,T = t + 1
12
r(TD t)
+ De
= 36. 26 + 5e0.08(8/12) = 41. 000

In this problem, TD =
P
St = Ft,T

u
u P

+ Der(TD t1/3) = 45. 296 + 5e0.08(8/121/3)


St+1/3 = Ft+1/3,T
50. 164
d

d P
St+1/3 = Ft+1/3,T
+ Der(TD t1/3) = 30. 616 + 5e0.08(8/121/3)
35. 484
uu

uu P
= Ft+2/3,T
+Der(TD t2/3) = 56. 584+5e0.08(8/122/3)
St+2/3
61. 584
ud

ud P
St+2/3
= Ft+2/3,T
+Der(TD t2/3) = 38. 246+5e0.08(8/122/3)
43. 246
dd

dd P
St+2/3
= Ft+2/3,T
+ Der(TD t2/3) = 25. 851 + 5e0.08(8/122/3)
30. 851
uuu

uuu P
= Ft+3/3,T
= 70. 685 (because TD < t + 3/3)
St+3/3
Similarly,
uud

uud P
St+3/3
= Ft+3/3,T
= 47. 777

udd

udd
P
St+3/3
= Ft+3/3,T
= 32. 293

ddd

ddd
P
St+3/3
= Ft+3/3,T
= 21. 828

=
=
=
=
=

11.3. STOCKS PAYING DISCRETE DIVIDENDS

103

So the stock price tree is:


70. 685
61. 584
50. 164
41. 000

47. 777
43. 246

35. 484

32. 293
30. 851
21. 828

After getting the stock price tree, we calculate the price of the American
call option as usual. We work backward from right to left. At each node, we
compare the backwardized value with the exercise value, taking the maximum
of the two.
The risk neutral probabilities are:
erh d
e0.08(1/3) 0.844 36
u =
=
= 0.451 2
ud
1. 249 20 0.844 36
d = 1 0.451 2 = 0.548 8
Payo tree:
Vuuu = 30. 685
Vuu = 21. 584
Vu = 11. 308
V = 5.770

Vuud = 7. 777
Vud = 3. 417

Vd = 1. 501

Vudd = 0
Vdd = 0
Vddd = 0

Vuuu = Cuuu = max (0, 70. 685 40) = 30. 685


Vuud = Cuud = max (0, 47. 777 40) = 7. 777
Vudd = Cudd = max (0, 32. 293 40 ) = 0
Vddd = Cddd = max (0, 21. 828 40) = 0
Cuu = (30. 685 0.451 2 + 7. 777 0.548 8) e0.08(1/3) = 17. 636
EVuu = max (0, 61. 584 40) = 21. 584
Vuu = max (17. 636 , 21. 584) = 21. 584
Cud = (7. 777 0.451 2 + 0 0.548 8) e0.08(1/3) = 3. 417
EVud = max (0, 43. 246 40) = 3. 246
Vud = max (3. 417 , 3. 246) = 3. 417
Cdd = 0
EVdd = max (0, 30. 851 40) = 0
Vud = 0
Cu = (21. 584 0.451 2 + 3. 417 0.548 8) e0.08(1/3) = 11. 308
EVu = max (0, 50. 164 40) = 10. 164

104

CHAPTER 11. BINOMIAL OPTION PRICING: II

Vu = max (11. 308, 10. 164) = 11. 308


Cd = (3. 417 0.451 2 + 0 0.548 8) e0.08(1/3) = 1. 501
EVd = max (0, 35. 484 40) = 0
Vd = max (1. 501 , 0) = 1. 501
C = (11. 308 0.451 2 + 1. 501 0.548 8) e0.08(1/3) = 5. 770
EV = max (0, 41. 000 40) = 1.0
V = max (5. 770, 1.0) = 5. 77

Chapter 12

Black-Scholes
Except the option Greeks and the barrier option price formula, this chapter is
an easy read.

12.1

Introduction to the Black-Scholes formula

12.1.1

Call and put option price

The price of a European call option is:


C (S, K, , r, T, ) = SeT N (d1 ) KerT N (d2 )

(12.1)

The price of a European put option is:


P (S, K, , r, T, ) = SeT N (d1 ) + KerT N (d2 )

S
1 2
ln
+ r+ T
K
2

d1 =
T

d2 = d1 T

(12.2)

(12.3)
(12.4)

Notations used in Equation 12.1, 12.3, and 12.4:


S, the current stock price (i.e. the stock price when the call option is
written)
K, the strike price
r, the continuously compounded risk-free interest rate per year
, the continuously compounded dividend rate per year
105

106

CHAPTER 12. BLACK-SCHOLES

, the annualized standard deviation of the continuously compounded


stock return (i.e. stock volatility)
T , option expiration time
N (d) = P (z d) where z is a standard normal random variable
C (S, K, , r, T, ), the price of a European call option with parameters
(S, K, , r, T, )
P (S, K, , r, T, ), the price of a European put option with parameters
(S, K, , r, T, )
Tip 12.1.1. To help memorize Equation 12.2, we can rewrite Equation 12.2
similar to Equation 12.1 as P (S, K, , r, T, ) = (S) eT N (d1 )+(K) erT N (d2 ).
In other words, change S,K,d1 ,and d2 in Equation 12.1 and youll get Equation
12.2.

Example 12.1.1. Reproduce the textbook example 12.1. This is the recap of
the information. S = 41, K = 40,r = 0.08, = 0.3, T = 0.25 (i.e. 3 months),
and = 0. Calculate the price of the price of a European call option.

S
1 2
ln
+ r+ T
K
2

d1 =

T
1
41
+ 0.08 0 + 0.32 0.25
ln
40
2

= 0.3730
=
0.3 0.25

d2 = d1 T = 0.3730 0.3 0.25 = 0.2230


N (d2 ) = 0.588 2
N (d1 ) = 0.645 4
0(0.25)
0.645 4 40e0.08(0.25) 0.588 2 = 3. 399
C = 41e

Example 12.1.2. Reproduce the textbook example 12.2. This is the recap of
the information. S = 41, K = 40,r = 0.08, = 0.3, T = 0.25 (i.e. 3 months),
and = 0. Calculate the price of the price of a European put option.
N (d1 ) = 1 N (d1 ) = 1 0.645 4 = 0.354 6
N (d2 ) = 1 N (d2 ) = 1 0.588 2 = 0.411 8
P = 41e0(0.25) 0.354 6 + 40e0.08(0.25) 0.411 8 = 1. 607

12.2. APPLYING THE FORMULA TO OTHER ASSETS

12.1.2

107

When is the Black-Scholes formula valid?

Assumptions under the Black-Scholes formula:


Assumptions about the distribution of stock price:
1. Continuously compounded returns on the stock are normally distributed
(i.e. stock price is lognormally distributed) and independent over time
2. The volatility of the continuously compounded returns is known and constant
3. Future dividends are known, either as a dollar amount (i.e. D and TD are
known in advance) or as a fixed dividend yield (i.e. is a known constant)
Assumptions about the economic environment
1. The risk-free rate is known and fixed (i.e. r is a known constant)
2. There are no transaction costs or taxes
3. Its possible to short-sell costlessly and to borrow at the risk-free rate

12.2

Applying the formula to other assets

12.2.1

Black-Scholes formula in terms of prepaid forward


price

P
The prepaid forward price for the stock is: F0,T
(S) = SeT
P
The prepaid forward price for the strike asset is: F0,T
(K) = P V (K) =
rT
Ke

Define V (T ) = T
The price of a European call option in terms of repaid forward is:

P
P
P
P
(S) , F0,T
(K) , V (T ) = F0,T
(S) N (d1 ) F0,T
(K) N (d2 )
C F0,T

(12.5)

The price of a European put option in terms of prepaid forward is:


P

P
P
P
P F0,T
(S) , F0,T
(K) , V (T ) = F0,T
(S) N (d1 ) + F0,T
(K) N (d2 ) (12.6)
ln
d1 =

P
F0,T
(S)

P
F0,T

1
+ V 2 (T )
(K) 2
V (T )

d2 = d1 V (T )

(12.7)
(12.8)

108

12.2.2

CHAPTER 12. BLACK-SCHOLES

Options on stocks with discrete dividends

When the stock pays discrete dividends, the prepaid forward price is:
P
(S) = S P V0,T (Div)
F0,T
P
(S) in Equation 12.5 and 12.6, you should get the price of the
Apply F0,T
European call and put where the stock pays discrete dividends.
Example 12.2.1. Reproduce the textbook example 12.3. Here is the recap of
the information. S = 41, K = 40, = 0.3, r = 0.08, and T = 0.25 (i.e. 3
months). The stock pays dividend of 3 in 1 month, but makes no other payouts
over the life of the option (so = 0). Calculate the price of the European call
and put.
P
(S) = S P V0,T (Div) = 41 3e(0.08)1/12 = 38. 020
F0,T
P
(K) = P V (K) = KerT = 40e0.08(0.25) = 39. 208
F0,T

V (T ) = T = 0.3 0.25 = 0.15


P
(S)
F0,T
1
38. 020 1
+ V 2 (T )
ln P
ln
+ 0.152
F0,T (K) 2
39.
208
2
d1 =
=
= 0.130 1
V (T )
0.15
d2 = d1 V (T ) = 0.130 1 0.15 = 0.280 1
N (d2 ) = 0.389 7
N (d1 ) = 0.448 2
N (d1 ) = 1 0.448 2 = 0.551 8
N (d2 ) = 1 0.389 7 = 0.610 3
P
P
(S) N (d1 ) F0,T
(K) N (d2 )
C = F0,T
= 38. 020 (0.448 2) 39. 208 (0.389 7) = 1. 76
P
P
P = F0,T
(S) N (d1 ) + F0,T
(K) N (d2 )
= 38. 020 (0.551 8) + 39. 208 (0.610 3) = 2. 95

12.2.3

Options on currencies

Notation
x, the current dollar value of 1
K, the strike price in dollars of 1
r, the continuously compounded risk-free rate earned by $1
rf , the continuously compounded risk-free rate earned by 1
, the annualized standard deviation of the continuously compounded
return on dollars
T , the option expiration date
C (x0 , K, , r, T, rf ), the price of a European call option with parameters
(x0 , K, , r, T, rf )

12.2. APPLYING THE FORMULA TO OTHER ASSETS

109

P (x0 , K, , r, T, rf ), the price of a European put option with parameters


(x0 , K, , r, T, rf )
The price of a European call option is:
C (x, K, , r, T, rf ) = xerf T N (d1 ) KerT N (d2 )

(12.9)

The price of a European put option is:


P (x, K, , r, T, rf ) = xerf T N (d1 ) + KerT N (d2 )
ln
d1 =

1
x
+ r rf + 2 T
K
2

d2 = d1 T

(12.10)

(12.11)
(12.12)

Tip 12.2.1. For currency options, just set S = x and = rf and apply the
Black-Scholes formulas on European call and put. The same thing happened in
Equation 10.26 and 10.27.
Example 12.2.2. Reproduce the textbook example 12.4. Here is the recap of the
information. The current dollar price of 1 is $0.92. The strike dollar price of
1 is $0.9. The annualized standard deviation of the continuously compounded
return on dollars is = 0.1. The continuously compounded risk-free rate earned
by dollars is r = 6%. The the continuously compounded risk-free rate earned
by 1 is rf = 3.2%. The option expires in 1 year. Calculate the price of the
European call and put on 1.

1 2
x
+ r rf + T
ln
K
2

d1 =

0.92
1
ln
+ 0.06 0.032 + 0.12 1
0.9
2

=
= 0.549 8
0.1 1

d2 = d1 T = 0.549 8 0.1 1 = 0.449 8


N (d1 ) = 0.708 8
N (d1 ) = 1 N (d1 ) = 1 0.708 8 = 0.291 2
N (d2 ) = 0.673 6
N (d2 ) = 1 N (d2 ) = 1 0.673 6 = 0.326 4
C = xerf T N (d1 ) KerT N (d2 )
= 0.92e0.032(1) 0.708 8 0.9e0.06(1) 0.673 6 = 0.06 06
P = xerf T N (d1 ) + KerT N (d2 )
= 0.92e0.032(1) 0.291 2 + 0.9e0.06(1) 0.326 4 = 0.017 2

110

CHAPTER 12. BLACK-SCHOLES

12.2.4

Options on futures

For a futures contract, the prepaid price is just the present value of the futures
P
P
price. Set F0,T
(F ) = F erT and F0,T
(K) = KerT , we get:
ln
d1 =

P
F0,T
(F )
P
F0,T

1
+ 2T
(K) 2

(12.13)

d2 = d1 T

(12.14)

C (F, K, , r, T ) = F erT N (d1 ) KerT N (d2 )

(12.15)

P (F, K, , r, T ) = F erT N (d1 ) + KerT N (d2 )

(12.16)

Example 12.2.3. Reproduce the textbook example 12.5. Here is the recap of
the information about the European option on a 1-year futures contract. The
current futures price for natural gas is $2.10. The strike price is K = 2.10. The
volatility is = 0.25. r = 0.055, T = 1. Calculate the price of the European
call and put.
2.10 1
F
1
ln
+ 2T
+ 0.252 (1)
K
2
2.10
2

=
d1 =
= 0.125
0.25 1
T


d2 = d1 T = 0.125 0.25 1 = 0.125
N (d2 ) = 0.450 3
N (d1 ) = 0.549 7
N (d1 ) = 1 N (d1 ) = 1 0.549 7 = 0.450 3
N (d2 ) = 1 N (d2 ) = 1 0.450 3 = 0.549 7
C = F erT N (d1 ) KerT N (d2 )
= 2.10e0.055(1) 0.549 7 2.10e0.055(1) 0.450 3 = 0.197 6
P = F erT N (d1 ) + KerT N (dd2 )
= 2.10e0.055(1) 0.450 3 + 2.10e0.055(1) 0.549 7 = 0.197 6
ln

12.3

Option the Greeks

The learning objectives in the SOAs syllabus is to Interpret the option Greeks.
The learning objective in CAS Exam 3 Financial Economics is to Interpret
the option Greeks and elasticity measures. How to derive the option Greeks
is explained in the textbook Appendix 12.B, but Appendix 21.B is excluded
from the SOA MFE and CAS FE. So you might want to focus on the learning
objective of the exam.
Of all the Greeks, delta and gamma are the most important.

12.3. OPTION THE GREEKS

12.3.1

111

Delta

Cu Cd
when we try to find the replicating
Su Sd
portfolio of a European call or put. Its the number of stocks you need to
own at time zero to replicate the discrete payo of a European call or put at
C
C
. Here =
expiration date T . If the payo is continuous, then =
S
S
is the number of stocks you need to have now to replicate the payo of the
next instant (i.e. the payo one moment later). The European call price is
C
C = SeT N (d1 ) KerT N (d2 ) and the delta for a call is call =
=
S
eT N (d1 ).
One less visible thing to know is that d1 is also a function of S. So itll tame
C
some work to derive call =
= eT N (d1 ). One naive approach is treat
S
C
N (d1 ) as a constant and get call =
= eT N (d1 ). Interestingly, this gives
S
the correct answer!
Since deriving delta is not on the syllabus, you dont need to go through the
C
= eT N (d1 ). Just memorize that call =
messy math and prove call =
S
eT N (d1 ) for a European call and put = eT N (d1 ) = eT [1 N (d1 )] =
call eT for a European put.
Other results you might want to memorize:
0 call 1
1 put 0
Delta . You already see =

Example 12.3.1. Calculate the delta of the following European call and put.
The information is: S = 25,K = 20, = 0.15, r = 6%, = 2%, and T = 1
year.

25
S
1 2
1
2
ln
ln
+ r+ T
+ 0.06 0.02 + 0.15 1
K
2
20
2

=
=
d1 =
0.15 1
T
1. 829 3
N (d1 ) = 0.966 3
N (d1 ) = 1 0.966 3 = 0.033 7
call = eT N (d1 ) = e0.02(1) 0.966 3 = 0.947 2
put = eT N (d1 ) = e0.02(1) 0.033 7 = 0.03 30

12.3.2

Gamma

Gamma is a measure of the change in delta regarding change in the underlying


stock price.

2C
=
=
S
S 2
If gamma is too large a small change in stock price will cause a big change
in . The bigger , the more often you need to adjust your holding of the
underlying stocks.

112

CHAPTER 12. BLACK-SCHOLES

Since put = call eT , then

12.3.3

put
call
=
or call = put .
S
S

Vega

Vega is the change of option price for 1% change of stock volatility (you can
think that the letter V stands for volatility).
C
V ega =
100

12.3.4

Theta

Theta is the change of option price regarding change in time when the option
is written (you can think that the letter T represents time). Let t represent the
time when the option is written and T the expiration date. Then
C (T t)
=
t

12.3.5

Rho

Rho is a measure of the change in option value regarding a 1% change in the


risk free interest rate (you can think the letter R represent r)
C
=
100r

12.3.6

Psi

Psi is a measure of the change in option value regarding a 1% change in the


dividend yield.
C
=
100

12.3.7

Greek measures for a portfolio

The portfolios Greek is just the sum of the individual Greek.


Example 12.3.2. A portfolio consists of 10 European calls and 50 otherwise
identical puts. The information about the European call and put is as follows:
S = 60,K = 65, = 0.25, r = 6%, = 4%, and T = 0.75 year. Calculate the
portfolio delta.

60
S
1 2
1
2
ln
ln
+ r+ T
+ 0.06 0.04 + 0.25 0.75
K
2
65
2

=
d1 =
=
0.25 0.75
T
0.192 2
N (d1 ) = N (0.192 2) = 1 N (0.192 2)
N (0.192 2) = 0.576 2
N (d1 ) = 1 0.576 2 = 0.423 8
N (d1 ) = N (0.192 2) = 0.576 2

12.3. OPTION THE GREEKS

113

call = eT N (d1 ) = e0.04(0.75) 0.423 8 = 0.411 3


put = eT N (d1 ) = e0.04(0.75) 0.576 2 = 0.559 2
portf olio = 10 (0.411 3) + 50 (0.559 2) = 23. 847
Example 12.3.3. You buy 20 European calls and simultaneously write 35 European puts on the same stock. The call expires in 3 months. The put both
expires in 9 months. The current stock price is 40. The call strike price is 35.
The put strike price is 45. The volatility is 20%. r = 8%, = 3%. Calculate
the delta of your portfolio.
T
Calculate call
N (d
1)

=e

40
S
1 2
1
2
ln
ln
+ r+ T
+ 0.08 0.03 + 0.2 0.25
K
2
35
2

d1 =
=
=
0.2 0.25
T
1. 510 3
N (d1 ) = 0.934 5
call = eT N (d1 ) = e0.03(0.25) 0.934 5 = 0.927 5
T
Calculate put
N (d
1)

= e

40
1 2
1
S
ln
+ r+ T
+ 0.08 0.03 + 0.22 0.75
ln
K
2
45
2

=
d1 =
=
0.2 0.75
T
0.376 9
N (d1 ) = 0.646 9
put = e0.03(0.75) 0.646 9 = 0.632 5
portf olio = 20 (0.927 5) 35 (0.632 5) = 40. 687 5
Since your write 35 puts, the delta of 35 puts is 35 (0.632 5).

12.3.8

Option elasticity and volatility

The elasticity is
% change in option price
=
% change in stock price
Suppose the stock price increase by where
Then the option price will change by .
% change in stock price=
S

% change in option price=


C

S
C
=
=
C
S

can be positive or negative.

(12.17)

The option volatility is


option = stock ||

(12.18)

114

CHAPTER 12. BLACK-SCHOLES

Example 12.3.4. Calculate the elasticity and the volatility of a European call
option and an otherwise identical European put option. The information is:
S = 80,K = 70, = 0.35, r = 5%, = 3%, and T = 0.5.

1
1
80
S
+ r + 2 T
+ 0.05 0.03 + 0.352 0.5
ln
ln
K
2
70
2

d1 =
=
=
0.35 0.5
T
0.703 7
N (d1 ) =
1 0.759 2 = 0.240 8
N (d1 ) = 0.759
2
d2 = d1 T = 0.703 7 0.35 0.5 = 0.456 2
N (d2 ) = 1 0.675 9 = 0.324 1
N (d2 ) = 0.675 9
C = SeT N (d1 )KerT N (d2 ) = 80e0.03(0.5) 0.759 270e0.05(0.5) 0.675 9 =
13. 687
P = SeT N (d1 )+KerT N (d2 ) = 80e0.03(0.5) 0.240 8+70e0.05(0.5)
0.324 1 = 3. 150
call = eT N (d1 ) = e0.03(0.5) 0.759 2 = 0.747 9
put = eT N (d1 ) = e0.03(0.5) 0.240 8 = 0.237 2
call S
0.747 9 80
call =
=
= 4. 371
C
13. 687
put S
0.237 2 80
=
= 6. 024
put =
P
3. 150
call = stock |call | = 0.35 (4. 371) = 1. 530
put = stock |put | = 0.35 (6. 024) = 2. 108

12.3.9

Option risk premium and Sharp ratio

At t = 0 the option is worth C = S + B. Suppose the option expires in h


years. Let represent the expected annual return on the stock, r the continuously compounded risk-free rate per year, and the expected continuously
compounded return earned on the option per year. According to Equation 11.3,
we have
S + B
eh =
Seh + Berh
However, S + B = C

Seh + Berh
S
S h
eh =
=
e + 1
erh
C
C
C
eh = eh + (1 ) erh

(12.19)

Equation 12.19 holds for any h. Using the Taylors expansion, we have:
1
1 + h + (h)2 + ...
2

1
1
= 1 + ah + (ah)2 + ... + (1 ) 1 + rh + (rh)2 + ...
2
2

12.4. PROFIT DIAGRAMS BEFORE MATURITY

115

For the above equation to hold for any h, it seems reasonable to assume that
1 = + (1 )
h = ah + (1 ) rh

1
1
1
2
2
2
(h) + ... =
(ah) + ... + (1 )
(rh) + ...
2
2
2
So we have h = ah + (1 ) rh or
r = ( r)

(12.20)

The Sharp ratio of an asset is the assets risk premium divided by the assets
volatility:
Sharp Ratio =

(12.21)

The Sharp ratio of an option is

Sharp Ratiooption =

( r)
r
=
= Sharp Ratiostock
stock
stock

(12.22)

So the Sharp ratio of an option equals the Sharp ratio of the underlying
stock.

12.3.10

Elasticity and risk premium of a portfolio

The elasticity of a portfolio is the weighted average of the elasticities of the


portfolio components. In contrast, the Greek of a portfolio is just the sum of
the Greeks of the portfolio components.
The risk premium of a stock portfolio is just the portfolios elasticity times
the stocks risk premium:
( r)portf olio = portf olio ( r)

12.4

Profit diagrams before maturity

12.4.1

Holding period profit

(12.23)

Example 12.4.1.
You buy a European call option that expires in 1 year and hold it for one
day. Calculate your holding profit. Information is:
The stock price is 40 when you buy the option.

116

CHAPTER 12. BLACK-SCHOLES

The stock price is still 40 one day later.


K = 40
r = 0.08
=0
= 30%
Solution.
At time zero, you buy a 1-year European option. Your purchase price is the
call price.

S
1 2
ln
+ r+ T
K
2

d1 =

T
40
1
ln
+ 0.08 0 + 0.32 1
40
2

=
= 0.416 7
0.3
1

d2 = d1 T = 0.416 7 0.3 1 = 0.116 7


N (d1 ) = 0.661 6
N (d2 ) = 0.546 5
C = 40e0(1) 0.661 6 40e0.08(1) 0.546 5 = 6. 285
One day later,your option is worth:

364
40
1
ln
+ 0.08 0 + 0.32
40
2
365
r
= 0.416 1
d1 =
364
0.3
365 r

364
d2 = d1 T = 0.416 1 0.3
= 0.116 5
365
N (d1 ) = 0.661 33
N (d2 ) = 0.546 37
C = 40e0(364/365) 0.661 33 40e0.08(364/365) 0.546 37 = 6. 274
Suppose you buy the option at t = 0 by paying 6. 285 and sell the option
one day later for 6. 274 . Your holding period profit is:
6. 274 6. 285e0.08(1/365) = 0.01 2
You buy a European call option that expires in 1 year and hold it for 6
months. Calculate your holding profit. Information is:
The stock price is 40 when you buy the option.
The stock price is 40 after 6 months.
K = 40
r = 0.08

12.4. PROFIT DIAGRAMS BEFORE MATURITY

117

=0
= 30%
At time zero, you buy a 1-year European option. Your purchase price is the
call price. As calculated before, the call price is 6. 285
6 months later,
the call is worth:
40
1
ln
+ 0.08 0 + 0.32 0.5
40
2

d1 =
= 0.294 6
0.3 0.5

d2 = d1 T = 0.294 6 0.3 0.5 = 0.082 5


N (d1 ) = 0.615 9
N (d2 ) = 0.532 9
C = 40e0(0.5) 0.615 9 40e0.08(0.5) 0.532 9 = 4. 156
Your holding profit is:
4. 156 6. 285e0.08(0.5) = 2. 385
You buy a European put option that expires in 1 year and hold it for 6
months. Calculate your holding profit. Information is:
The stock price is 40 when you buy the option.
The stock price is 42 after 6 months.
K = 40
r = 0.08
= 0.02
= 30%
At time zero, you buy a 1-year European option. Your purchase price is the
call price.

1
S
+ r + 2 T
ln
K
2

d1 =

T
40
1
2
ln
+ 0.08 0.02 + 0.3 1
40
2

=
= 0.35
0.3
1

d2 = d1 T = 0.35 0.3 1 = 0.05


N (d1 ) = 0.363 2
N (d2 ) = 0.480 1
P = 40e0.02(1) 0.363 2 + 40e0.08(1) 0.480 1 = 3. 487
One day later, your

option is worth:
42
1
ln
+ 0.08 0.02 + 0.32 0.5
40
2

= 0.477 5
d1 = =
0.3 0.5

d2 = d1 T = 0.477 5 0.3 0.5 = 0.265 4

118

CHAPTER 12. BLACK-SCHOLES

N (d1 ) = 0.316 5
N (d2 ) = 0.395 4
P = 42e0.02(0.5) 0.316 5 + 40e0.08(0.5) 0.395 4 = 2. 035
Suppose you buy the option at t = 0 by paying 3. 487 and sell the option 6
months later for 2. 035. Your holding period profit is:
2. 035 3. 487e0.08(0.5) = 1. 59

12.4.2

Calendar spread

The textbook has an intimidating diagram (Figure 12.14). Dont worry about
this diagram. Just focus on understanding what a calendar spread is.
A calendar spread (also called time spread or horizontal spread) is an option
strategy that takes advantage of the deteriorating time value of options. A
calendar spread involves selling one option that has a shorter expiring date and
simultaneously buying another option that has a longer expiration date, with
both options on the same stock and having the same strike price.
Suppose that Microsoft is trading for $40 per share. To have a calendar
spread, you can sell a $40-strike call on a Microsoft stock with option expiring
in 2 month. Simultaneously, you buy a $40-strike call on a Microsoft stock
with option expiring in 3 months. Suppose the price of a $40-strike 2-month to
expiration call is $2; the price of a $40-strike 3-month to expiration call is $5.
So your net cost of having a calendar spread at time zero is $3.
Then as time goes by, suppose the stock price doesnt move much and is still
around $40, then the value of your sold call and purchased call both deteriorate
but at a dierent deteriorating speed. The value of the $40-strike 2-month to
expiration call deteriorates much faster. With each day passing, this option has
less and less value left. If there are only several days left before expiration, the
value of the sold call will be close to zero.
With each day passing, the value of the $40-strike 3-month to expiration call
also decreases but at a slower speed.
For example, one month later, the sold call has 1 month to expiration and
is worth only $1. The purchased call has 2-month to expiration and is worth
$4.5. Now the calendar spread is worth $3.5. You can close out your position by
buying a $40-strike 1-month to expiration call (price: $1) and sell a $40-strike
2-month to expiration call (price: $4.5). If you close out your position, youll
get $3.5.
At time zero, you invest $3 to set up a calendar spread. One month later,
you close out your position and get $3.5. Your profit (assuming no transaction
cost) is $0.5.
Time zero: your cost is $3
$40-strike call $40-strike call
Value
$2
$5
Time to expiration 2 months
3 months

12.5. IMPLIED VOLATILITY

119

One month later: your total value is $3.5


$40-strike call $40-strike call
Value
$1
$4.5
Time to expiration 1 month
2 months
A calendar spread can create value because as time passes the sold option
(which is your liability) can quickly become worthless yet the purchased option
(your asset) is still worth something.
For more examples, please refer to
http://www.optionsxpress.com/educate/strategies/calendarspread.
aspx
http://www.highyieldstrategy.com/artclndrsprds.htm.

12.5

Implied volatility

12.5.1

Calculate the implied volatility

Volatility cannot be observed. One approach to estimating volatility is use past


returns to calculate the historical volatility. However, past volatility may be a
poor estimate of future volatility because the market condition may change.
Another approach to estimating volatility is to calculate the implied volatility. The call and put values depend on (S, K, T, r, , ). Given (S, K, T, r, )
and the option price, we can calculate . This is called the implied volatility.
Calculate the implied volatility given the following information about a European call.
CEuropean = 7.25
S = 60
K = 55
T = 0.75 (i.e. 9 months)
r = 0.06
= 0.02
Solution.
This is a dicult problem to solve manually. However, the calculation procedure is conceptually simple.
Implied volatility is solved by trial and error. You use a trial and see
whether the computed option price under the trial reproduces the actual
option price. If the computed option price is lower than the observed option
price, use a higher trial and try again; if the computed option price is higher

120

CHAPTER 12. BLACK-SCHOLES

than the observed option price, use a lower trial and try again. Keep doing
this until you find a such the computed option price equals the observed option
price.
First, lets try = 10%

60
1 2
ln
+ 0.06 0.02 + 0.1 0.75
55
2

d1 =
= 1. 3944
0.1 0.75

d2 = d1 T = 1. 3944 0.1 0.75 = 1. 307 8


N (d1 ) = 0.918 4
N (d2 ) = 0.904 5
0.02(0.75)
C = 60e
0.918 4 55e0.06(0.75) 0.904 5 = 6. 725
6. 725 < CEuropean = 7.25. So increase and try again.
Try = 20%

1 2
60
+ 0.06 0.02 + 0.2 0.75
ln
55
2

d1 =
= 0.762 2
0.2 0.75

d2 = d1 T = 0.762 2 0.2 0.75 = 0.5890


N (d1 ) = 0.777 0
N (d2 ) = 0.722 1
C = 60e0.02(0.75) 0.777 0 55e0.06(0.75) 0.722 1 = 7. 958
7. 958 > CEuropean = 7.25. So decrease and try again
Try = 15%

1
60
+ 0.06 0.02 + 0.152 0.75
ln
55
2

d1 =
= 0.965 7
0.15 0.75

d2 = d1 T = 0.965 7 0.15 0.75 = 0.835 8


N (d1 ) = 0.832 9
N (d2 ) = 0.798 4
C = 60e0.02(0.75) 0.832 9 55e0.06(0.75) 0.798 4 = 7. 25
7. 25 = CEuropean = 7.25.
So the implied is 15%.

12.5.2

Volatility skew

Volatility skew refers to


Options on the same stock with dierent strike price and expiration date
should have the same implied volatility. However, in reality, options on
the same stock with dierent strike price and expiration date dont have
the same implied volatility.
In addition, the dierent implied volatilities often form a pattern such as
"smiles," "frowns," and "smirks."
Volatility is explained more in Derivatives Markets Chapter 23.

12.6. PERPETUAL AMERICAN OPTIONS

12.5.3

121

Using implied volatility

Implied volatility is important because it helps us


We can generate option price thats consistent with the price of other
similar options
We can quote the option in terms of volatility rather than a dollar price
Volatility skew helps us see how well an option pricing formula works.
Volatility skew shows that the Black-Scholes formula and assumptions are
not perfect.
This is all you need to know about how to use the implied volatility.

12.6

Perpetual American options

Perpetual American options are excluded from the exam syllabus. Please ignore
this chapter.
I included this section for completeness, but you dont need to read it.

12.6.1

Perpetual calls and puts

Our task here is to derive the price formula for a perpetual American call with
strike price K.
A perpetual American option never expires and the option holder can exercise the option at any time. Its a typical American option where the expiration
date T = +.
The theoretical framework behind the perpetual option formula is the BlackScholes partial dierential equation (called the Black-Scholes PDE). The BlackScholes PDE is Derivatives Market Equation 21.11 (page 682):
1
Vt + 2 S 2 VSS + (r ) SVS rV = 0
2

(Textbook 20.11)

The above equation is also presented in Derivatives Market Equation 13.10


(page 430):
1
+ 2 St2 t + rSt t rV (St ) = 0
2

(Textbook 13.10)

Please note that Equation 13.10 assumes = 0. In addition,it uses C (instead of V ) to represent the option price.
The Black-Scholes PDE is commonly written as:
V (t, St ) 1 2 2 2 V (t, St )
V (t, St )
+ (r ) St
rV (t, St ) = 0 (12.24)
+ St
t
2
St2
St

122

CHAPTER 12. BLACK-SCHOLES

In the above equation, V (t, St ) is the option price at time t where the stock
price is St .
If you are interested in learning how to derive the Black-Scholes PDE, refer
to the textbook. For now lets accept Equation 12.24.
V
For a perpetual option, its value doesnt depends on time. Hence
= 0.
t
The Black-Scholes PDE becomes an ordinary dierential equation:
1 2 2 d2 V (t, St )
dV (t, St )
+ (r ) St
rV (t, St ) = 0
St
2
dSt2
dSt

(12.25)

To find the solution to Equation 12.25, lets simplify the equation as


d2 V (t, St )
dV (t, St )
+ St
V (t, St ) = 0
St2
dSt2
dSt
We can guess the solution is in the form of V (t, St ) = Sth . Then
d2 V (t, St )
dV (t, St )
= hSth1
= h (h 1) Sth2
dSt
dSt2
d2 V (t, St )
dV (t, St )
St2
+ St
V (t, St )
dSt2
dSt
= St2 h (h 1) Sth2 + St hSth1 Sth
= Sth [h (h 1) + h 1] = Sth h2 1

d2 V (t, St )
dV (t, St )
So as long as h2 1 = 0, or h = 1, Equation St2
+St

2
dSt
dSt
V (t, St ) = 0 has a solution.
Of course, if Sth is a solution, aSth must also be a solution.
Similarly, we can guess that the solution to Equation 12.25 is in the form of
V (t, St ) = Sth . Some brilliant thinker guessed the following solution:

h
St
H K h

V (t, St ) = (H K)
=
St = aSth
h
H
(H )
Here H the stock price where exercise is optimal (H is a constant). H K

h
St
is the terminal payo at exercise time.
is an indicator telling us how
H

close the stock price approaches H .


d h
d2 V (t, St )
dV (t, St )
=
= ah (h 1) Sth2
aSt = ahSth1
dSt
dS
dSt2
Equation 12.25 becomes:
1 2 2
St ah (h 1) Sth2 + (r ) St ahSth1 raSth = 0
2
1 2
h (h 1) + (r ) h r = 0
2

1 2 2
1 2
h + r hr =0
2
2
s
2
1
1 2
r 2 + 2 2 r
(r )
2
2
h=
2

12.6. PERPETUAL AMERICAN OPTIONS


1 r
= 2
2

1 r
= 2
2

s
s

r 1

2
2
1 r
2
2

2r
2

2r
2

123

But how can we find H ? Since the perpetual American option can be exercised at any time, the option holder will choose H such that V (t, St ) =

h
St
dV (t, St )
reaches its maximum value. This requires setting
=
(H K)

H
dH
0.
"

h #
dV (t, St )
St
d

(H K)
=
dH
dH
H
h
i
d
1h
h
)

K
(H
)
(H
= Sth
hdH
i
h
h1
h
= St (1 h) (H ) K (h) (H )
=0
(1 h) (H )h K (h) (H )h1 = 0
(1 h) H K (h) = 0
K (h)
h
H =
=
K
1h
h1
h
1
H K =
K K =
K
h1
h1

So V (t, St ) = (H K)

St
H

K
=
h1

h 1 St
h K

Set t = 0. Let S represent the stock price at time zero (i.e. S = S0 ). Then
the option value at time zero is

h
S
K
h1 S
V (0, S) = (H K)
=
H
h1
h K
But how do we choose h since there are two possible value of h? H K =
1
K. To avoid h 1 becoming negative, we choose the bigger h:
h1
s
2
1 r
1 r
2r
+
+ 2
hcall =
2
2
2
2

A similar logic can be applied to a perpetual American put. We can guess


the put value is:
h

St
K H h

V (t, St ) = (K H )
=
St = bSth
h
H
(H )
Once again, we get the following equation:

124

CHAPTER 12. BLACK-SCHOLES

1 2
h (h 1) + (r ) h r = 0
2

1
1 2 2
h + r 2 h r = 0
2
s2
2
2r
1 r
1 r
+ 2
h= 2

2
2

dV (t, St )
= 0:
dH
"
h #

h
i
d
dV (t, St )
S
t
h d

h
(K

H
=
(S
=
)
)
)
(H
)
(K

H
t
dH
dH
H
dH
h
i
d
K (H )h (H )1h
= (St )h
hdH
i
h
h1
h
= (St ) K (h) (H )
(1 h) (H )
=0

Set

K (h) (H )

h1

(1 h) (H )

=0

h
H = K
K (h) (1 h) H = 0
h

h
1
K=
K
K H = 1
h1
1h
To avoid 1 h becoming
we choose the smaller h:
s negative,
2
1 r
2r
1 r

+ 2

hput =
2
2
2
2

Summary of the formulas for perpetual American calls and perpetual American puts:

Cperpetual =

(HCall

K)

HCall

hcall

K
=
hcall 1

hcall 1 S
hcall K

hcall

(12.26)

hcall
K
hcall 1

s
2
1 r
2r
1 r
=
+ 2
+

2
2
2

=
HCall

hcall

Pperpetual = K Hput

Hput

hput

Hput
=

K
1 hput

hput
K
hput 1

hput 1 S
hput K

(12.27)

(12.28)
hput

(12.29)

(12.30)

12.6. PERPETUAL AMERICAN OPTIONS

hput =

1 r
2
2

hcall and hput satisfies:

1 r
2
2

125
2

2r
2

1 2
h (h 1) + (r ) h r = 0
2

(12.31)
(12.32)

Example 12.6.1. Calculate the price of a perpetual American call and the price
of an otherwise identical perpetual American put. The information is as follows.
The current stock price is S = 50. The strike price is K = 45. The continuously
compounded risk-free rate is r = 6%. The continuously compounded dividend
yield is 2%. The stock volatility is = 25%.
Solution.
Solve for h.
1 2
h (h 1) + (r ) h r = 0
2
1
0.252 h (h 1) + (0.06 0.02) h 0.06 = 0
2
h2 = 1. 532 7
h1 = 1. 252 7
Use the bigger h for call and the smaller h for put.
Next, calculate the stock price where exercising the option is optimal.
hcall
1. 252 7

HCall
=
K=
45 = 223. 08
hcall 1
1. 252 7 1
hput
1. 532 7

=
Hput
K=
45 = 27. 23
hput 1
1. 532 7 1

hcall
1. 252 7
S
50

= (223. 08 45)
=
Cperpetual = (HCall K)

HCall
223. 08
27. 35
hput

1. 532 7

S
50

Pperpetual = K Hput
=
(45

27.
23
)
= 7.

Hput
27. 23
00
Tip 12.6.1. The CD attached to the textbook Derivatives Markets has a spreadsheet that calculates the price of a perpetual American call and a perpetual American put. The spreadsheet is titled "optbasic2." You can use this spreadsheet to
double check your solution.

12.6.2

Barrier present values

Consider the following barrier option. If the stock price first reaches a preset
price H from below, then the payo of $1 is received. This is a special case of a

perpetual American call option by setting the terminal payo HCall


K as $1

and by setting HCall


= H.

126

CHAPTER 12. BLACK-SCHOLES

The value at time zero of $1 received when the stock price first reaches H
from below (i.e. the stock first rises to H) is

h1 =

1 r
2
2

S
H
+

h1

(12.33)
1 r
2
2

2r
2

(12.34)

Similarly, the value at time zero of $1 received when the stock price first
reaches H from above (i.e. the stock first falls to H) is

h2 =

1 r
2
2

h1 and h2 satisfies:

S
H

h2

(12.35)
1 r
2
2

2r
2

1 2
h (h 1) + (r ) h r = 0
2

(12.36)
(12.37)

Example 12.6.2.
Calculate the value of a $1 paid if the stock price first reaches $100 and $60
respectively. The information is:
S = 80
r = 6%
= 2%
= 30%
Solution.
calculate the value of a $1 paid if the stock price first reaches $100
H = 100 > S. So we need to calculate the price of $1 payo when the stock
price first rises to H from below
1 2
h (h 1) + (r ) h r = 0
2
1 2
0.3 h (h 1) + (0.06 0.02) h 0.06 = 0
2
h2 = 1. 100 5
h1 = 1. 211 6
Use the bigger h
h1
1. 211 6
80
S
=
= 0.763
H
100

12.6. PERPETUAL AMERICAN OPTIONS

127

calculate the value of a $1 paid if the stock price first reaches $90
H = 60 < S.So we need to calculate the price of $1 payo when the stock
price first falls to H from above.
h1 = 1. 211 6
h2 = 1. 100 5
Use
the
smaller
h
h2 1. 100 5
S
80
=
= 0.729
H
60
Tip 12.6.2. The CD attached to the textbook Derivatives Markets has a spreadsheet that calculates the price of a barrier option. The spreadsheet is titled "optbasic2." You can use this spreadsheet to double check your solution.

128

CHAPTER 12. BLACK-SCHOLES

Chapter 13

Market-making and
delta-hedging
13.1

Delta hedging

V
Here is the main idea behind delta hedging. Delta of an option is =
.
S
Hence for a small change in stock price, the change of the option value is approximately V1 V0 (S1 S0 ). Suppose you sell one European call option.
If you hold shares of stock, you are immunized against a small change of the
stock price.
If the stock price goes up from S0 to S1 , then the call will be more valuable
to the buyer and your are exposed to more risk. Suppose the value of the call
goes up from V0 to V1 as the stock price goes up from S0 to S1 , then your
liability will increase by V1 V0 . At the same time, the value of your shares
of stock will go up by (S1 S0 ). Because V1 V0 (S1 S0 ), the increase
of your liability will be roughly oset by the increase of your asset.
However, under delta hedging, you are immunized against only a small
change of the stock price (just like immunization by duration matching assets
and liabilities is only good for a small change of interest rate). If a big change
of stock price knocks o your hedging, youll need to rebalance your hedging.
However, in the real world, continuously rebalancing the hedging portfolio
is impossible. Traders can only do discrete rebalancing.

13.2

Examples of Delta hedging

Make sure you can reproduce the textbook calculation of delta-hedging for 2
days. In addition, make sure you can reproduce the textbook table 13.2 and
13.3
Exam problems may ask you to outline hedging transactions or calculate the
hedging profit.
129

130

CHAPTER 13. MARKET-MAKING AND DELTA-HEDGING

The major diculty many candidates face is not knowing how to hedge.
They wonder "Should the market-maker buy stocks? Should he sell stocks?"
To determine how to hedge a risk, use the following ideas:
The goal of hedging is to break even. If a trader makes money
on option, he must lose money on stock; if he loses money on
option, he must make money on stock.
To determine whether a trader should buy stocks or sell stocks,
ask "If the stock price go up (or down), will the trader make
money or lose money on the option?"
If a trader loses money on the option as the stock price goes up,
then the trader needs to initially own (i.e. buy) stocks. This
way, the value of the traders stocks will go up and the trader
will make money on his stocks. He can use this profit to oset
his loss in the option.
If a trader makes money on the option as the stock price goes
up, then the trader needs to initially short sell stocks. This way,
as the stock price goes up, the trader will lose money on the
short sale (because he needs to buy back the stocks at a higher
price). His loss in short sale can oset his profit in option.
Example 13.2.1. The trader sells a call. How can he hedge his risk,?
If the stock price goes up, the call payo is higher and the trader will lose
money. To hedge this risk, the trader should buy stocks. This way, if the stock
price goes up, the trader makes money in the stocks. This profit can be used to
oset the traders loss in the written call.
You can also ask the question "If the stock goes down, will the trader make
money or lose money on the option?" If the stock goes down, the call payo is
lower and the trader will make money. To eat up his profit in the option, the
trader needs to buy stocks. This way, as the stock price goes down, the value
of the traders stocks will go down too and the trader will lose money in his
stocks. This loss will oset the traders profit in the written call.
Example 13.2.2. The trader sells a put. How can he hedge his risk?
If the stock price goes down, the put payo is higher and the trader will
make money. To hedge his risk, the trader should short sell stocks. This way,
if the stock price goes down, the trader can buy back stocks at lower price,
making a profit on stocks. This profit can be used to oset the traders loss in
the written put.
You can also ask the question "If the stock goes up, will the trader make
money or lose money on the option?" If the stock goes up, the put payo is
lower and the trader will make money on the written put. To eat up his profit
in the option, the trader needs to short sell stocks. This way, as the stock price

13.2. EXAMPLES OF DELTA HEDGING

131

goes up, the trader needs to buy back stocks at a higher price. The trader will
lose money in his stocks. This loss will oset the traders profit in the written
put.
Example 13.2.3. The trader buys a call. How can he hedge his risk?
If a trader buys a call, the most he can lose is his premium and theres no
need to hedge. This is dierent from selling a call, where the call seller has
unlimited loss potential.
However, if a trader really wants to hedge his limited risk, he can short sell
stocks.
Example 13.2.4. The trader buys a put. How can he hedge his risk?
If a trader buys a put, the most he can lose is his premium and theres no
need to hedge. This is dierent from selling a put, where the call seller has a
big loss potential.
However, if a trader really wants to hedge his limited risk, he can buy stocks.

Example 13.2.5.
Reproduce the textbook example of delta-hedging for 2 days. Here is the
recap of the information. At time zero the market maker sells 100 European
call options on a stock. The option expires in 91 days.
K = $40
r = 0.08
=0
= 0.3
The stock price at t = 0 is $40
The stock price at t = 1/365 (one day later) is $40.50
The stock price at t = 2/365 (two days later) is $39.25
The market-maker delta hedges its position daily. Calculate the marketmakers daily mark-to-market profit

Solution.
First, lets calculate the call premium and delta at Day 0, Day 1, and Day 2.
I used my Excel spreadsheet to do the following calculation. If you cant fully
match my numbers, its OK.

132

CHAPTER 13. MARKET-MAKING AND DELTA-HEDGING


Time t
Expiry T (Yrs)
St

1
St
+ 0.08 + 0.32 T
ln
40
2

d1 =
0.3 T
N (d
1 )
d2 = d1 0.3 T
N (d2 )
C = St N (d1 ) 40e0.08T N (d2 )
= e(T t) N (d1 )

Day 0 (t = 0)
91/365
40

Day 1
90/365
40.50

Day 2
89/365
39.25

0.208048

0.290291

0.077977

0.582404
0.058253
0.523227
2.7804
0.58240

0.614203
0.141322
0.556192
3.0621
0.61420

0.531077
0.070162
0.472032
2.3282
0.53108

The above table can be simplified as follows:


Time t
Day 0 t = 0 Day 1 t = 1/365
Expiry T
T0 = 91/365
T1 = 90/365
St
S0 = 40
S1 = 40.50
Ct C0 = 100 2.7804 = 278. 04
C1 = 306.21
t 0 = 100 0.58240 = 58. 24
1 = 61.420

Day 2 t = 2/365
T2 = 89/365
S2 = 39.25
C2 = 232.82
2 = 53.108

Beginning of Day 0
Trader #0 goes to work
Day 0 t = 0
T0 = 91/365
S0 = 40
C0 = 278.04
0 = 58.240
Trader #0 goes to work. The brokerage firm (i.e. the employer of Trader
#0) gives Trader #0 C0 = $278. 04. This is what the call is worth today.
Trader #0 needs to hedge the risk of the written call throughout Day 0.
To hedge the risk, Trader #0 buys 0 stocks, costing 0 S0 = 58. 2440 = $
2329. 6. Since Trader #0 gets $278. 04 from the brokerage firm, he needs to
borrow:
0 S0 C0 = 2329. 6 278. 04 = $2051. 56.
The trader can borrow $2051. 56 from a bank or use this own money. Either
way, this amount is borrowed. The borrowed amount earns a risk free interest
rate.
Now Trader #0s portfolio is:
Component
Value
0 = 58.24 stocks 2487. 51
call liability
278. 04
borrowed amount 2051. 56
Net position
0
End of Day 0 (or Beginning of Day 1)

13.2. EXAMPLES OF DELTA HEDGING

133

Mark to market without rebalancing the portfolio


Day 1 t = 1/365
T1 = 90/365
S1 = 40.50
C1 = 306.21
Method 1
To cancel out his position, Trader #0 can at t = 0
buy a call (we call this the 2nd call) from the market paying C1 = $306.
21. At expiration, the payo of this 2nd call will exactly oset the payo
of the 1st call. The 1st call is the call sold by the brokerage firm at t = 0
to the customer who bought the call. For example, if at expiration the
stock price is ST = 100, then both calls are exercised. The trader gets
ST K = 100 40 = 60 from the 2nd call. The liability of the first call
is also $60. These two calls cancel each other out.
sell out 0 = 58. 24 stocks for 0 S1 = 58. 24 40.5 = $2358. 72
pay o the loan. The payment is (0 S0 C0 ) erh = 2051. 56e0.081/365 =
2052. 01
At the end of Day 0, the traders profit is C1 + 0 S1 (0 S0 C0 ) erh =
306. 21 + 2358. 72 2052. 01 = 0.5
Trader #0 hands in $0.5 profit to his employer and goes home.
Method 2

We consider the change between Day 0 and Day 1.

In the beginning of Day 0, the traders stock is worth 0 S0 = 58. 24 (40) =


2329. 6. In the end of Day 0 (or the beginning of Day 1), the traders stock
is worth 0 S1 = 58. 24 (40.5) = 2358. 72. The value of the traders 58. 24
stocks goes up by 58. 24 (40.5 40) = 29. 12. This is good for the trader.
In the beginning of Day 0, the call is worth C0 = $278. 04. In the end of
Day 0 (or the beginning of Day 1), the call is worth C1 = 306.21. The call
value is the traders liability. Now the traders liability increases by 306.
21 278. 04 = 28. 17. So the trader has a loss 28. 17 (or a gain of 28.75).
Recall under Method 1, the trader has to buy a call for 306. 21 to cancel
out the call he sold. So call value increase is bad for the trader.
In the beginning of Day 0, the borrowed amount is 0 S0 C0 = 2051. 56.
In the end of Day 0 (or the beginning of Day 1), this borrowed amount
grows to (0S0 C0 ) erh = 2051. 56e0.081/365 = 2052. 01. The increase
(0 S0 C0 ) erh 1 = 2051. 56e0.081/365 2051. 56 = 0.45. This is
the interest paid on the amount borrowed. No matter the trader borrows
money from a bank or uses his own money, the borrowed money needs to
earn a risk free interest rate.

134

0.5

CHAPTER 13. MARKET-MAKING AND DELTA-HEDGING

At the end of Day 0, the traders profit is:

0 S1 0 S0 (C1 C0 ) (0 S0 C0 ) erh 1 = 29. 12 28. 17 0.45 =

You can verify that

0 S1 0 S0 (C1 C0 )(0 S0 C0 ) erh 1 = C1 +0 S1 (0 S0 C0 ) erh


Method 3

On Day 0, the trader owns 0 = 58.240 stocks to hedge the call liability
C0 = $278. 04. The traders net asset is
M V (0) = 0 S0 C0 = 58. 24 40 278. 04 = $2051. 56
In the end of Day 0 (or the beginning of Day 1) before the trader rebalances
his portfolio, the traders asset is:
M V BR (1) = 0 S1 C1 = 58. 24 40.5 100 3.0621 = 2052. 51
BR stands for before rebalancing.
The traders profit at the end of Day 0 is:
M V BR (1) M V (0) e0.081/365 = (0 S1 C1 ) (0 S0 C0 ) erh = 2052.
51 2051. 56e0.081/365 = 0.50
Please note that Trader #0 doesnt need to rebalance the portfolio. The
portfolio is rebalanced by the next trader.
Beginning of Day 1
Trader #1 goes to work
Day 1 t = 1/365
T1 = 90/365
S1 = 40.50
C1 = 306.21
1 = 61.420
Trader #1 goes to work. He starts from a clean slate. The portfolio is
automatically rebalanced since Trader #1 starts from scratch.
The brokerage firm gives Trader #1 C1 = $306.21.
Trader #1 needs to hedge the risk of the written call throughout Day 1.
To hedge the risk, Trader #1 buys 1 = 61.420 stocks, costing 1 S1 =
61.42 40.5 = $2487. 51. Since Trader #1 gets $306.21 from the brokerage firm,
he needs to borrow:
1 S1 C1 = 2487. 51 306.21 = 2181. 3
The trader can borrow $2181. 3 from a bank or use this own money. Either
way, this amount is borrowed. The borrowed amount earns a risk free interest
rate.
Now Trader #1s portfolio is:

13.2. EXAMPLES OF DELTA HEDGING


component
1 = 61.42 stocks
call liability
borrowed amount
Net position

135

value
2487. 51
306.21
2181. 3
0

One question arises, "What if Trader #1 doesnt start from a clean slate?"
Next, well answer this question.
Instead of starting from scratch, Trader #1 can start o with Trader #0s
portfolio. At the end of Day 0, Trader #0 has
0 = 58.24 stocks
a borrowed amount (0 S0 C0 ) erh = 2051. 56e0.081/365 = 2052. 01
0.5 profit
At the end of Day 0 or the beginning of Day 1, 1 = 61.420. So Trader #1
buys additional shares:
1 0 = 61.42 58.24 = 3. 18
The cost of these additional shares is (1 0 ) S1 = 3. 18 40.5 = 128. 79
Since these additional shares are bought at the current market price, Trader
#1 can sell these shares at the same price he bought them. This doesnt aect
the mark-to-market profit.
Trader #1 can borrow 128. 79 to pay for the purchase of 3. 18 stocks.
Now Trader #1 has a total of 1 = 61.42 shares worth 1 S1 = 61.4240.5 =
2487. 51
His liability is now C1 = 306.21
The borrowed amount is now 2052. 01 + 128. 79 + 0.5 = 2181. 3. The 0.5 (the
profit made by Trader #0) is the amount of money Trader #1 borrows from
Trader #0.
Now Trader #1s portfolio is:
component
value
1 = 61.42 stocks $2487. 51
call liability
306.21
borrowed amount 2181. 3
Net position
0
The portfolio is the same as the portfolio if Trader #1 starts from scratch.
Calculation is simpler and cleaner if we start from scratch.
End of Day 1 (or Beginning of Day 2)
Mark to market without rebalancing the portfolio
Day 2 t = 2/365
T2 = 89/365
S2 = 39.25
C2 = 232.82

136

CHAPTER 13. MARKET-MAKING AND DELTA-HEDGING

Method 1
To cancel out his position, the trader can
buy a call from the market paying C2 = 232.82. The payo of this call
will exactly oset the payo of the call sold by the brokerage firm to the
customer. These two call have the common expiration date T1 = 89/365
and the same payo. They will cancel each other out.
sell out 1 = 61.420 stocks for 1 S2 = 61.420 39.25 = 2410. 735
pay o the loan. The payment is (1 S1 C1 ) erh = 2181. 3e0.081/365 =
2181. 778
The traders profit at the end of Day 1 is
C2 + 1 S2 (1 S1 C1 ) erh = 232.82 + 2410. 735 2181. 778 = 3. 863
Trader #1 hands in 3. 863 profit to his employer and goes home.
Method 2
We consider the change between the beginning of Day 1 and
the end of Day 1 (or the beginning of Day 2).
In the beginning of Day 1, the traders stock is worth 1 S1 ; In the end of
Day 1 (or the beginning of Day 2), the traders stock is worth 1 S2 . The
value of the traders stocks goes up by 1 S2 1 S1
In the beginning of Day 1, the call is worth C1 ; In the end of Day 1 (or the
beginning of Day 2), the call is worth to C2 . The call value is the traders
liability. Now the traders liability increases by C2 C1
In the beginning of Day 1, the borrowed amount is 1 S1 C1 . In the
end of Day 1 (or the beginning of Day 2), this borrowed

amount grows
to (1 S1 C1 ) erh . The increase is (1 S1 C1 ) erh 1 . This is the
interest paid on the amount borrowed. No matter the trader borrows
money from a bank or uses his own money, the borrowed money needs to
earn a risk free interest rate.
The traders profit at the end of Day 1:

(1 S2 1 S1 ) (C2 C1 ) (1 S1 C1 ) erh 1
= C2 + 1 S2 (1 S1 C1 ) erh = 3. 863
Method 3
In the beginning of Day 1, the traders net asset is
M V (1) = 1 S1 C1 = 61.420 40.5 306.21 = 2181. 3
In the end of Day 1 before the trader rebalances his portfolio, the traders
asset is:
M V BR (2) = 1 S2 C2 = 61.420 39.25 232.82 = 2177. 915
The traders profit at the end of Day 1 is:
M V BR (2) M V (1) erh = 2177. 915 2181. 3e0.081/365 = 3. 863

13.2. EXAMPLES OF DELTA HEDGING

137

You can verify that


M V BR (2) M V (1) erh

= (1 S2 1 S1 ) (C2 C1 ) (1 S1 C1 ) erh 1
= C2 + 1 S2 (1 S1 C1 ) erh
Method 3 is often faster. Under this method,
Profit during a day=Asset at the end of Day before rebalancing the portfolio
- Future value of the asset at the beginning of the day
Asset at the end of Day before rebalancing the portfolio = delta at the
beginning of the day stock price at the end of the day
Profit at the end of Day t = M V BR (t + 1)M V (t) erh = (t St+1 Ct+1 )
(t St Ct ) erh
Now this completes the textbook example on page 417 to 418.
Next, lets do additional calculations and calculate the profit at the end of
Day 2, 3, and 4 (or the profit at the beginning of Day 3,4,5)
Day
Time t
Expiry T
St
Ct
t
Profit end of Day

Day 2
t = 2/365
T2 = 89/365
S2 = 39.25
C2 = 232.82
2 = 53.108
0.40

Day 3
t = 3/365
T3 = 88/365
S3 = 38.75
C3 = 205. 46
3 = 49. 564
4. 0

Day 4
t = 4/365
T4 = 87/365
S4 = 40
C4 = 271.04
4 = 58.06
1. 32

Day 5
t = 5/365
T5 = 86/365
S5 = 40
C5 = 269.27
5 = 58.01

Profit at the end of Day 2 (or beginning of Day 3).


Asset at the beginning of Day 2: M V (2) = 2 S2 C2 = 53.108 39.25
232.82 = 1851. 669
Asset at the end of Day 2 before rebalancing: M V BR (3) = 2 S3 C3 =
53.108 38.75 205. 46 = 1852. 475
Profit at the end of Day 2: M V BR (3) M V (2) erh = 1852. 475 1851.
669e0.08/365 = 0.400 1

Profit at the end of Day 3 (or beginning of Day 4).


Asset at the beginning of Day 3: M V (3) = 3 S3 C3 = 49. 564 38.75
205. 46 = 1715. 145
Asset at the end of Day 3 before rebalancing: M V BR (4) = 3 S4 C4 =
49. 564 40 271.04 = 1711. 52
Profit at the end of Day 3: M V BR (4) M V (2) erh = 1711. 52 1715.
145e0.08/365 = 4. 000 96

138

CHAPTER 13. MARKET-MAKING AND DELTA-HEDGING

Profit at the end of Day 4 (or beginning of Day 5).


Asset at the beginning of Day 4: M V (4) = 4 S4 C4 = 58.06 40
271.04 = 2051. 36
Asset at the end of Day 4 before rebalancing: M V BR (3) = 4 S5 C5 =
58.06 40 269.27 = 2053. 13
Profit at the end of Day 4: M V BR (5) M V (4) erh = 2053. 13 2051.
36e0.08/365 = 1. 320 3
You can verify that the profit at the end of Day 2, 3, 4 calculated above
matches Derivatives Markets Table 13.2. However, in Table 13.2, the profit at
the end of Day 0 is posted in Day 1 column. Similarly, the profit at the end of
Day 1 is posted in Day 1 column. So on and so forth.

13.3

Textbook Table 13.2

Next, we want to reproduce the textbook Table 13.2.


Profit at the end of Day t (or the beginning of Day t + 1) can be broken
down into two parts:
= M V BR (t + 1) M V (t) erh = M V BR (t + 1) M V (t) M V (t) erh +
M V (t)

= M V BR (t + 1) M V (t) M V (t) erh 1

M V (t) erh 1
= M V BR (t + 1) M V (t) +
{z
}
|
{z
}
|
capital gain at end of Day t

Interest earned at the end of Day t

Define M V BR (t + 1) M V (t) = CapitalGain earned at the end of Day t


CapitalGain (t) is also equal to:
M V BR (t + 1)M V (t) = (t St+1 Ct+1 )(t St Ct ) = t (St+1 St )
(Ct+1 Ct )

Define M V (t) erh 1 as the interest earned at the end of Day t. If the
trader invests
money
at the beginning of Day t, then M V (t) is positive and

M V (t) erh 1 is negative. The negative interest earned is just the interest
expense incurred by the trader.
Define M V (t) as the investment made at the beginning of Day t. If M V (t)
is negative, then it means that the trader receives money.
Example 13.3.1.
Reproduce the textbook Table 13.2
We already reproduced the daily profit in Table 13.2. We just need to
reproduce the investment, interest, and the capital gain.

13.3. TEXTBOOK TABLE 13.2

139

Day
Time t
Expiry T
St
Ct
t
Investment (beginning of the day)
interest earned during the day
Capital gain (end of the day)
Profit (end of the day)

0
t=0
T0 = 91/365
S0 = 40
C0 = 278.04
0 = 58.2404
2051. 56
0.45
0.95
0.50

1
t = 1/365
T1 = 90/365
S1 = 40.5
C1 = 306.21
1 = 61.42
2051. 56
3. 385
0.478
3. 863

2
t = 2/365
T2 = 89/365
S2 = 39.25
C2 = 232.82
2 = 53.108
1851. 669
0.41
0.81
0.40

Day 0
We already know:
M V (0) = 0 S0 C0 = 58. 24 40 278. 04 = $2051. 56
M V BR (1) = 0 S1 C1 = 58. 24 40.5 100 3.0621 = 2052. 51
The traders profit at the end of Day 0 is:
M V BR (1) M V (0) erh = 2052. 51 2051. 56e0.081/365 = 0.50
To find the capital gain and the interest earned at the end of Day 0, we just
need to break down the profit M V BR (1) M V (0) erh into two parts:
M V BR (1) M V (0) erh
= M V BR (1) M V (0) M V (0) erh +
M V (0)
rh

BR
= MV
(1) M V (0) + M V (0) e 1
{z
}
|
{z
}
|
capital gain

interest earned

The capital gain at the end of Day 0:


M V BR (1) M V (0) = 2052. 51 2051. 56 = 0.95

The interest
credited
at the endof Day 0:

M V (0) erh 1 = 2051. 56 e0.081/365 1 = 0.449 7 = 0.45


Investment at the beginning of Day 0:
M V (0) = 2051. 56
Please note the textbook shows the interest credited at the end of Day 0,
capital gain earned at the end of Day 0, and daily profit at the end of Day 0 in
Day 1 column.
Day 1
M V (1) = 1 S1 C1 = 61.420 40.5 306.21 = 2181. 3
M V BR (2) = 1 S2 C2 = 61.420 39.25 232.82 = 2177. 915
The traders profit at the end of Day 1 is:
M V BR (2) M V (1) erh = 2177. 915 2181. 3e0.081/365 = 3. 863

140

CHAPTER 13. MARKET-MAKING AND DELTA-HEDGING

To find the capital gain and the interest earned at the end of Day 0, we just
need to break down the profit M V BR (1) M V (0) erh into two parts:

M V BR (2) M V (1) erh = M V BR (2) M V (1) + M V (1) erh 1


{z
}
|
{z
}
|
capital gain

interest earned

Capital gain at the end of Day 1: M V BR (2) M V (1) = 2177. 915 2181.
3 = 3. 385

Interest earned at the end of Day 1: M V (1) erh 1 = 2181. 3 e0.081/365 1 =


0.478
Investment at the beginning of Day 1:
M V (1) = 1 S1 C1 = 61.420 40.5 306.21 = 2181. 3
You should be able to reproduce Table 13.2 for the other days.
Day
3
4
Time t
t = 3/365
t = 4/365
Expiry T
T3 = 88/365 T4 = 87/365
St
S3 = 38.75
S4 = 40
Ct C3 = 205. 46 C4 = 271.04
t 3 = 49. 564
4 = 58.06
Investment (beginning of the day)
1715. 15
2051. 36
interest earned during the day
0.38
0.45
Capital gain (end of the day)
3. 63
1. 77
Profit (end of the day)
4. 01
1. 32

13.4

5
t = 5/365
T5 = 86/365
S5 = 40
C5 = 269.27
5 = 58.01

Textbook Table 13.3

The stock price of Table 13.3 follows the binomial tree with = 0.3.
On Day 0, the stock price is S0 = 40
On Day 1 thestock moves up 1
S1 = S0 erh+ h = 40e0.08/365+0.3 1/365 = 40.642
Day 2 the stock
moves down 1

rh h
S2 = S1 e
= 40.642e0.08/3650.3 1/365 = 40. 018
Day 3 the stock
moves down 1

S3 = S2 erh h = 40. 018e0.08/3650.3 1/365 = 39. 403


Day 4 the stock
moves down 1

rh h
S4 = S3 e
= 39. 403e0.08/3650.3 1/365 = 38. 797
Day 5 the stock
moves up 1

S5 = S4 erh+ h = 38. 797e0.08/365+0.3 1/365 = 39. 420


If you use the same method for reproducing Table 13.2, you should be able
to reproduce Table 13.3. When reading Table 13.3, remember the interest, the
capital gain, and the daily profit on Day 1 is the interest, the capital gain, and
the daily profit at the end of Day 0 (or the beginning of Day 1). Similarly, the

13.5. MATHEMATICS OF DELTA HEDGING

141

interest, the capital gain, and the daily profit on any other day is the interest,
the capital gain, and the daily profit at the end of the previous day or the
beginning of that day.
The author of the textbook uses Table 13.3 to show us that if the stock price
moves up or down 1 daily, then the traders profit is zero.

13.5

Mathematics of Delta hedging

13.5.1

Delta-Gamma-Theta approximation

First, lets understand the textbook Equation 13.6:


1
V (St+h , T t h) V (S0 , T t) + t + h + t
2

(Textbook 13.6)

Lets consider an option written at time t expiring on date T (i.e. expiring


in T t years). The value of this option is V (St , T t), where St is the stock
price at t. Suppose a tiny time interval h (such as 0.00001 second) has passed
and we are now standing at t + h. Now the option has a remaining life T t h
years and is worth V (St+h , T t h), where St+h is the stock price at t + h.
Suppose St+h = St + .
Time
t
t+h
T
Stock price
St
St+h = St +
Option value V (St , T t) V (St+h , T t h)
We want to estimate V (St+h , T t h). By Taylor series, we have:
f (x0 , y0 )
f (x0 , y0 )
f (x0 + x , y0 + y ) f (x0 , y0 ) +
x+
y
x
y
2
2
1 f (x0 , y0 ) 2 1 f (x0 , y0 ) 2
+
x+
y
2
x2
2
y 2
Similarly,
V (St+h , T t h) = V [St+h , (t + h) + T ]
V (St , T t)
V (St , T t)
V (St , T t) +
+
h
S
t
2
2
1

V (St , T t) 2
1 V (St , T t) 2
+
h
+
2
S 2
2
t2
However,
V (St , T t)
= t
S

2 V (St , T t)
= t
S 2

1 2 V (St , T t) 2
h since its close to zero. However,
2
t2
1 2 V (St , T t) 2
is not close to zero. The reason that
h
2
t2

We decide to ignore
1 2 V (St , T t)
2
S 2

V (St , T t)
=
t

142

CHAPTER 13. MARKET-MAKING AND DELTA-HEDGING

1 2 V (St , T t) 2
is close to zero but
is not close to zero will be explained
2
S 2
in Derivatives Markets Chapter 20 when we derive Itos Lemma. For now just
accept it.
Now we have:
1
V (St+h , T t h) V (St , T t) + t + h + t
2

13.5.2

Understanding the market makers profit

Suppose a trader sets up a hedging portfolio at time t. The traders profit after
a short interval h (i.e. at time t + h) is:
P rof it (t + h)
= M V BR (t + h) M V (t) erh
= (t St+h Ct+h ) (t St Ct ) erh

= (t St+h Ct+h ) (t St Ct ) erh 1 + (


Ct )
t St
rh
= t (St+h St ) (Ct+h Ct ) (t St Ct ) e 1
For a small h, using Taylor series, we get erh 1 + rh
P rof it (t + h) = t (St+h St ) (Ct+h Ct ) rh (t St Ct )
According to the textbook Equation 13.6,
1
Ct+h Ct = t + h + t 2
2

1
P rof it (t + h) = t (St+h St ) t + h + t 2
2
Since St+h = St + , we have:
P rof it (t+ h)

1 2
= t t + h + t
rh (t St Ct )
2

1 2
rh (t St Ct )
= h + t
2

1 2
t + h rh [t St Ct ]
P rof it (t + h)
2
However, St+h = St erh+

rh (t St Ct )

(Textbook 13.7)

Using the Taylor series, we have: erh+


...

1
2
= 1+ rh + h + rh + h +
2

2
1
rh + h and higher order terms all approach zero.
2

In addition, as h approaches 0, h is much larger than h. For example,


0.0001 = 0.01 is much larger than 0.0001.Hence, we can discard rh but keep

h.
As h 0,

13.5. MATHEMATICS OF DELTA HEDGING

143

1

2
= St 1 + rh + h +
rh + h + St 1 + h
2

= St+h St St h
2 St2 2 h
Plug the above equation in Textbook Equation 13.7, we get:
St+h = St erh+

P rof it (t + h)

1 2 2
St ht + h rh [t St Ct ]
2

(Textbook 13.9)

From the textbook Table 13.3, we know that if the stock price moves up or
down
by 1 , the traders profit is zero. So
1 2 2

S ht + h rh [t St Ct ] = 0
2 t
This gives us the Black-Scholes PDE:
1 2 2
S t + r [t St Ct ] = 0
2 t

(Textbook 13.10)

Next, the textbook has the following topics:


Delta-hedging of American options
The advantage to frequent rehedging
Delta-hedging in practice
Market making as insurance
These topics are minor ideas. I recommend that you skip them.

144

CHAPTER 13. MARKET-MAKING AND DELTA-HEDGING

Chapter 14

Exotic options: I
Any option that is not a plain vanilla call or put is called an exotic option.
There are usually no markets in these options and they are purely bought OTC
(over-the-counter). They are much less liquid than standard options. They often
have discontinuous payos and can have huge deltas near expiration which make
them dicult to hedge.
Before studying this chapter, make sure you understand the learning objective.
SOAs learning outcome for this chapter:
Explain the cash flow characteristics of the following exotic options: Asian,
barrier, compound, gap and exchange

If you want to cut corners, you can skip the pricing formula for exotic options
because calculating the exotic option price is out of the scope of the learning
outcome or learning objective.

14.1

Asian option (i.e. average options)

14.1.1

Characteristics

Asian options (also called average options) have payos that are based on the
average price of the underlying asset over the life of the option. The average
price can be the average stock price or the average strike price. The average
can be arithmetic or geometric.
The unique characteristic of an average price option is that the underlying
asset prices are averaged over some predefined time interval.
Averaging dampens the volatility and therefore average price options are
less expensive than standard options
145

146

CHAPTER 14. EXOTIC OPTIONS: I

Average price options are path-dependent, meaning that the value of the
option at expiration depends on the path by which the stock arrives at its
final price. The price path followed by the underlying asset is crucial to
the pricing of the option.
Average price options are useful in situations where the trader/hedger
is concerned only about the average price of a commodity which they
regularly purchase.

14.1.2

Examples

Examples are based on

A 9-month European average price contract calls for a payo equal to the
dierence between the average price of a barrel of crude oil and a fixed
exercise price of USD18. The averaging period is the last two months of
the contract. The impact of this contract relative to a standard option
contract is that the volatility is dampened by the averaging of the crude oil
price, and therefore the option price is lower. The holder gains protection
from potential price manipulation or sudden price spikes.
A Canadian exporting firm doing business in the U.S. is exposed to Can$/US$
foreign exchange risk every week. For budgeting purposes the treasurer
must pick some average exchange rate in which to quote Can$ cash flows
(derived from US$ revenue) for the current quarter. Suppose the treasurer
chooses an average FX rate of Can$1.29/US$1.00. If the US$ strengthens, the cash flows will be greater than estimated, but if it weakens, the
companys Can$ cash flows are decreased.
Arithmetic average
We record the stock price every h periods from time 0 to time T . There are
N = T /h periods. The arithmetic average is:
n
1 X
A (T ) =
Sih
N i=1

14.1.3

Geometric average

G (T ) = (Sh S2h ...SN h )1/N


1 Based on
http://www.fintools.com/doc/exotics/exoticsAbout_Average_Options.html

14.2. BARRIER OPTION

14.1.4

147

Payo at maturity T

Let AV G (T ) represent the average stock price. For arithmetic average, AV G (T ) =


n
1 X
A (T ) =
Sih . For geometric average, AV G (T ) = (Sh S2h ...SN h )1/N .
N i=1
Option
average
average
average
average

14.2

price call
price put
strike call
strike put

Payo
max [0, AV G (T ) K]
max [0, K AV G (T )]
max [0, ST AV G (T )]
max [0, AV G (T ) ST ]

Barrier option

Barrier options are similar to standard options except that they are extinguished
or activated when the underlying asset price reaches a predetermined barrier or
boundary price. As with average options, a monitoring frequency is defined as
part of the option which specifies how often the price is checked for breach of
the barrier. The frequency is normally continuous but could be hourly, daily,
etc.

14.2.1

Knock-in option

"In" option starts its life worthless and becomes active only if the barrier
price is reached
Technically, this type of contract is not an option until the barrier price
is reached. So if the barrier price is never reached it is as if the contract
never existed.
down-and-in (spot price starts above the barrier level and has to move
down for the option to become activated.)
up-and-in (spot price starts below the barrier level and has to move up
for the option to become activated.)

14.2.2

Knock-out option

"Out" option starts its life active and becomes null and void if the barrier
price is reached
The option will expire worthless if the asset price exceeds the barrier price
down-and-out (spot price starts above the barrier level and has to move
down for the option to become null and void)
up-and-out (spot price starts below the barrier level and has to move up
for the option to be knocked out)

148

14.2.3

CHAPTER 14. EXOTIC OPTIONS: I

Rebate option

Barrier options are sometimes accompanied by a rebate, which is a payo


to the option holder in case of a barrier event.
Rebates can either be paid at the time of the event or at expiration.

14.2.4

Barrier parity

Knock in option + Knock out option = Ordinary Option

(14.1)

Example 14.2.1.
Explain why
up-and-in option + up-and-out option = Ordinary option.
T =option expiration time
tH =time when the stock price first reaches the barrier, where the barrier
price is greater than the current stock price
During the time interval [0, tH )
the up-and-out option is alive
the up-and-in option is dead
During the time interval [tH , T ]
the up-and-out option is dead (because the barrier is reached)
the up-and-in option is alive (because the barrier is reached)
If we have a down-and-in option and a down-and-out option with two options
on the same stock, having the same barrier price and the same strike price, then
at any moment in the interval [0, T ], well always have exactly one ordinary
option alive. Hence down-and-in option + down-and-out option = Ordinary
option.
Similarly, down-and-in option + down-and-out option = Ordinary option.

14.2.5

Examples

1. A European call option is written on an underlying with spot price $100,


and a knockout barrier of $120. This option behaves in every way like
a vanilla European call, except if the spot price ever moves above $120,
the option "knocks out" and the contract is null and void. Note that the
option does not reactivate if the spot price falls below $120 again. Once
it is out, its out for good.

14.3. COMPOUND OPTION

149

2. A bank purchases an at-the-money 9-month Nikkei call option struck at


17,000 with a down-and-out barrier price of 16,000. If the price of the
Nikkei falls to 16,000 or below, during the 9-month period, the bank will
no longer have the benefit of Nikkei price appreciation since the call option
will have been knocked out.
3. An airline is concerned that events in the Middle East might drive up
the price of fuel. An up-and-in call would allow the airline to buy crude
oil futures at a fixed price if some knock-in boundary price is reached.
The price of the U&I call would be less than a standard call with the
same expiration and exercise price so it might be viewed as a cost eective
hedging instrument.

14.3

Compound option

Definition
With a compound option one has the right to buy an ordinary option at
a later date
Compound options are more expensive to purchase than the underlying
option, as the purchaser has received a price guarantee and eectively
extended the life of the option
Make sure you understand the textbook Figure 14.2.
For a compound call (i.e. call on call) to be valuable, the following two
conditions need to be met:
St1 > S
St1 > K
Compound option parity
The key formula is DM 14.12:
CallOnCall P utOnCall + xert1 = BSCall

(DM 14.12)

This formula looks scary but is actually easy to remember. We know the
put-call parity C + KerT = P + S0 . If we treat the standard BSCall as the
underlying asset, then applying the standard put-call parity, we get:
CallOnCall + xert1 = P utOnCall + BSCall
This is DM 14.12.
Similarly, we have:
CallOnP ut + xert1 = P utOnP ut + BSP ut

150

CHAPTER 14. EXOTIC OPTIONS: I

Options on dividend-paying stocks


This section is a minor part of Exam MFE. I recommend that you skip it.
However, if you dont want to skip, here is the main idea.
At t1 , the value of an American call option CA (St1 , T t1 ) is (see the textbooks explanation for this formula):

CA (St1 , T t1 ) = max [CE (St1 , T t1 ) , St1 + D K]

(DM 14.13)

Here CE (St1 , T t1 ) is a European call option. Under the put-call parity,


we get:
CE (St1 , T t1 ) = PE (St1 , T t1 ) + St1 Ker(T t)
So

CA (St1 , T t1 ) = max PE (St1 , T t1 ) + St1 Ker(T t) , St1 + D K

= St1 +DK+max PE (St1 , T t1 ) + St1 Ker(T t) (St1 + D K) , 0


= St1 + D K + max PE (St1 , T t1 ) D K 1 er(T t) , 0

i
CA (St1 , T t1 ) = St1 +DK+max PE (St1 , T t1 ) D K 1 er(T t) , 0
(DM 14.14)


max PE (St1 , T t1 ) D K 1 er(T t) , 0 is payo of a call on put

with strike price equal to D K 1 er(T t) .


Next, the author made two points:
Exercising call on put at t1 is the same as not exercising the American call
at
t
1 . If you
exercise the
call on put, then it must be true that PE (St1 , T t1 )
D K 1 er(T t) 0 (otherwise you wont exercise it). Then DM 14.14
becomes

CA (St1 , T t1 ) = St1 + D K + PE (St1 , T t1 ) D K 1 er(T t)


= St1 + PE (St1 , T t1 ) Ker(T t) = CE (St1 , T t1 )
So exercising the call on put at t1 means that not early exercising the American call option at t1 (so the American call option becomes a European call
option).
The second point. If you dont
the call on
exercise

put at t1 , then it must


be true that PE (St1 , T t1 ) D K 1 er(T t) < 0.
This gives us CA (St1 , T t1 ) = St1 + D K. This equation means that we
receive dividend D at t1 and immediately exercise the call at t1 (so the American
call option at t1 is equal to St1 + D K). So not exercising the call on put at
t1 is the same as early exercising the American call option at t1 .
This is all you need to know about this section.

14.4. GAP OPTION

151

DM Example 14.2 shows you how to use DM 14.14 to calculate the price of
an American call option with a single dividend. When reading this example,
please note two things:
(1) DM Example 14.2 has errors. Make sure you download the errata. The
DM textbook errata can be found at the SOA website.
(2) The call on put price is calculated using the Excel spreadsheet (so dont
worry about how to manually calculate the call on put).

14.4

Gap option

14.4.1

Definition

An option in which one strike price K1 determines the size of the payo
and another strike price K2 determines whether or not the payo is made.
Example. A call option pays ST 5 if ST > 8 and pays zero otherwise.
The strike price K1 = 5 determines the size of the payo but a dierent
strike price K2 = 8 determines whether or not the payo is made. K2 is
called the payment trigger.
Example. A put option pays 8 ST if ST > 5 and pays zero otherwise.
The strike price K1 = 8 determines the size of the payo but a dierent
strike price K2 = 8 determines whether or not the payo is made.
As the footnote in Page 457 of the textbook indicates, a gap call or put
option is really not a option; once the payment trigger is satisfied, the
owner of a gap option MUST exercise the option. Hence the premium of
a gap option can be negative.

14.4.2

Pricing formula

To find the price of a gap option, you can modify a standard options formula
by changing d1 :
C (K1 , K2 ) = SeT N (d1 ) K1 erT N (d2 )
rT
T
N (d
N (d1 )
P (K1 , K2 ) = K
2 ) Se
1 e
1 2
S
+ r+ T
ln
K2
2

d1 =
T
d2 = d1 T

14.4.3

How to memorize the pricing formula

The call payo is 0 if ST K2 and ST K1 if ST > K2 . The call option


price is just the risk-neutral expected discounted value of the payo. So
the call price is SeT N (d1 ) K1 erT N (d2 ). Similarly, the put price is
K1 erT N (d2 ) SeT N (d1 ).

152

CHAPTER 14. EXOTIC OPTIONS: I

You are given the following information on a gap option on a stock


The gap call expires in 6 months
The stock is currently selling for $40
The payment trigger is $50
The strike price is $60
The continuously compounded risk-free interest rate is 6% per year
The continuously compounded dividend yield is 2% per year
The stock volatility = 30%
Calculate
the price of the gap call option.
the price of the gap put option
K2 = 50
r = 0.06
K1 = 60
T = 0.5

1 2
40
S
1
2
+ r+ T
ln
ln
+ 0.06 0.02 + 0.3 0.5
K2
2
50
2

=
d1 =
=
0.3 0.5
T
0.851 6

d2 = d1 T = 0.851 6 0.3 0.5 = 1. 063 7


N (d1 ) = 0.197 2
N (d2 ) = 0.143 7
N (d1 ) = 0.802 8
N (d2 ) = 0.856 3
C = SeT N (d1 ) K1 erT N (d2 ) = 40e0.020.5 0.197 2 60e0.060.5
0.143 7 = 0.56
S = 40
= 0.02

P = K1 erT N (d2 ) SeT N (d1 ) = 60e0.060.5 0.856 3 40e0.020.5


0.802 8 = 18. 07

14.5

Exchange option

Allows the holder of the option to exchange one asset for another
Pricing
p formula. Use the standard Black-Scholes formula except changing
to 2S + 2K 2 S K . Also, make sure you know which is the stock
asset and which is the strike asset. If you give up Asset 2 and receive
Asset 1, Asset 1 is the stock and Asset 2 is the strike asset.

14.5. EXCHANGE OPTION

153

Lets walk through DM Example 14.3.


S = 95.92
K = 3.5020 27.39 = 95. 92 = S (i.e. at-the-money call)
S = 0.75%
K = 1.17%
= 0.1694
T =1
95.92e0.00751 1
+ 0.16942 1
0.01171
95.92e
2
d1 =
= 0.109 5
0.1694 1

d2 = d1 T = 0.109 5 0.1694 1 = 0.059 9


N (d1 ) = 0.543 6
N (d2 ) = 0.476 1
C = SeS T N (d1 ) K1 eK T N (d2 )
= 95.92e0.00751 0.543 6 95.92e0.01171 0.476 1 = 6. 616
ln

This is slightly dierent from the textbook call price 6.6133 due to rounding.
You can verify the call price using the Excel worksheet "Exchange."
Inputs:
Underlying Asset
Price
95.92
Volatility
20.300%
Dividend Yield
0.750%
Strike Asset
Price
95.92
Volatility
22.270%
Dividend Yield
1.170%
Other
Correlation
0.6869
Time to Expiration (years)

Output:
Exchange Option
Black-Scholes
Call
Put
Price
6.6144
6.2154
So the exchange call price is 6.6144; the exchange put price is 6.2154.

154

CHAPTER 14. EXOTIC OPTIONS: I

Chapter 18

Lognormal distribution
This chapter is an easy read. Im going to highlight the major points.

18.1

Normal distribution

The normal distribution has the following pdf (probability density function)
(DM 18.1):
"

2 #
1
1 x
(x; , ) = exp
2

2
In the standard normal
distribution,
= 0 and = 1. Its pdf is:

1
1 2
(x; 0, 1) = exp x
2
2
Its cdf (cumulative probability density function) is DM18.2:
Ra
Ra
1
1
P (z 6 a) = N (a) = (x; 0, 1) dx = exp x2 dx
2
2
A frequently used formula is DM 18.3:
N (a) = 1 N (a)
How to convert a normal
random variable to the standard normal random
variable. If x N , 2 , then we can transform x into a standard normal
random variable using DM 18.4:
x
z=

1
1
x is normal

(linear combination
ofnormal random variables are also normal). Next,

x
1
E (z) = E
= [E (x) ] = 0

We can verify that z N (0, 1). First, notice that z =

155

156

CHAPTER 18. LOGNORMAL DISTRIBUTION

V ar (z) = V ar
So z N (0, 1)

1
1
1
V ar (x ) = 2 V ar (x) = 2 2 = 1
2

DM 18.4 can be rewritten as DM 18.7: x = z +


variables
is still normal. For example, if x1
Sum of

normal random

2
N 1 , 21 and x2
N

,
then
2
2

ax1 + bx2 N a1 + b2 , a 21 + b 22 + 2ab1 2

Next, the textbook briefly explains the central limit theorem, a concept you
most likely already know.

18.2

Lognormal distribution

If x is normal, then y = ex is lognormal. If you say y is lognormal, it means


that ln y is normal.
Let R (0, t) represent the not-annualized continuously compounded return
over the interval [0, t], the stock price at time t is (DM 18.12):
St = S0 eR(0,t)
St
This leads to DM 18.11:
R (0, t) = ln
S0
Next comes an important formula:

2
2
If x N m, v 2 , then E (ex ) = em+0.5v and V ar (ex ) = e2m+v ev 1 .
You dont need to know how to derive these formulas. Just memorize them.

18.3

Lognormal model of stock prices

Define:
, the expected (annualized) continuously compounded return earned by
the stock
, the (annualized) continuously compounded dividend yield
, the stocks volatility
z, the standard normal random variable
Then the (not annualized) continuously
compounded
return earned during

[0, t] is normally distributed with mean 0.5 2 t and standard deviation

t (DM 18.18):

St
N 0.5 2 t, 2 t
ln
S0
random variable with mean
Using DM2 18.7, we can rewrite the normal

0.5 t and standard deviation t as:

18.4. LOGNORMAL PROBABILITY CALCULATION

157

x = 0.52 t + tz

Then we have DM 18.19:

St
ln
= x = 0.5 2 t + tz
S0

The stocks price at time t is (DM 18.20):


St = S0 ex = S0 exp 0.5 2 t + tz

Using DM 18.13,
we
get DM 18.22:

E (St ) = S0 E exp 0.5 2 t + tz = S0 exp 0.52 t + 0.5 2 t =


S0 e()t

18.4

Lognormal probability calculation

Lets go through some formulas. First, lets look at DM 18.23:


"

ln K ln S0 0.5 2 t

P (St < K) = N
= N d 2
t

S0

ln
+ 0.5 2 t
ln K ln S0 0.5 2 t
K

=
where
=
t
t

Heres how to derive it. ln St N ln S0 + 0.5 2 t, 2 t

d2

The real world probability of St < K


" is:

ln K ln S0 + 0.52 t

P (St < K) = P (ln St < ln K) = N


t

In the above formula, if we set = r, well get the risk neutral probability
of St < K:
"

ln K ln S0 + r 0.5 2 t

P (St < K) = N
= N (d2 )
t
Next, lets calculate the real world probability of P (St > K). Please note
that P (St = K) = 0. This is because St is a continuous random variable; the
probability for a continuous random variable to take on a specific value is zero.
So P (St > K) = 1 P (St < K) P (St = K) = 1 P (St < K). The real
world probability of St > K is:


S0
2



ln
t
+

0.5

= N d2 = N K
P (St > K) = 1N d2 = 1 1 N d2

158

CHAPTER 18. LOGNORMAL DISTRIBUTION

If we set = r, well get the risk neutral probability of P (St > K):


S0
ln
+ r 0.52 t

P (St > K) = N (d2 ) = N K

18.4.1

Lognormal confidence interval

The textbooks explanation is a bit confusing. Here is how to build a confidence


interval of the stock price.
First, lets consider building a 95% confidence interval for the standard normal random variable z. We need to find a value a positive value b such that
P (b < z < b) = 0.95.
P (b < z < b) = N (b) N (b) = N (b) [1 N (b)] = 2N (b) 1 = 0.95
1 + 0.95
= N (b) =
= 0.975
2
b = N 1 (0.975) = 1.96.
Then the 95% confidence interval is 1.96 < z < 1.96.
Alternatively, we can find b.
P (b < z < b) = N (b) N (b) = [1 N (b)] N (b) = 1 2N (b) =
0.95
1 0.95
Then N (b) =
= 0.025
2
1
b = N (0.025) = 1.96
So the 95% confidence interval is 1.96 < z < 1.96.
Lets generalize the result.
The
interval
p confidence
of the standard normal random variable z is

1p
1+p
1
1
N
<z<N
.
2
2
For example, if p = 0.95, then the confidence interval is:

1 0.95
1 + 0.95
< z < N 1
, which is N 1 (0.025) < z <
2
2
N 1 (0.975) , which is 1.96 < z < 1.96.
N 1

We can also specify the confidence interval as 1 p. For example, if we want


to build a 95% confidence interval, we just set p = 5%.
The (1 p) confidence
interval of
variable z
the standard

normal random

1 (1 p)
1 + (1 p)
1
1
1 p
is N
< z < N
, which is N
< z <
2
2
2

p
N 1 1
.
2
For example, if p = 5%, then the 1 5% = 95% confidence interval is:

18.4. LOGNORMAL PROBABILITY CALCULATION

159

0.05
0.05
< z < N 1 1
, which is N 1 (0.025) < z < N 1 (0.975),
2
2
which is 1.96 < z < 1.96.
N 1

Next, lets consider

building the 95% confidence interval for normal random


variable x N , 2 . Since x = + z,the p
confidence interval of x is:
1

p
1
+
p
+ N 1
< x < + N 1
.
2
2

For example, the 95% confidence interval of x N 1, 22 is: 1+2 (1.96) <
x < 1 + 2 (1.96), which is 2. 92 < x < 4. 92.

If we specify the confidence interval in terms of 1p, then the 1p confidence


interval is:
p

p
+ N 1
< x < + N 1 1
2
2

1p
You dont need to memorize messy formulas such as + N 1
<
2

1+p
p
x < + N 1
or + N 1
< x < + N 1 1
. The
2
2
2

following is a simple way to find a confidence interval for x N , 2 .


Step 1. Find the confidence interval b < z < b for the standard normal
random variable z. Just solve the equation P (b < z < b) = N (b)
N (b) = 2N (b) 1 =the confidence interval.

Step 2. After finding b, the confidence interval for x is b < x < + b

Example. Find the 90% confidence interval for x N 1, 22 . First, we find


the 90% confidence interval for the standard normal random variable z. We
solve the equation:
P (b < z < b) = N (b) N (b) = N (b) [1 N (b)] = 2N (b) 1 = 0.9
1 + 0.9
So N (b) =
= 0.95 and b = 1.645.
2
Then the 90% confidence interval for z is 1.645 < z < 1.645; for x is
1 1.645 2 < x < 1 + 1.645 2, which is 2. 29 < x < 4. 29.
Similarly, the 99% confidence interval is calculated as follows:
1 + 0.99
2N (b) 1 = 0.99, 2N (b) =
= 0.995, b = 2.576
2

The 99% confidence interval for z is 2.576 < z < 2.576; for x N 1, 22
is 1 2.576 2 < x < 1 + 2.576 2, which is 4. 152 < x < 6. 152.
Now lets walk through DM example 18.6. Here we need to find the 95%
confidence interval for the stock price St . The inputs are:
S0 = 100
t=2

160

CHAPTER 18. LOGNORMAL DISTRIBUTION

= 0.1
= 0.3
=0

St
We know that ln
N a 0.5 2 t, 2 t .

S0

a 0.5 2 t = 0.1 0 0.5 0.32 2 = 0.11


2 t = 0.32 (2) = 0.18
S2
=
ln
N (0.11, 0.18)
100
How to find the 95% confidence interval for S2 .
Step 1
Whats the 95% confidence interval for z N (0, 1)?
Answer: 1.96 < z < 1.96.
St
Whats the 95% confidence interval for ln
N (0.11, 0.18)?
100

S2
< 0.11 + 1.96 0.18,
Answer: 0.11 1.96 0.18 < ln
100
S2
or 0.721 56 < ln
< 0.941 56.
100
Step 2

S2
Take exponentiation of 0.721 56 < ln
< 0.941 56
100

S2
exp (0.721 56) < exp ln
< exp (0.941 56)
100

Step 3

exp (0.721 56) = 0.485 994


S2
100
=

exp (0.941 56) = 2. 563 98

S2
exp ln
=
100

S2
< 2. 563 98
48.60 < S2 < 256.40
100
So the 95% confidence interval for S2 is (48.60, 256.40) .
0.485 994 <

Now lets walk through Row 1 (i.e. 1 day horizon) in DM Table 18.1. The
inputs are:
S0 = 50
t = 1/365 (i.e. 1 day)
= 0.15
=0
= 0.3

18.4. LOGNORMAL PROBABILITY CALCULATION

161

We need to find the confidence interval that corresponds to 1 < z < 1


and the confidence interval that corresponds to 2 < z < 2. First, let consider
1 < z < 1.

St
N a 0.5 2 t, 2 t
S0

2
S1/365
0.15 0 0.5 0.32 0.32
4 0.3
ln
N
,
= N 2. 876 71 10 ,
50
365
365
365
ln

Whats the confidence interval for the standard normal random variable z?
Answer: 1 < z < 1.
Whats the corresponding confidence interval for ln
Answer:

S1/365
?
50

S1/365
0.32
< ln
< 2. 876 71 104 + 1
2. 876 71 10 1
365
50
S1/365
1. 541 505 102 < ln
< 1. 599 04 102
50
2
2
50e1. 541 50510 < S1/365 < 50e1. 599 0410
4

0.32
365

49. 24 < S1/365 < 50. 81


So the confidence interval for S1/365 is 49. 24 < S1/365 < 50. 81.
What confidence interval is that?
P (1 < z < 1) = N (1) N (1) = 2N (1) 1 = 2 0.841 34 1 = 0.682 7
So the confidence interval is 68.27%. So theres 68.27% chance that S1/365
will fall in the range (49. 24, 50. 81).
Similarly, we can calculate the confidence interval that corresponds to 2 <
z < 2.
r
r
S1/365
0.32
0.32
4
4
2. 876 71 10 2
< ln
< 2. 876 71 10 + 2
365
50
365
S1/365
3. 111 776 102 < ln
< 3. 169 311 102
50
2
2
50e3. 111 776 10 < S1/365 < 50e3. 169 31110
48. 47 < S1/365 < 51. 61
So the confidence interval for S1/365 is 48. 47 < S1/365 < 51. 61.
What confidence interval is that?
P (2 < z < 2) = 2N (2) 1 = 2 (0.977 25) 1 = 0.954 5
So the 95.45% confidence interval for S1/365 is 48. 47 < S1/365 < 51. 61.

162

CHAPTER 18. LOGNORMAL DISTRIBUTION

18.4.2

Conditional expected prices

One key formula is DM 18.27. The partial expectation of St on the condition


St < K is:

RK
()t
St g (St ; S0 ) dSt = E (St ) N d1 = S0 e
N d 1
0

S0
+ + 0.5 2 t
K

=
t
g (St ; S0 ) is the probability density function of St . Here St ; S0 indicates that
the initial stock price for St is S0 .
ln

d1

DM 18.27 is a little dicult to derive. I recommend that you dont bother


how to prove DM 18.27. Just memorize it. Once you memorize DM 18.27, you
can derive DM 18.28:

RK
S0 e()t N d1
St g (St ; S0 ) dSt

E (St |St < K) = 0


=

P (St < K)
N d2
Once you memorize DM 18.27, you can also derive DM 18.29:
R
K

R
0

RK
0

St g (St ; S0 ) dSt =

R
0

St g (St ; S0 ) dSt

RK
0

St g (St ; S0 ) dSt

St g (St ; S0 ) dSt = E (St ) = S0 e()t

St g (St ; S0 ) dSt = E (St ) N d1 = S0 e()t N d1

= K St g (St ; S0 ) dSt = S0 e
1 N d1

= S0 e()t 1 1 N d1
= S0 e()t N d1
R

()t

Then you can derive DM 18.30:


E (St |St > K) =

18.4.3

R
K

()t
N d1
1 N d 1
S0 e

1 N d2
N d2

()t

St g (St ; S0 ) dSt
=
P (St > K)

S0 e

Black-Scholes formula

This section derives the Black-Scholes formula with simple math. The BlackScholes formula was originally derived using stochastic calculus and partial differential equation. Many years after the Black-Scholes formula was published,
someone came up with this simple proof (hindsight is always 20-20).

18.5. ESTIMATING THE PARAMETERS OF A LOGNORMAL DISTRIBUTION163


The European
R call option price in the risk-neutral world, is:
C = ert K (St K)g (St ; S0 ) dSt = ert E (St K|St > K) P (St > K)
Here any term with the sign indicates the term is valuated in the riskneutral world (i.e. by setting = r).
E (St K|St > K) P (St > K) = E (St |St > K) P (St > K)E (K|St > K) P (St > K)
E (St |St > K) =
=

S0 e(r)t N (d1 )
N (d2 )

P (St > K) = N (d2 )

E (St |St > K) P (St > K) = S0 e(r)t N (d1 )

E (K|St > K) = K
=
E (K|St > K) P (St > K) = KN (d2 )
=

C = ert S0 e(r)t N (d1 ) KN (d2 ) = S0 et N (d1 )Kert N (d2 )

Similarly, the European put price is:


P = Kert N (d2 ) S0 et N (d1 )

18.5

Estimating the parameters of a lognormal


distribution

Consider the stock price over the time interval [t h, t].


2
St = Sth e(0.5 )h+

hz

where z is the standard normal random variable with


E (z) = 0 and V ar (Z) = 1
=

ln

St
= 0.5 2 h + hz
Sth

h
i

St
=
E ln
= E 0.52 h + hz = 0.5 2 h+
Sth

2
h
hE (z) = 0.5
h
i

St
=
V ar ln
= V ar 0.5 2 h + hz
Sth

Since 0.5 2 h is a constant, we have:


h
i 2

V ar 0.5 2 h + hz = h V ar (z) = 2 h
Now lets go through DM Table 18.2.

164

CHAPTER 18. LOGNORMAL DISTRIBUTION

week

1
2
3
4
5
6
7
sum

100
105.04
105.76
108.93
102.5
104.8
104.13
0

St
ln
St1
0.049171
0.006831
0.029533
0.060843
0.022191
0.006414
0.040470

St
ln
St1

0.0024178
0.0000467
0.0008722
0.0037018
0.0004924
0.0000411
0.0075721

Wehave:

St
0.040470
E ln
= 0.5 2 h =
= 0.006 745
Sth
6
0.006 745
= = + 0.5 2 +
h
In the above equations,
is the stocks expected (annual) continuously compounded return
is the stocks (annual) continuously compounded dividend yield ( = 0 in
this problem)
is the stocks (annual) volatility
h = 1/52 (52 weeks in one year)

2
P
St

ln

St
n
St1
1 P ln St
=
V ar ln

= 2h =


Sth
n 1 n
St1
n
6
5

1
0.0075721 0.006 7452
6

=
=
0.388 7

= 0.001 459 83

0.001 459 83
h
0.006 745
0.001 459 83 0.006 745
= + 0.5 2 +
= 0 + 0.5
+
=
h
1/52
1/52

2 =

The textbook talks about making two adjustments before calculating . I


found this approach
that you use my approach.
r confusing. I recommend
r
0.001 459 83
0.001 459 83
=
=
=
= 0.275 5
h
1/52
So the stocks expected return per year is = 38.8 7% and its annual volatility is = 0.275 5.

1P
1P
St
St
=
=
Please note that E ln
ln
(ln St ln Sth ) =
Sth
n
Sth
n

1 Sn
1
= 0.5 2 h
(ln Sn ln S0 ) = ln
n
n S0

18.6. HOW ARE ASSET PRICES DISTRIBUTED

165

In this problem,
we could have estimated

St
1 104.13
1 Sn
E ln
= ln
= 0.52 h = ln
= 0.006745
Sth
n S0
6
100
In addition, we have:
1
1
Sn
Sn
= ln
0.5 2 =
ln
nh S0
T
S0
where T = nh is the length of the observation.
We see that when we estimate 0.5 2 , only the following 3 factors
matter:
the length of the observation T
the stocks starting price S0
the stocks ending price Sn
The stock prices between S0 and Sn are irrelevant. And for a given T ,
increasing the number of observations n doesnt aect our estimate of
0.5 2 (because as n goes up, h goes down, and T = nh is a constant). Frequent
observations dont improve our estimate of 0.5 2 . To improve our
estimate of 0.5 2 , we need to increase T .
When we estimate , the in-between stock prices do matter and more frequent observations do improve our estimate.

18.6

How are asset prices distributed

The lognormal stock price model assumes that the stock returns are normally
distributed. Are stock returns really normally distributed?

18.6.1

Histogram

One method to assess whether stock returns are normally distributed is to plot
the continuously compounded returns as a histogram. Look at DM Figure 18.4.
The histograms in Figure 18.4 dont appear normal. The textbook oers
two explanations:
Stock prices can jump discretely from time to time
Stock returns are normally distributed, but the variance of the return
changes over time.
Thats all your need to know about this section.

166

18.6.2

CHAPTER 18. LOGNORMAL DISTRIBUTION

Normal probability plots

Another method to check whether stock returns are normally distributed is to


draw a normal probability plot.
To see what a normal probability plot is, lets go through an example. We
know that the 99% confidence interval for a standard normal random variable is
[2.58, 2.58]. Suppose we take the following samples from the range [2.58, 2.58]
with the step equal to 0.01:
2.58, 2.57, 2.56, 2.55, ..., 0, 0.01, 0.02, ..., 2.57, 2.58
The for each number xi sampled, we calculate yi = P (z < xi ) = N (xi ). If
we plot the data pairs (xi , yi ), what do we get? Answer: well get roughly a
straight line.
You can experiment this using Excel. Here is a screen shot:
A
B
1
xi yi = P (Z < xi )
2 2.58
0.00494
3 2.57
0.005085
4 2.56
0.005234
...
...
...
260
0
0.5
261
0.01
0.503989
262
0.02
0.507978
...
...
...
517
2.57
0.994915
518
2.58
0.99506
Sample formulas.
Cell A2 = 2.58
A3 = A2 + 0.01
...
A518 = A517 + 0.01
B2 = N ORM DIST (A2, 0, 1, 1)
B3 = N ORM DIST (A3, 0, 1, 1)
N ORM DIST (A518, 0, 1, 1)

...

Next, plot the range "A2:B518" as xy diagram in Excel. The plot you got
should look like the 2nd diagram in DM Figure 18.2 (see DM page 589). You
can see that from diagram from z = 2.58 to z = 2.58 is roughly a straight line.
The point of this Excel experiment: if you plot samples from a normal
distribution in a normal probability plot, youll get roughly a straight-line. If
you plot samples from an unknown distribution in a normal probability plot and
get roughly a straight line, then the samples are from approximately normal
distribution.
Steps on how to construct a normal probability plot:
1. Sort the samples from smallest to biggest. Number the samples as i =
1, 2, 3, ..., n. So you samples are x1 , x2 , ..., xn .

B518 =

18.7. SAMPLE PROBLEMS

167

i 0.5
.Here
n
1
0.5 is the continuity adjustment. Calculate the corresponding zi = N (yi ).

2. Assign the cumulative probability to xi as yi = P (x < xi ) =

3. Graph the data pairs (xi , yi ) or (xi , zi )


If the graph is roughly a straight line, then x1 , x2 , ..., xn are approximately
normal.
Heres a table to summarize the 3 steps for building a normal probability
plot:
index

xi

x1

x2

x3

...

...

xn

yi = P (x < xi ) =
0.5
y1 =
n
1.5
y2 =
n
2.5
y3 =
n
...
n 0.5
yn =
n

i 0.5
n

zi = N 1 (yi )

0.5
z1 = N 1
n
1.5
z2 = N 1
n
2.5
z3 = N 1
n
...

n 0.5
1
zn = N
n

Lets walk through DM Example 18.9.


i 0.5
yi = P (x < xi ) =
zi = N 1 (yi )
n
0.5
1 3
N 1 (0.1) = 1.2816
= 0.1
5
1.5
2 4
N 1 (0.3) = 0.5244
= 0.3
5
2.5
3 5
N 1 (0.5) = 0
= 0.5
5
3.5
4 7
N 1 (0.7) = 0.5244
= 0.7
5
4.5
5 11
N 1 (0.9) = 1.2816
= 0.9
5
Next, plot the data pairs (xi , yi ) or (xi , zi ). The result is the top two diagrams in DM Figure 18.6.
i

18.7

xi

Sample problems

Problem 1

Stock prices follow the lognormal distribution


S0 = 100

168

CHAPTER 18. LOGNORMAL DISTRIBUTION

= 0.12
= 0.06
= 0.25
T =2
Construct the 95% confidence interval of ST .
Solution
First we find the 95% confidence interval for the standard normal random
variable z. We need to b such that P (b < z < b) = 0.95.
P (b < z < b) = N (b) N (b) = N (b) [1 N (b)] = 2N (b) 1 = 0.95
1 + 0.95
N (b) =
= 0.975
b = N 1 (0.975) = 1. 96
2
So the 95% confidence interval for z is (1.96, 1.96).
ln

ST
N a 0.5 2 T, 2 T
S0

a 0.5 2 T = 0.12 0.06 0.5 0.252 2 = 0.057 5


2 T = 0.252 2
ST
The 95% confidence interval for ln
is:
S0

0.057 5 1.960.252 2 = 0.635 46


0.057 5 + 1.96 0.252 2 = 0.750 46
The 95% confidence interval for ST is:
100e0.635 46 = 52. 97
100e0.750 46 = 211. 80
The 95% confidence interval for ST is (52. 97, 211. 80)
Problem 2 (Spring 2007 Exam C #4)
You are given the following information for a stock with current price 0.25:
The price of the stock is lognormally distributed with continuously compounded expected annual rate of return = 0.15.
The dividend yield of the stock is zero.
The volatility of the stock is = 0.35 .

18.7. SAMPLE PROBLEMS

169

Using the procedure described in the McDonald text, determine the upper
bound of the 90% confidence interval for the price of the stock in 6 months.
Solution
90% confidence interval for a standard normal random variable is (1.645, 1.645).

ST
ln
N a 0.5 2 T, 2 T
S0

a 0.5 2 T = 0.15 0 0.5 0.352 0.5 = 0.044 375


2 T = 0.352 0.5

ST
is 0.044 375 1.645 0.352 0.5, 0.044 375 + 1.645 0.352 0.5 .
S0
So the upper boundfor ST is:
2
0.25e0.044 375+1.645 0.35 0.5 = 0.392 7

90% confidence interval for ln

Problem 3
A European call option and a European put option are written on the same
stock. You are given:
S0 = 100
K = 105
= 0.08
=0
= 0.3
T = 0.5
Calculate the probability that the call option will be exercised.
Calculate the probability that the put option will be exercised.
Solution
just need to use
The call will be exercised is S
T > K. We
DM 18.24.

S
0

ln
+ 0.5 2 t

P (ST > K) = N d2 = N K

100
ln
+ 0.08 0 0.5 0.32 0.5

= N 105

0.3 0.5

= N (0.147 5 ) = 1 N (0.147 5 ) = 1 NormalDist (0.147 5 ) = 0.441 4

The put option will


exercise is S
be
T <
K

P (ST < K) = N d2 = 1 N d2 = 1 0.441 4 = 0.558 6

170

CHAPTER 18. LOGNORMAL DISTRIBUTION

Alternatively,
P (ST < K) = 1 P (ST > K) = 0.558 6
Problem 4
S0 = 50
= 0.1
=0
= 0.4
Calculate the conditional expected value of the stock at T = 3 given ST > 75.
Calculate the conditional expected value of the stock at T = 3 given ST < 75.
Solution
Need to use DM 18.28 and 18.29.

S0 e()t N d1

E (St |St < K) =

N d 2

d1

ln
=

S0 e()t N d1

E (St |St > K) =

N d2

S0
50
ln
+ + 0.5 2 t
+ 0.1 0 + 0.5 0.42 3
K
75

= 0.194 2
=
t
0.4 3

S0
50
ln
+ 0.5 2 t
+ 0.1 0 0.5 0.42 3
K
75

=
= 0.498 6
=
t
0.4 3
50e(0.10)3 N (0.194 2)
50e(0.10)3 0.423 0
=
= 41. 32
E (St |St < K) =
N (0.498 6)
0.6910

d2

03

ln

E (St |St > K) =

50e(0.10)3 N (0.194 2)
50e(0.10)3 (1 0.423 0)
==
= 126.
N (0.498 6)
1 0.6910

Problem 5
Stock prices follow lognormal distribution. You are given:
S0 = 100
= 0.08
=0
= 0.3
T =5

18.7. SAMPLE PROBLEMS

171

Calculate the 75 percentile of ST . Calculate the median of ST .


Solution
75 percentile of a standard normal random variable z is N 1 (0.75) = 0.674 5.

ST
ln
N a 0.5 2 T, 2 T
S0

a 0.5 2 T = 0.08 0 0.5 0.32 5 = 0.175


2 T = 0.32 5
ST
is
S0
x = 0.175 + 0.674 5 0.32 5 = 0.627 47
75 percentile of ln

75 percentile of ST is:
100ex = 100e0.627 47 = 187. 29
Median is 50 percentile, which corresponds to z = 0 and x = 0.175.
Median of ST is: 100e0.175 = 119. 12
Problem 6 (Spring 2007 C #34)
The price of a stock in seven consecutive months is:
Month Price
1
54
2
56
3
48
4
55
5
60
6
58
7
62
Based on the procedure described in the McDonald text, calculate the annualized expected return of the stock.
(A) Less than 0.28
(B) At least 0.28, but less than 0.29
(C) At least 0.29, but less than 0.30
(D) At least 0.30, but less than 0.31
(E) At least 0.31
Solution

172

CHAPTER 18. LOGNORMAL DISTRIBUTION


month

price

1
2
3
4
5
6
7
sum

54
56
48
55
60
58
62

St
St1

x2

0.036368
0.154151
0.136132
0.087011
0.033902
0.066691
0.138150

0.001323
0.023762
0.018532
0.007571
0.001149
0.004448
0.056785

x = ln

0.138150
St
= 0.5 2 h =
E ln
= 0.023 025
Sth
6
0.023 025
= = + 0.5 2 +
h
In the above equations,
is the stocks expected (annual) continuously compounded return
is the stocks (annual) continuously compounded dividend yield ( = 0 in
this problem)
is the stocks (annual) volatility
h = 1/12 (12 months in one year)

2
P
St

ln

St
n
St1
1 P ln St
=
V ar ln

= 2h =

Sth
n1 n
St1
n
6
5

1
0.056785 0.023 0252
6

= 0.010 720 8

0.010 720 8
h
0.023 025
0.010 720 8 0.023 025
=
= + 0.5 2 +
= 0 + 0.5
+
=
h
1/12
1/12
0.340 624 8
2 =

The answer is E.

Chapter 19

Monte Carlo valuation


In this chapter, Im going to walk you through some examples. If you understand
my examples, youll know enough for the exam.

19.1

Example 1 Estimate E (ez )

Lets forget about option pricing for now and focus on a random variable X.
Suppose we want to find out E (X), the mean of X, but we dont have an easy
formula to do so. For example, we have a random variable X = ez where z is a
standard normal random variable with mean 0 and variance 1.

R
R
1
E (X) = x (z) f (z) dz = exp (z) exp 0.5z 2 dz
2

Suppose we havent heard of DM 18.13 E (ex ) = exp m + 0.5v 2 .

R
1
We dont know how to do the integration exp (z) exp 0.5z 2 dz.
2
How can we calculate E (X)?
One approach is to create n instances of z ,
calculate
the corresponding
n
1 P
z
Xi . Based on the cenvalue of X = e , and find the sample mean =
n i=1

n
1 P
Xi is normally distributed with mean E () =
tral limit theorem, =
n i=1
n

n
P
P
1
1
1
Xi = (nE (X)) = E (X) and variance V ar () = 2 V ar
Xi =
E
n
n
n
i=1
i=1
1
V ar (X)
nV ar (X) =
. Since E () = E (X), we can calculate the mean of
2
n
n
the sample mean E () and use it to approximate the population mean E (X).
Lets come back to the example of estimating E (ez ). Im going to produce 10
sample means and the use the average of these 10 sample means as an estimate
to E (ez ). To produce each sample mean, Im going to randomly draw 10, 000
z 0 s (so I have 10, 000 values of ez to produce each sample mean). Another way
173

174

CHAPTER 19. MONTE CARLO VALUATION

to describe this process is to say that Im going to have 10 trials with each trial
having 10, 000 samples.
Im going to do this in Microsoft Excel. The following table shows how to
calculate the sample mean for one trial.

1
2
3
4
5
6
7
8
......
10001
10002
10003

A
simulation i
1
2
3
4
5
6
7
...
10000
Total
sample mean

B
zi
0.042457
0.253383
0.017632
1.167590
0.273333
0.206665
0.305234
...
1.506862

C
Xi = ezi
0.958432
1.288376
1.017788
0.311116
1.314338
0.813292
1.356942
...
0.221604
16, 479.279610
1.647928

Sample formulas in Excel:


B2=NORMINV(RAND(),0,1) = 0.042457

C2 = e0.042457 = 0.958 432

Lets walk through B2. Rand() is Excels formula for a random draw in a
uniform distribution over (0, 1). Norminv(p, , ) is Excels formula for the inverse normal distribution. For example, for a standard normal random variable
z N (0, 1), P (z 1.96) = N (1.96) = 0.975. Then Norminv(0.975, 0, 1) =
N 1
(0.975)
= 1.96. Another example. For a normal random variable Z
N 2, 0.32 , P (Z 2) = 0.5. Then Norminv(0.5, 2, 0.3) = 2.
NORMINV(RAND(),0,1) works this way. First, Rand() generates a number
from a uniform distribution over (0, 1). Say this random number is 0.4831. Then
Excel finds that P (z 0.042457) = 0.4831. Hence a =NORMINV(0.4831,0,1) =
0.042457.
Please note that Excel has a similar formula normsinv for calculating the
inverse normal random variable. The "s" in normsinv stands for standard. So
normsinv produces the inverse standard normal distribution, while norminv produces the inverse normal distribution with mean and standard deviation . In
other words, normsinv(p) =norminv(p, = 0, = 1). So normsinv(0.975) =norminv(0.975, 0, 1) =
1.96.
Similarly, the formula for B3 and C3 are:
B3=NORMINV(RAND(),0,1) = 0.253383
C3 = e0.253383 = 1. 288 376
B10001=NORMINV(RAND(),0,1) = 1.506862
C10001 = e1.506862 =
0.221 604
C10002=sum(C2:C10001)=16, 479.279610


19.1. EXAMPLE 1 ESTIMATE E E Z

175

The samplemean
is
n
P
1
16, 479.279610
=
Xi =
= 1.647928
10000 i=1
10000

Next, Im going to produce 9 more trials. Here is the snapshot of Excel:

i
1
2
3
...
10000
P
X

Trial 1
zi
0.042457
0.253383
0.017632
1.506862

Now I have 10
Trial

1
1.6479
2
1.6512
3
1.6163
4
1.6325
5
1.6821
6
1.6626
7
1.6656
8
1.5994
9
1.6459
10 1.6554
total 16.4589

Xi = ezi
0.958432
1.288376
1.017788

Trial 2
zi
0.985719
1.324269
0.157924

Xi = ezi
0.373171
0.265997
0.853915

0.221604
16, 479

1.673002

0.187683
16, 512

1.6479

...

Trial 10
zi
0.783098
0.186120
0.504305
1.177804

1.6512

Xi = ezi
0.456988
1.204567
1.655835
3.247236
16, 554
1.6554

sample means (k = 10):


2
2.715574
2.726461
2.612426
2.665056
2.82946
2.764239
2.774223
2.55808
2.708987
2.740349
27.09486

E (X) = E () '

1.6479 + 1.6512 + ... + 1.6554


1 P
i =
= 1.64589
10
10

So E (X) ' 1.64589. The correct value of E (X) according to DM 18.13 is


E (X) = e0.5 = 1. 648 721.
We can also construct the confidence interval of E (). The estimated variance of the sample
mean

is P
P
P

2 !
2
i
1P
2i k ( i )2
k
1P 2
V ar () '
=
=
=
i
i
k1
k1
k1 k
k

10
2.709486 1.645892 = 0.000 591 2
9
p

V ar () ' 0.000 591 2 = 0.02 431


The 95% confidence interval is [1. 598 24, 1. 693 54]. This is calculated as
follows:

176

CHAPTER 19. MONTE CARLO VALUATION

First, we calculate the critical value z where P (z z ) =


0.025 . This gives us z = 1.96.
Then lower bound of the 95% confidence interval is
1.64589 1.96 (0.02 431) = 1. 598 242 4

1 0.95
=
2

Then upper bound of the 95% confidence interval is


1.64589 + 1.96 (0.02 431) = 1. 693 537 6
So we are 95% certain that the sample mean E () is in the range [1. 598 24, 1. 693 54].
Since E (X) = E (), we are 95% certain that E (X) is in the range [1. 598 24, 1. 693 54].
The 99% confidence interval is calculated as follows:
1 0.99
First, we calculate the critical value z where P (z z ) =
=
2

0.005 . This gives us z = 2.58.


Then lower bound of the 99% confidence interval is
1.64589 2.58 (0.02 431) = 1. 583 170 2
Then upper bound of the 99% confidence interval is
1.64589 + 2.58 (0.02 431) = 1. 708 609 8
So we are 99% certain that E (X) is in the range [1. 583 2, 1. 708 6]
Some other key points in this example.
This method to generate a normal random variable is called the inversion
method (DM textbook page 622 and 623). This method works as follows:
Step 1. Generate a random number p from the uniform distribution over
(0, 1). Say p = 0.4831.
Step 2. Ask "For a normal random variable Z with mean and standard
deviation , whats the number a such that P (Z a) = p?" In this example, we find P (z a) = 0.4831. Then a = 0.042457 is the simulated
normal random variable.
We can also generate the normal random variable as the sum of 12 uniformly
distributed random variables minus 6 (DM textbook 622):

Z = u1 + u2 + ... + u12 6 where ui is an independent identically distributed


uniform random variable over (0, 1).
R1
R1
R1
R1
E (ui ) = 0 uf (t) du = 0 udu = 1/2
E u2i = 0 u2 f (t) du = 0 u2 du =
1/3
2
1
1
1
V ar (ui ) =
=
3
2
12
Based the central limit theorem, u1 + u2 + ... + u12 is approximately normal.
The normal random variable with a constant 6 added is still normal. Hence

Z is normal.

19.2. EXAMPLE 2 ESTIMATE

177

1
E Z = E (u1 + u2 + ... + u12 ) 6 = 12 6 = 0
2

1
V ar Z = V ar (u1 + u2 + ... + u12 ) = 12
=1
12

So Z roughly a standard normal random variable.


The following snapshot shows how to create 5 standard normal random variables in Excel:
ui
trial 1
trial 2
trial 3
trial 4
trial 5
1
0.6316
0.3308
0.0062
0.3634
0.3207
2
0.0247
0.6775
0.0296
0.8678
0.7361
3
0.4223
0.3489
0.8446
0.4283
0.8308
4
0.1326
0.5315
0.3436
0.3937
0.5658
5
0.1944
0.6660
0.2083
0.4484
0.6079
6
0.5996
0.3129
0.5807
0.3446
0.2882
7
0.7959
0.7749
0.9899
0.5553
0.9242
8
0.5468
0.2398
0.7499
0.5410
0.7900
9
0.1719
0.0645
0.0679
0.8638
0.0394
10
0.0964
0.4656
0.1779
0.8405
0.5718
11
0.1059
0.9432
0.3354
0.6266
0.4902
12
0.6106
0.0310
0.6455
0.8713
0.7856
sum
4.3328
5.3868
4.9794
7.1447
6.9508
sum-6
-1.6672
-0.6132
-1.0206
1.1447
0.9508
Each uniform random variable is created using Excel formula Rand().
The last row show the 5 standard normal random variable created.
For example, the first standard normal random variable is created as follows:
(0.6316 + 0.0247 + 0.4223 + ... + 0.6106) 6 = 1.6672
This method is more cumbersome than the inversion method.

19.2

Example 2 Estimate

We decide to estimate using the Monte Carlo simulation (we can treat as
a random variable that happens to be a constant). Well estimate using a
classic "throwing the dart" method.
Imagine a square whose size is two units. The center of this square is the
origin (0, 0). Inside this square sits a unit circle x2 + y 2 = 1. The center of
this circle is also (0, 0). If we randomly throw a dart at the square, whats the
probability that the dart falls within the unit circle?

178

CHAPTER 19. MONTE CARLO VALUATION

1.0
0.8
0.6
0.4
0.2

-1.0 -0.8 -0.6 -0.4 -0.2


-0.2

0.2

0.4

0.6

0.8

1.0

-0.4
-0.6
-0.8
-1.0

Of all the darts falling into the square (we ignore the darts falling out of the
square), the probability that the dart falls within the circle
is the area of the
12

circle divided by the area of the square, which is P =


= .
22
4
We can also focus on the first quadrant. Of all the darts falling in square in
1 2
1
the first quadrant, the probability that they fall in the circle is P = 4 2
=
1

. Then = 4P .
4
Next, we need to simulate darts falling in the first quadrant. Once again,
well do the simulation in Excel. We plan to produce 20 estimates of . For
each estimate, well use 10, 000 simulations. Heres how to produce one trial
(one trial=10, 000 simulations).
Create the first point (x1 , y1 ), where 0 < x1 < 1 and 0 < y1 < 1. We can
create x1 and y1 by randomly drawing two numbers from U (0, 1), a
uniform distribution over [0, 1].
If x21 + y12 1, then (x1 , y1 ) falls in the circle in the first quadrant.
Similarly, create the 2nd point (x2 , y2 ) by randomly drawing two numbers
from U (0, 1). Determine whether (x2 , y2 ) falls in the circle in the first
quadrant.
......

19.2. EXAMPLE 2 ESTIMATE

179

Create n-th point (xn , yn ) by randomly drawing two numbers from U


[0, 1]. Determine whether (xn , yn ) falls in the circle in the first quadrant.
Set n = 65, 000.
Count m, the total number of points that fall in the circle.
P

. Then = 4P .
n

Heres the snapshot


A
1 throw i
2
1
3
2
4
3
5
4
6
5
7
6
8
7
...
...
10, 001 10, 000

of Excel.
B
C
xi
yi
0.2854
0.1694
0.5147
0.2787
0.1915
0.2699
0.6501
0.8420
0.0007
0.6580
0.1520
0.9772
0.3179
0.5713
...
...
0.5560 0.5679

D
x2i + yi2
0.1102
0.3426
0.1095
1.1317
0.4330
0.9780
0.4274
...
0.6317

E
Fall in the circle? (1=Yes, 0=No)
1
1
1
0
1
1
1
...
1

The number of simulations: n = 10, 000.


We find that the number of darts falling in the circle in the first quadrant:
m = 7, 894
m
7, 894
P
=
= 0.7894
n
10, 000

= 4 (0.7894) = 3. 157 6
Sample formulas in the above Excel spreadsheet are:
B2=rand()=0.2854
C2=rand()=0.1694
D2=B2^2+C2^2=0.1102
E2=if(D2>=1,1,0)=1
B3=rand()=0.5147
C3=rand()=0.2787
D3=B3^2+C3^2=0.3426
E3=if(D3>=1,1,0)=1
m=SUM(E2:E10001)=7, 894
We produce 19 more trials. The total number of trials is k = 20. Here is the
snapshot of Excel:
2

trial mi
pi
i
i
1
7,894
0.7894
3.1576
9.97043776
2
7,818
0.7818
3.1272
9.77937984
3
7,830
0.783
3.1320
9.80942400
4
7,752
0.7752
3.1008
9.61496064
5
7,879
0.7879
3.1516
9.93258256
6
7,879
0.7879
3.1516
9.93258256
7
7,778
0.7778
3.1112
9.67956544

180
8
9
10
11
12
13
14
15
16
17
18
19
20
total

CHAPTER 19. MONTE CARLO VALUATION


7,878
7,891
7,896
7,844
7,880
7,920
7,821
7,825
7,870
7,886
7,822
7,839
7,762

0.7878
0.7891
0.7896
0.7844
0.788
0.792
0.7821
0.7825
0.787
0.7886
0.7822
0.7839
0.7762

3.1512
9.93006144
3.1564
9.96286096
3.1584
9.97549056
3.1376
9.84453376
3.1520
9.93510400
3.1680
10.03622400
3.1284
9.78688656
3.1300
9.79690000
3.1480
9.90990400
3.1544
9.95023936
3.1288
9.78938944
3.1356
9.83198736
3.1048
9.63978304
62.7856
197.10829728

The estimated mean of the sample mean is

P
62.7856
i
E =
=
= 3. 139 28
k
20

The estimated variance


of the sample mean
is:

2 !

P
P
k
1
20 197.10829728
1
=
2i
i
3. 139 282 =
V ar '
k1 k
k
19
20
0.0003536 27
p

V ar () ' 0.0003536 27 = 0.01880 5


The 95% confidence interval is [3. 102 4, 1. 3. 176 1]. This is calculated as
follows:
3. 139 28 1.96 (0.01880 5) = 3. 102 422 2
3. 139 28 + 1.96 (0.01880 5) = 3. 176 137 8
The 99% confidence interval is [3. 090 8, 3. 187 8]. This is calculated as follows:
3. 139 28 2.58 (0.01880 5) = 3. 090 763 1
Then upper bound of the 99% confidence interval is
3. 139 28 + 2.58 (0.01880 5) = 3. 187 796 9
Now you have some ideas about the Monte Carlo simulation. Lets look at
the textbook.

19.3

Example 3 Estimate the price of European


call or put options

Monte Carlo simulation can be used to price European options, especially when
theres no simple formula for the option price such as the arithmetic Asian
option.

19.3. EXAMPLE 3 ESTIMATE THE PRICE OF EUROPEAN CALL OR PUT OPTIONS181


First, lets use the Monte Carlo method to calculate the price of a European
call option and put option. The inputs are (See DM page 377 Example 12.1):
S0 = 41
K = 40
= 0.3
r = 8%
=0
T = 0.25 (3 months)
Solution
Well use DM Equation 19.6 to calculate the option price:
n
n 1

P
1 rT P
V STi , T = erT
e
V STi , T
n
t=1
t=1 n
n 1
i
P
V ST , T is the average terminal payo. So the price of the European
t=1 n
option is just the discounted value of the terminal payo.

V (S0 , 0) =

Were are going to produce 5 sample means. For each sample mean, well
use 10 simulations. So we have 5 trials with 20 simulations per trial. For each
trial, we need to generate the stocks terminal price at T = 0.25.
We generate hthe terminal stock price using
DM 19.3:
i

ST = S0 exp 0.5 2 T + T Z

Please note that the above formula produces the real world stock price at
T . If we use the real world terminal stock price to calculate the option price,
you have to use the real world discount rate, which is path-dependent (see DM
Table 19.1). Path dependent discount rates are dicult to calculate. To avoid
the diculty of finding the path-dependent discount rates, well want to live in
the risk neutral world where everything earns the risk-free rate. To move into
the risk neutral hworld, we just set = r andi change DM 19.3 into:

ST = S0 exp r 0.52 T + T Z

S0.25 = 41 exp 0.08 0.5 0.32 0.25 + 0.3 0.25Z = 41 exp (0.15Z + 0.008 75)

Next, well produce 10 simulations of Z and S0.25 . The following is the


snapshot of Excel simulation:
index
Z
1
0.67830
2
0.65327
3
1.21158

S0.25
Call payo
45.7901
5.7901
45.6185
5.6185
49.6034
9.6034

put payo
0.0000
0.0000
0.0000

182

CHAPTER 19. MONTE CARLO VALUATION

4
1.50347
51.8234
11.8234
0.0000
5
0.62495
45.4251
5.4251
0.0000
6
-1.89286
31.1369
0.0000
8.8631
7
0.57282
45.0712
5.0712
0.0000
8
-0.84887
36.4154
0.0000
3.5846
9
-0.34767
39.2586
0.0000
0.7414
10 -0.34540
39.2720
0.0000
0.7280
Total
429.4146
43.3317
13.9171
Sample calculations.
If Z = 0.67830, S0.25 = 41 exp (0.15 0.67830 + 0.008 75) = 45. 790 1
The call payo is 45. 790 1 40 = 5. 790 1
The put payo is zero.
43.3317
= 4. 333 17
10
rT
So the call price is 4. 333 17e
= 4. 333 17e0.080.25 = 4. 247 4

The average call payo at T is:

13.9171
= 1. 391 71
10
0.080.25
= 1. 364 2
So the put price is 1. 391 71e
The average put payo at T is:

Now Im going to repeat this process 4 more times. This is my final result:
trials
call price
put price
1
4. 247 4
1. 364 2
2
3.9132
1.2391
3
0.5436
2.4875
4
1.6850
1.8342
5
2.4911
1.9250
The mean of the sample mean for the call price is:
4. 247 4 + 3.9132 + 0.5436 + 1.6850 + 2.4911
= 2. 58
5
Compare this with the correct price using the Black-Scholes formula (see
DM page 377): C = 3.40
The mean of the sample mean for the put price is:
1. 364 2 + 1.2391 + 2.4875 + 1.8342 + 1.9250
= 1. 77
5
Compare this with the correct price using the Black-Scholes formula (see
DM page 377): P = 1.61
The prices from the Monte Carlo simulations are o because the number of
simulations per trial is small.
After running 10 trials (5,000 simulations per trial), I got the following:
trials
call
call^2
put
put^2
1
3.4132
11.6502
1.5976
2.5523

19.3. EXAMPLE 3 ESTIMATE THE PRICE OF EUROPEAN CALL OR PUT OPTIONS183


2
3
4
5
6
7
8
9
10

3.3931
3.4085
3.4002
3.4019
3.4224
3.4089
3.4140
3.3998
3.4018

11.5134
11.6182
11.5614
11.5729
11.7129
11.6206
11.6551
11.5587
11.5725

1.6003
1.6007
1.6249
1.5815
1.6056
1.6158
1.6039
1.5879
1.6216

2.5611
2.5622
2.6402
2.5012
2.5778
2.6108
2.5725
2.5214
2.6297

Sum 34.0639 116.03585


16.0398 25.7292
So the mean of the sample mean for call price is:
34.0639
= 3. 40639
10
Thevariance of the sample
mean for the call price is:
10 116.03585
3. 406 392 = 1. 024 1 104
9
10

1. 024 1 104 = 0.01


So the 95% confidence interval for the call price is [3. 39, 1. 3. 43]. This is
calculated as follows:
3. 40639 1.96 (0.01) = 3. 39
3. 40639 + 1.96 (0.01) = 3. 43
So the mean of the sample mean for put price is:
16.0398
= 1. 603 98
10
Thevariance of the sample
mean for the put price is:
10 25.7292
1. 603 982 = 0.000 186 844
9
10

0.000 186 844 = 0.014


So the 95% confidence interval for the call price is [1.58, 1.63]. This is calculated as follows:
1. 603 98 1.96 (0.014) = 1. 58
1. 603 98 + 1.96 (0.014) = 1. 63
By the way, theres no need for us to use Monte Carlo summations to calculate a European call or put option. The Black-Scholes formula can produce
the price of a European call or put option. Here we calculate European option
prices to illustrate how to use the Monte Carlo simulation.
Please note that using the Monte Carlo simulation to calculate the American
option price is covered in DM 1936, which is out of the scope of exam MFE.

184

19.4

CHAPTER 19. MONTE CARLO VALUATION

Example 4 Arithmetic and geometric options

Lets use the Monte Carlo method to calculate the price of an arithmetic call
option and an arithmetic put option. The inputs are (See DM page 629 Table
19.3):
S0 = 40
K = 40
= 0.3
r = 8%
=0
T = 0.25 (3 months)
The average stock price is the average stock prices at the end of Month 1,
Month2, and Month 3
Solution
The monthly interval is h = T /3 = 0.25/3
The stock prices (the real world price)
at the end of Month 1, 2, and 3 are:

end of Month 1:
Sh = S0 exp 0.5 2 h + hz1

end of Month 2:
S2h = Sh exp 0.5 2 h + hz2

end of Month 3:
S3h = ST = S2h exp 0.5 2 h + hz3
where z1 , z2 , and z3 are three separate random draws of the standard normal
distribution.
Since our goal is to calculate the option price, we dont need the real world
stock price. We just need the risk-neutral stock price. We change the stocks
expected return into the risk free rate r. The risk neutral stock prices at the
end of Month 1,2, and 3 are:

Sh = S0 exp r 0.5 2 h + hz1

S2h = Sh exp r 0.5 2 h + hz2

S3h = S2h exp r 0.5 2 h + hz3


The average stock price is S =
The arithmetic call price is:
The arithmetic call price is:

Sh + S2h + S3h
3

C = erT max S K, 0

P = erT max K S, 0

19.4. EXAMPLE 4 ARITHMETIC AND GEOMETRIC OPTIONS

185

The following snapshot of Excel shows one trial (5000 simulations per trial)
of the call/put price:
i
z1
Sh
z2
S2h
z3
S3h
S
1
0.8424 43.1529
0.258 44.2568 2.4041 36.0434 41.1510
2
0.066 40.3468 1.4415 35.7157 2.0925 29.8830 35.3152
3
0.5357 42.0218
0.1255 42.6051 0.8295 39.7677 41.4649
4
0.0014 40.1217
1.0027 43.8893 0.8446 40.9128 41.6413
5
1.598 34.9321 1.5906 30.5258
1.2832 34.2134 33.2238
6
1.2843 44.8364 0.6896 42.3605 0.3961 41.0516 42.7495
7 0.0712 39.8702
0.2363 40.8134
0.0147 40.9848 40.5561
8 1.0075 36.7649 0.5005 35.3082
0.6539 37.4745 36.5159
...
...
...
...
...
...
...
...
5000 0.0168 40.0585 1.2203 36.1464
0.6609 38.3874 38.1974

C pay
1.1510
0.0000
1.4649
1.6413
0.0000
2.7495
0.5561
0.0000
...
0.0000

P pay
0.0000
4.6848
0.0000
0.0000
6.7762
0.0000
0.0000
3.4841
...
1.8026

Sample calculations for the first row.


First, we draw z1 = 0.8424, z2 = 0.258,and z3 = 2.4041 from the standard
normal distribution.

!
r

0.25
0.25
Sh = 40 exp 0.08 0 0.5 0.32
+ 0.3
0.8424 = 40 exp (0.075 87) =
3
3
43. 152 9

!
r

0.25
2 0.25
+ 0.3
0.258 =
S2h = 43. 152 9 exp 0.08 0 0.5 0.3
3
3

2
= 44. 256 8
43. 152 9 exp 2. 526 01
10
!
r

0.25
2 0.25
S3h = 44. 256 8 exp 0.08 0 0.5 0.3
+ 0.3
(2.4041) =
3
3
44. 256 8 exp (0.205 28) = 36. 0434
43. 152 9 + 44. 256 8 + 36. 0434
S=
= 41. 151 0
3
index
1
2
3
4
5
6
7
8
...
5000
sum

Call payo
1.1510
0.0000
1.4649
1.6413
0.0000
2.7495
0.5561
0.0000
...
0.0000
10, 111.9410

put payo
0.0000
4.6848
0.0000
0.0000
6.7762
0.0000
0.0000
3.4841
...
1.8026
7, 390.6006

Sample calculations for the first row.

C
1.1282
0.0000
1.4359
1.6088
0.0000
2.6951
0.5451
0.0000
...
0.0000
9, 911.7119

P
0.0000
4.5920
0.0000
0.0000
6.6420
0.0000
0.0000
3.4151
...
1.7669
7, 244.2548

C2
1.2728
0.0000
2.0618
2.5882
0.0000
7.2636
0.2971
0.0000
...
0.0000
61, 373.4417

P2
0.0000
21.0865
0.0000
0.0000
44.1162
0.0000
0.0000
11.6629
...
3.1219
32, 916.8904

186

CHAPTER 19. MONTE CARLO VALUATION

Call price: erT call payo= e0.08(0.25) 1.1510 = 1. 128 2


Put payo: erT put payo= e0.08(0.25) 0.0000 = 0
C 2 = 1. 128 22 = 1. 272 8
P 2 = 02 = 0
The estimated option prices are calculated as follows.

index
Call payo put payo
C
sum 10, 111.9410 7, 390.6006 9, 911.7119
The average call price of these 5000 simulations:

erT

or

P
7, 244.2548

C2
61, 373.4417

P2
32, 916.8904

1 P
10111.9410
call payo= e0.08(0.25)
= 1. 982 34
5000
5000

9911.7119
1 P
C=
= 1. 982 34
5000
5000

The average put price of these 5000 simulations:

erT

or

1 P
7390.6006
put payo= e0.08(0.25)
= 1. 448 9
5000
5000

1 P
7244.2548
P =
= 1. 448 9
5000
5000

The estimated variance of the call price per simulation:

P 2 !

C
5000
1 P 2
1
n
=
C
61373.4417 1. 982 342 =
n1 n
n
5000 1 5000
8. 346 7

The estimated variance of the put price per simulation:

P 2 !

2 !
n
5000
P
1 P 2
1
7244.2548
=
P
32916.8904
n1 n
n
5000 1 5000
5000
= 4. 485 1

The following is the snapshot of Excel for 30 trials (5000 simulations per
trial) for the arithmetic European call and put prices:

19.4. EXAMPLE 4 ARITHMETIC AND GEOMETRIC OPTIONS


i
1
2
3
4
5
6
7
8
9
10
11
12
13
14
15
16
17
18
19
20
21
22
23
24
25
26
27
28
29
30
sum

C
1.9823
1.9409
1.9467
2.0098
1.9894
1.9515
1.9825
2.0462
2.1270
1.9583
1.9878
1.9664
2.0147
1.9664
1.9915
1.9504
1.9414
1.9804
1.9666
2.0276
1.9940
1.9307
1.9221
1.9710
1.9185
1.9529
1.9791
1.9776
1.9713
1.9689
59.3139

P
1.4489
1.4486
1.4920
1.4211
1.4447
1.4886
1.4813
1.4408
1.3565
1.4675
1.4691
1.4496
1.3785
1.4327
1.4253
1.4592
1.4800
1.4186
1.4724
1.3779
1.4217
1.4815
1.4903
1.4491
1.4526
1.4638
1.4538
1.4676
1.5092
1.4640
43.5069

C2
3.929513
3.767093
3.789641
4.039296
3.957712
3.808352
3.930306
4.186934
4.524129
3.834939
3.951349
3.866729
4.059016
3.866729
3.966072
3.804060
3.769034
3.921984
3.867516
4.111162
3.976036
3.727602
3.694468
3.884841
3.680642
3.813818
3.916837
3.910902
3.886024
3.876567
117.3193

187

P2
2.099311
2.098442
2.226064
2.019525
2.087158
2.215930
2.194250
2.075905
1.840092
2.153556
2.158255
2.101340
1.900262
2.052629
2.031480
2.129265
2.190400
2.012426
2.167962
1.898608
2.021231
2.194842
2.220994
2.099891
2.110047
2.142710
2.113534
2.153850
2.277685
2.143296
63.1309

For the remaining part of the calculation, all you need to know is the following:
Trial
C
P
C2
P2
sum 59.3139 43.5069 117.3193 63.1309

The estimated arithmetic call price is:


59.3139
= 1. 977 13
E C =
30
The estimated variance of the call price (per trial) is:

188

CHAPTER 19. MONTE CARLO VALUATION

Ci


P 2
n
1
1 n=30
i=1
2
= 30
Ci
V ar C =
=

117.3193

1.
977
1

n 30 1 30
n 1 n
i=1
n=30
P

0.001 778

The standard deviation is: 0.001 778 = 0.04 217

We can also estimate the variance of the call price as follows:


For each trial, the average call price of the 5, 000 simulations is used to
P
1 5,000
estimate the call price. C =
Ci .
5, 000 i=1
So the variance of the
priceper trial is:
call
5,000

P
1
1
1
V ar C =
Ci =
5, 000V ar (C) =
V ar
V ar (C) =
2
2
5, 000
5, 000
5000
i=1
8. 346 7
= 0.001 669
5000

The standard deviation is: 0.001 669 = 0.0406


Please note that the two methods of estimating the variance of the call price
per trial often produce close, but not identical, results. The results are not
identical because the estimated variance per trial changes depending how many
trial you have and how many simulations per trial.
The estimated arithmetic put price is:
43.5069
E P =
= 1. 450 23
30
The estimated variance of the put price (per trial) is:

n=30
2
P

P
i


P 2
1
30
n
1 n=30

i=1
2

Pi
63.1309 1. 450 23 =
V ar P =

=
n 1 n
n 30 1 30
i=1

0.001 237 5

The standard deviation is: 0.001 237 5 = 0.03 518

We can also estimate the variance of the put price as follows:


For each trial, the average call price of the 5, 000 simulations is used to
P
1 5,000
estimate the call price. P =
Pi .
5, 000 i=1

5,000

P
1
1
1
V ar P =
V ar
Pi =
5, 000V ar (P ) =
V ar (P ) =
2
2
5, 000
5, 000
5000
i=1
4. 4668
= 0.000 89
5000

The standard deviation is: 0.000 89 = 0.0298


Now lets use Monte Carlo simulation to estimate the geometric call and put
prices. Im going to use the same inputs:

19.4. EXAMPLE 4 ARITHMETIC AND GEOMETRIC OPTIONS

189

S0 = 40
K = 40
= 0.3
r = 8%
=0
T = 0.25 (3 months)
The average stock price is the average stock prices at the end of Month 1,
Month2, and Month 3

In addition, Im going to use the same random draws of standard normal


random variables z1 , z2 , and z3 as used for simulating the arithmetic call/put
prices. Though I dont have to use the z1 , z2 , and z3 used for estimating the
arithmetic call/put prices, reuse saves me time. Here is a snapshot of Excel
doing one trial:
i
z1
Sh
z2
S2h
1
0.8424 43.1529
0.258 44.2568
2
0.066 40.3468 1.4415 35.7157
3
0.5357 42.0218
0.1255 42.6051
4
0.0014 40.1217
1.0027 43.8893
5
1.598 34.9321 1.5906 30.5258
6
1.2843 44.8364 0.6896 42.3605
7 0.0712 39.8702
0.2363 40.8134
8 1.0075 36.7649 0.5005 35.3082
...
...
...
...
...
5000 0.0168 40.0585 1.2203 36.1464
sum
Sample calculations.
The 1st simulation.
S = (43.1529 44.2568 36.0434)1/3 = 40. 983 1
Call payo: 40. 983 1 40 = 0.983 1
Put payo: 0
The 5000-th simulation:
i
z1
Sh
5000 0.0168 40.0585

z2
1.2203

S2h
36.1464

S = (40.0585 36.1464 38.3874)1/3 = 38. 163 6


Call payo: 0
Put payo: 40 38. 163 6 = 1. 836 4

z3
2.4041
2.0925
0.8295
0.8446
1.2832
0.3961
0.0147
0.6539
...
0.6609

z3
0.6609

S3h
36.0434
29.8830
39.7677
40.9128
34.2134
41.0516
40.9848
37.4745
...
38.3874

S3h
38.3874

S
40.9831
35.0508
41.4466
41.6101
33.1662
42.7209
40.5532
36.5047
...
38. 163 6

S
38. 163 6

C pay
0.9831
0
1.4466
1.6101
0
2.7209
0.5532
0
....
0
9, 911.3146

C pay
0

P pay
0
4.9492
0
0
6.8338
0
0
3.4953
...
1. 836 4
7, 528.5134

P pay
1. 836 4

190

CHAPTER 19. MONTE CARLO VALUATION

The next snapshot:


i call payo
1
0.9831
2
0
3
1.4466
4
1.6101
5
0
6
2.7209
7
0.5532
8
0
...
...
5000
0
sum 9, 911.3146

put payo
0
4.9492
0
0
6.8338
0
0
3.4953
...
1. 836 4
7, 528.5134

C
0.9636
0
1.418
1.5782
0
2.667
0.5422
0
...
0
9, 715.0581

P
0
4.8512
0
0
6.6985
0
0
3.4261
...
1. 800 0
7, 379.4385

C2
0.9285
0
2.0107
2.4907
0
7.1129
0.2940
0

P2
0
23.5341
0
0
44.8699
0
0
11.7382

0
59, 337.3153

3.24
33, 852.0593

Sample calculation.
The 1st simulation.
Call price: 0.9831e0.080.25 = 0.963 6
Put price: 0 e0.080.25 = 0
The 5000-th simulation.
Call price: 0
Put price: 1. 836 4e0.080.25 = 1. 800 0
The average call price of these 5000 simulations:
1 P
10111.9410
erT
call payo= e0.08(0.25)
= 1. 982 34
5000
5000
or

1 P
9911.7119
C=
= 1. 982 34
5000
5000

The following calculation uses this part of the table:


i call payo put payo
C
P
sum 9, 911.3146 7, 528.5134 9, 715.0581 7, 379.4385

C2
59, 337.3153

P2
33, 852.0593

The average call price of these 5000 simulations:


1 P
9911.3146
erT
call payo= e0.08(0.25)
= 1. 943 0
5000
5000
or

1 P
9715.0581
C=
= 1. 943 0
5000
5000

The estimated
variance of the call
price per simulation is:

P 2 !

C
5000
1 P 2
1
n
=
C
59337.3153 1. 943 02 =
n1 n
n
5000 1 5000
8. 093 8

19.4. EXAMPLE 4 ARITHMETIC AND GEOMETRIC OPTIONS


The average put price of these 5000 simulations:
erT
or

1 P
7528.5134
put payo= e0.08(0.25)
= 1. 475 9
5000
5000

1 P
7379.4385
P =
= 1. 475 9
5000
5000

The estimated
variance of the put price per
simulation is:

1
5000
33852.0593 1. 475 92 = 4. 593 0
5000 1 5000
Next is the snapshot of the 30 trials (1 trial=5,000 simulations):
i
1
2
3
4
5
6
7
8
9
10
11
12
13
14
15
16
17
18
19
20
21
22
23
24
25
26
27
28
29
30
sum

C
1. 943 0
1.9001
1.9080
1.9698
1.9506
1.9112
1.9436
2.0058
2.0854
1.9191
1.9487
1.9281
1.9748
1.9272
1.9519
1.9116
1.9039
1.9424
1.9274
1.9867
1.9535
1.8924
1.8845
1.9328
1.8804
1.9133
1.9399
1.9398
1.9318
1.9279
58.1356

P
1.4759
1.4754
1.5200
1.4476
1.4727
1.5171
1.5091
1.4675
1.3830
1.4958
1.4959
1.4775
1.4058
1.4608
1.4523
1.4863
1.5072
1.4453
1.5007
1.4049
1.4483
1.5100
1.5182
1.4771
1.4809
1.4916
1.4814
1.4947
1.5376
1.4921
44.3327

C2
3.775249
3.610380
3.640464
3.880112
3.804840
3.652685
3.777581
4.023234
4.348893
3.682945
3.797432
3.717570
3.899835
3.714100
3.809914
3.654215
3.624835
3.772918
3.714871
3.946977
3.816162
3.581178
3.551340
3.735716
3.535904
3.660717
3.763212
3.762824
3.731851
3.716798
112.7048

P2
2.178281
2.176805
2.310400
2.095546
2.168845
2.301592
2.277383
2.153556
1.912689
2.237418
2.237717
2.183006
1.976274
2.133937
2.109175
2.209088
2.271652
2.088892
2.252100
1.973744
2.097573
2.280100
2.304931
2.181824
2.193065
2.224871
2.194546
2.234128
2.364214
2.226362
65.5497

191

192

CHAPTER 19. MONTE CARLO VALUATION

C2
P2
112.7048 65.5497
58.1356
The estimated call price is:
= 1. 937 85
30
44.3327
= 1. 477 76
The estimated put price is:
30
trial
sum

C
58.1356

P
44.3327

The estimated
variance of the call price
per trial:

30
1
2
112.7048 1. 937 85 = 0.001618
30 1 30

The standard deviation is: 0.001618 = 0.040 22


The estimated variance of the call per trial can also be calculated as follows:
For each trial, the average call price of the 5, 000 simulations is used to
P
1 5,000
Ci .
estimate the call price. C =
5, 000 i=1
So the variance of the
priceper trial is:
call
5,000

P
1
1
8. 093 8
V ar C =
Ci =
V ar
V ar (C) =
= 0.001 619
2
5, 000
5000
5000
i=1
The standard deviation is:

0.001 619 = 0.04 02

The estimated
variance of the putprice per trial:

30
1
65.5497 1. 477 762 = 0.001 257 3
30 1 30
The standard deviation is:

0.001 257 3 = 0.03545 8

The estimated variance of the call per trial can also be calculated as follows:
For each trial, the average put price of the 5, 000 simulations is used to
P
1 5,000
Pi .
estimate the put price. P =
5, 000 i=1
So the variance of the
priceper trial is:
put
5,000

P
1
1
4. 593 0
V ar P =
Pi =
V ar
V ar (P ) =
= 0.000 918 6
2
5, 000
5000
5000
i=1
The standard deviation is:

0.000 918 6 = 0.0303

Heres a question. We know that the estimated standard


variance of the put
price per simulation is 4. 593 0 (the standard deviation is 4. 593 0 = 2. 143 1).
If we want the standard deviation of the put price per trial is 0.02, how many
simulations should be performed in one trial? Here is how to find it.

1
V ar C = 2
n

V ar

n
P

Ci

i=1

1
V ar (C)
nV ar (C) =
n2
n

19.5. EFFICIENT MONTE CARLO VALUATION

Set
5

V ar (C)
= 0.02. We have:
n

193

4.5930
4. 593 0
= 11482.
= 0.02 or n =
n
0.022

We need to have about 11,500 simulations per trial.

19.5

Ecient Monte Carlo valuation

19.5.1

Control variance method

Suppose we have n simulations per trial. These n simulations produce n option


prices V1 , V2 , ..., Vn . We can use the sample mean of these option prices to
estimate the true option price. So the true option is estimated as:
V1 + V2 + ... + Vn
V =
n
The variance of V is:

V ar V = V ar

V ar (V )
nV ar (V )
=
2
n
n
V
The standard deviation of the sample mean is V = , where n is the numn
ber of the simulations and V is the standard deviation of the option price per
simulation. To decrease V , we need to increase n by doing more simulations.
Doing more simulations costs time.
However, there are techniques out there to reduce V without increasing n.
One method is called the control variate method. It goes like this.
Suppose we have two random similar variables X and Y . We need to calculate E (X) and E (Y ). We can calculate E (X) easily because theres a formula
for E (X). E (Y ), on the other hand, doesnt have a formula and needs to be
calculated through the Monte Carlo simulation. Since X and Y are similar, we
expect that our errors in estimating E (X) are similar to our errors in estimating
E (Y ):

V1 + V2 + ... + Vn
n

E (X) X E (Y ) Y
In the above formula, E (X) and E (Y ) are the true means of X and Y ;

X and Y are the estimated means of X and Y based on the Monte Carlo
simulation.
Since our goal his to find E (Y
i ), we
arrangethe above formula into:

E (Y ) Y + E (X) X = Y X + E (X)

So we can use Y X + E (X) to estimate E (Y ). We define this new

estimate as Y = Y X + E (X).

194

CHAPTER 19. MONTE CARLO VALUATION

Why is the new estimate E (Y ) Y = Y X + E (X) better than

the old estimate E (Y ) Y ? It turns out the new estimate often has lower

variance if X and Y are positively correlated.

The
variance
estimate
is: i
of the
hnew

V ar Y = V ar Y X + E (X) = V ar Y X = V ar X +


V ar Y 2Cov X ,Y
The above formula holds because E (X) is a constant.
Now lets go through an example. Theres no formula for the arithmetic
European call option price; theres a formula for the geometric European call
option price. So well use the Monte Carlo simulation to estimate the arithmetic
European call option. Well use the geometric European price as the control
variate (i.e. a dummy variable).
Let AC=arithmetic European call price
price.

AC = AC GC + E (GC)

GC=geometric European call

Well find AC and GC through Monte Carlo simulation. Well calculate


E (GC) using the equation DM 14.18 and DM 14.19 (see Derivatives Markets
Appendix 14.A). Once again, the inputs are:
S0 = 40
K = 40
= 0.3
r = 8%
=0
T = 0.25 (3 months)
The average stock price is the average stock prices at the end of Month 1,
Month2, and Month 3 (so N = 3)
First, well calculate the geometric European call option price. By the way,
the appendix 14.5 is not on the syllabus of the exam MFE. You can ignore my
calculation and just accept that the geometric call option price is 1. 938 5.
Using DM 14.18, we get:

4 0.32 4 7
1
2
=
0.08 + 0 + 0.5 0.32
2
= 0.03 333
2
3
3
3
6
Using DM 14.19, we have:

19.5. EFFICIENT MONTE CARLO VALUATION


=

0.3
3

195

47
= 0.216 025
6

C (S, K, , r, T, ) = Se T N (d1 ) KerT N (d2 )

1
40
1
S
ln
+ r + ( )2 T
+ 0.08 0.03 333 + 0.216 0252 0.25
ln
K
2
40
2

d1 =
=
=
0.216 025 0.25
T
0.162 026

N (d1 ) = NormalDist (0.162 026) = 0.564 357

d2 = d1 T = 0.162 026 0.216 025 0.25 = 0.054 013 5

N (d2 ) = NormalDist (0.054 013 5) = 0.521 538

C = 40e0.033330.25 0.564 357 40e0.080.25 0.521 538 = 1. 938 5

So the geometric call option price is 1. 938 5.

Now we have:

AC = AC GC + 1. 938 5

Next, we perform 30 trials (1 trial =5,000 simulations).

196

CHAPTER 19. MONTE CARLO VALUATION


AC GC
0.0393
0.0408
0.0387
0.0400
0.0388
0.0403
0.0389
0.0404
0.0416
0.0392
0.0391
0.0383
0.0399
0.0392
0.0396
0.0388
0.0375
0.0380
0.0392
0.0409
0.0405
0.0383
0.0376
0.0382
0.0381
0.0396
0.0392
0.0378
0.0395
0.0410
1.1783

(AC GC)2
0.001544
0.001665
0.001498
0.001600
0.001505
0.001624
0.001513
0.001632
0.001731
0.001537
0.001529
0.001467
0.001592
0.001537
0.001568
0.001505
0.001406
0.001444
0.001537
0.001673
0.001640
0.001467
0.001414
0.001459
0.001452
0.001568
0.001537
0.001429
0.001560
0.001681
0.046313

AC
GC AC GC
59.3139 58.1356
1.1783
59.3139
=
= 1. 977 13
30

(AC GC)2
0.046313

i
1
2
3
4
5
6
7
8
9
10
11
12
13
14
15
16
17
18
19
20
21
22
23
24
25
26
27
28
29
30
sum

AC
1.9823
1.9409
1.9467
2.0098
1.9894
1.9515
1.9825
2.0462
2.1270
1.9583
1.9878
1.9664
2.0147
1.9664
1.9915
1.9504
1.9414
1.9804
1.9666
2.0276
1.9940
1.9307
1.9221
1.9710
1.9185
1.9529
1.9791
1.9776
1.9713
1.9689
59.3139

GC
1.9430
1.9001
1.9080
1.9698
1.9506
1.9112
1.9436
2.0058
2.0854
1.9191
1.9487
1.9281
1.9748
1.9272
1.9519
1.9116
1.9039
1.9424
1.9274
1.9867
1.9535
1.8924
1.8845
1.9328
1.8804
1.9133
1.9399
1.9398
1.9318
1.9279
58.1356

So
i
sum

AC

58.1356

GC =
= 1. 937 85
30
Hence the updated estimated price of the geometric European call option is:

AC = AC GC + 1. 938 5 = 1. 977 13 1. 937 85 + 1. 938 5 = 1. 977 78


Alternatively,

19.6. ANTITHETIC VARIATE METHOD

197

1.1783
= 0.03927 67
30
= 0.03927 67 + 1. 938 5 = 1. 977 78

AC GC =

AC

The variance of AC is:


V ar AC = V ar AC GC + 1. 938 5 = V ar AC GC =

1. 145 7 106

30
30 1

In contrast, the estimated variance of the geometric European call price


without usingthe control variate method is, as calculated before,

V ar GC = 0.001618


We can see that V ar AC is much smaller than V ar AC .

Boyle points out that Y = Y X + E (X) doesnt always produce

lower variance than the variance of the original estimate Y . Boyle recommends
the following newestimate:

Y = Y + E (X) X
If we use the textbook
notation, the above new estimate is DM 19.10:
A = A + G G

Since G is constant (here G is equivalent to E (X)), we have DM 19.11:



V ar (A ) = V ar A + G G = V ar A G = V ar A + 2 V ar G

2Cov A, G

Cov A, G
.
The textbook says that V ar (A ) is minimized if we set =
V ar G
How do we find ? Typically, we perform a small number of Monte Carlo
simulations, run regression of DM 19.10, and estimate . Then we apply the
estimated to DM 19.10, run more simulations, and calculate A .
This is all you need to know about the Boyles improved control variate
method.

19.6

Antithetic variate method

Antithetic variate method is simple. Suppose we want to estimate E (X) and


have generated random numbers from a symmetric distribution such as a standard normal random variables. We have n random draws z1 , z2 ,...,zn from the

1
0.046313 0.03927 672
30

198

CHAPTER 19. MONTE CARLO VALUATION

standard normal distributions. Since z1 , z2 ,...,zn are random draws from the
standard normal distribution, z1 , -z2 ,...,zn are also random draws from the
standard normal distribution. Next, we calculate two samples means, X 1 (using
z1 , z2 ,...,zn ) and X 2 (using z1 , -z2 ,...,zn ). Then we can estimated E (X) as
the average of X 1 and X 2 :
X1 + X2
E (X)
2
This method is called the antithetic variate method.
Heres an example. We want to estimate E (ez ) where z is the standard
normal random variable. We have generated the following 10 standard normal
random variables:
i
zi
1 0.0183
2
2.0478
3 0.4849
4 0.6583
5 0.4666
6 0.3757
7
1.1392
8
2.1566
9
0.1096
10 1.5389
This is
i
1
2
3
4
5
6
7
8
9
10
sum

how to estimate E (ez ) using the antithetic variate method:


zi Xi = exp (zi )
zi Yi = exp (zi )
0.0183
0.981866
0.0183
1.018468
2.0478
7.750831 2.0478
0.129018
0.4849
0.615759
0.4849
1.624013
0.6583
0.517731
0.6583
1.931506
0.4666
0.627131
0.4666
1.594563
0.3757
0.686808
0.3757
1.456010
1.1392
3.124268 1.1392
0.320075
2.1566
8.641706 2.1566
0.115718
0.1096
1.115832 0.1096
0.896193
1.5389
0.214617
1.5389
4.659462
24.276548
13.745027

X=

24.276548
= 2. 427 654 8
10

Y =

13.745027
= 1. 374 502 7
10

E (ez )

2. 427 654 8 + 1. 374 502 7


= 1. 901 078 75
2

19.7. STRATIFIED SAMPLING

19.7

199

Stratified sampling

In stratified sampling, we divide the population into several non-overlapping


group. Each group is called a strata. Then we take a sample from each group.
Example. We divide the range [0, 1] into 100 groups: [0, 0.01), [0.01, 0.02),
[0.02, 0.03),...,[0.99, 1). Next, we draw 100 numbers u1 , u2 , ..., u100 from the
uniform distribution [0, 1]. All these 100 numbers are divided by 100. So we
u1 u2 u3
u100
i1
ui
have
,
,
, ...,
.Next, we add
to the i-th number
. Now
100 100 100
100
100
100
we have:
u3
u100
u1 u2
,
+ 0.01,
+ 0.02, ...,
+ 0.99
100 100
100
100
ui
< 0.01.
Since 0 ui < 1, 0
100
u1
So
falls in the group [0, 0.01).
100
u2
u2
+0.01 < 0.02. So
+0.01 falls into group [0.01, 0.02).
Similarly, 0.01
100
100
u3
u3
Similarly, 0.02
+0.02 < 0.03. So
+0.01 falls into group [0.02, 0.03).
100
100
So on and so forth.
The end result is that we take one sample from [0, 0.01), one sample from
[0.01, 0.02), ..., and one sample from [0.99, 1).
This is all you need to know about the stratified sample for the purpose of
passing Exam MFE.

19.7.1

Importance sampling

In the importance sampling, we perform simulations from a conditional distribution, not from the original distribution. For example, we want to estimate
the price of an option that is deep out of the money. If we perform simulations
from the original distribution, then most of the simulated payos will be zero.
This is a waste of our time. To make the simulations more ecient, well draw
random numbers from the conditional distribution where the payo is not zero.
This is all you need to know about the importance sampling.
The textbook also mentioned Latin hypercube sampling and low discrepancy
sequences. Since the textbook merely mentioned these terms without providing
much explanation, I dont think SOA expects you to know much about term.
Skip these terms and move on.

19.8

Sample problems

Problem 1

200

CHAPTER 19. MONTE CARLO VALUATION

You are simulating the standard normal random variable z by taking random
draws from a uniform distribution over (0, 1). Let a represent the simulated
value of z. Calculate P (z a). You are given:
i
1
2
3
4
5
6
7
8
9
10
11
12

ui
0.3763
0.1349
0.414
0.0405
0.5225
0.0423
0.2041
0.9282
0.6792
0.3368
0.1535
0.157

Solution
P
ui = 0.3763+ 0.1349 + 0.414 + 0.0405+ 0.5225 + 0.0423 + 0.2041+ 0.9282 +
0.6792 + 0.3368 + 0.1535 + 0.157 = 3. 989 3
P
The simulated value is a = ui 6 = 3.9893 6 = 2. 010 7
P (z a) = N (2. 010 7) = 1 N (2. 010 7) = 0.022

Problem 2
You are simulating E (ez ) by taking random 10 draws from a uniform distribution over (0, 1). Calculate the simulated value of E (ez ). You are given:
i
1
2
3
4
5
6
7
8
9
10

ui
0.878
0.762
0.069
0.8
0.048
0.22
0.178
0.661
0.191
0.258

Solution

19.8. SAMPLE PROBLEMS

201

zi = N 1 (ui )
1.17
0.71
1.48
0.84
1.66
0.77
0.92
0.42
0.87
0.65

xi = ezi
3.221993
2.033991
0.227638
2.316367
0.190139
0.463013
0.398519
1.521962
0.418952
0.522046
11.314619
11.314619
The estimated value is E (ez ) =
= 1. 131 461 9
10
Sample calculation. u1 = 0.878. Look at the normal table. You see that
roughly P (z < 1.17) = 0.878 so z1 = 1.17.
i
1
2
3
4
5
6
7
8
9
10
sum

ui
0.878
0.762
0.069
0.8
0.048
0.22
0.178
0.661
0.191
0.258

By the way, Im using Excel to find zi = N 1 (ui ) for me so I dont have to


look at the normal table. So if you cant match my z, thats OK. For example,
for u1 = 0.878, you might get z1 = 1.16 or z1 = 1.165. However, your final
estimated E (ez ) should be close to mine.
x1 = e1.17 = 3. 221 993
u3 = 0.069. Since 0.069 < 0.5, you know that z3 = N 1 (u3 ) < 0. You
cant directly look up z3 from the normal table (the normal table lists only the
positive z values). Use the formula N (z3 ) = 1 N (z3 ) = 1 0.069 = 0.931 .
From the normal table you see that P (z < 1.48) = 0.931. So z3 = 1.48 or
z3 = 1.48.
Problem 3
You are simulating the real-world price ST . Inputs are:
Stock prices are lognormally distributed
S0 = 100
= 0.06
r = 0.08
= 0.02
T = 0.5
= 0.3

202

CHAPTER 19. MONTE CARLO VALUATION


i
1
2
3
4
5

ui U (0, 1)
0.6515
0.8839
0.7621
0.3922
0.1748

Calculate the average of the simulated real-world stock prices at T .


Solution
The real world
price at T is:


2
ST = S0 exp 0.5 2 T + T z = 100e(0.060.020.50.3 )0.5+0.3 0.5z =

0.5z

100e0.002 5+0.3
i
1
2
3
4
5
Total

u1 U (0, 1)
0.6515
0.8839
0.7621
0.3922
0.1748

zi = N 1 (u1 )
0.39
1.19
0.71
0.27
0.94

0.002 5+0.3 0.5z


ST = 100e

0.002 5+0.3 0.50.39


100e
= 108. 353 8
128. 394 5
115.9645
94.1976
81.7173
528.6276

The average of the simulated stock prices is:

528.6276
= 105. 73
5

Please note that the risk free rate r is not needed for solving this problem.
Problem 4
You are simulating the price of an arithmetic average stock price European
call option and put option. You are given:
Stock prices are lognormally distributed
S0 = 40
K = 40
= 0.06
r = 0.08
=0
T =1
= 0.3

19.8. SAMPLE PROBLEMS

203

Stock prices at the end of Month 4, 8, and 12 are averaged


The 3 random draws from a uniform distribution (0, 1) are 0.4828, 0.6177,
and 0.9345, which are used to simulate the stock price at the end of Month
4, 8, and 12 respectively.
Calculate the simulated values of this arithmetic Asian call option and put
option.
Solution
The option
is:
price

+ S2/3
+ S1
S1/3
rT
e
max
K, 0
3
stock price at t.
St is the risk-neutral

2
St = Sth exp r 0.5 2 h + hz = Sth e(0.0800.50.3 )h+0.3 hz
u1
0.4828

z1
0.04

S1/3
40.1900

u2
0.6177

z2
0.3

S2/3
42.8303

u3
0.9345

z3
1.51

S1
56.2864

Sample calculation.

= 40e(0.0800.50.3 )1/3+0.3 1/3(0.04) = 40. 1900


S1/3

= 40.1900e(0.0800.50.3 )1/3+0.3 1/3(0.3) = 42. 830 3


S2/3

2
S = 42.8303e(0.0800.50.3 )1/3+0.3 1/3(1.51) = 56. 286 4
1

40. 1900 + 42.8303 + 56.2864


= 46. 435 6
3
The call payo: max (46. 435 6 40, 0) = 6. 435 6
The put payo: max (40 46. 435 6, 0) = 0
Average stock price:

The simulated value of the call price is: e0.081 6. 435 6 = 5. 940 8
The simulated value of the put price is: e0.081 0 = 0
Problem 5 (spring 2007 Exam C #19)
The price of a non dividend-paying stock is to be estimated using simulation.
It is known that:
The price St follows the lognormal distribution
S0 = 50, = 0.15, and = 0.30.

204

CHAPTER 19. MONTE CARLO VALUATION

Using the following uniform (0, 1) random numbers and the inversion method,
three prices for two years from the current date are simulated 0.9830, 0.0384, 0.7794.
Calculate the mean of the three simulated prices.
(A) Less than 75
(B) At least 75, but less than 85
(C) At least 85,
but less than 95
(D) At least 95, but less than 115
(E) At least 115
Solution
ST = S0 exp

2z

50e0.21+0.3

2
0.5 2 T + T z = 50e(0.1500.50.3 )2+0.3 2z =
zi = N 1 (u1 )
2. 12
1. 77
0.77

ST = 50e0.21+0.3 2z
22.12
50e0.21+0.3
= 151. 63

0.21+0.3 2(1. 77)


50e
= 29. 11

0.21+0.3 2(0. 77)


50e
= 85. 52
151. 63 + 29. 11 + 85. 52 = 266. 26
266. 26
The average of the simulated prices is
= 88. 75. The answer is C.
3
i
1
2
3
Total

u1 U (0, 1)
0.9830
0.0384
0.7794

Chapter 20

Brownian motion and Itos


Lemma
20.1

Introduction

According to Wikipedia, Brownian motion (named in honor of the botanist


Robert Brown) is either the random movement of particles suspended in a fluid
or the mathematical model used to describe such random movements, often
called a Wiener process.
In 1827, while examining pollen grains suspended in water under a microscope, Brown observed minute particles in the pollen grains executing a continuous jittery motion. He observed the same motion in particles of dust, enabling
him to rule out the hypothesis that the motion was due to pollen being alive.
Although he did not provide a theory to explain the motion, the phenomenon
is now known as Brownian motion in his honor.
Brownian motion is a useful tool for modeling the stock price. The price
of a stock is constantly hit by random events, just as a particle in the water is
constantly hit by water molecules. In fact, the Brownian motion is to stochastic
processes as the standard normal distribution to random variables.
Brownian motion is an abstract concept. The first step toward learning the
Brownian motion is to see it and experiment it. You can easily find Brownian
motion simulations in the internet. Here are some simulations in the internet:
http://www.phy.ntnu.edu.tw/java/gas2d/gas2d.html
http://www.aip.org/history/einstein/brownian.htm
http://www.stat.umn.edu/~charlie/Stoch/brown.html
http://www.matter.org.uk/Schools/Content/BrownianMotion
Theres a large body of knowledge on the internet about the Brownian motion. For example, you can check out Wikipedias explanation at:
205

206

CHAPTER 20. BROWNIAN MOTION AND ITOS LEMMA

http://en.wikipedia.org/wiki/Brownian_motion
To help you experiment with the Brownian motion, I designed a spreadsheet
titled "Simulate Brownian Motion." Download this spreadsheet.
My spreadsheet uses the following Excel functions:
Rand() returns a random number equal to or greater than 0 but less than 1.
In other words, Rand() simulates a random variable uniformly distributed
over [0, 1).
NORMINV(probability,mean,standard_dev) returns the inverse of the normal cumulative distribution for the specified mean and standard deviation.

20.1.1

Big picture

When studying the Brownian motion and Itos lemma, remember the following
big picture. The Black-Scholes option pricing formula Equation 12.1 relies on
the Black-Scholes PDE (Equation 12.24). Equation 12.24 assumes the following
price model for a risk-free asset and a risky asset (i.e. stock):
The price of a risk free asset (i.e. the savings account or a bond) is
B (t) = ert

(20.1)

The price of a risky asset (i.e. stock) at time t is:


#

S (t) = S (0) e

1 2

t+ tY (t)

(20.2)

In the above equation:


S (0) is the stock price at time zero
S (t) is the stock price at time t
is the expected (annualized continuously compounded)
return
of the
1
S
(t)
stock. In other words, E [S (t)] = S (0) et or = ln E
t
S (0)
r is the (annualized continuously compounded) risk-free interest rate.
is the stocks volatility.
Y (t) is a random draw of a standard normal random variable. Y (t1 ) and
Y (t2 ) are independent for t1 6= t2
Equation 20.2 is another form of Equation 20.1 in the textbook:
dS (t)
= dt + dZ (t)
S (t)

(Textbook 20.1)

20.2. BROWNIAN MOTION

207

20.2

Brownian motion

20.2.1

Stochastic process

A stochastic process is a family of random variables indexed by time. For


example, the temperature out side your house X (t) is a stochastic process. Let
t = 0 represent now and t = 1 represent the next time (such as next hour, next
day, next week). Then for a series of time points t = 1, 2, 3, ... there is a family
of random temperatures X (0), X (1), X (2), X (3) ....
This is the major dierence between a stochastic process and a deterministic
process. In a stochastic process youll see a series of random variables; for
each time t theres a corresponding random variable X (t). In contrast, in a
deterministic process, youll see only one random variable.

20.2.2

Definition of Brownian motion

Consider a particle that jumps, at discrete times, up or down along the vertical
line. At t = 0 the particle is at position zero. After each h-long time period,
the particle jumps up or down by a constant distance of k and with equal
probability of 0.5. That is, at t = h, 2h, 3h, ..., nh, the particle either moves
up by k or moves down by k, with up and down movements having an equal
probability of 0.5. Let Z (t) represent the height of the article from the position
zero at time t. Clearly Z (0) = 0. We like to find Z (T ), the height of the article
at time T = nh.
The particles height
4k
3k
2k
k
0

0
k

time 0

2k
k

2k
2h

0
k
3k
3h

k
4k
4h

Lets walk through the above table. At t = 0 the particle is at the position
zero. At t = h, the particles height is either k or k. At t = 2h, the k node
either goes up to 2k or goes down to 0. Similarly, the k node either goes up
to 0 or goes down to 2k. So on and so forth.
The jump at t = h is: Z (h) Z (0) = Y (h) k
Here Y (h) is a direction indicator. If Y (h) = 1, then the particle moves up
by k; if Y (h)
= 1, then the particle moves down by k:
1 Probability 0.5
Y (h) =
1 Probability 0.5

208

CHAPTER 20. BROWNIAN MOTION AND ITOS LEMMA

The jumpat t = 2h is: Z (2h) Z (h) = Y (2h) k


1 Probability 0.5
Y (2h) =
1 Probability 0.5
The jump at t = nh = T is: Z (nh) Z [(n 1) h] = Y (nh) k
Here Y (nh)
is a direction indicator:
1 Probability 0.5
Y (nh) =
1 Probability 0.5
The direction indicators Y (h), Y (2h), Y (3h), ..., Y (nh) are independent
identically distributed binomial random variables. For i = 1 to n,
E h(ih) =i(1) 0.5 + (1) 0.5 = 0

E (ih)2 = (1)2 0.5 + (1)2 0.5 = 1


i
h
2
V ar (ih) = E (ih) E 2 (ih) = 1
The particles height at time T is:
Z (T ) = [ Z (h) Z (0)]+[ Z (2h) Z (h)]+[ Z (3h) Z (2h)]+...+[ Z (nh = T ) Z [(n 1) h]]
= Y (h) k + Y (2h) k + Y (3h) k + ... + Y (nh) k
= [Y (h) + Y (2h) + Y (3h) + ... + Y (nh)] k

According to the central limit theorem, Z (T ) is approximately normal with


mean and variance as:
E [Z (T )] = kE [Y (h) + Y (2h) + Y (3h) + ... + Y (nh)] = 0
V ar [Z (T )] = k 2 V ar [Y (h) + Y (2h) + Y (3h) + ... + Y (nh)]
k2
T
= k2 n = k2 =
T
h
h
2
k
We want V ar [Z (T )] =
T to exist (i.e. not to become infinite) as n .
h
2
k
k2
To achieve this, we set
to a positive constant:
= c. To make our
h
h
model simple, we set c = 1. Hence

k2
=1
k= h
h
Now we have:

Z (ih) Z [(i 1) h] = Y (ih) h


Now Z (T ) is approximately normal with mean zero and variance
Z (T ) N (0, T )

(20.3)
k2
T = T:
h

Please note that another way to specify the model is treat Y (ih) as a random
draw of a standard normal random variable (instead of a binomial random
variable):

20.2. BROWNIAN MOTION

Z (ih) Z [(i 1) h] = Y (ih) h and Y (ih) N (0, 1)

209

(20.4)

My spreadsheet for simulating the Brownian motion uses both Equation 20.3
and Equation 20.4.
The random process Z (t) as n is called the Brownian motion or the
Wiener process.
Next, lets formally define the Brownian motion.
Definition 20.2.1.
A stochastic process Z (t) is a Brownian motion or a Wiener process if
1. Z (0) = 0 . Brownian motion starts at zero (this is merely for our convenience).
2. Z (t + h) Z (t) is normally distributed with mean 0 and variance h. This
means that the increments over a time interval h is normally distributed
with mean 0 and variance h. This stands true no matter how small or big
h is.
3. Z (t + s1 ) Z (t) is independent of Z (t) Z (t s2 ) where s1 , s2 > 0.
4. Z (t) is continuous.

20.2.3

Martingale

The following part is based on Wikipedia.


A martingale is a stochastic process (i.e., a sequence of random variables)
such that the conditional expected value of an observation at some time t, given
all the observations up to some earlier time s, is equal to the observation at that
earlier time s.
Originally, martingale referred to a betting strategy popular in 18th century
France. The rule of the game is that the gambler wins his stake if a coin comes
up heads and loses it if the coin comes up tails. The martingale strategy had
the gambler double his bet after every loss, so that the first win would recover
all previous losses plus win a profit equal to the original stake. Since eventually
a gambler will win at least once, the martingale betting strategy was thought
to be sure way of winning. In reality, however, the exponential growth of the
bets would eventually bankrupt the gambler.
A stochastic process (i.e., a sequence of random variables) X (t) is a martingale if the following holds:
E [X (t) |X (s)] = X (s) for t > s.
It can be shown that the Brownian motion Z (t) is martingale.

210

CHAPTER 20. BROWNIAN MOTION AND ITOS LEMMA

For t > s, we have:


E [Z (t) |Z (s)]
= E [Z (t) Z (s) + Z (s) |Z (s)]
= E [Z (t) Z (s) |Z (s)] + E [Z (s) |Z (s)]
E [Z (s) |Z (s)] = Z (s)
Z (t) Z (s) is independent of Z (s).
E [Z (t) Z (s) |Z (s)] = E [Z (t) Z (s)] = 0
E [Z (t) |Z (s)] = E [Z (t) Z (s) |Z (s)] + E [Z (s) |Z (s)] = Z (s)
Hence Z (t) is martingale.
E [Z (t) |Z (s)] = Z (s) means that the best estimate of the future value of a
Brownian motion is its current value.

20.2.4

Properties of Brownian motion

1. The Brownian motion is continuous everywhere yet dierentiable nowhere.


2. The first-order variation is infinite: lim |Z (h)Z (0) |+|Z (2h)Z (h) |+
n

... + |Z (nh) Z [(n 1) h] |

3. The second-order variation (called quadratic variation) is equal to the


2
2
length of the time interval: lim {[Z (h) Z (0)] + [Z (2h) Z (h)] +
n

... + (Z (nh) Z [(n 1) h])2 } = T

4. Cov [Z (s) , Z (t)] = min (s, t). The covariance of two Brownian motions is
the shorter time interval.

0
if n is odd
5. The higher moments of Z (t) is: E [Z n (t)] =
tn/2 (n 1) (n 3) ...1 if n is even
Lets look at the first property.
If you look at the simulation of Brownian motion over the internet, youll
find that the Brownian motion is always continuous yet its not dierentiable
anywhere. Can you imagine that a function is continuously anywhere yet dierentiable nowhere? If I hadnt studied the Brownian motion, I would have never
thought that such a function exists.
We can explain the non-dierentiality using the following equation (its textbook Equation 20.4):

dZ (t) = Y (t) dt

(20.5)

The above equation is another form of


Equation 20.3 or 20.4:
dZ (t) = Z (t + dt) Z (t) = Y (t) dt
The textbook explains in the footnote that you can treat Y (t) as a binomial

20.2. BROWNIAN MOTION

211

random variable or a standard normal random variable. Either way, dZ (t) is a


normal random variable.
From Equation 20.5, we have:
dZ (t)
Y (t)
= as dt 0
dt
dt
Lets look at the 2nd property. The following is an intuitive but not rigorous
proof.
According to the definition of the Brownian motion, Z (h) Z (0), Z (2h)
Z (h), ..., Z (nh) Z [(n 1) h] are independent identically distributed normal
random variable with mean 0 and variance h. On average, |Z [(i + 1) h]Z (ih) |
approaches E|X| where X N (0, h). Let f (x) represent the probability density functionR of X, then

E|X| = |x|f (x) dx > 0 (since |x| 0, its mean must be positive)
Then lim (|Z (h) Z (0) | + |Z (2h) Z (h) | + ... + |Z (nh) Z [(n 1) h] |)
n

approaches nE|X|, which approaches (since n , a positive constant E|X|


times n must also approach infinity.)

Property 2 should be easy to understand. Since Z (t)= Y (t) t, for a

0.000001
tiny interval t, t is much large than t. For example,
= 1, 000.
0.000001
Hence during any short interval, the Brownian motion can move up or down by
an infinitely large amount.
In contrast, for a continuously dierentiable function y = f (t), we have:
y = f 0 (t) t 0 as t 0
Lets get an intuitive
feel
of the 3rd property. On average, {Z [(i + 1) h]
2
2
approaches
E
X
.
Z (ih)}

E X 2 = E 2 (X) + V ar (X) = 02 + h = h
2
2
2
lim [Z (h) Z (0)] + [Z (2h) Z (h)] + ... + (Z (nh) Z [(n 1) h]) apn
proaches
h + h + ... + h = nh = T
In contrast, the 2nd order variation of a dierentiable function is zero. For
example, we can find the 2nd order variation of the function y = x is zero.
Divide the interval [0, T ] into [0, h],[h, 2h],...,[(n 1) h, nh = T ]
2
T
2
2
2
2
lim {(h 0) + (2h h) +... +[nh (n 1) h]} = nh = lim n
=
n
n
n
0
It can be proven that the quadratic variation of any continuously dierentiable function is zero. The quadratic variation of any continuously dierentiable
function is zero because such a function is roughly linear at any point. For a
continuously dierentiable function y = f (t), we have:
2
y = f 0 (t) t
(y)2 = [f 0 (t)] (t)2
2
2
For a tiny interval t, (t) 0 must faster than t 0. Hence (y) 0

212

CHAPTER 20. BROWNIAN MOTION AND ITOS LEMMA

2
In contrast, for a Brownian motion we have Z (t) = Y (t) t and [Z (t)] =
P
P
2
2
[Y (t)] t t. Hence
[Z (t)] t = T
Brownian motion Property 3 reconfirms the idea that Brownian motion is
not dierentiable anywhere. If its dierentiable, then its second order variation
would be zero.

Tip 20.2.1. Just memorize Equation 20.5 dZ (t) = Y (t) dt. This equation
tells you that the Brownian motion Z (t) is not dierentiable anywhere, its second order variation is t, and its first order variation is infinite.

Tip 20.2.2. To get an intuitive feel of the equation dZ (t) = Y (t) dt, imagine
you are looking at the Brownian motion under a magnifying class. If you zoom
in on the Brownian motion by shrinking the time interval dt, no matter how
much you reduce dt, youll see a jigsaw. In comparison, if you zoom in on a
continuously dierentiable function such as y = t2 , youll see a straight line.
Please note that Derivatives Markets explains Property 2 and 3 using the
following formula:

Z [(i + 1) h] Z (ih) = Y [(i + 1) h] h


Since Y [(i + 1) h] is abinomial random variable having a value of 1, then
|Z [(i + 1) h] Z (ih) | = h and {Z [(i + 1) h] Z (ih)} = h. Hence

lim (|Z (h) Z (0) | + |Z (2h) Z (h) | + ... + |Z (nh) Z [(n 1) h] |) = n h


n

2
2
2
lim {[Z (h) Z (0)] + [Z (2h) Z (h)] + ... + (Z (nh) Z [(n 1) h]) } =
n
nh = T
The problem with this explanation is that it works if we treat Y [(i + 1) h]
as binomial random variable whose value is 1. Such explanation wont work if
we treat Y [(i + 1) h] as a random draw of a standard normal random variable.
The explanation I provided here works no matter if you treat Y [(i + 1) h] as a
binomial random variable or a standard normal random variable.
Lets look at Property 4. Suppose s t
Cov [Z (s) , Z (t)] = Cov{Z (s) , Z (s) + [Z (t) Z (s)]}
Using the formula Cov (a, b + c) = Cov (a, b) + Cov (a, c), we get:
Cov{Z (s) , Z (s)+[Z (t) Z (s)]} = Cov{Z (s) , Z (s)}+Cov{Z (s) , [Z (t) Z (s)]}
Cov{Z (s) , Z (s)} = V ar [Z (s)] = s
Since Z (s) and [Z (t) Z (s)] are independent (Brownian motion definition
Point #3), Cov{Z (s) , [Z (t) Z (s)]} = 0
Cov [Z (s) , Z (t)] = s = min (s, t)
Property 5 is based on the moment formula for a standard normal random
variable

0
if n is odd
E (n ) =
(20.6)
(n 1) (n 3) ...1 if n is even
We can find the n-th moment of a random variable X using the moment
generating function (MGF):

20.2. BROWNIAN MOTION

213

dn
E (X ) =
MX (t)
dtn
n

(20.7)

t=0

The MGF of a normal random variable X with mean and the standard
deviation is:
1
tX
t+ 2 t2
2
MX (t) = E e
=e

(20.8)

1 2
t
M (t) = e 2
Using Equation 20.7, you can verify that Equation 20.6 holds.
Since Z (t) is a normal random variable with mean 0 and variance t, then
Z (t)
is a standard normal random variable. Hence
t

0
if n is odd
(20.9)
E [Z n (t)] =
n/2
(n 1) (n 3) ...1 if n is even
t
Example 20.2.1. Calculate P [Z (3) > 1]
Z (3) is a normal random variable with mean
0 andvariance 3.
10
P [Z (3) > 1] = 1 P [Z (3) 1] = 1
= 1 (0.577 35) =
3
0.281 9
Example 20.2.2. Calculate P [Z (1) 0 Z (2) 0]
Z (1) is a normal random variable with mean 0 and variance 1. Let X = Z (1)
Z (2) = Z (1) + [Z (2) Z (1)]
Z (2) Z (1) is a normal random variable with mean 0 and variance 1.
Z (2) = Z (1) + [Z (2) Z (1)] 0
[Z (2) Z (1)] Z (1)
Let Y = Z (2) Z (1).
X and Y are independent.
P [Z (1) 0 Z (2) 0] = P (X 0 Y X)
To have X 0 Y X, we first fix X at a tiny interval (x, x + dx)
where < x < 0. Next, we set Y < x. Then we are guaranteed to have
X 0Y X. Let f (x) and (x) represent the probability density function
(pdf) and the cumulative density function (cdf) of a standard normal random
variable.
R0
R0
P (X 0 Y X) = P (x < X < x + dx) P (Y < x) = [f (x) dx] P (Y < x)
However, f (x) dx = d (x) and P (Y < x) = (x) = 1 (x)
R0
R0
R0
[f (x) dx] P (x) = [d (x)] [1 (x)] = [1 (x)] d (x) =

R0
[1 (x)] d (x)

0
1 2
= (x) (x)
2

214

CHAPTER 20. BROWNIAN MOTION AND ITOS LEMMA

1 2
= [ (0) ()]
(0) 2 ()
2
#

" 2
1
1 1
2
=
0
0
2
2 2
2
1 1 1
3
=
=
2 2 2
8
Example 20.2.3. Calculate Cov [Z (5) , Z (2)]
Cov [Z (5) , Z (2)] = min (5, 2) = 2

Example 20.2.4. Calculate E Z 4 (t) .

E Z 4 (t) = t4/2 (4 1) = 3t2

20.2.5

Arithmetic Brownian motion and Geometric Brownian motion

A standard Brownian motion is normally distributed with mean 0 and variance


1. Now we want to extend the Brownian motion to allow for non-zero mean and
an arbitrary variance. Define a stochastic
process:
X (t + h) X (t) = h + Y (t + h) h
Breaking down [0, T ] into small intervals [0, h],[h, 2h],...,[(n 1) h, nh = T ],
we have:
i
i
Pn h
Pn h
X (T ) X (0) = i=1 h + Y (ih) h = T + i=1 Y (ih) h
i
P h
As n , ni=1 Y (ih) h Z (T )
X (T ) X (0) = T + Z (T )

(20.10)

dX (t) = dt + dZ (t)

(20.11)

Equation 20.10 and 20.11 are called arithmetic Brownian motion. is the
instantaneous mean per unit of time; is the instantaneous standard deviation
per unit of time.
Equation 20.10 and 20.11 indicate that X (T )X (0) is normally distributed.
Its mean and variance are:
E [X (T ) X (0)] = E [T + Z (T )] = T + E [Z (T )] = T + 0 = T
V ar [X (T ) X (0)] = V ar [T + Z (T )] = V ar [Z (T )] = 2 V ar [Z (T )] =
2T
The textbook lists the major properties and weaknesses of Equation 20.11.
Major properties:
1. X (t) is normally distributed.

20.2. BROWNIAN MOTION

215

2. We can change the variance by changing the parameter .


3. We can change the mean by changing the parameter . Now the mean is
no longer zero if 6= 0. And we have E [X (T )] E [X (0)] = T . This
means that after time T , the stock price drifts away from the price at time
zero.
Major weaknesses:
1. The stock price X (t) can be negative. Since X (t) is normally distributed, <
X (t) < . Equation 20.11 allows a negative stock price. Of course, the
stock price cant become negative.
2. The expected change of the stock price does not depend on the stock price.
In reality, the expected change of the stock price should be proportional
to the stock price. The higher the stock price, the higher the expected
change. So we like to have E [dX (t)] = X (t). We need to modify
Equation 20.11 to allow E [dX (t)] = X (t).
3. The variance of the stock price does not depend on the stock price. In
reality, the variance should be proportional to the stock price. So we need
to modify Equation 20.11 to allow [X (t) , t] = X (t).
Major Weakness #2 and #3 can also be stated this way. Equation 20.11 can
dX (t)

dX (t)
be rewritten as
=
dt +
dZ (t). This indicates that
,
X (t)
X (t)
X (t)
X (t)
the percentage return on the stock depends on the stock price X (t). However,
in reality, we think that the stock return on average shouldnt depend on the
stock price. In other words, instead of Equation 20.11, we like to see
dX (t)
= dt + dZ (t)
X (t)

(20.12)

dX (t) = X (t) dt + X (t) dZ (t)

(20.13)

or

Equation 20.12 and 20.13 are called the geometric Brownian motion.

20.2.6

Ornstein-Uhlenbeck process

We can modify Equation 20.11 to allow for mean reversion. Its reasonable for
us to assume that the stock price or the interest rate will revert to the mean.
For example, if the stock price is too high, then it might go down; if the stock
price is too low, it might go up. We modify the drift term in Equation 20.11:
dX (t) = [ X (t)] dt + dZ (T )

(20.14)

216

CHAPTER 20. BROWNIAN MOTION AND ITOS LEMMA

If = 0, 20.14 is called the Ornstein-Uhlenbeck process:


dX (t) = X (t) dt + dZ (T )

(20.15)

The Ornstein-Uhlenbeck process is the most widely used mean reverting


stochastic process in financial modeling.

20.3

Definition of the stochastic calculus

We can rewrite Equation 20.12 as:


RT
RT
RT
dX (t) = 0 X (t) dt + 0 X (t) dZ (t)
0
RT
RT
RT
RT
or X (T )X (0) = 0 X (t) dt+ 0 X (t) dZt = 0 X (t) dt+ 0 X (t) dZt
RT
But whats the meaning of 0 X (t) dZ (t)? Or generally, whats the meaning
Rb
of a g (t) dZ (t)?
To answer this question, lets
R 1take a step back and find out the meaning of
a simple deterministic calculus 0 x2 dx.
R1 2
x dx is the area of the function x2 bounded by x = 0 and x = 1. To find
0
this area, we divide the interval [0, 1] into n intervals [0, h], [h, 2h], [2h, 3h], ...,
[(n 1) h, nh = 1]. Then we approximate the area with the sum of n rectangles.
The area of function x2 over the interval [(i 1) h, ih] is roughly the area of the
rectangular with height [(i 1) h]2 and width ih (i 1) h = h.
R ih

x2 dx [(i 1) h]2 h = (i 1)2 h3


Pn R ih
Pn
2 3
2
2
= lim
i=1 (i1)h x dx = lim
i=1 (i 1) h = lim {0 h +

(i1)h
R1 2
x dx
0
2
2

h h + (2h) h + ... + [(n 1) h] h}


02 h +hh2 h + (2h)2 h + ... + [(n 1)i h]2 h
= h3 02 + 12 + 22 + ... + (n 1)2
Using the famous formula:
n (n + 1) (2n + 1)
12 + 22 + 32 + ... + n2 =
h
i6
(n 1) (n) (2n 1)
2
3
2
2
2
h 0 + 1 + 2 + ... + (n 1) = h3
6
1
Since h = , we have:
n
R1

1 (n 1) (n) (2n 1)
x2 dx = lim 3
n
n

6
1
1
1
1
= lim
1
(1) 2
=
n 6
n
n
6
R ih
There are other ways to approximate (i1)h x2 dx. For example, the area of
function x2 over the interval [(i 1) h, ih] is roughly the area of the rectangular
with height (ih)2 and width ih (i 1) h = h.
R ih
2
x2 dx (ih) h = i2 h3
(i1)h
0

20.3. DEFINITION OF THE STOCHASTIC CALCULUS


R1
0

x2 dx = lim

R ih
i=1 (i1)h

Pn

x2 dx = lim

Pn

i=1

217

n (n + 1) (2n + 1) 3
h
n
6

i2 h3 = lim

1
n (n + 1) (2n + 1) 1
= lim
=
n
6
n3 R 6
ih
The 3rd way to approximate (i1)h x2 dx is to take the average height of
the rectangular. So the area of function x2 over the interval [(i 1) h, ih] is
2
2
(i 1) h2 + (ih)
and width
roughly the area of the rectangular with height
2
ih (i 1) h = h.
2
2
2
(i 1) h2 + (ih)
(i 1) + i2 2
=
h
2
2 #
"
2
R ih
(i 1) + i2 2
2
x
dx

h h
(i1)h
2
"
#
2
2
R1 2
Pn R ih
P
(i

1)
+
i
n
2
x dx = lim
lim
h2 h
i=1 (i1)h x dx = n
i=1
0
n
2
i
Pn h
Pn 2 2
1
1
2 2
h
(i

1)
h + lim
i h h
=
lim
i=1
n
2
2 n i=1


1 1
1 1
1
=
+
=
2 6
2 6
6
It seems natural that we extend this logic of deterministic integration to a
define a stochastic integration.
Suppose we partition [a, b] into a = t0 < t1 < t2 < ... < tn = b. We can
make the partition intervals [t0, t1 ], [t1, t2 ], ..., [tn1, tn ] have the same length
ba
tk+1 tk =
. We can also have the partition intervals [t0, t1 ], [t1, t2 ], ...,
n
[tn1, tn ] have dierent lengths.
Rb
Pn1
g (t) dZ (t) = lim
k=0 g (tk ) [Z (tk+1 ) Z (tk )]
a
n

Definition 20.3.1.
Suppose g (t) is a simple process, meaning that g (t) is piecewise-constant

Rb
but may have jumps at a = t0 < t1 < t2 < ... < tn = b. If a E g 2 (t) dt < ,
Rb
then the stochastic integral a g (t) dZ (t) is defined as
Z

g (t) dZ (t) =

n1
X
k=0

g (tk ) [Z (tk+1 ) Z (tk )]

(20.16)

Rb
In the above definition, a E g 2 (t) dt < is the sucient condition for
Pn1
k=0 g (tk ) [Z (tk+1 ) Z (tk )] to exist.
Example 20.3.1. Calculate
Solution.

RT
0

dZ (t)

218

CHAPTER 20. BROWNIAN MOTION AND ITOS LEMMA

Here g (t) = 1.
RT
R T 2
E 1 dt = T < . So 0 dZ (t) exists.
0
RT
Pn1
dZ (t) = k=0 [Z (tk+1 ) Z (tk )] = Z (T )
0

Example 20.3.2.

1 if 0 t 1
2 if 1 < t 2
X (t) =

3 if 2 < t 3
R3
Calculate 0 X (t) dZ (t)

Solution.

Dividend [0, 3] into (0, 1), (1, 2), and (2, 3). Then X (t) is constant during
the Rinterval and jumps at t = 1 and t = 2. Hence
3
X (t) dZ (t)
0
= X (0) [Z (1) Z (0)] + X (1) [Z (2) Z (1)] + X (2) [Z (3) Z (2)]
= 1 [Z (1) Z (0)] + 2 [Z (2) Z (1)] + 3 [Z (3) Z (2)]
= 1Z (1) + 2 [Z (2) Z (1)] + 3 [Z (3) Z (2)]
= 3Z (3) Z (2) Z (1)
If g (tk ) is not a simple process, then we define the stochastic integral as
follows.
If E lim (X a)2 = 0, we say that X approach a in mean square.
n
Explain why the sample mean approaches the population mean in mean
square.
Suppose we take n random samples X1 , X2 ,...,Xn from the population X.
1 Pn
Xk . It
Let represent the population mean. Then the sample mean is
n k=1
can be shown that
2

1 Pn
=0
E lim
k=1 Xk
n n
Tosee why, notice

Pn
1 Pn
1
1
E
Xk = E ( k=1 Xk ) = n =
n k=1
n
n
2
1 Pn
Xk
E
n k=1

P
1 Pn
1
= V ar
= 2 V ar ( nk=1 Xk )
k=1 Xk
n
n
V ar (X)
1
= 2 nV ar (X) =
n
n

2
1 Pn
E lim
X

=0
k
k=1
n n
So the sample mean approaches the population mean in mean square.

20.3. DEFINITION OF THE STOCHASTIC CALCULUS

219

Definition 20.3.2.
Let a = t0 < t1 < t2 < ... < tn = b represent a partition of [a, b]. Define
P
random variable In = n1
k=0 g (tk ) [Z (tk+1 ) Z (tk )], where g (tk ) is a simple
or a complex process and g (tk ) and Z (tk+1 ) Z (tk ) are independent. If
Rb 2
E g (t) dt < and the mean squared dierence between In and U is zero
a
as n :
2

E lim (In U ) = 0
n

(20.17)

Then we say

Rb
a

g (t) dZ (t) = U

In converges to

Rb
a

g (t) dZ (t) in mean square

The above definition holds whether g (t) is a simple or complex process.


Please note that in the term g (tk ) [Z (tk+1 ) Z (tk )], g is evaluated at the
left of the interval [tk , tk+1 ]:
Rb
Pn1
g (t) dZ (t) = lim
k=0 g (tk ) [Z (tk+1 ) Z (tk )]
a
n
Rb
Pn1
g (t) dZ (t) 6= lim
k=0 g (tk+1 ) [Z (tk+1 ) Z (tk )]
a
n
Rb
Pn1 g (tk ) + g (tk+1 )
g (t) dZ (t) 6= lim
[Z (tk+1 ) Z (tk )]
k=0
a
n
2
R ih
This is dierent from the deterministic calculus (i1)h x2 dx, which can be
approximated using 3 heights:
The left height [(i 1) h]2
The right height (ih)2
The average height

(i 1)2 h2 + (ih)2
2

In addition, we require that g (tk ) and Z (tk+1 ) Z (tk ) are independent. In


other words, we require that g (tk ) not depend on the future Brownian increment Z (tk+1 ) Z (tk ). The requirements that we evaluate g at the left of the
interval [tk , tk+1 ] and that g (tk ) and Z (tk+1 ) Z (tk ) are independent agree
with our intuition. We calculating g (tk ) [Z (tk+1 ) Z (tk )], g (tk ) is based on
the information available to us during [0, tk ] and is independent of the future
Brownian increment Z (tk+1 ) Z (tk ).
RT
2
Example 20.3.3. Calculate 0 [dZ (t)] .

220

CHAPTER 20. BROWNIAN MOTION AND ITOS LEMMA

RT

RT
2
2
[dZ (t)] = 0 g (t) [dZ (t)] where g (t) = 1
Partition [0, T ] into [0, h], [h, 2h],...,[(n 1) h, nh = T ].
P
Pn
2
2
Let In = n1
k=0 {Z [(k + 1) h] [Z (kh)]} =
k=1 {Z (kh) [Z (k 1) h]}
Z (kh) [Z (k 1) h] is a normal random variable with mean 0 and variance
h. Using Equation 20.9, we have
2
E{Z (kh)
Pn [Z (k 1) h]} = h
In = k=1 {Z (kh) [Z (k 1) h]}2
RT
2
Since E (In ) = nh = T , we guess that 0 [dZ (t)] = T
The dicult part is to verify that the mean square error is zero as n :
hP
i2
2
lim E ( nk=1 {Z (kh) [Z (k 1) h]}) T = 0
0

Let k = Z (kh) [Z (k 1) h]
Pn
2
( k=1 {Z (kh) [Z (k 1) h]}) = 21 + 22 + ... + 2n
hP
i2
2
n
( k=1 {Z (kh) [Z (k 1) h]}) T

2
2

= 21 + 22 + ... + 2n T = 21 + 22 + ... + 2n +T 2 2T 21 + 22 + ... + 2n

2
2
1 + 22 + ... + 2n = 41 + 42 + ...4n + 221 22 + 221 23 + ...

E 21 + 22 + ... + 2n = E 41 + 42 + ...4n +2E 21 22 + 21 23 + ...


k isnormal
with mean 0 and variance h

E 4k = 3h2

E 41 + 42 + ...4n = 3nh2
i and j where i 6= j are two independent normal random variables (Point
3 of the Brownian

motion
definition)

E 2i 2j = E 2i E 2j = h h = h2
1
There are n (n 1) pairs of i and j where i 6= j
2

1
2E 21 22 + 21 23 + ... = 2 n (n 1) h2 = n (n 1) h2
2

2
E 21 + 22 + ... + 2n = 3nh2 + n (n 1) h2

E 2T 21 + 22 + ... + 2n = 2T E 21 + 22 + ... + 2n = 2T (nh) = 2T 2


hP
i2
2
n
E ( k=1 {Z (kh) [Z (k 1) h]}) T

= 3nh2 + n (n 1) h2 + T 2 2T = [3n + n (n 1)] h2 T 2


However, h =

T
n

2
T
3n + n (n 1) 2
[3n + n (n 1)] h T = [3n + n (n 1)]
T 2 =
T
n
n2
2

T2

As n ,

3n + n (n 1)
1
n2

3n + n (n 1) 2
T T2 0
n2

20.3. DEFINITION OF THE STOCHASTIC CALCULUS

221

hP
i2
2
lim E ( nk=1 {Z (kh) [Z (k 1) h]}) T = 0
n
RT
Hence 0 [dZ (t)]2 = T .
RT
Example 20.3.4. Calculate 0 Z (t) dZ (t)

Partition [0, T ] into [0, h], [h, 2h],...,[(n 1) h, nh = T ].


Pn1
Pn
Let In = k=0 Z (kh) {Z [(k + 1) h]Z (kh)} = k=1 Z [(k 1) h] {Z (kh)
Z [(k 1) h]}

2
(a + b) a2 + b2
Use the formula: ab =
2
Let a = Z [(k 1) h]
b = Z (kh) Z [(k 1) h]
a
P+n b = Z (kh)
k=1 Z [(k 1) h] {Z (kh) Z [(k 1) h]}
2
2
P
[Z (kh)] Z [(k 1) h] {Z (kh) Z [(k 1) h]}2
= nk=1
2
1 Pn
1 Pn
2
2
{[Z
(kh)]

Z
[(k

1)
h]
}
{Z (kh) Z [(k 1) h]}2
=
k=1
2
2 k=1
1
1 Pn
2
= [Z (nh)]
{Z (kh) Z [(k 1) h]}2
2
2 k=1
1
1 Pn
2
= [Z (T )]
{Z (kh) Z [(k 1) h]}2
2
2 k=1
In =

1
1 Pn
2
{Z (kh) Z [(k 1) h]}2
[Z (T )]
2
2 k=1

Pn
lim
lim In
k=1 Z [(k 1) h] {Z (kh) Z [(k 1) h]} = n
P
1
1
2
n
= [Z (T )] lim
{Z (kh) Z [(k 1) h]}2
2
2 n k=1
Pn
From the previous example, we know that k=1 {Z (kh)Z [(k 1) h]}2 apRT
RT
proaches 0 [dZ (t)]2 = T in the mean square. So we guess that 0 Z (t) dZ (t) =
1
1
[Z (T )]2 T
2
2

2
1
1
2
Next, we need to prove that E lim In
=0
[Z (T )] T
n
2
2

1
1
1
1 Pn
2
In
{Z (kh) Z [(k 1) h]}2
[Z (T )] T = T
2
2
2
2 k=1
In E (In )

1
1 Pn
1
1
2
2
= [Z (T )]2
{Z
(kh)

Z
[(k

1)
h]}

[Z
(T
)]
T
2
2 k=1
2
2

Pn
1
2
T k=1 {Z (kh) Z [(k 1) h]}
=
2
From the previous example, we found that

2
Pn
lim E T k=1 {Z (kh) Z [(k 1) h]}2 = 0
n

2
1
1
E lim In
=0
[Z (T )]2 T
n
2
2
n

222

CHAPTER 20. BROWNIAN MOTION AND ITOS LEMMA

So we have

Z (t) dZ (t) =

1
1
2
[Z (T )] T
2
2

(20.18)

Equation 20.18 is surprising. In the deterministic calculus, we have


1 2
T .
2

RT
0

xdx =

1
Equation 20.18 has an extra term T . This is why this extra term is
2
needed. Taking expectation of Equation 20.18:
i 1
hR
1
T
2
E 0 Z (t) dZ (t) = E [Z (T )] T
2
2

i
hR
T
As to be explained later, E 0 Z (t) dZ (t) = 0. We already know that
hR
i 1
1
T
2
2
E [Z (T )] = T . Hence E 0 Z (t) dZ (t) = E [Z (T )] T = 0. The extra
2
2
1
term T is needed so the expectations of both sides of Equation 20.18 are
2
equal.

20.4

Properties of the stochastic calculus

1. Lineality.

RT
0

[c1 g (t) + c2 h (t)] dZ (t) = c1

2. Zero mean property. If


0

RT
0

RT
0

g (t) dZ (t)+c2

RT
0

h (t) dZ (t)

T
E X 2 (t) dt < , then E 0 X (t) dZ (t) =

The proof is complex. However, for a simple process g (t) and h (t), Property
#1 holds due to the definition of the stochastic integral. For a simple process
g (t), Property #2 can be easily established.
RT
P
g (t) dZ (t) = n1
k=0 g (tk ) [Z (tk+1 ) Z (tk )]
0
g (tk ) is constant during each partition interval and independent of Z (tk+1 )
Z (tk ). Then
E{g (tk ) [Z (tk+1 ) Z (tk )]} = E [g (tk )] E [Z (tk+1 ) Z (tk )] = E [g (tk )] 0 =
0
RT

RT
RT
Lets apply Property 2 to 0 Z (t) dZ (t). Since 0 E Z 2 (t) dt = 0 tdt =
h
i
RT
1 2
T < , we have E 0 Z (t) dZ (t) = 0
2

20.5. ITOS LEMMA

223

20.5

Itos lemma

20.5.1

Multiplication rules

dZ
dt
0

dZ
dt

dt
0
0

The above table means:


[dZ (t)]2 = dt
dZ (t) dt = 0

(dt)2 = 0

Example 20.5.1. Explain why [dZ (t)]2 = dt


dZ (t) is a normal random variable with mean 0 and variance dt (see the
footnote of Derivatives Markets Page 652). Hence E [dZ (t)]2 = dt
2
E [dZ (t) dt] = V ar [dZ (t)] = dt 0
2
2
Hence dt approach [dZ (t)] in mean square. So [dZ (t)] = dt
Example 20.5.2. Explain why dZ (t) dt = 0
E [dZ (t) dt] = E [dZ (t)] dt = 0 (here dt is treated as a constant)
2
2
2
3
E [dZ (t) dt 0] = E [dZ (t)] (dt) = (dt) 0
Hence 0 approaches dZ (t) dt in mean square. dZ (t) dt = 0.
2

Example 20.5.3. Explain why (dt) = 0


2

(dt) doesnt contain any Brownian motion term dZ. So we need to calculate
(dt)2 according to the deterministic calculus. In the deterministic calculus,
2
2
(dt) 0 as dt 0. Hence (dt) = 0.
The textbook Derivatives Markets also gives the following formula:
0

dZ dZ = dt

(20.19)

The above formula will be explained later.

20.5.2

Itos lemma

In essence, Itos lemma is a Taylor series applied to Brownian motion.

Suppose that a stock has an expected instant return [S (t) , t], dividend

yield S (t) , t, and instant volatility [S (t) , t] follows geometric Brownian motion:

dS (t) = dt + dZ (t)
(20.20)

Here , , and are function of the stock price S (t) and time t.

224

CHAPTER 20. BROWNIAN MOTION AND ITOS LEMMA

Let C [S (t) , t] represent the value of a call or put option. We want to find
out the change of the option value given theres a small change of the stock price
and a small change of time. Using Taylor series, we have:
C
1 2C
2C
C
1 2C
2
2
(dS)
+
(dt)
+
dS +
dt+
dSdt (20.21)
S
t
2 S 2
2 t2
St

2
2
[dS (t)] =
dt + dZ (t) = (dt) +2 dZ (t) dt+

dC [S (t) , t] =

[dZ (t)]

2
dSdt = (dt) + dZ (t) dt

Using the multiplication rules: (dt) = 0


2

[dS (t)] = dt
Now we have:

[dZ (t)] = dt

dZ (t) dt = 0

dSdt = 0

C
C
C
1 2C
C
1 2 C 2
2
(dS)
=
dt
dS +
dt +
dS
+
dt
+
S
t
2 S 2
S
t
2 S 2
(20.22)
Next, apply Equation
20.20 toEquation ??:

C
C
1 2 C 2

dC [S (t) , t] =
dt + dZ (t) +
dt
dt +
S
t
2 S 2

C
C
1 2 2 C
C

+
dZ (t)
+
dt +
S
2 S 2
t
S

C
C
1 2 2 C
C

+
dZ (t)
(20.23)
+
dt +
dC [S (t) , t] =
2
S
2 S
t
S

dC [S (t) , t] =

Equation 20.23 is called the Itos lemma.


Tip 20.5.1. Dont bother memorizing Equation 20.23. Just derive Equation
20.23 on the spot. First, write down the Taylor series Equation 20.22. Next,
apply the multiplication rules. Then youll get Equation 20.23.
If S (t) follows a geometric Brownian motion, we have:

[S (t) , t] = S (t)

[S (t) , t] = S (t)

[S (t) , t] = S (t)
Equation 20.23 becomes:

C
C
1
C
2C
dC [S (t) , t] = ( ) S
+ 2S 2 2 +
dt +
SdZ (t) (20.24)
S
2
S
t
S

20.6. GEOMETRIC BROWNIAN MOTION REVISITED

225

Tip 20.5.2. Dont bother memorizing Equation 20.24. Just derive Equation
20.24 on the spot.
Please note that
C
C
2C
=
=
=
S
S 2
t
Then Equation 20.22 becomes:

(option Greeks)

1
2
dC [S (t) , t] = dS + dt + (dS)
2

(20.25)

Consider a tiny interval dt h. We have:


dC [S (t) , t] C [S (t + h) , t + h] C [S (t) , t]
dS S (t + h) S (t)
Equation 20.25 becomes:
1
2
C [S (t + h) , t + h]C [S (t) , t] [S (t + h) S (t)]+h+ [S (t + h) S (t)]
2
(20.26)
Equation 20.26 is just the textbook Equation 13.6 (Derivatives Markets page
426).
If S (t) were deterministic (i.e. if Equation 20.20 didnt have dZ (t) term),
2C
then
0 and Equation 20.22 would be:
S 2
C
C
dC [S (t) , t] =
dS +
dt
S
t

20.6

Geometric Brownian motion revisited

There are two minor concepts under geometric Brownian motion. SOA can
easily write a question on these concepts. So lets study them.

20.6.1

Relative importance of drift and noise term

Consider a discrete geometric Brownian motion:


X (t + h) X (t) =

X (t) h
| {z }

deterministic component

+ X (t) Y (t) h
{z
}
|
random comp onent

One phrase you need to know is called "the ratio of the standard deviation
to the drift" (the drift is actually the deterministic component). The ratio of
the standard deviation to the drift is defined as:

X (t) h

(20.27)
=
X (t) h
h

226

20.6.2

CHAPTER 20. BROWNIAN MOTION AND ITOS LEMMA

Correlated Ito processes

Let W1 (t) and W2 (t) represent two independent Brownian motions. Suppose
Z (t) = W1 (t)
0

Z (t) = W1 (t) +

p
1 2 W2 (t)

(20.28)
(20.29)

Please note that Z (t) is normally distributed with mean 0 and variance t
because W1 (t) is normally distributed with mean 0 and variance
p t.
You might wonder why Equation 20.29 has constants and 1 2 . These
0

two constants are needed to make Z (t) normally distributed with variance t.
Because W1 (t) and W2 (t) areptwo independent normal random variables, the
2
linearhcombination
i
hW1 (t) + p1 W2 (t) iis also normally distributed.
0
E Z (t) = E W1 (t) + 1 2 W2 (t)
hp
i
= E [W1 (t)] + E
1 2 W2 (t)
p
= E [W1 (t)] + 1 2 E [W2 (t)]
p
= 0 + 1 2 0 = 0
h 0 i
h
i
p
V ar Z (t) = V ar W1 (t) + 1 2 W2 (t)
hp
i
= V ar [W1 (t)] + V ar
1 2 W2 (t)

= 2 V ar [W1 (t)] + 1 2 V ar [W2 (t)]

= 2 t + 1 2 t = t

h
i
0
0
The covariance between Z (t) and Z (t) is: Cov Z (t) , Z (t)

Using the
formula:
Yi) = E (XY ) hE (X)iE (Y )
h standard
i
h Cov (X,
0
0
0
Cov Z (t) , Z (t) = E Z (t) Z (t) E [Z (t)] E Z (t)
h
i
h
i
0
0
= E Z (t) Z (t) 0 0 = E Z (t) Z (t)

h
i
p
p
0
Z (t) Z (t) = W1 (t) W1 (t) + 1 2 W2 (t) = [W1 (t)]2 + 1 2 W1 (t) W2 (t)
h
i

hp
0
E Z (t) Z (t) = E [W1 (t)]2 + E
1 2 W1 (t) W2 (t)
p
2
= E [W1 (t)] + 1 2 E [W1 (t) W2 (t)]
E [W1 (t)]2 = t
E [W1 (t) W2 (t)] = E [W1 (t)] E [W2 (t)] = 0 0 = 0
i
h
0
(20.30)
E Z (t) Z (t) = t

20.6. GEOMETRIC BROWNIAN MOTION REVISITED

227

h
i
h
i
0
0
The textbook says calls Cov Z (t) , Z (t) = E Z (t) Z (t) = t the corre0

lation between Z (t) and Z (t). However, the term correlation between X and
Y typically means the following:
Cov (X, Y )
X,Y =
X Y
0
If wehuse the typical
i definition, the correlation between Z (t) and Z (t) is:
0

Cov Z (t) , Z (t)

t
= =
Z(t) Z 0 (t)
t t
Since Derivatives Markets is the textbook, we have to adopt its definition
0
that the correlation between Z (t) and Z (t) is t.
0

dZ (t) and dZ (t) are both normal random variables with mean 0 and variance dt. Applying Equation 20.30 and replacing t with dt, we have:
i
h
0
(20.31)
E dZ (t) dZ (t) = dt
0

Finally,
lets explain
why Equation 20.19 dZ dZ = dt holds.
i
h
0
E dZ (t) dZ (t) = dt
i2
h
i
h
i2
i
h
h
0
0
0
0
E dZ (t) dZ (t) dt = V ar dZ (t) dZ (t) = E dZ (t) dZ (t) E 2 dZ (t) dZ (t)
0

dZ (t) dZ (t) h
i
p
= dW1 (t) d W1 (t) + 1 2 W2 (t)
h
i
p
= dW1 (t) dW1 (t) + 1 2 dW2 (t)
p
= [dW1 (t)]2 + 1 2 dW1 (t) dW2 (t)
h
i2
0
dZ (t) dZ (t)
p

4
2
2
3
= 2 [dW1 (t)] + 1 2 [dW1 (t)] [dW2 (t)] +2 1 2 [dW1 (t)] dW2 (t)
h
i2
0
E dZ (t) dZ (t)


= 2 E [dW1 (t)]4 + 1 2 E [dW1 (t)]2 [dW2 (t)]2 +2 1 2 E [dW1 (t)]3 dW2 (t)
p

= 2 E [dW1 (t)]4 + 1 2 E [dW1 (t)]2 E [dW2 (t)]2 +2 1 2 E [dW1 (t)]3 E [dW2 (t)]
p

= 2 3 (dt)2 + 1 2 dt dt + 2 1 2 0 0

2
= 2 3 (dt) + 1 2 dt dt

= 32 + 1 2 (dt)2 = 22 + 1 (dt)2
h
i
0
E dZ (t) dZ (t) = dt
i2
i
h
h
0
0
E dZ (t) dZ (t) E 2 dZ (t) dZ (t)

2
2
2
= 22 + 1 (dt) 2 (dt) = 2 + 1 (dt)

228

CHAPTER 20. BROWNIAN MOTION AND ITOS LEMMA


2

(dt) = 0
h
i2
i
h
0
0
E dZ (t) dZ (t) E 2 dZ (t) dZ (t) = 0
h
i2
h
i
0
0
E dZ (t) dZ (t) dt = V ar dZ (t) dZ (t) = 0
0

Hence dZ (t) dZ (t) approach dt in mean square. Hence


0

dZ dZ = dt

(20.32)

Suppose the stock price S (t) follows a geometric Brownian motion:


dS (t)
= dt + dZ (t)
S (t)
1. Apply Itos lemma to ln S (t)
and derive that
ln S (t) is a normal random
variable with mean ln S (0) + 0.5 2 t and variance 2 t.
2. Derive that the mean of S (t) is E [S (t)] = S (0) et

As explained in my book and my solution manual, you dont need to mem2


orize Itos lemma. Just use the Taylor expansion but keep the (dZ) term.
First, use Taylor expansion:
ln S
ln S
1 2 ln S
2
d ln S =
(dS)
dS +
dt +
S
t
2 S 2
ln S
ln S
In the deterministic calculus, you just write d ln S =
dS +
dt.
S
t
2
2
However, in the stochastic calculus, we cant ignore (dZ) since (dZ) = dt.
In this problem, d ln S is a linear function of dZ, so:
h
i
2
2
2
2
(dS) = [Sdt + dZ] = S 2 2 (dt) + 2dtdZ + 2 (dZ)
Using the multiplication
rule (DM 20.17 a, ib, c), we get:
h

2
2
2
2
2
(dS) = S (dt) + 2dtdZ + 2 (dZ) = S 2 2 0 + 2 0 + 2 dt =

S 2 2 dt

So we have to keep the term

1 2 ln S
2
(dS) .
2 S 2

Anyway, Taylor expansion gives us:


ln S
1 2 ln S 2 2
ln S
S dt
dS +
dt +
d ln S =
S
t
2 S 2
2
1
ln S
1
ln S
ln S
=
= 2
=0
Next,
S
S
S
S
t

1 2
1 2
dS
1 2
d ln S =
dt = dt + dZ dt = dt + dZ
S
2
2
2
During a tiny time interval [t, t + dt], the change of ln S is d ln S.
d ln S is a normal random variable. This is why.

First, dZ N (0, dt) .According to DM 20.4, dZ = Y dt. So dZ is a normal


random variable with mean 0 and variance dt.

20.6. GEOMETRIC BROWNIAN MOTION REVISITED

229

During the fixed interval [t, t + dt], dt is a constant. Since a constant plus a
random variable is also a random variable,

1
1
d ln S = 2 dt+dZ is a normal random variable with mean 2 dt
2
2
and variance 2 dt
Now consider the time interval [0, t].

Rt
Rt
Rt
1 2
ln S (t) = ln S (0) + 0 d ln S = ln S (0) + 0 ds + 0 dZ
2

Rt
1 2 Rt
= ln S (0) +
ds + 0 dZ = ln S (0) + 0.5 2 t + Z
0
2
Z is a standard
normal random variable with mean 0 and variance 1, that

is, Z N 0, 2 t . Hence Z is a normal random variable with mean 0 and


variance 2 t. And ln S (t) is normal with mean ln S (0) + 0.5 2 t and
variance 2 t:

ln S (t) N ln S (0) + 0.5 2 t, 2 t

Next, from ln S (t) = ln S (0) + 0.5 2 t + Z, we get:


2
eln S(t) = eln S(0)+(0.5 )t+Z h
i
2
2
S (t) = S (0) e(0.5 )t+Z = S (0) e(0.5 )t eZ
h
i

2
E [S (t)] = S (0) e(0.5 )t E eZ

We can use DM Equation 18.13 to calculate E eZ . DM Equation


18.13

says that if x is normal with mean m and variance v 2 , that is x N m, v 2 ,


then
E (ex ) = em+0.5v

(DM 18.13)

2
E eZ = e0+0.5 t
h
i
2
2
E [S (t)] = S (0) e(0.5 )t e0+0.5 = S (0) et

Alternative method to calculate E [S (t)]. Since ln S (t) N S (0) + 0.5 2 t, 2 t ,


using DM 18.13, we have:
2
2
E [S (t)] = eS(0)+(0.5 )t+0.5 t = S (0) et

230

CHAPTER 20. BROWNIAN MOTION AND ITOS LEMMA

I want you to memorize the following results (these results are used over and
over in Exam MFE):
dZ N (0, dt)

(20.33)

Z N (0, t)

(20.34)

x N m, v 2

E (ex ) = em+0.5v

S (t) is geometric Brownian motion

(20.35)

(20.36)

(20.37)

ln S (t) N ln S (0) + 0.5 2 t, 2 t (20.38)

dS (t)
= dt + dZ (t)
S (t)

2
S (t) = S (0) e(0.5 )t+Z

dS (t)
= dt + dZ (t)
S (t)

20.7

dS (t)
= dt + dZ (t)
S (t)

d ln S (t) = 0.5 2 dt + dZ

dS (t)
= dt + dZ (t)
S (t)
dS (t)
= dt+dZ (t)
S (t)

E [S (t)] = S (0) et

(20.39)

(20.40)

Sharpe ratio

An assets Sharpe ratio is equal to the assets risk premium r divided by


the assets volatility :
r
(20.41)

If two non-dividend paying assets are perfectly corrected (i.e. they are driven
by the same Brownian motion Z (t)), then their Sharpe ratios are equal. Lets
derive this.
The price processes of Asset 1 and Asset 2 are:
SR =

dS1 = 1 S1 dt + 1 S1 dZ

(20.42)

dS2 = 2 S2 dt + 2 S2 dZ

(20.43)

We can form a riskless portfolio by removing the random Brownian motion


dZ. Rewrite Equation 20.42 and 20.43 as:

20.7. SHARPE RATIO

231

1
1 S1
1
2 S2

dS1 =

1
dt + dZ
1

(20.44)

dS2 =

2
dt + dZ
2

(20.45)

Suppose at time zero we buy N1 =

1
units of Asset 1 and short sell
1 S1

1
units of Asset 2. If we hold one unit of Asset 1 at time zero, then
2 S2
after a tiny interval dt, the value of Asset 1 increases by the amount dS1 . If
1
units of Asset 1 at time zero, then after dt the value of
we hold N1 =
1 S1

1
1
1
N1 =
units of Asset 1 will increase by
dt + dZ. Notice
dS1 =
1 S1
1 S1
1
that the increase of the value of Asset 1 has a random component dZ, where dZ
is a normal random variable with mean 0 and variance dt.
1
Similarly, if we short sell N2 =
units of Asset 2 at time zero, after dt,

2 S2

1
2
the value of Asset 2 will increase by
dt + dZ. The increase of
dS2 =
2 S2
2
the value of Asset 2 has a random component dZ, where dZ is a normal random
variable with mean 0 and variance dt.
Suppose at time zero we simultaneously buy N1 units of Asset 1 and short
sell N2 units of Asset 2. Then at dt, we close our position by selling N1 units
of Asset 1 in the open market and buying N2 units of Asset 2 from the open
market.
The cash flow at time zero:

1
1
dollar to buy N1 units of Asset 1.
S1 =
We pay N1 S1 =
1 S1
1
N2 =

We receive N2 S2 =

1
dollars for short selling N2 units of Asset 2
2

1
1
1
1

dollars. If

< 0, then we receive


The net cost is
2
1
2
1

1
1

net cash. To avoid tying up our capital, we go to a bank

1 2

1
1
1
1
1
1
and borrow

dollars. If

< 0, then we lend

.
1 2
1 2
1 2
Our net cash outgo is zero.
Our payo at time dt
At time dt, we sell o N1 units of Asset 1 in the open market for the price
1
1
of S1 + dS1 , receiving N1 (S1 + dS1 ) = N1 S1 + N1 dS1 =
+
dt + dZ
1
1

232

CHAPTER 20. BROWNIAN MOTION AND ITOS LEMMA

At time dt we buy N2 units of Asset 2 in the open market at the price


1
2
+
dt + dZ
S2 + dS2 . We pay N2 (S2 + dS2 ) = N2 S2 + N2 dS2 =
2
2
At time dt, we pay back the bank both the principal
and theaccrued
1
1
interest. The sum of the principal and the interest is

erdt
1 2
Our profit at dt is:
P rof it =

1
1
1
2
1
1
+
dt + dZ
+
dt + dZ

erdt (20.46)
1
1
2
2
1
2

1
1
1
1

< 0, well lend

at t = 0. Then at dt, well receive


If
1
2
1 2
1
1

erdt from the borrower.


1 2
The Taylor expansion:
r2
r3
2
3
erdt = 1 + rdt +
(dt) +
(dt) + ... = 1 + rdt
2!
3!
Equation
20.46
can
be rewritten
as:

1
1
2
1
1
1
+
dt
+
dt

erdt
1
1 2
2

1
2

1
1
1
1 2
1
=

+
dt
erdt
2
1 2 1
1 2

1
1
1
1
1 2
1
1
=

+
dt

rdt
1
2
1 2
1
2
1 2
1 2
1
1
=

dt
rdt
2
1
1 2
1 r 2 r
=

dt
1
2
= (SR1 SR2 ) dt

1 r 2 r
P rof it =
dt = (SR1 SR2 ) dt

1
2

(20.47)

Equation 20.46 and 20.47 dont have the random term dZ (t), indicating that
the profit is surely made.
Our cash outgo is zero at t = 0, but well have the profit indicated by 20.47
at dt.

1 r 2 r
Wealth 0

dt = (SR1 SR2 ) dt
1
2
Time
0
dt
To avoid arbitrage, the profit needs to be zero:
1 r
2 r

=
SR1 = SR2
1
2

20.8. RISK NEUTRAL PROCESS

20.8

233

Risk neutral process

This section is dicult because the author tired to put too many complex concepts in this small section. It seems that the author was in a big hurry to finish
this chapter. The author mentioned many concepts (such as martingale, Girsanovs theorem) but he didnt really explain them, leaving us hanging in the
air asking why.
The only thing worth studying is how to transform a standard geometric
Brownian motion into a risk neutral process.
The standard geometric Brownian motion is:
dS (t)
= ( ) dt + dZ (t)
S (t)
This is how to transform:
dS (t)
= ( ) dt + dZ (t)
S (t)
= (r ) dt + dZ
(t) + ( ) dt

dt
= (r ) dt + dZ (t) +

dt
Define dZ (t) = dZ (t) +

dS (t)

= ( ) dt + dZ (t) = (r ) dt + dZ (t)
S (t)
Just learn this transformation and move on.

20.9

Valuing a claim on S a

In a call or put, the payo at T is a linear function of the stock price S (T ).


What if the payo is linear? For example, what if the payo at T is S (T ) raised
to some power a?

20.9.1

Process followed by S a

Though the textbook uses S a , Im going to use S A . The reason is explained


later.
Suppose the stock price follows geometric Brownian motion:
dS
= ( ) dt + dZ
S
This is why I use S A . If you use S a , the letter a in S a is very similar to
dS
the (alpha) in the equation
= ( ) dt + dZ. This can easily lead to
S
confusion and mistake. As a matter of fact, when I was deriving the formula for
E (S a ), I couldnt match the textbooks formula. It took me a while to figure
out that I accidentally switched a and .
We need to determine the process followed by payo at T is S A .

234

CHAPTER 20. BROWNIAN MOTION AND ITOS LEMMA

S A
1
1 2S A
2
2
(dS) = AS A1 dS + A (A 1) S A2 (dS)
dS +
S
2 S 2
2

2
2
However, (dS) = [( ) dt + dZ] S 2 = 2 dt S 2 = 2 S 2 dt

dS A =

1
dS A = AS A1 dS + A (A 1) S A2 2 S 2 dt
2
1
= AS A1 dS + A (A 1) S A 2 dt
2

dS A
dS
1
=A
+ A (A 1) 2 dt
SA
S
2
2
=A
( ) dt + AdZ + 0.5A2(A
1) dt
= A ( ) + 0.5A (A 1) dt + AdZ

We see that S A follows a geometric Brownian motion with drift A ( ) +


0.5A (A 1) 2 and risk component AdZ.

20.9.2

Formula for S A (t) and E S A (t)

Next, we apply Equation 20.37. Replace with A ( ) + 0.5A (A 1) 2 and


with A.

d ln S A (t) = A ( ) + 0.5A (A 1) 2 0.5A2 2 dt + AdZ


= A ( ) dt 0.5A2 dt + AdZ
Using Equation
20.38, we get:

ln S A (t) N ln S A (0) + A ( ) 0.5A 2 t, A2 2 t


Using Equation 20.39:
2
S A (t) = S A (0) e[A()0.5A ]t+AZ

Using Equation 20.40 just replacing with A ( ) + 0.5A (A 1) 2

2
E S A (t) = S A (0) e[A()+0.5A(A1) ]t

2
We can also derive E S A (t) using S A (t) = S A (0) e[A()0.5A ]t+AZ

2
E S A (t) = E S A (0) e[A()0.5A ]t+AZ

2
= S A (0) E e[A()0.5A ]t+AZ

2
= S A (0) E e[A()0.5A ]t eAZ

2
= S A (0) e[A()0.5A ]t E eAZ

AZ is a normal random variable with mean 0 and variance V ar [AZ (t)] =


A2 2 V ar [Z (t)] = A2 2 t. Using Equation 20.35, we get:

2 2
E eAZ = e0.5A t
A
2
2
2 2
E S (t) = S A (0) e[A()0.5A ]t e0.5A t = S A (0) e[A()+0.5A(A1) ]t

20.9. VALUING A CLAIM ON S A

235

Alternative method to calculate E S A (t) . Using Equation 20.39, we get:


2
dS
= ( ) dt + dZ
S (t) = S (0) e(0.5 )t+Z
S
2
S A (t) = S A (0)eA(0.5 )t+AZ

A
2
E S (t) = E S A (0) eA(0.5 )t+AZ

2
= S A (0) eA(0.5 )t E eAZ
2
2 2
= S A (0) e[A()0.5A ]t e0.5A t
2
= S A (0) e[A()+0.5A(A1) ]t

20.9.3

Expected return of a claim on S A (t)

Consider two geometric Brownian motions S (t) and S A (t).


dS
= ( ) dt + dZ
S
where is the expected return on a claim on S (t) and is the continuous
dividend yield earned by S (t)

dS A
= A ( ) + 0.5A (A 1) 2 dt + AdZ
A
S
= ( ) dt + AdZ
where is the expected return on a claim on S A (t) and is the continuous
dividend yield earned by S A (t). The textbook calls the lease rate.
These two processes have the same risk dZ. Consequently, they have the
same Sharpe ratio:
r
r
=
= r + A ( r)

A
where r is the continuously compounded risk-free interest rate.

20.9.4

Specific examples

The textbook keeps mentioning Jensens inequality. So lets first talk about
Jensens inequality. Jensens inequality is in the appendix C of the textbook.
You can also find information at http://en.wikipedia.org/wiki/Convex_
function
Jensens inequality says
1. if f (x) is convex, then for any probability distribution, we have E [f (x)]
f [E (x)]
2. if f (x) is concave, then for any probability distribution, we have E [f (x)]
f [E (x)]

236

CHAPTER 20. BROWNIAN MOTION AND ITOS LEMMA

Whats a convex function? Whats a concave function?

If at any point you draw a tangent line, the function f (x) stays above the
tangent line, then f (x) is a convex function.

If at any point you draw a tangent line, the function f (x) stays below the
tangent line, then f (x) is a concave function.

A twice dierentiable function of one variable is convex on an interval if and


only if its second derivative is non-negative there.

A twice dierentiable function of one variable is concave on an interval if


and only if its second derivative is negative there.

Heres a simple explanation of Jensens inequality.

20.9. VALUING A CLAIM ON S A

237

d2 y
= 2 > 0, y = x2 is a convex function. Suppose
dx2
wetake two points A (1, 1) and
B (4, 16). The mid point of the line AB is
1+4
1 + 16
C
= 2.5,
= 8.5 . The fact that y = x2 is a convex function means
2
2
that y = x2 curves up. Then it follows that the point C must be above the point

12 + 42
D 2.5, 2.52 = 6. 25 . From the graph below, we clearly sees that
= 8.5
2
2

1+4
= 6.25 (which
(which is the height of the point C) is greater than
2
is the height of the point D). This is an example where the mean of a convex
function is greater than the function of the average.
Consider y = x2 . Sine

25

20

B
15

10

C
D

A
0
0

12 + 42
>
2

1+4
2

, an example of E [f (x)] f [E (x)].

238

CHAPTER 20. BROWNIAN MOTION AND ITOS LEMMA

Next, lets consider a concave function


two points!A (1, 1)
y = x. Consider

1+4
1+ 4
and B (4, 2). The mid point of AB is C
= 2.5,
= 1.5 . Since
2
2

1+4
y = x curves down, then it follows that C must be lower than D
= 2.5, 2.5 = 1. 58 .
2

1+ 4
From the graph below, we clearly sees that
= 1.5 (which is the height
2
r
1+4
= 1. 58 (which is the height of the point
of the point C) is less than
2
D). This is an example where the mean of a concave function is less than the
function of the average.

2.0

D
C

1.5

1.0

0.5

0.0
0

1+ 4
1+4
<
, an example of E [f (x)] f [E (x)]
2
2

By now you should have intuitive feel of Jensens inequality. Lets move on.

20.9. VALUING A CLAIM ON S A

239

The textbook considers the following examples: A = 1, 0, 1.


If A = 1, the time 0 value of the claim S (T ) at T is:
P
[S (T )] = erT S (0) e[(r)]T = S (0) eT
V (0) = F0,T
This is just DM Equation 5.4.
If A = 0, then the claim is just one dollar: S 0 (T ) = 1
P
[1] = erT
V (0) = F0,T
So the time 0 value of getting $1 at T is erT . V (0) is equal $1 discounted
back to time 0 at the risk-free rate.
If A = 2, the time 0 value of the claim S 2 (T ) at T is:

2
2
P
S (T ) = erT S 2 (0) e[2(r)+ ]T
V (0) = F0,T
As a general rule, the forward price of any asset at T is just the prepaid
forward price accumulating at the risk-rate from time 0 to T :
P
erT
F0,T = F0,T
We use the risk-free rate r in the above equation. To get an asset at T , we
P
as a buyer can either pay the seller F0,T
at time 0 or pay the seller F0,T at time
T . To avoid the arbitrage, the two payments should dier only in timing. So
P
F0,T = F0,T
erT .

2
2
2
P
S (T ) erT = S 2 (0) e[2(r)+ ]T
F0,T S (T ) = F0,T
P
F0,T [S (T )] = F0,T
[S (T )] erT = S (0) eT erT = S (0) e(r)T

2
2
2
F0,T S (T ) = S 2 (0) e[2(r)+ ]T = (F0,T [S (T )])2 e T

2
Since e T 1, we have F0,T S 2 (T ) (F0,T [S (T )])2 . This agrees with
Jensens inequality. Roughly speaking 1 , F0,T = E (ST ).
d2 2
S = 2 > 0. Hence S 2 is convex.
dS

2
According to Jensens inequality, we have E S 2 (T ) (E [S (T )]) . This

2
2
2
leads to F0,T S (T ) = E S (T ) (F0,T [S (T )]) = (E [S (T )]) .
S 2 is twice dierentiable and

1
at T is:
If A = 1, the time 0 value of the claim
S (T )

1
1 [(r)+2 ]T
1
2
P
= erT
e T
V (0) = F0,T
= erT
e
(r)T
S
(T
)
S
(0)
S
(0)
e

1
1
1
2
P
F0,T
e T
= F0,T
erT =
S (T )
S (T )
S (0) e(r)T
(r)T
However,
F0,T [S
(T )] = S (0) e
1
1
1
2
F0,T
=
e T
S (T )
F0,T [S (T )]
F0,T [S (T )]

1 Experts dont agree whether F


0,T = E (ST ). Some say the forward price is the unbiased
estimate of the expected future spot price, that is, F0,T = E (ST ). Others disagree. However,
its safe to see that F0,T is very close to E (ST ).

240

CHAPTER 20. BROWNIAN MOTION AND ITOS LEMMA

1
Since
is concave, according to Jensens inequality, we have:
S (T )


2
2
1
1
1
1
= E
.
F0,T
=E
F0,T
S (T )
S (T )
S (T )
S (T )
d2 1
2
S = 3 > 0. Hence S 2 is convex.
2
dS
S

2
1
1
According to Jensens inequality, we have E
. This
E
S (T )
S (T )


2
2
1
1
1
1
leads to F0,T
= E
.
=E
F0,T
S (T )
S (T )
S (T )
pS (T )
If A = 0.5, the
the claim S (T ) at T is:
hptime 0ivalue of p
2
P
rT
S (T ) = e
S (0)e[0.5(r)+0.50.5(0.51) ]T
V (0) = F0,T
p
2
= herT S i(0)e[0.5(r)0.125 ]T
p
p
p
2
F0,T
S (T ) = S (0)e[0.5(r)0.125 ]T = S (0)e0.5(r)T
p
p
2
2 T
= S (0) e(r)T e0.125
=
F0,T [S (T )]e0.125 T
i
h
p
p
2
Since e0.125 T 1, we see that F0,T
S (T ) F0,T [S (T )]
This agrees with Jensens inequality.
p
p
d2
S (T ) is twice dierentiable.
S = 4S 1.5 < 0. Hence S (T ) is
dS 2
concave. hp
i
hp
i p
hp
i
S (T ) = E
S (T ) F0,T [S (T )] =
E (S (T ))
F0,T
S 1 is twice dierentiable and

Example 20.9.1.
The price of a stock follows a geometric Brownian motion:
dS (t)
= (0.1 0.04) dt + 0.3dZ
S (t)
The current price of the stock is 10.
The continuously compounded dividend yield is 0.04 per year.
The continuously compounded risk-free rate is 0.06 per year.
The seller and the buyer enter a forward contract. The contract requires the
seller to pay the buyer S 2 (5) five years from now.
Calculate
the prepaid forward price
the forward price

20.9. VALUING A CLAIM ON S A

241

The probability that S 2 (5) 160.


Solution.
dS (t)
= (0.1 0.04) dt + 0.3dZ
S (t)

A
2
P
S (T ) = erT S A (0) e[A(r)+0.5A(A1) ]T
F0,T
2
2
P
F0,5
S (5) = e0.065 102 e(2(0.060.04)+0.52(21)0.3 )5 = 141. 906 8

P
F0,5 S (5) = F0,5
S 2 (5) e0.065 = 141. 906 8e0.065 = 191. 554 1
= r + A ( r) = 0.06 + 2 (0.1 0.06)
= 0.14
= A ( ) + 0.5A (A 1) 2

= 0.14 2 (0.1 0.04) + 0.5 2 (2 1) 0.32


= 0.07
Its OK for to be negative.

ln S A (t) N ln S A (0) + A ( ) 0.5A 2 t, A2 2

2
2
2
2
2
ln
S (5) N ln 10 + 2 (0.1 2 0.04) 0.5 2 0.3 5, 2 0.3 5
2 (0.1 0.04) 0.5 2 0.3 5 = 0.15

ln 102 + 2 (0.1 0.04) 0.5 2 0.32 5 = ln 100 + 0.15 = 4. 755 17

P S 2 (5) 160 = P ln S 2 (5) ln 160 = (z)


ln 160 4. 755 17
= 0.238 5
z=
22 0.32 5

(z) = 0.594 3
P S 2 (5) 160 = 0.594 3
Example 20.9.2.

The price of a stock follows a geometric Brownian motion:


dS (t)
= (0.12 0.05) dt + 0.25dZ
S (t)
The current price of the stock is 20.
The continuously compounded dividend yield is 0.05 per year.
The continuously compounded risk-free rate is 0.07 per year.
The seller and the buyer enter a forward contract. The contract requires the
1
four years from now.
seller to pay the buyer
S (4)
Calculate
the prepaid forward price
the forward price

242

CHAPTER 20. BROWNIAN MOTION AND ITOS LEMMA


The probability that

1
0.03.
S (4)

Solution.
dS (t)
= (0.12 0.05) dt + 0.25dZ
S (t)

2
P
F0,T
S A (T ) = erT S A (0) e[A(r)+0.5A(A1) ]T

2
1
1
P
F0,4
= e0.074 e(1(0.070.05)+0.5(1)(11)0.25 )4 = 0.04479
S (4)
20
1
1
P
F0,4
= F0,4
e0.074 = 0.04479e0.074 = 0.05 926
S (4)
S (4)
= r + A ( r) = 0.07 1 (0.12 0.07)
= 0.02

= A ( ) + 0.5A (A 1) 2

= 0.02 1 (0.12 0.05) + 0.5 (1) (1 1) 0.252 = 0.027 5

ln S A (t) N ln S A (0) + A ( ) 0.5A 2 t, A2 2

1
1
2
ln
N ln
+ 1 (0.12 0.05) 0.5 (1) 0.252 4, (1) 0.252 4
20
S (4)

1 (0.12 0.05) 0.5 (1) 0.252 4 = 0.155

1
1
ln + 1 (0.12 0.05) 0.5 (1) 0.252 4 = ln 0.155 = 3. 150 7
20
20

1
1
0.03 = P ln
ln 0.03 = (z)
P
S (4)
S (4)
ln 0.03 (3. 150 7)
= 0.711 7
z=q
2
(1) 0.252 4

1
(z) = 0.238 3
P
0.03 = 0.238 3
S (4)

Example 20.9.3.
The price of a stock follows a geometric Brownian motion:
dS (t)
= (0.15 0.04) dt + 0.35dZ
S (t)
The current price of the stock is 10.
The continuously compounded dividend yield is 0.04 per year.
The continuously compounded risk-free rate is 0.08 per year.
The seller and the buyer
p enter a forward contract. The contract requires the
seller to pay the buyer S (6) six years from now.
Calculate

20.9. VALUING A CLAIM ON S A

243

the prepaid forward price


the forward price


The probability that

p
S (6) 4.

Solution.
dS (t)
= (0.15 0.04) dt + 0.35dZ
S (t)

2
P
F0,T
S A (T ) = erT S A (0) e[A(r)+0.5A(A1) ]T
i
hp

2
P
F0,6
S (6) = e0.086 10e(0.5(0.080.04)+0.50.5(0.51)0.35 )6 = 2. 012 6
i
i
hp
hp
P
F0,6
S (6) = F0,6
S (6) e0.086 = 2. 012 6e0.086 = 3. 252 5
= r + A ( r) = 0.08 + 0.5 (0.15 0.08) = 0.115
= A ( ) + 0.5A (A 1) 2

= 0.115 0.5 (0.15 0.04) + 0.5 0.5 (0.5 1) 0.352 = 0.075 3

ln S A (t) N ln S A (0) + A ( ) 0.5A 2 t, A2 2

ln
S (6) N ln 10 + 0.5 (0.15 0.04) 0.5 0.5 0.352 5, 0.52 0.352 6

0.5 (0.15 0.04) 0.5 0.5 0.352 6 = 0.146 25

ln 10 + 0.5 (0.15 0.04) 0.5 0.5 0.352 5 = ln 10 + 0.146 25 = 1.


297 5
hp
i
h p
i
P
S (6) 4 = P ln S (6) ln 4 = (z)
ln 4 1. 297 5
= 0.207 1
z=
0.52 0.352 6 h
i
p
S (6) 4 = 0.582 0
(z) = 0.582 0
P

244

CHAPTER 20. BROWNIAN MOTION AND ITOS LEMMA

Chapter 21

Black-Scholes equation
21.1

Dierential equations and valuation under


certainty

21.1.1

Valuation equation

Lets consider a risk-free world where all assets just earn the risk free interest
rate. Support at time t we spend S (t) to buy one share of a stock. Then at t+h,
we receive D (t + h) h amount of the dividend, where D (t + h) represents the
dividend accumulated per unit of time during [t, t + h]. At t + h after receiving
D (t + h) h dividend, we sell the stock and receive S (t + h). The total amount
of money we have at t + h is D (t + h) h + S (t + h). Had we put S (t) amount of
money in a savings account, we would have S (t) (1 + rh ) amount of money at
t+h, where rh represent the (not-annualized) risk-free interest rate per h period.
To avoid arbitrage, we need to have S (t) (1 + rh ) = D (t + h) h + S (t + h).
Rearranging this equation, we get:
D (t + h) h + S (t + h)
1 + rh

(DM 21.1)

S (t + h) S (t) + D (t + h) h = rh S (t)

(DM 21.2)

S (t) =

hr
i
S (t + h) S (t)
dS (t)
h
= lim
= lim
S (t) D (t + h) = rS (t) D (t)
h0
h0 h
dt
h
(DM 21.3)
1
In Equation 21.3, r = rh represents the annualized continuously comh
pounded interest rate per year. rh is the interest rate per h period; the total
1
1
number of h lengths in a year is . Hence r = rh represents the annualized
h
h
continuously compounded interest rate per year.
245

246

21.1.2

CHAPTER 21. BLACK-SCHOLES EQUATION

Bonds

For a zero-coupon bond, D (t) = 0. DM Equation 21.3 becomes


This gives us
S (t) = S (T ) er(T t)

dS (t)
= rS (t).
dt
(DM 21.4)

Since the boundary condition is S (T ) = 1, we have S (t) = er(T t) . This


equation says that $1 at T is worth er(T t) at time t.

21.1.3

Dividend paying stock

At time t, you spend S (t) and buy one share of a stock. This gives you two
things:
at time T you can sell the stock and get S (T ), which is worth S (T ) er(T t)
at t
you accumulate dividend at a continuous rate of D (s), where D (s) is the
instant dividend earned per unit of time at time s. The total value of
the continuous dividend earned during the interval [s, s + ds] is D (s) ds,
which is worth [D (s) ds] er(st) = D (s) er(st) ds at time t. The present
value at time t of the total continuous dividend earned during the interval
RT
[t, T ] is t D (s) er(st) ds The PV of this continuous flow of dividend is
1 er(T t)
DaT t|r = D
r
RT
To avoid arbitrage, we have S (t) = S (T ) er(T t) + t D (s) er(st) ds.
RT
Please note that if D (s) = D is a constant, then t D (s) er(st) ds =
RT
1 er(T t)
D t er(st) ds = DaT t|r = D
.
r
RT
Anyway, t D (s) er(st) ds is a continuous annuity. If you have trouble
RT
understanding t D (s) er(st) ds, refer to your FM book.

21.2

Black-Scholes equation

21.2.1

How to derive Black-Scholes equation

This section derives DM Equation 21.11.


Vt + 0.5 2 S 2 VSS + (r ) SVS rV = 0
This is the outline of how to derive this formula.
At time t
We buy one option on the stock. We pay V

(DM 21.11)

21.2. BLACK-SCHOLES EQUATION

247

We buy N shares of the stock (a negative N means short selling stocks).


We pay N S .
We deposit W means into a savings account. We pay W .
To have zero-financing, we set our total initial cost to zero
I = V + NS + W = 0

(DM 21.7)

Next, lets consider the change of I during [t, t + dt]:


dI = dV + N (dS + Sdt) + dW

(DM 21.8)

dI is the interest earned on the option during [t, t + dt]. Since W is invested
in a savings account, we have dW = rW dt. This says that the interest earned
on W during [t, t + dt] is rW dt. Notice that the change of S is dS + Sdt (the
sum of the change of the stock price dS and the dividend received Sdt). Apply
Ito lemma:
dV = Vt dt + VS dS + 0.5 2 S 2 VSS dt
dI = Vt dt + VS dS + 0.52 S 2 VSS dt + N (dS + Sdt) + dW = Vt dt +
(VS + N ) dS + 0.5 2 S 2 VSS dt + N Sdt + dW
Set N = VS . Then dS term becomes zero and W = (V VS S).
Because our initial cost is zero, the interest we earned dI should be zero.
dI = Vt dt + 0.5 2 S 2 VSS dt VS Sdt r (V VS S) dt = 0
This leads to DM 21.11.
Vt + 0.52 S 2 VSS + (r ) SVS rV = 0

21.2.2

Verifying the formula for a derivative

Simple PV calculation
Verification that the price of a zero-coupon bond satisfies the Black-Scholes
equation DM 21.11.
$1 at time T is worth V (t) = er(T t) at t.
=
VS = VSS = 0
Vt = rer(T t) = rV
2 2
=
Vt + 0.5 S VSS + (r ) SVS rV = 0
Verification that the price of a prepaid forward contract satisfies the BlackScholes equation DM 21.11.
V (S, t) = S (t) e(T t)
=
VS = e(T t)
VSS = 0
Vt = S (t) e(T t)
=
Vt +0.5 2 S 2 VSS +(r ) SVS rV = Se(T t) +(r ) Se(T t)
rSe(T t) = 0

248

CHAPTER 21. BLACK-SCHOLES EQUATION

Call option
The textbook explains that the price of a European call option satisfies (1) the
boundary condition and, (2) DM 21.11.
V = S (t) e(T t) N (d1 ) Ker(T t) N (d2 )
Verification that the call price formula meets the boundary condition.
The boundary condition is that at the call expiration date T the call is worth
S (T ) K (T ) if S (T ) > K (T ) and zero otherwise.

1
S (t)
+ r + 2 (T t)
K
2

d1 =
T t

d2 = d1 T t

S (T )
1 2
ln
+ r + (T t)
K
2

, d2 = d1 ,
If t approaches T , then d1 =
T t
and

1 2
S (T )
r+
T t
ln
S (T )
S (T )
2
If S (T ) > K, then
> 1, ln
> 0, d1 = K +
=
K
K

T t
+ , N (d1 ) = N (d2 ) = 1, V = S T.
ln

S (T )
If S (T ) < K, then ln
< 0, d1 = , N (d1 ) = N (d2 ) = 0 and
K
V =0
By the way, we dont need to worry about S (T ) = K because the probability of S (T ) = K is zero. The probability that a continuous random variable
takes on a fixed value is zero. When we talk about the probability regarding a
continuous random variable X, we talk about the probability that X falls in a
range [a, b], not the probability that X takes on a single value. If the probability
that X takes a single value is not zero, then the total probability that a < X < b
will be infinite because there are infinite number of single values in the range
[a, b].
We have proved that the call price satisfies the boundary condition.
For the verification that the call price satisfies DM 21.11, see my solution to
DM Problem 21.5, 21.6, and 21.7.
We can also verify that the European put price satisfies DM 21.11. The put
price is
V = Ker(T t) N (d2 ) S (t) e(T t) N (d1 )
Using the formula N (x) = 1 N (x), we can rewrite the put price as
V = Ker(T t) [1 N (d2 )] S (t) e(T t) [1 N (d1 )] = Ker(T t)
S (t) e(T t) + S (t) e(T t) N (d1 ) Ker(T t) N (d2 )

21.2. BLACK-SCHOLES EQUATION

249

You can also derive the above equation using the put-call parity. Notice that
S (t) e(T t) N (d1 ) Ker(T t) N (d2 ) is the call price.
Each of the three terms, Ker(T t) , S (t) e(T t) , and S (t) e(T t) N (d1 )
r(T t)
Ke
N (d2 ), satisfies the BS PDE. Hence the put price satisfies the BS
PDE.
I wont prove that the put price satisfies the boundary condition V (t = T ) =
K S (T ) if K > S (T ) and zero otherwise. You can easily prove this yourself.
Key formula to remember:
If t T , then d1 = d2 and
If S (T ) > K, N (d1 ) = N (d2 ) = 1
If S (T ) < K, N (d1 ) = N (d2 ) = 0
All or nothing option
The textbook points out that S (t) e(T t) N (d1 ) and er(T t) N (d2 ) each satisfy the BS PDE (see my solution to DM Problem 21.5, 21.6). So S (t) e(T t) N (d1 )
can be a price of a derivative; er(T t) N (d2 ) can be a price of another derivative. What derivatives are priced as S (t) e(T t) N (d1 ) and er(T t) N (d2 )
respectively?
Lets check the boundary condition. As t T ,
S (t) e(T t) N (d1 ) S (T ) if S (T ) > K
S (t) e(T t) N (d1 ) 0 if S (T ) < K
S (t) e(T t) N (d1 ) must be the price of an option that pays S (T ) if S (T ) >
K and zero otherwise. Such an option is an asset-or-nothing option.
Similarly, as t T
er(T t) N (d2 ) 1 if S (T ) > K
er(T t) N (d2 ) 0 if S (T ) < K
er(T t) N (d2 ) must be the price of an option that pays 1 if S (T ) > K and
zero otherwise. Such an option is an cash-or-nothing option.
We can break down a European call option into one asset-or-nothing option
and several cash-or-nothing options. Buying a European call option is equivalent
to buying one asset-or-nothing option and selling K units of cash-or-nothing
option (you can verify that they have the same payo). Hence the price of the
European call option is S (t) e(T t) N (d1 ) Ker(T t) N (d2 ).
Similarly, we can break down a gap option (gap option is explained in Chapter 14). In a gap call, the payo is S (T ) K1 if S (T ) > K2 . This is equivalent

250

CHAPTER 21. BLACK-SCHOLES EQUATION

to buying one asset-or-nothing call and selling K1 units of cash-or-nothing option. So the price
of this gap call is S (t) e(T t) N (d1 ) K1 er(T t) N (d2 ),
(T
t)

/K2 er(T t) + 0.5 2 (T t)


ln St e

and d2 = d1 T t.
where d1 =
T t
P (ST > K) = N (d2 ). This is the risk-neutral probability that the call
will be exercised (i.e. call will finish in the money)
P (ST < K) = 1 N (d2 ) = N (d2 ). This is the risk-neutral probability
that the call will NOT be exercised.
Ker(T t) N (d2 ) is the strike price multiplied by the risk-neutral probability that the strike price will ever be paid. This is the expected value of
the strike price *if* the call will be exercised
S (t) e(T t) N (d1 ) is the expected value of the stock price *if* the call
will be exercised (i.e. if ST > K)
N (d1 ) is the expected fractional share of the stock *if* the call will be
exercised
If an option pays one stock when ST > K and zero otherwise, then this
option is worth S (t) e(T t) N (d1 )
If an option pays $1 when ST > K and zero otherwise, then this option is
worth PV of $1 (which is er(T t) ) multiplied by P (ST > K)

21.2.3

Black-Scholes equation and equilibrium returns

This section derives the BS PDE using the idea that the actual expected return
on an option should be equal to the equilibrium expected return.
The expected continuously compounded return on an option is option =
E (dV )
. Here dV is the increase of the option value (i.e. the interest earned
V dt
on the option) during [t, t + dt]. At time t, if you spend V dollars and buy an
option, then during the tiny interval [t, t + dt], your option value will go up by
dV
dV . The (not annualized) return earned on the option per dt period is
.
V
1
Since the number of dt periods in one year is , the annualized (continuously
dt
dV
1
dV
compounded) rate of return per $1 invested in the option is

=
.
V
dt
V
dt

E (dV )
dV
=
. Here V and dt are
The expected return on the option is E
V dt
V dt
treated as constants. dV is a random variable.
To calculate E (dV ), we use Itos lemma:

21.2. BLACK-SCHOLES EQUATION

251

dV = 0.5 2 S 2 VSS + ( ) SVS + Vt dt + SVS dZ


E [dV ] = E 0.5 2 S 2 VSS + ( ) SVS + Vt dt + E (SVS dZ)

= 0.5 2 S 2 VSS + ( ) SVS + Vt dt + SVS E (dZ)


We know that E (dZ) = 0. Recall dZ (t) is a normal random variable with
mean 0 and variance dt (see the footnote of Derivatives Markets Page 652).
E (dV )
0.5 2 S 2 VSS + ( ) SVS + Vt
=
V dt
V
SVS dZ
The (not annualized) unexpected return on the option is
. Here we
V
didnt divide SVS dZ by dt because the unexpected return is not annualized.
SVS
Define
= option .
V
Hence the (not annualized) unexpected return on the option is option dZ.
The Itos lemma can be written as:
dV
= option dt + option dZ
V
Hence option =

By the way, it seems that the textbook has a typo in DM 21.18:


E (dV ) dV
SVS dZ

=
V
V
V

(DM 21.18)

E (dV )
SVS dZ
dV

=
V
V
V
Consider two assets, an stock and an option on the stock.
dS
The process of the stock price:
= dt + dZ
(DM 20.1)
S
dV
The process of the option price:
= option dt + option dZ
V
These two processes are driven by the same dZ. According to Chapter 20,
the stock and the option on the stock must have the same Sharpe ratio. Hence
r
option r
=

option
The correct formula should be:

=
=

V
r
option r
r =
(option r)
=
SV

SVS
S
V

V
0.5 2 S 2 VSS + ( ) SVS + Vt
r =
r
SVS
V
( r) SVS = 0.5 2 S 2 VSS + ( ) SVS + Vt rV
Vt + 0.5 2 S 2 VSS + (r ) SVS rV = 0 (BS PDE)

252

CHAPTER 21. BLACK-SCHOLES EQUATION

SVS
By the way, according to DM 12.8, the option elasticity is =
. Hence
V
SVS
option =
= (DM 21.20). DM 21.20 is slightly dierent from DM
V
12.9:
(DM 12.9)
option = ||
option =
(DM 21.20)
Obviously, the author of Derivatives Markets changed his definition of option .
In Chapter 12, option is non negative; in Chapter 21, option can be positive,
zero, or negative.

21.3

Risk-neutral pricing

This section repeats the old idea of risk neutral pricing. Under risk neutral
pricing, we can set the expected return on the stock to r and get the correct
price of a derivative.
This section contains many formulas. Make sure you understand the meaning
of each formula.
The textbook lists the following equations:

dS
= (r ) dt + dZ
S

d
E (dV ) = Vt + 0.52 S 2 VSS + (r ) SVS
dt
d
E (dV ) = rV
dt

(DM 21.28)
(DM 21.30)
(DM 21.31)

These equations are risk-neutral version of similar formulas.


Next, the textbook introduced a new symbol f (ST : St ). f (ST : St ) is the
conditional probability of the stocks terminal price ST given that the price
today is St .
Youll want to memorize
the following formulas:
R
P (ST > K) = K f (ST : St ) dST (this holds whether f (ST : St ) is riskneutral probability or the true probability)
The price of a derivative is Vt = er(T t) E (VT )
The price of a European call is
RK
R
Vt = er(T t) E (VT ) = er(T t) 0 0 f (ST : St ) dST +er(T t) K [S (T ) K]
R

f (ST : St ) dST = er(T t) K [S (T ) K] f (ST : St ) dST


Please note that the price of the call at expiration is VT = 0 if S (T ) < K
and S (T ) K if S (T ) > K.
Similarly, the price of a European put is
RK
Vt = er(T t) E (VT ) = er(T t) 0 [K S (T )] f (ST : St ) dST

Chapter 22

Exotic options: II
This is an easy chapter.

22.1

All-or-nothing options

Cash-or-nothing option.
A cash call pays $1 at T is ST > K and $0 if ST K
A cash put pays $1 at T is ST < K and $0 if ST > K
Asset-or-nothing option.
An asset-or-nothing call pays ST at T is ST > K and $0 if ST K
An asset-or-nothing put pays ST at T is ST < K and $0 if ST > K
Price of cash-or-nothing option.
The price of a cash call option is er(T t) N (d2 ), where N (d2 ) = P (ST > K)
(i.e. the risk neutral probability of ST > K)
The price of a cash put option is er(T t) [1 N (d2 )] = er(T t) N (d2 ),
where N (d2 ) = P (ST < K) (i.e. the risk neutral probability of ST <
K)
Price of asset-or-nothing option.
The price of an asset call option is St e(T t) N (d1 )
The price of an asset put option is St e(T t) [1 N (d1 )] = St e(T t) N (d1 )
253

254

CHAPTER 22. EXOTIC OPTIONS: II

Price of a European call and put


Buying an ordinary European call option is equivalent to buying one
asset-or-nothing call and selling K cash-or-nothing calls. The call price is
St er(T t) N (d1 ) Ker(T t) N (d2 )
Buying an ordinary European put option is equivalent to buying K cashor-nothing puts and selling one asset-or-nothing put. The put price is
Ker(T t) N (d2 ) St er(T t) N (d1 )
Price of a gap call and put
Buying a gap call option is equivalent to buying one asset-or-nothing call
with strike price K2 and selling K1 cash-or-nothing calls with strike price
K2 . The gap call price is St er(T t) N (d1 ) K1 er(T t) N (d2 ) .
Buying a gap put option is equivalent to buying K1 asset-or-nothing puts
with strike price K2 and selling one asset-or-nothing put with strike price
K2 . The gap put price is K1 er(T t) N (d2 ) St er(T t) N (d1 ) .

ln St e(T t) /K2 er(T t) + 0.5 2 (T t)

For both a gap call and put, d1 =


T t

and d2 = d1 T t.
Delta-hedging all-or-nothing options
Its dicult to hedge an all-or-nothing option because the payo is not continuous. The textbook explains this well. Refer to the textbook.

Chapter 23

Volatility
The concept of implied volatility is explained in DM section 12.5. The implied
volatility of an option is the volatility implied by the market price of the option
based on the Black-Scholes formula. If you plug the implied volatility into the
Black-Scholes formula, the formula should produce the option price that is equal
to the current market price of the option.
The Black-Scholes formula assumes that a given stock has a constant volatility. Under the BS formula, the stocks volatility is a inherent characteristic of
the stock; it doesnt depend on other factors (such as the stick price K and the
options expiry T ). In reality, however, the implied volatility of a stock depends
on K and T .
The volatility smile is a long-observed pattern where at-the-money options
tend to have lower implied volatilities than in- or out-of-the-money options.
The implied volatility is lowest when the option is at the money
The implied volatility gets higher and higher as the option gets more and
more in-the-money
The implied volatility gets higher and higher as the option gets more and
more out-of-the money.
If we plot the implied volatility in a 2-D plane (setting the implied volatility
as Y and the strike price K as X), the diagram looks like a letter U (the letter
U looks like a smile).
To see a diagram of the volatility smile, refer to
http://www.optiontradingpedia.com/volatility_smile.htm
http://en.wikipedia.org/wiki/Volatility_smile
Volatility surface is a 3-D diagram of the implied volatility as a function
of the strike price K and time to maturity T .
255

256

CHAPTER 23. VOLATILITY

To account for the factor that the implied volatility depends on K and T ,
dS (t)
dS (t)
we can rewrite DM 20.25
= ( ) dt + dZ (t) as DM 23.1
=
S (t)
S (t)
( ) dt+ (St , Xt, t) dZ (t). In DM 23.1, the instant volatility (St , Xt, t) dZ (t)
is a function of the stock price St , another factor Xt, , and the
timent.
1
1 P 2
2
Historical volatility. Please note that DM 23.2 =
is not
h n 1 i=1 i
new. This formula is already used in DM Chapter 11 "Estimating Volatility." In
n
1 P
2 is the estimated variance per h period. Since the number
DM 23.2,
n 1 i=1 i

n
1
1
1 P
2i .
of h periods in one year is , the variance per year is
h
h n 1 i=1
This is all you need to know about Chapter 23.

Chapter 24

Interest rate models


24.1

Market-making and bond pricing

24.1.1

Review of duration and convexity

Duration and convexity are explained in Derivatives Markets Section 7.3. Section 7.3 is excluded from the MFE syllabus. However, Derivatives Markets page
781 mentions the duration hedging:
...hedging a bond portfolio based on duration does not result in a perfect
hedge. Recall that the duration of a zero-coupon bond is the bonds time to
maturity...
Because Chapter 24 mentions duration and Chapter 24 is on the syllabus,
SOA may test your knowledge about duration hedging. So read DM Section
7.3 and take a quick review of duration and convexity. Next, lets solve a few
problems.
Example 24.1.1.
The yield to maturity of a 5-year zero-coupon bond is 6%. The face amount
of the bond is 100.
Calculate
the value of the bond
the Macaulay duration
the modified duration
the convexity
257

258

CHAPTER 24. INTEREST RATE MODELS

In addition, calculate the bond value if the yield to maturity


moves up to 7%
moves down to 5%.
Solution.
The bond value is P0 = 100 1.065 = 74. 725 8
The (Macaulay) duration of a zero-coupon bond is its maturity T . So the
(Macaulay) duration is
DMac = T = 5 years.
The modified duration is
DMac
5
Dmod =
=
= 4. 716 98 years
1 + yield
1.06
The convexity of a zero-coupon bond is:
56
T (T + 1)
C=
2 = 1.062 = 26. 699 893
(1 + y)
If the yield to maturity is now 7%, the bond value is:
P1 = 100 1.075 = 71. 298 6
We can also use duration and convexity to approximate the bond value under
the new yield.
The new bond value is

P
P1 = P0 + P = P0 1 +
P0
1
1
P
= Dmod y+ C (y)2 = 4. 716 980.01+ 26. 699 893(0.01)2 =
P0
2
2
0.045 8
P1 = 74. 725 8 (1 0.045 8) = 71. 303 3
This is very close to the correct amount of 71. 298 6.
If the yield to maturity is now 5%, the bond value is:
P1 = 100 1.055 = 78. 352 6
We can also use duration and convexity to approximate the bond value under
the new yield.
P
1
1
2
= Dmod y + C (y) = 4. 716 98 (0.01) + 26. 699 893
P0
2
2
(0.01)2 = 0.048 5
P1 = 74. 725 8 (1 + 0.048 5) = 78. 350 0
This is very close to the correct amount of 78. 352 6.
Example 24.1.2.

24.1. MARKET-MAKING AND BOND PRICING

259

The yield to maturity of a 4-year zero-coupon bond is 8%. The face amount
of the bond is 100.
Calculate
the value of the bond
the Macaulay duration
the modified duration
the convexity
In addition, calculate the bond value if the yield to maturity
moves up to 9%
moves down to 7%.
Solution.
The bond value is P0 = 100 1.084 = 73. 5030
DMac = T = 4 years.
DMac
4
Dmod =
=
= 3. 703 7 years
1 + yield
1.08
45
T (T + 1)
C=
2 = 1.072 = 17. 468 8
(1 + y)
If the yield to maturity is now 9%, the bond value is:
P1 = 100 1.094 = 70. 842 5
We can also use duration and convexity to approximate the bond value under
the new yield.
The new bond value is

P
P1 = P0 + P = P0 1 +
P0
1
1
P
2
2
= Dmod y + C (y) = 3. 703 7 0.01 + 17. 468 8 (0.01) =
P0
2
2
0.036 2
P1 = 73. 5030 (1 0.036 2) = 70. 842 2
This is very close to the correct amount of 70. 842 5.
If the yield to maturity is now 7%, the bond value is:
P1 = 100 1.074 = 76. 289 5
We can also use duration and convexity to approximate the bond value under
the new yield.
P
1
1
= Dmod y + C (y)2 = 3. 703 7 (0.01) + 17. 468 8
P0
2
2
2
(0.01) = 0.037 9
P1 = 73. 5030 (1 + 0.037 9) = 76. 288 8
This is very close to the correct amount of 76. 289 5.

260

CHAPTER 24. INTEREST RATE MODELS

Example 24.1.3.
Portfolio A consists of one unit of 4-year zero coupon bond.
Portfolio B consists of x units of 1-year zero coupon bond and y units of
8-year zero coupon bond. Portfolio B is used to duration hedge Portfolio A.
Each of the three zero-coupon bonds above has 100 face amount.
The current annual eective interest rate is 5% for all three bonds.
Assume that the yield curve changes by a uniform amount if theres a change.
Demonstrate why duration-hedging leads to arbitrage under two scenarios:
the annual eective interest rate moves up to 6% immediately after the
three bonds are issued.
the annual eective interest rate instantly moves down to 4% immediately
after the three bonds are issued.

Solution.
To duration hedge Portfolio A, we need to satisfy two conditions:
Portfolio A and B have the same present value
Portfolio A and B have the same duration
So we set up the following equations:
100
100
100
x+
y=
1.051
1.058
1.054
100
100
x
y
8
1.051
1.05
1+
8=4
100
100
100
100
x
+
y
x
+
y
1.051
1.058
1.051
1.058
This is the meaning of the second equation. Portfolio B consists of two
bonds. If a portfolio consists of multiple bonds, then the portfolios duration is
just the weighted average of the each bonds duration, with weight equal to the
present value of each bond.
The 2nd equation can be simplified as

100
100
100
x 1+
y 8 = 4
1
8
1.05
1.05
1.054

100
100
100
x+
y=
1
8
1.05
1.054
1.05

100
100
100
x 1+
y 8=4
1.051
1.058
1.054

x = 0.493 62 , y = 0.520 93

24.1. MARKET-MAKING AND BOND PRICING

261

So Portfolio B needs to consist of 0.493 62 unit of 1-year zero coupon bond


and 0.520 93 units of 8-year zero coupon bond.
If the interest rate moves up to 6%:
100
Portfolio A is worth:
= 79. 2094
1.064
100
100
0.493 62 +
0.520 93 = 79. 251 7
1.061
1.068
To arbitrage, at t = 0 (when the interest rate is 5%), we buy low and sell
high:
Portfolio B is worth:

buy Portfolio B. We pay

100
100
100
x+
y=
= 82. 270 2
1.051
1.058
1.054

sell Portfolio A. We receive

100
= 82. 270 2
1.054

So our net cost is zero.


Then instantly later at t = 0+ , the interest rate moves up to 6%. Then
Portfolio B (our asset) is worth 79. 251 7; Portfolio A (our liability) is worth 79.
2094.
Our net profit is 79. 251 7 79. 2094 = 0.042 3
If the interest rate moves down to 4%:
100
Portfolio A is worth:
= 85. 480 4
1.044
100
100
0.493 62 +
0.520 93 = 85. 527 3
1
1.04
1.048
To arbitrage, at t = 0 (when the interest rate is 5%), we buy low and sell
high:
Portfolio B is worth:

buy Portfolio B. We pay

100
100
100
x+
y=
= 82. 270 2
1
8
1.05
1.05
1.054

sell Portfolio A. We receive

100
= 82. 270 2
1.054

So our net cost is zero.


Then instantly later at t = 0+ , the interest rate moves down to 4%. Then
Portfolio B (our asset) is worth 85. 527 3; Portfolio A (our liability) is worth 85.
480 4.
Our net profit is 85. 527 3 85. 480 4 = 0.046 9
So no matter the interest rate moves up to 6% or moves down to 4%, we can
always make free money by buying Portfolio B and selling Portfolio A.

262

CHAPTER 24. INTEREST RATE MODELS

Why is Portfolio B better than Portfolio A? It turns out B has high convexity.
Convexity of A:
TA (TA + 1)
45
=
= 18. 1406
CA =
2
2
1.05
(1 + y)
Convexity of B is just the weighted average convexities of the two bonds,
with weights beingthe present valueof the bond.

100
89
100
12

0.493
62
+

0.520
93
1.052
1.051
1.052
1.058
CB =
100
100
0.493 62 +
0.520 93
1.051
1.058

12
100
89
100

0.493 62 +

0.520 93
1.052
1.051
1.052
1.058
=
= 29. 024 9
100
1.054
CB > CA
If the interest rate moves
up by

y, then the present value of a portfolio is:


P
P1 = P0 + P = P0 1 +
P0
1 A
1
P B
P A
2
A
B
=
D
y
+
(y)
= Dmod
y + C B (y)2
C
mod
A
B
2
2
P0
P0
4
A
B
B
A
However, Dmod = Dmod =
= 3. 809 5
C >C
1.05
P B
P A
>
B
P0
P0A
A
Since P0 = P0B
P B > P A
B
A
P1 > P1
So Portfolio B always worth more than Portfolio A under
a flat yield curve.
For example, if the interest rate moves up to 6%, then
P A
1
= 3. 809 5 0.01 + 18. 1406 0.012 = 0.037 2
2
P0A
1
P B
= 3. 809 5 0.01 + 29. 024 9 0.012 = 0.036 6
B
2
P0

P1A =

100
(1 0.037 2) = 79. 2098
1.054

P1B =

100
(1 0.036 6) = 79. 259 2
1.054

P1B P1A = 79. 259 2 79. 2098 = 0.049 4


Key point to remember:

24.1. MARKET-MAKING AND BOND PRICING

263

1. Two portfolios can have the same present value, the same duration, but
dierent convexities.
2. Under the assumption of the parallel shift of a flat yield curve, we can
always make free money by buying the high-convexity portfolio and sell
the low-convexity portfolio.
3. The parallel shift of a flat yield curve assumption leads to arbitrage.
Information about three zero-coupon bonds:
Maturity (Yrs) Face
2
100
4
100
7
100
The current interest rate is 7% for all three bonds. Assume a parallel shift
of a flat yield curve. Design an arbitrage strategy.
We need to form 2 portfolios. These two portfolios have the same PV, the
same duration, but dierent convexity. We can make free money by buying the
high convexity portfolio and selling the low convexity portfolio.
The low convexity portfolio is the 4-year bond (Portfolio A).
The high convexity portfolio consists of x unit of 2-year bond and y unit
of 7-year bond (Portfolio B). This is called a barbell. A barbell bond portfolio
combines short maturities (low duration) with long maturities (high duration)
for a blended, moderate maturity (moderate duration)
Portfolio

Maturity (Yrs)

PV
100
= 76. 29
1.074
100
= 87. 34
1.072
100
= 62. 27
1.077

100
100
100
x+
y=
1.072 1.077
1.074

100
100
100
x 2+
y 7=4
2
7
1.07
1.07
1.074

Duration
4
2
7

Convexity
45
= 17. 468 8
1.072
23
= 5. 240 6
1.072
78
= 48. 912 6
1.072

x = 0.524 1, y = 0.490 0

At time 0, we buy Portfolio B (which consists of 0.524 1 unit of 2-year bond


and 0.490 0 unit of 7-year bond). Simultaneously, we sell Portfolio A.
The convexity of Portfolio A is:
C A = 17. 468 8
The convexity of Portfolio B is:

Units
1
x
y

264

CB

CHAPTER 24. INTEREST RATE MODELS


100
78
100
23

0.524 1 +
0.490 0
2
2
2
7
1.07
1.07
1.07
1.07
=
= 22. 708 9
100
1.074

For example, if the new interest rate is 7.25% for all bonds with dierent
maturities, then
100
100
P1B =
0.524 1 + 0.490 0
= 75. 584 1
1.07252
1.07257
100
= 75. 580 9
1.07254
B
A
P1 > P1
Our profit is 75. 584 1 75. 580 9 = 0.003 2
P1A =

If the new interest rate is 6.5% for all bonds with dierent maturities, then
100
100
P1B =
0.524 1 + 0.490 0
= 77. 739 6
2
1.065
1.0657
100
= 77. 732 3
1.0654
P1B > P1A
Our profit is 77. 739 6 77. 732 3 = 0.007 3
P1A =

Either way, we make money.


By now you should see that the parallel shift of a flat yield curve is a bad
model.

24.1.2

Interest rate is not so simple

We all know what an interest rate is, yet a derivative on interest rate is surprisingly dicult. To get a sense of the diculty, suppose we want to calculate the
price of a European call on a 1-year zero-coupon bond that pays $100 one year
from now. Here are the inputs:
The call expires in one year
The strike price is $100
The continuous risks-free rate is r = 6% per year
The current price of the bond is 100e0.06 = 94. 18.
The volatility of the bond return is = 30%
Using the Black-Scholes formula, we find:
d2 = 0.15
N (d1 ) = 0.5596
N (d2 ) = 0.4404
d1 = 0.15
C = 94. 18 0.5596 94. 18 0.4404 = 11. 23

24.1. MARKET-MAKING AND BOND PRICING

265

Everything looks fine. However, after further thinking, you realize that the
call price C should be zero. At T = 1, the bond pays $100. Hence the 100-strike
call value is zero. Why should anyone buy a 100-strike call on an asset worth
$100 at call expiration?
What went wrong? It turns out that we cant use the Black-Scholes formula
to calculate the price of an interest rate derivative:
The Black-Scholes option formula assume that the term structure of the
interest rate is flat and deterministic (see DM page 379). However, If the
interest rate is known and constant, there wont be any need for interest
rate derivatives. This is similar to the idea that if the stock price is known
and constant, there wont be any need for call or put option.
The standard deviation of the return is a constant. However, the standard deviation of the return is not constant. Unlike a stock, a bond has
a finite maturity. At the maturity date, the bond value is its face amount.
Hence the standard deviation of a bonds return decreases as the bond
approaches its maturity.
The key point. Its much harder to calculate the price of an option interest
rate because interest rate is not a tradeable underlying asset. For call and put
on stocks, we can buy or short sell stocks to set up the hedge portfolio. However,
we cant go out and buy a 5% interest rate to hedge an option on interest rate.
Now lets go to the textbook.

24.1.3

Impossible bond pricing model

A bond is a derivative on interest rate. We normally dont think this way, but
a bond derives its value from interest rate. If the market interest rate goes up,
the bond value goes down; if the market interest rate goes down, the bond value
goes up.
So our starting point is to set up a stochastic random variable called the
short interest rate. The textbook keeps using the phrase "short interest
rate" or "short rate" without giving a clear definition. Here is the definition:
Definition 24.1.1. A short interest rate or short rate is just the instantaneous
interest rate r (t) over a short (hence the name "short rate") interval [t, t + dt].
First, we assume that the short interest rate r (t) follows the Itos process:
dr = (r) dt + (r) dZ

(24.1)

Next, the text book explains that we cant assume r (t) follows a flat yield
curve ("impossible bond pricing model").
A flat yield curve means that the interest rate is independent of time. To
have a flat yield curve, the following two conditions are met:

266

CHAPTER 24. INTEREST RATE MODELS

The initial interest rate r is a constant regarding time, that is, r (t) = t
If the interest rate changes, the change is also independent of time (i.e.
parallel shift of the yield curve)
If the continuously compounded interest rate r is constant, then the present
value at time t of $1 to be received at T is:
P (t, T ) = er(T t)

(24.2)

For example, if a zero coupon bond pays us $1 at T = 2, the PV of this bond


at time zero is P (0, 2) = e2r . The PV of this bond at t = 1 is P (1, 2) = er .
Next, lets review the textbooks proof and find out why Equation 24.1 and
24.2 wont work together.
Suppose we want to delta hedge a bond. Whereas delta hedging a call means
buying shares of a stock, delta hedging a bond means buying units of a
bond with a dierent maturity date.
Suppose at time t we buy a bond maturing at T2 . "A bond maturing at T2 "
just means that we, the bond holder, will receive $1 at T2 . So the price of this
bond at time t is P (t, T2 ) = er(T2 t) .
To delta hedge this bond, at t, we buy bonds maturing at T1 (please note
the textbook uses N instead of ). The cost is P (t, T1 ) = er(T1 t)
The total cost of buying two bonds is P (t, T1 ) + P (t, T2 ) = er(T1 t) +
r(T2 t)
e
. To avoid tying up our capital, at time t we borrow er(T1 t) +
r(T2 t)
e
from a bank to finance the purchase of two bonds. So at time t, our
portfolio is:
buy 1 bond maturing at T2
buy bonds maturing at T1
borrow er(T2 t) + er(T1 t) from a bank
Our net position is zero:
I = P (t, T1 ) + P (t, T
2 ) + W = 0 where W = [P (t, T1 ) + P (t, T2 )] =
er(T1 t) + er(T2 t) . As seen before, if we borrow money, we use a negative number. If we lend money, we use the a positive number. This is why W
is negative.
In the next instant dt, the interest rate moves up to r + dr. Now change of
our portfolio value is:
dI = dP (t, T1 ) + dP (t, T2 ) + dW
Using Itos lemma, we have:
2
2
1)
1)
1)
dt + P (t,T
dr + 12 Pr(t,T
(dr)
dP (t, T1 ) = P (t,T
2
t
r
P (t,T1 )
r(T1 t)
= t
e
= rP (t, T1 )
t
P (t,T1 )
r(T1 t)
=
e
= P (t, T1 ) (T1 t)
r
r

24.1. MARKET-MAKING AND BOND PRICING


2 P (t,T1 )
r2
2

267

r(T1 t)
= r
= er(tT1 ) (T1 t) = P (t, T1 ) (T1 t)
2e
2
2
(dr) = (dt + dZ) = 2 (dZ) = 2 dt
dP (t, T1 ) = rP (t, T1 ) dt P (t, T1 ) (T1 t) dr + 12 P (t, T1 ) (T1 t)2 2 dt

Similarly,
2
dP (t, T2 ) = rP (t, T2 ) dt P (t, T2 ) (T2 t) dr + 12 P (t, T2 ) (T2 t) 2 dt
However, dW = rW dt. To see why, notice that dW is the interest earned on
W during the interval [t, t + dt]. During [t, t + dt], the continuous interest rate
r can be treated as a discrete interest rate. The interest earned is just
the initial capital interest rate length of the interval = rW dt
Now wehhave DM Equation 24.6:
i
2
dI = rP (t, T1 ) dt P (t, T1 ) (T1 t) dr + 12 P (t, T1 ) (T1 t) 2 dt
h
i
+ rP (t, T2 ) dt P (t, T2 ) (T2 t) dr + 12 P (t, T2 ) (T2 t)2 2 dt
+rW dt
Well want to have dI = 0. If dI = 0, then the value of our portfolio wont
change during the same during [t, t + dt]. To make dI = 0, we first choose
such that the dr term is zero.
[P (t, T1 ) (T1 t)] + [P (t, T2 ) (T2 t)] = 0
2 t)P (t,T2 )
= (T
(T1 t)P (t,T1 )

This negative delta means that we need to sell


maturing at T1 .

(T2 t)P (t,T2 )


(T1 t)P (t,T1 )

units of bond

2 t)P (t,T2 )
By the way, = (T
(T1 t)P (t,T1 ) is similar to DM Equation 7.13 (Derivatives
Markets page 227):

N =

D1 B1 (y1 ) / (1 + y1 )
D2 B2 (y2 ) / (1 + y2 )

(DM 7.13)

So buying bonds is really duration-hedging.


Next,
h we want to set the dt term to zero:
i h
i
2
2
rP (t, T1 ) + 12 P (t, T1 ) (T1 t) 2 + rP (t, T2 ) + 12 P (t, T2 ) (T2 t) 2 +
rW = 0
2

r [P (t, T1 ) + P (t, T2 ) + W ]+ 12 P (t, T1 ) (T1 t) 2 + 12 P (t, T2 ) (T2 t) 2 =


However,P (t, T1 ) + P (t, T2 ) + W = 0
2
2
12 P (t, T1 ) (T1 t) 2 + 12 P (t, T2 ) (T2 t) 2 = 0
2
(T
t)P
(t,T
)
12 (T21 t)P (t,T21 ) P (t, T1 ) (T1 t) 2 + 12 P (t, T2 ) (T2 t)2 2 = 0
2

12 (T2 t) P (t, T2 ) (T1 t) 2 + 12 P (t, T2 ) (T2 t) 2 = 0


12 (T2 t) P (t, T2 ) [(T1 t) (T2 t)] 2 = 0

268

CHAPTER 24. INTEREST RATE MODELS

12 (T2 t) P (t, T2 ) (T1 T2 ) 2 = 0


T1 = T2
= 1
= 1 means that sell one bond.
If we buy a bond maturing in T2 years, the only way to hedge the risk is to
sell this bond! So one moment you buy a bond. The next moment you sell it.
Your net position is zero. Of course you are hedged against all risks, but this
hedging is really doing nothing type of hedging.
This tells us that the bond pricing model based on Equation 24.1 and 24.2
are impossible. In our words, Equation 24.1 is OK. But Equation 24.2 is bad.
This confirms that a parallel shift of a flat yield curve is a bad assumption for
pricing a bond.

24.1.4

Equilibrium equation for bonds

Since its bad to assume that r is a flat yield curve, we switch our gears and
assume that r isnt a flat yield curve. Now we just assume that Equation 24.1
holds.
dP (r, t, T ) =

P
P
1 2P
(dr)2
dr +
dt +
r
t
2 t2

(dr) = [ (r) dt + (r) dZ] = 2 (r) (dZ) = 2 (r) dt


P
P
P
1 2P 2
(r) dt =
dP (r, t, T ) =
dr +
dt +
[ (r) dt + (r) dZ] +
r
t
2 t2
r
2
1 P 2
P
(r) dt
dt +
t
2 t2

P
P
1 2P 2
P

(r)
+
dP (r, t, T ) = (r)
+
dt +
(r) dZ
r
2 t2
t
r

(24.3)

Define
(r, t, T ) =

P
P
1 2P 2
1

(r)
+
(r)
+
P (r, t, T )
r
2 t2
t

(24.4)

P
1
(r)
P (r, t, T ) r

(24.5)

q (r, t, T ) =

Now Equation 24.3 becomes:


dP (r, t, T )
= (r, t, T ) dt + q (r, t, T ) dZ
P (r, t, T )

(24.6)

24.1. MARKET-MAKING AND BOND PRICING

269

dP (r, t, T )
is the bonds return. Equation 24.6 says that the bonds return
P (r, t, T )
is the sum of a drift term (r, t, T ) dt and a random component q (r, t, T ) dZ.
P
is
Please also note that generally q (r, t, T ) is negative. This is because
r
negative. If r goes up, the bond price goes down; if r goes down, P goes up.
Next, we are going to derive DM Equation 24.16:
2 (r, t, T2 ) r
1 (r, t, T1 ) r
=
q1 (r, t, T1 )
q2 (r, t, T2 )

(24.7)

Suppose we have two bonds, Bond #1 and Bond #2 maturing at T1 and T2


respectively. Each bond follows DM Equation 24.6:
dP (r, t, T1 )
= 1 (r, t, T1 ) dt + q1 (r, t, T1 ) dZ
P (r, t, T1 )
dP (r, t, T2 )
= 2 (r, t, T2 ) dt + q2 (r, t, T2 ) dZ
P (r, t, T2 )
At t = 0, we form a self-financing portfolio consisting of
buying units of Bond #1
buying 1 Bond #2
borrowing P (r, t, T1 ) + P (r, t, T2 ) from a bank at the riskless short rate
r
The value of this portfolio is I
dP (r, t, T1 )
dP (r, t, T2 )
dI = P (r, t, T1 )
+ P (r, t, T2 )
P (r, t, T1 )
P (r, t, T2 )
[P (r, t, T1 ) + P (r, t, T2 )] rdt
= P (r, t, T1 ) [1 (r, t, T1 ) dt + q1 (r, t, T1 ) dZ]
+P (r, t, T2 ) [2 (r, t, T2 ) dt + q2 (r, t, T2 ) dZ][P (r, t, T1 ) + P (r, t, T2 )] rdt
= P (r, t, T1 ) [1 (r, t, T1 ) r] dt + P (r, t, T2 ) [2 (r, t, T2 ) r] dt
+ [P (r, t, T1 ) q1 (r, t, T1 ) + P (r, t, T2 ) q2 (r, t, T2 )] dZ
Choose such that P (r, t, T1 ) q1 (r, t, T1 )+P (r, t, T2 ) q2 (r, t, T2 ) = 0. This
removes the stochastic random term dZ. Now dI is deterministic. Since the
portfolio is self-financing and riskless, it earns zero interest rate. Hence
P (r, t, T1 ) [1 (r, t, T1 ) r] dt + P (r, t, T2 ) [2 (r, t, T2 ) r] dt = 0
P (r, t, T2 ) q2 (r, t, T2 )

P (r, t, T1 ) [1 (r, t, T1 ) r] dt+P (r, t, T2 ) [2 (r, t, T2 ) r] dt =


P (r, t, T1 ) q1 (r, t, T1 )
0
q2 (r, t, T2 )

[1 (r, t, T1 ) r] dt + [2 (r, t, T2 ) r] dt = 0
q1 (r, t, T1 )
2 (r, t, T2 ) r
1 (r, t, T1 ) r
=

q1 (r, t, T1 )
q2 (r, t, T2 )

270

CHAPTER 24. INTEREST RATE MODELS

Define the following term as Sharp ratio:


(r, t, T ) r
= (r, t)
q (r, t, T )

(24.8)

Apply Equation
24.4 and 24.6 to 24.8:

P
P
1 2P 2
1

(r)
+
(r)
+
r
P (r, t, T )
r
2 t2
t
= (r, t)
P
1
(r)
P (r, t, T ) r

P
1 2P 2
P
P
(r) +
+
rP = (r, t)
(r)
(r)
r
2 t2
t
r
2P
P
P
1 2
(r) 2 + [ (r) (r) (r, t)]
+
rP = 0
(24.9)
2
r
r
t
Any interest-dependent securities (not just zero coupon bonds) must satisfy
Equation 24.9.
To solve the bond price P (r, t, T ), we need to use Equation 24.9 together
with the following boundary condition:
P (r, T, T ) = 1

(24.10)

The solution to Equation 24.9 and 24.10 is:

P (r, t, T ) = Et

" Z
exp

#!

r (s) ds

= Et (exp [R (t, T )])

(24.11)

where E means that the expectation is based on the risk neutral probability
and
Z
T

R (t, T ) =

r (s) ds

(24.12)

Equation 24.11 is the general formula for a bond price.

24.1.5

Delta-Gamma approximation for bonds

The key formula is:


1
(24.13)
E (dP ) = rP
dt
Dont worry about how to prove Equation 24.13. Just memorize its meaning.
Equation 24.13 says that under the risk neutral distribution, the bond is priced
to earn a risk-free rate.

24.2. EQUILIBRIUM SHORT-RATE BOND PRICE MODELS

271

24.2

Equilibrium short-rate bond price models

24.2.1

Arithmetic Brownian motion (i.e. Merton model)

A simple model is to assume that the short rate r (t) follows arithmetic Brownian
motion:
dr (t) = dt + dZ

(24.14)

In addition, we assume (r, t) =


Advantage:
The model is simple.
Disadvantages:
r (t) can go negative. Since r (t) is normally distributed with mean r0 + t
and variance 2 t, r (t) can be negative. A negative interest rate wont
make any sense.
r (t) is not mean reverting.
V ar [r (t)] = 2 t. This is undesirable because the volatility of the interest
should depend on the interest rate. If the interest rate is high, then the
volatility is high.
We can derive the bond price under the Merton model. Under this model,
Equation 24.9 now becomes:
1 2 2P
P
P
2 r2 + [ ] r + t rP = 0
We guess the solution is P (r, t, T ) = A (T t) eB(T t)r(t) where A (T t)
and B (T t) are two functions of T t.
2P
P
P
0
= AB 2 eBr
= ABeBr
= rAB eBr A0 eBr
2
r
r
t
0
12 2 AB 2 eBr [ ] ABeBr + rAB eBr A0 eBr rAeBr = 0

0
1 2
AB 2 eBr [ ] ABeBr A0 eBr = rA eBr B eBr
2

(24.15)

Equation 24.15 should hold for any r. The only way to make it work for any
r is:
1 2
2
0
2 AB [ ] AB A = 0
A 1B

=0

Using the boundary condition is P (r, T, T ) = 1, we get A (0) eB(0)r = 1.


For this equation
to hold
for any r, we need to have A (0) = 1 and B (0) = 0.
0
Hence from A 1 B = 0, we get:

272

CHAPTER 24. INTEREST RATE MODELS


0

B =1
B =T t
From 12 2 AB 2 [ ] AB A0 = 0. we get:
A0
2
= 12 2 B 2 [ ] B = 12 2 (T t) [ ] (T t)
A
2
d ln A (T t) = 12 2 (T t) ( ) (T t)
1
3
2
ln A (T t) = 16 2 (T t) ( ) (T t) + C where C is a con2
stant.

1
3
2
1 2
A = A (0) exp 6 (T t) ( ) (T t)
2

1
3
2
1 2
= exp 6 (T t) ( ) (T t)
2
Finally, we have:

1
3
2
1 2
P (r, t, T ) = exp 6 (T t) ( ) (T t) e(T t)r(t)
2
You dont need to memorize the above formula. Just understand how to
derive the formula in case SOA or CAS gives you a tough problem.

24.2.2

Rendleman-Bartter model

This model assume that r (t) follows a geometric Brownian motion:


dr (t)
= dt + dZ
r (t)

(24.16)

Advantage:
r (t) cant be negative
V ar [r (t)] = r2 (t) 2 t. The variance increases if r (t) increases. This is
desirable because the volatility of the interest is high if the interest rate
is high.
Disadvantage:
not mean-reverting

24.2.3

Vasicek model

According to Wikipedia, Vasiceks model was the first one to capture mean reversion, an essential characteristic of the interest rate that sets it apart from
other financial prices. Stock prices can rise indefinitely. However, interest rates
cannot. Excessively high interest rate would hamper economic activity, prompting a decrease in interest rates. Similarly, interest rates can not decrease indefinitely. If interest rates are too low, few people are willing to lend their money.
This tends to push up the interest rate. As a result, interest rates move in a
limited range, showing a tendency to revert to a long run value.

24.2. EQUILIBRIUM SHORT-RATE BOND PRICE MODELS

273

The model is:


dr (t) = (b r) dt + dZ

(24.17)

The variance of r (t) is t


Advantage:
mean reverting
Disadvantage:
can produce a negative interest rate
V ar [r (t)] = 2 t. So the variance doesnt increase if r (t) increases.
We can also derive the bond price P (t, T ) under the Vasicek model. However, the derivation is far more complex than in the Merton model. Here is the
outline of the derivation.
Under the Vasicek model, Equation 24.9 becomes:
1 2 2P
P
P
2 r2 + [ (b r) ] r + t rP = 0
subject to the boundary condition P (r, T, T ) = 1
Once again, we guess the solution is P (r, t, T ) = A (T t) eB(T t)r(t)
P
2P
P
0

= AB 2 eBr
= ABeBr
= rAB eBr A0 eBr
2
r
r
t0
12 2 AB 2 eBr [ (b r) ] ABeBr +rAB eBr A0 eBr rAeBr =
0
0

12 2 AB 2 [ (b r) ] AB + rAB A0 rA = 0

0
12 2 AB 2 (b ) AB + AB A0 = A AB aAB r
For the above equation to hold for any r, we need to have:
0
A AB AB = 0
1 2
2
0
2 AB (b ) AB + AB A = 0
0

1 B B = 0
A AB AB = 0
The boundary condition is B (0) = 0
1 e(T t)
This gives us: B =

Equation 12 2 AB 2 (b ) AB + AB A0 = 0 is hard to solve.


Someone solved this equation for us. The result is DM Equation 24.26. You
dont need to memorize the solution for A or B. Just memorize P (r, t, T ) =
A (T t) eB(T t)r(t) .
Example 24.2.1. May 2007 SOA MFE #13
Let P (r, t, T ) denote the price at time t of $1 to be paid with certainty at
time T , t T , if the short rate at time t is equal to r. For a Vasicek model
you are given:
P (0.04, 0, 2) = 0.9445

274

CHAPTER 24. INTEREST RATE MODELS

P (0.05, 1, 3) = 0.9321
P (r , 2, 4) = 0.8960
Calculate r .
The price of any bond must satisfy Equation 24.9. We guess that the solution
to Equation 24.9 is P (r, t, T ) = A (T t) eB(T t)r(t)
P (0.04, 0, 2) = 0.9445
A (2) eB(2)0.04 = 0.9445
P (0.05, 1, 3) = 0.9321
A (2) eB(2)0.05 = 0.9321
B(2)0.04
0.9445
0.9445
A (2) e
=
eB(2)0.01 =

B(2)0.05
0.9321
0.9321
A (2) e

1
100
0.9445 0.01
0.9445
eB(2) =
= 3. 749 26
=
0.9321
0.9321

0.04

A (2) = 0.9445eB(2)0.04 = 0.9445 eB(2)


= 0.9445 3. 749 260.04 =
0.995 77
P (r , 2, 4) = 0.8960

A (2) eB(2)r = A (2) eB(2)


= 0.8960

r
= 0.8960
0.995 77 3. 749 26

0.8960
r
3. 749 26
=
= 0.899 81
0.995 77

r ln 3. 749 26 = ln 0.899 81
ln 0.899 81
r =
= 0.07988 4 = 0.08
ln 3.
749 26
Please note r can be solved without using any specifics of the Vasicek model.
As a matter of fact, this problem can be rewritten as:
Let P (r, t, T ) denote the price at time t of $1 to be paid with certainty at
time T , t T , if the short rate at time t is equal to r. You are given:
P (0.04, 0, 2) = 0.9445
P (0.05, 1, 3) = 0.9321
P (r , 2, 4) = 0.8960
Calculate r .

24.2.4

CIR model

The model assumes that the short interest rate r (t) follows the stochastic differential equation:
p
dr (t) = (b r) dt + r (t)dZ
(24.18)
In CIR model, V ar [r (t)] = 2 r (t) t.
The drift factor (b r) in the CIR model is the same as the drift factor
in the Vasicek model. It ensures mean reversion of the interest rate towards
the long run value b, with speed of adjustment governed by the strictly positive
parameter a.
p
The standard deviation factor, r (t), corrects the main drawback of Vasiceks model, ensuring that the interest rate cannot become negative. Thus,
at low values of the interest rate, the standard deviation becomes close to zero,

24.3. BOND OPTIONS, CAPS, AND THE BLACK MODEL

275

canceling the eect of the random shock on the interest rate. Consequently,
when the interest rate gets close to zero, its evolution becomes dominated by
the drift factor, which pushes the rate upwards.
Advantage:
Mean-reverting
Not allow a negative interest rate
Variance of V ar [r (t)] = 2 r (t) t. The higher the r (t), the higher the
V ar [r (t)]. This is a desirable feature.
To solve the bond price under CIR model, again we guess the solution is
P (r, t, T ) = A (T t) eB(T t)r(t) . The solution is listed in DM page 788. You
dont need to memorize the solution. Just memorize P (r, t, T ) = A (T t) eB(T t)r(t) .

24.3

Bond options, caps, and the Black model

24.3.1

Black formula

Notations:
Pt (T, T + s). This is the price agreed upon at time t, which will be paid at
T in order to receive $1 at T + s. Simply put, Pt (T, T + s) is the present
value of $1 discounted from T + s to T using the interest rate available
at t. For example, at time zero we know that the annual interest rate
from t = 1 to t = 3 is 10% compounded continuously. Then P0 (1, 1 + 2)
is just PV of $1 discounted from t = 3 to t = 1 using 10% continuously
compounded interest rate. So P0 (1, 1 + 2) = e0.1(2) = 0.818 73
PT (T, T + s). This is PV $1 discounted from T + s to T . Generally, we
simplify the symbol PT (T, T + s) as P (T, T + s)
Consider a call option with strike price K, expiring at time T , on a zerocoupon bond paying $1 at time T + s. If you buy this call option, then at T you
have the right to buy a bond maturing at time T + s for the guaranteed price
K. Since time T cost of a bond maturing in T + s is PV of $1 discounted from
T + s to T , buying a bond maturing at time T + s for the guaranteed price K
really means "give up K and receive PV of $1 discounted from T + s to T ." The
PV of $1 discounted from T + s to T is PT (T, T + s). So the call payo is
Call P ayof f = max [0, PT (T, T + s) K]

(DM 24.30)

Next, the textbook has a dicult formula:


Ft,T [P (T, T + s)] =

P (t, T + s)
P (t, T )

(DM 24.31)

276

CHAPTER 24. INTEREST RATE MODELS

To understand the meaning of DM 24.31, use an example. Suppose t = 0,


T = 1, and s = 2. In addition, assume that the interest rate is always 10%
compounded continuously.
Ft,T [P (T, T + s)] = F0,1 [P (1, 3)] is the price agreed upon at t = 0 to be
paid at T = 1 in order to receive $1 at T + s = 3. In other words, if you
pay F0,1 [P (1, 3)] at T = 1, you should receive PV of $1 discounted from
T + s = 3 to T = 1. To avoid arbitrage, F0,1 [P (1, 3)] = e0.1(2) = e0.02
P (t, T + s) = P (0, 3) is PV of $1 discounted from T + s = 3 to time zero.
P (0, 3) = e0.1(3) = e0.03
P (t, T ) = P (0, 1) is PV of $1 discounted from T = 1 to time zero.
P (0, 1) = e0.1
0.03

Clearly we see that e0.02 = ee0.1 .


Next, the textbook gives us the price of the call on a bond. Consider a call
option is written at time zero with strike price K, expiring at time T , on a
zero-coupon bond paying $1 at time T + s. If you buy this call option at time
zero, then at T you have the right to pay K and receive the PV of $1 discounted
from T + s to T .
To find the call price, well use DM Equation 12.5:
P
P
C = F0,T
(S) N (d1 ) F0,T
(K) N (d2 )
ln

d1 =

P (S)
F0,T
+0.5 2 T
F P (K)
0,T

d2 = d1 T

This is how I memorize DM Equation 12.5:


C =Time zero cost of what you get at T N (d1 ) Time zero cost of what
you giveat T N (d2 )

Tim e zero cost of what you get at T
2
d1 = ln Time
zero cost of what you give at T + 0.5 T / T

d2 = d1 T

Lets apply DM Equation 12.5 to call on bond.


What we get at T is P (T, T + s), PV of $1 discounted from T + s to T
Time zero cost of P (T, T + s) is P (0, T + s), PV of $1 discounted from
T + s to time 0
What we give at T is K
Time zero cost of what we give at T is KP (0, T ), PV of K discounted
from T to time 0

24.3. BOND OPTIONS, CAPS, AND THE BLACK MODEL

277

C =Time zero cost of what you get at T N (d1 ) Time zero cost of what
you giveat T N (d2 )

Time zero cost of what you get at T
2
d1 = ln Time
+
0.5
T
/ T
zero cost of what you give at T

d2 = d1 T

This gives us the price of the call on a bond:


C = P (0, T + s) N (d1 ) P (0, T ) KN (d2 )
d1 =

ln

P (0,T +s)
2
P (0,T ) +0.5 T

d2 = d1 T

We can also calculate the price of the put on the bond:


P = P (0, T ) KN (d2 ) P (0, T + s) N (d1 )
Please note that DM Equation 24.32 uses the following formula:
C = P (0, T ) [F N (d1 ) KN (d2 )]
d1 =

ln

2
F
T
K +0.5

This is how to derive these formulas. Notice, F = F0,T [P (T, T + s)] =


P (0,T +s)
P (0,T )

C = P (0, T + s) N (d1 ) P (0, T ) KN (d2 )


= F P (0, T ) N (d1 ) P (0, T ) KN (d2 )
= P (0, T ) [F N (d1 ) KN (d2 )]
Example 24.3.1. SOA May 2007 MFE #7
You are given the following information:
Bond maturity (years)
1
2
Zero-coupon bond price 0.9434 0.8817
A European call option, that expires in 1 year, gives you the right to purchase
a 1-year bond for 0.9259.
The bond forward price is lognormally distributed with volatility = 0.05.
Using the Black formula, calculate the price of the call option.
Solution.
If you buy this option, then at T = 1, you can pay K = 0.9259 and buy a
1-year bond. This 1-year bond will give you $1 at time T + s = 2. .

278

CHAPTER 24. INTEREST RATE MODELS

The value of this bond at T = 1 is PV of $1 discounted from T + s = 2 to


T = 1.Time zero cost of PV of $1 discounted from T + s = 2 to T = 1 is
just PV of $1 discounted from T + s = 2 to t = 0. The cost is 0.8817
Time zero cost of the strike price K at T = 1 is just PV of K discounted
from T = 1 to time zero. So P V (K) = K 0.9434 = 0.9259 0.9434
C =Time zero cost of what you get at T N (d1 ) Time zero cost of what
you giveat T N (d2 )

Tim e zero cost of what you get at T
2
d1 = ln Time
zero cost of what you give at T + 0.5 T / T
d1 =

ln

2
0.8817
0.92590.9434 +0.50.05 1

= 0.212 0
0.05 1

d2 = d1 T = 0.212 0 0.05 1 = 0.162 0

N (d2 ) = 0.564 3
N (d1 ) = 0.583 9
C = 0.8817 0.583 9 0.9259 0.9434 0.564 3 = 0.021 9
We can also calculate the put price.
P =Time zero cost of what you give at T N (d2 ) Time zero cost of what
you get at T N (d1 )
P = 0.9259 0.9434 (1 0.564 3) 0.8817 (1 0.583 9) = 0.013 7
If you want to use the formula C = P (0, T ) [F N (d1 ) KN (d2 )], this is
how:
P (0, T ) = P (0, 1) is PV of $1 discounted from T = 1 to time zero. So
P (0, 1) = 0.9434
F = F0,T [P (T, T + s)] = F0,1 [P (1, 2)] is the forward price of 1-year bond.
This is the price agreed up at time zero, paid at T = 1 in order to receive
$1 at T + s = 2. Using Equation DM 24/31, we have F0,1 [P (1, 2)] =
P (0,2)
0.8817
P (0,1) = 0.9434
d1 =

ln

F
K

+0.5 2 T

ln

0.8817
0.9434
0.9259

+0.50.052 1

0.05 1

= 0.212 0

d2 = 0.212 0 0.05 1 = 0.162 0


N (d1 ) = 0.583 9
N (d2 ) = 0.564 3
C = P (0,T ) [F N (d1 ) KN (d2 )]

= 0.9434 0.8817
0.9434 0.583 9 0.9259 0.564 3 = 0.0220

P = P (0,T ) [KN (d2 ) F N (d1 )]


= 0.9434 0.9259 (1 0.564 3)
= 0.013 7

0.8817
0.9434

(1 0.583 9)

24.4. BINOMIAL INTEREST RATE MODEL

24.3.2

279

Interest rate caplet

Notation
Rt (T, T + s). The (not annualized) interest rate pre-agreed upon at time
t where t T that applies to the future time interval [T, T + s].
RT (T, T + s). The (not annualized) interest rate agreed upon at time T
that applies to the time interval [T, T + s].
Caplet. A caplet gives the buyer the right to buy the time-T market
interest rate RT (T, T + s) by paying a fixed strike interest rate KR . If
KR RT (T, T + s), the caplet expires worthless. The payo of the caplet
at T + s is max [0, RT (T, T + s) KR ]. The payo of the caplet at T is
max [0, RT (T, T + s) KR ]
1 + RT (T, T + s)
To calculate the price of the caplet, we first modify
the payo:

max [0, RT (T, T + s) KR ]


RT (T, T + s) KR
= (1 + KR ) max 0,
=
1 + RT (T, T + s)
(1 + RT (T, T + s)) (1 + KR )

1
1
1

PT (T, T + s)
(1 + KR ) max 0,
= (1 + KR ) max 0,
1 + KR
1 + RT(T, T + s)
1 + KR

1
max 0,
PT (T, T + s) is the payo of a put on a bond. This put
1 + KR
gives the buyer the right, at T , to sell a bond that matures at T + s for a
1
guaranteed price 1+K
. Let P represent the price of this put. Hence the price
R
of the caplet is (1 + KR ) P .

24.4

Binomial interest rate model

Binomial interest rate tree is really simple. Unfortunately, the author of the
textbook uses too many math symbols and formulas, making this section hard
to read. What I will do here is to walk you through a few examples. If you
understand these examples, you are fine.
Example 24.4.1. (DM Example 24.3)
The following is the 3-period interest rate tree (DM Figure 24.3)
t=0 t=1 t=2
0.18
0.14
0.10
0.10
0.10
0.06
0.02

280

CHAPTER 24. INTEREST RATE MODELS

Make sure you understand the above table. The 10% interest rate at t = 0
applies to the interval [t = 0, t = 1]. The 14% and 6% interest rates apply to
the interval [t = 1, t = 2]. The interest rates 0.18, 0.10, and 0.02 at t = 2 apply
to the interval [t = 2, t = 3].
The price of the 1-year bond is just the PV of $1 discounted from t = 1 to
t = 0. Hence P (0, 1) = e0.1
The price of the 2-year bond is just the PV of $1 discounted from t = 2 to
t = 0. Whats tricky is that we have two interest rates during [t = 1, t = 2].
If the path is u, then $1 at t = 2 travels back to t = 0 through 2 interest
rates: 0.14, and 0.1. The PV of $1 discounted from t = 2 to t = 1 is
e0.14 . The PV of e0.14 discounted from t = 1 to t = 0 is e0.14 e0.11 =
e(0.14+0.1) . So the 2-year bond is worth e(0.14+0.1) at t = 0
If the path is d, then $1 at t = 2 travels back to t = 0 through 2 interest
rates: 0.06 and 0.1. The 2-year bond is worth e(0.06+0.1) at t = 0
Since the risk-neutral probability of up or down is 50%, then the 2-year bond
is worth the following at t = 0:
P (0, 2) = 0.5e(0.14+0.1) + 0.5e(0.06+0.1) = 0.819 4
The term 0.5e(0.14+0.1) + 0.5e(0.06+0.1)
can be irewritten as:
h S
(0.14+0.1)
(0.06+0.1)

2i=0 ri h
0.5e
+ 0.5e
=E e
with h = 1
The price of the 3-year bond is just the PV of $1 discounted from t = 3 to
t = 0.
If the path is uu, then $1 at t = 3 travels back to t = 0 through 3 interest
rates: 0.18, 0.14, and 0.1. The PV of $1 discounted from t = 3 to t = 0 is
e(0.18+0.14+0.1) . So the 3-year bond is worth e(0.18+0.14+0.1) at t = 0
If the path is ud, then $1 at t = 3 travels back to t = 0 through 3 interest
rates: 0.1, 0.14, and 0.1. The 3-year bond is worth e(0.1+0.14+0.1) at t = 0
If the path is du, then $1 at t = 3 travels back to t = 0 through 3 interest
rates: 0.1, 0.06, and 0.1. The 3-year bond is worth e(0.1+0.06+0.1) at t = 0
If the path is dd, then $1 at t = 3 travels back to t = 0 through 3 interest
rates: 0.02, 0.06, and 0.1. The 3-year bond is worth e(0.02+0.06+0.1) at
t=0
Since the risk-neutral probability for each is 0.52 = 0.25, the price of a 3-year
bond is the following
at t = 0:

P (0, 3) = 0.25 e(0.18+0.14+0.1) + e(0.1+0.14+0.1) + e(0.1+0.06+0.1) + e(0.02+0.06+0.1) =


0.743 8

The term 0.25 e(0.18+0.14+0.1) + e(0.1+0.14+0.1) + e(0.1+0.06+0.1) + e(0.02+0.06+0.1)


can be rewritten as:

24.4. BINOMIAL INTEREST RATE MODEL

281

(0.18+0.14+0.1)

+ e(0.1+0.14+0.1) + e(0.1+0.06+0.1) + e(0.02+0.06+0.1) =


h0.25Se3
i
E e i=0 ri h
The general formula is DM 24.44:

h Sn
i
P (0, n) = E e i=0 ri h

(DM 24.31)

Option pricing example

Example 24.4.2. (SOA May 2007 #9)


You use a binomial interest rate model to evaluate a 7.5% interest rate cap
on a $100 three-year loan. You are given:
(i) The interest rates for the binomial tree are as follows:
r0 = 6%
ru = 7.704%
rd = 4.673%
ruu = 9.892%
rud = rdu = 6.000%
rdd = 3.639%
(ii) All interest rates are annual eective rates.
(iii) The risk-neutral probability that the annual eective interest rate moves
up or down is 0.5.
(iv) The loan interest payments are made annually.
Using the binomial interest rate model, calculate the value of this interest
rate cap.
t=0 t=1
t=2
9.892%
7.704%
6.000%
6%
6.000%
4.673%
3.639%
Once again, the interest rate at t in the above table applies to the period
[t, t + 1]. For example, the 6% rate applies to [0, 1] (i.e. Year 1).
First, lets understand whats an interest rate cap. Imagine you borrowed
$100 from a bank. Your interest accrued on the loan each year is not based
on a fixed interest rate (such as 8%), but is based on the then market interest
rate. For example, if the market interest rate is 6% in Year 1, then your interest
payment at the end of Year 1 is 100 0.06 = 6.0.

282

CHAPTER 24. INTEREST RATE MODELS

As a borrower, you are worried that the market interest may go up. For
example, if the Year 1 interest rate is 20%, then your interest payment at the
end of Year 1 is 100 0.2 = 20.
How can you reduce your risk? One thing you can do is to buy an interest
cap. Suppose you buy an interest rate cap of 7.5%. This is what happens:
If the market interest rate is at or below 7.5%, then the cap doesnt kick
in. So you get nothing from the cap
if the market interest rate is above 7.5%, then the party who sold you
the cap will pay you the excess of the market interest rate over the cap
rate. For example, if the market interest rate in Year 1 is 10%, then the
seller of the cap will pay you 100 (0.1 0.075) = 2. 5 at the end of Year
1. You still own the bank 100 0.1 = 10 at the end of Year 1, but you
pay 100 0.075 = 7. 5 out of your own pocket. The cap seller pays you
100 (0.1 0.075) = 2. 5. So together you get 7. 5 + 2.5 = 10. You mail a
$10 check to the bank.
In summary, if you buy an interest capped of 7.5%, then the interest on
your loan is capped at 7.5% regardless of the market interest rate (because any
excess of the market rate over the cap 7.5% is paid by the cap seller).
Now you know what an interest cap is, lets solve this problem.

The first year market rate 6% is below the cap rate. At the end of Year 1,
you get nothing from the cap

If the 2nd year market rate is 7.704%, then at the end of year 2 the cap
seller pays you 100 (0.07704 0.075) = 0.204 . Notice that 0.204 occurs at
t = 2. To help keep track of payments, well discount this payment to t = 1.
0.204
The discounted value is
at t = 1.
1 + 0.07704
If the 2nd year market rate is 4.673%, then the payment at the end of year
2 is zero.

Lets calculate the cap payo in Year 3. Of the 4 interest rates during
[t = 2, t = 3] , only when the market interest rate is 9.892% do we get a payo
of 100 (0.09892 0.075) = 2. 392. This payment occurs at t = 3. The PV of
2. 392
.
this payment at t = 2 is
1 + 0.09892
Now we can draw a payo table:

24.5. BLACK-DERMAN-TOY MODEL


t=0

t=1

283
t=2
9.892% Payo:

7.704% Payo:

100 (0.07704 0.075)


1 + 0.07704

100 (0.09892 0.075)


1 + 0.09892

6.000%
6%
6.000%
4.673%
3.639%
100 (0.07704 0.075)
= 0.189 4 1 at t = 1
1 + 0.07704
100 (0.07704 0.075)
t = 0 is
. The risk neutral probability
(1 + 0.09892) (1 + 0.06)
node is 0.5.
100 (0.09892 0.075)
= 2. 176 68 at t = 2
The PV of payo
1 + 0.09892
100 (0.09892 0.075)
t = 0 is
. The risk neutral
(1 + 0.09892) (1 + 0.07704) (1 + 0.06)
reaching uu node is 0.52 = 0.25
The PV of payo

discounted to
of reaching u
discounted to
probability of

Hence risk neutral based expected present value of the cap payo is:
100 (0.07704 0.075)
100 (0.09892 0.075)
0.5+
0.52 =
(1 + 0.07704) (1 + 0.06)
(1 + 0.09892) (1 + 0.07704) (1 + 0.06)
0.565 99 = 0.57
So the price of the cap is $0.57

24.5

Black-Derman-Toy model

The BDT model is a procedure to produce a discrete binomial interest rate


tree that matches the observed term structure of interest rates. Once again,
to quickly explain the essence of the BDT model, Im walk you through an
example. If you understand this example, you are ready for the exam.
Suppose we gathered the following data from the market (DM Table 24.2):
Maturity n (Yrs) YTM Bond Price Volatility in Yr 1 on (n 1)-Yr bond
1
10%
0.9091
2
11%
0.8116
10%
3
12%
0.7118
15%
4
12.5% 0.6243
14%
We want to produce an interest rate tree that will match the above table.
What should we do?
First, lets go through some terms.

284

CHAPTER 24. INTEREST RATE MODELS

YTM (yield to maturity) is the discrete annual eective interest rate earned
by a bond. The formula is
P (0, T ) = (1 + Y T M )T
For example, a 2-year bond in the table is worth 0.8116. This means that
PV of $1 discounted from t = 2 to t = 0 is 0.8116. To find YTM, we solve the
following equation:
2
0.8116 = (1 + i)
i = 0.11
So the YTM for this 2-year bond is 11%
The volatility in Yr 1 on (n 1) bond is the standard deviation of the natural
log of the YTM on an (n 1)-year bond issued at t = 1 and maturity at time
n. For example, 10% volatility in the table means that standard deviation of
the natural log of the YTM on a 1-year bond issued at t = 1 maturing in t = 2
is 10%. This concept will be clear to you later.
Next, next use the BDT model to build an interest rate tree. Well first find
the interest rate ru and rd , the two possible interest rates for Year 2 (i.e. for
the time interval [t = 1, t = 2]).
t=0
t=h=1
ru
r0 = 10%
rd

Once again, rt is the interest for the interval [t, t + 1]. For example, r0 = 10%
is the interest rate for Year 1. ru and rd are the two interest rates for Year 2
(i.e. from t = 1 to t = 2).
We assume the risk neutral probability of up and down is 0.5. We want to
find ru and rd to satisfy the condition that standard deviation of the natural
log of the YTM on a 1-year bond issued at t = 1 maturing in t = 2 is 10%.
The price of a 1-year bond issued at t = 1 maturing in t = 2 is
1
1
P (1, 2) =
or
1 + ru
1 + rd
The YTM is:
1
1
Y T Mu = ru
(1 + Y T Mu ) =
1 + ru
1
1
or (1 + Y T Md ) =
Y T Md = rd
1 + rd
We know that the standard deviation of ln Y T Mu = ln ru and ln Y T Md =
ln rd is 10%.
The mean of the log of the YTM on a 1-year bond issued at t = 1 maturing
in t = 2 rate is:
E (ln r1 ) = 0.5 ln ru + 0.5 ln rd = 0.5 (ln ru + ln rd )

24.5. BLACK-DERMAN-TOY MODEL

285
2

We use the variance formula V ar (X) = E [X E (X)] to calculate the


variance of ln Y T Mu = ln ru and ln Y T Md = ln rd :
0.5 [ln ru E (ln r1 )]2 + 0.5 [ln rd E (ln r1 )]2
2
2
= 0.5 (ln ru 0.5 ln ru 0.5 ln rd ) + 0.5 (ln rd 0.5 ln ru 0.5 ln rd )
2
2
= 0.5 (0.5 ln ru 0.5 ln rd ) + 0.5 (0.5 ln rd 0.5 ln ru )
= 0.5 (0.5 ln ru 0.5 ln rd )2 + 0.5 (0.5 ln ru 0.5 ln rd )2

2
ru
2
= (0.5 ln ru 0.5 ln rd ) = 0.5 ln
rd
The standard deviation of ln Y T Mu = ln ru and ln Y T Md = ln rd is 1 =
10%.
ru
ru
= 1
ln
= 2 1
ru = rd e21 = rd e0.2
0.5 ln
rd
rd
Now we see that matching volatility requires ru = rd e2 . This relationship
holds for every node.
Next, we want to reproduce the 2-year bond price of 0.8116.
If the Year 2 interest rate is ru , then the PV of $1 discounted from t = 2 to
1
t = 0 is
(1 + ru ) (1 + r0 )
If the Year 2 interest rate is rd , then the PV of $1 discounted from t = 2 to
1
t = 0 is
(1 + rd ) (1 + r0 )
The risk-neutral probability of up and down is 0.5. The expected 2-year
bond price is:
1
1
0.5
+ 0.5
(1 + ru ) (1 + r0 )
(1 + rd ) (1 + r0 )
To match the observed price P (0, 2) = 0.8116, we have:
1
1
0.5
+ 0.5
= P (0, 2)
(1 + ru ) (1 + r0 )
(1 + rd ) (1 + r0 )
To
sum up, we have two equations:
ru = rd e21
1
1
+ 0.5
= P (0, 2)
0.5
(1 + ru ) (1 + r0 )
(1 + rd ) (1 + r0 )
Or

ru = rd e0.2
0.5

1
1
+ 0.5
= 0.8116
(1 + ru ) (1 + 0.1)
(1 + rd ) (1 + 0.1)

1
1
+
= 2 0.8116 (1 + 0.1)
1 + rd 1 + rd e0.2
rd = 0.108 265 = 10.83%

286

CHAPTER 24. INTEREST RATE MODELS

ru = 0.108 265e0.2 = 0.132 2 = 13.22%


We can use the same procedure to calculate ruu , rud , rdu , rdd . To simplify
our model, we arbitrary set rud = rdu (i.e. we assume the tree is recombining).
t=0
t=h=1
t = 2h = 2
ruu = rdd e42
ru = 13.22%
r0 = 10%
rud = rdu = rdd e22
rd = 10.83%
rdd
We want to choose rdd such that the standard deviation of the natural log
of the YTM on a 2-year bond issued at t = 1 and maturing at t = 3 is 15%.
First, we calculate P (1, 3), the price of a 2-year bond issued at t = 1 and
maturing at t = 3.
If the 2nd year interest rate is ru = 13.22%, then the 3rd year rate is either
ruu or rud with equal risk-neutral probability of 0.5. Then the expected PV of
$1 discounted from t = 3 to t = 1 is
1
1
+ 0.5
P (1, 3, ru ) = 0.5
(1
+
r
)
(1
+
r
)
(1
+
r
)
u
uu
u (1 + rud )

1
0.5
1
+
=
1.1322 1 + rdd e42
1 + rdd e22
The YTM can solved as follows:
P (1, 3, ru ) = (1 + i)2
i = P (1, 3, ru )0.5 1
If the 2nd year interest rate is rd = 10.83%, then the 3rd year rate is either
rdu or rdd with equal risk-neutral probability of 0.5. Then the expected PV of
$1 discounted from t = 3 to t = 1 is
1
1
P (1, 3, rd ) = 0.5
+ 0.5
(1 + rd ) (1 + r
(1 + rd ) (1 + rdd )
du )

1
0.5
1
+
=
1.1083 1 + rdd e22
1 + rdd
The YTM can solved as follows:
2

P (1, 3, rd ) = (1 + i)

i = P (1, 3, rd )
0.5

0.5

P (1, 3, ru )
1
0.15 = 0.5 ln
0.5
P (1, 3, rd )
1
0.5

1
0.5
1
+
1
1.1322 1 + rdd e42
1 + rdd e22
= 0.5 ln
0.5

0.5
1
1
+
1
2
2
1.1083 1 + rdd e
1 + rdd
By the way, please note that 2 is not 10%.

24.5. BLACK-DERMAN-TOY MODEL

287

We should also match the 3-year bond price P (0, 3) = 0.7118


The PV of $1 at t = 3 discounted to t = 0 is

1
if the path is 0 u uu
(1 + r0 ) (1 + ru ) (1 + ruu )

1
if the path is 0 u ud
(1 + r0 ) (1 + ru ) (1 + rud )

1
if the path is 0 d du
(1 + r0 ) (1 + rd ) (1 + rdu )

1
if the path is 0 d dd
(1 + r0 ) (1 + rd ) (1 + rdd )

Each path has a risk neutral probability of 0.25. Now we have:


0.25
0.25
+
(1 + r0 ) (1 + ru ) (1 + ruu ) (1 + r0 ) (1 + ru ) (1 + rud )
0.25
0.25
+
+
= P (0, 3)
(1 + r0 ) (1 + rd ) (1 + rdu ) (1 + r0 ) (1 + rd ) (1 + rdd )
0.25
0.25
+
(1.1) (1.1322) (1 + rdd e4 ) (1.1) (1.1322) (1 + rdd e2 )
0.25
0.25
+
+
= 0.7118
(1.1) (1.1083) (1 + rdd e2 ) (1.1) (1.1083) (1 + rdd )
Now our equations are:
0.5

1
1
0.5
+
1
1.1322 1 + rdd e42
1 + rdd e22
0.15 = 0.5 ln
0.5

1
0.5
1
+
1
1.1083 1 + rdd e22
1 + rdd
0.25
0.25
+
4
2
(1.1) (1.1322) (1 + rdd e ) (1.1) (1.1322) (1 + rdd e22 )
0.25
0.25
+
+
= 0.7118
2
2
(1.1) (1.1083) (1 + rdd e ) (1.1) (1.1083) (1 + rdd )
These equations are hard to solve manually. Special software is needed to
solve them.
2 = 0.195 0
You can verify that the solutions are: rdd = 0.0925
0.5

1
1
0.5
+
1
1.1322 1 + 0.0925e40.1950 1 + 0.0925e20.1950
= 0.149 97 =
0.5 ln

0.5
0.5
1
1
+
1
1.1083 1 + 0.0925e20.1950 1 + 0.0925
0.15

288

CHAPTER 24. INTEREST RATE MODELS

0.25
0.25
+
(1.1) (1.1322) (1 + 0.0925e40.1950 ) (1.1) (1.1322) (1 + 0.0925e20.1950 )
0.25
0.25
+
+
(1.1) (1.1083) (1 + 0.0925e20.1950 ) (1.1) (1.1083) (1 + 0.0925)
= 0.711 755 = 0.7118

Then the year 3 interest rates are:


rdd = 0.0925
rdu = rud = 0.0925e20.1950 = 0.136 6
ruu = 0.0925e40.1950 = 0.201 8
We can use the same logic and calculate the Year 4 interest rates. However,
Im not going to do the calculation because the calculation is overly intensive.
Make sure you can reproduce the Yr 2 and Yr 3 rates. Yr 2 rates can be
easily reproduced. SOA or CAS can ask you to calculate the Year 2 rates using
the BDT model. Yr 3 rates are harder. A full calculation of Year 3 rates by
hand is dicult. However, SOA or CAS can give you some partial information
on Year 3 rates and ask you to calculate the rest.

Solution to Derivatives Markets: SOA Exam


MFE and CAS Exam 3 FE
Yufeng Guo
October 26, 2009

www.actuary88.com

c
Yufeng
Guo

ii

Contents
Introduction

vii

9 Parity and other option relationships

10 Binomial option pricing I

23

11 Binomial option pricing II

91

12 Black-Scholes formula

107

13 Market making and delta hedging

125

14 Exotic options: I

149

18 Lognormal distribution

161

19 Monte Carlo simulation

177

20 Brownian motion and Itos lemma

187

21 The Black-Scholes equation

193

22 Exotic options: II

203

23 Volatility

205

24 Interest rate models

209

iii

CONTENTS

www.actuary88.com

CONTENTS

c
Yufeng
Guo

iv

Preface
This is Guos solution to Derivatives Markets (2nd edition ISBN 0-321-28030-X)
for SOA MFE or CAS Exam 3 FE. Unlike the ocial solution manual published
by Addison-Wesley, this solution manual provides solutions to both the evennumbered and odd-numbered problems for the chapters that are on the SOA
Exam MFE and CAS Exam 3 FE syllabus. Problems that are out of the scope
of the SOA Exam MFE and CAS Exam 3 FE syllabus are excluded.
Please report any errors to yufeng_guo@msn.com.
This book is the exclusive property of Yufeng Guo. Redistribution of this
book in any form is prohibited.

PREFACE

www.actuary88.com

PREFACE

c
Yufeng
Guo

vi

Introduction
Recommendations on using this solution manual:
1. Obviously, youll need to buy Derivatives Markets (2nd edition) to see the
problems.
2. Make sure you download the textbook errata from http://www.kellogg.
northwestern.edu/faculty/mcdonald/htm/typos2e_01.html

vii

CHAPTER 8. INTRODUCTION

www.actuary88.com

c
Yufeng
Guo

viii

Chapter 9

Parity and other option


relationships
Problem 9.1.
S0 = 32
C = 2.27

T = 6/12 = 0.5
K = 35
r = 0.04
= 0.06

C + P V (K) = P + S0 eT
2.27 + 35e0.04(0.5) = P + 32e0.06(0.5)

P = 5. 522 7

Problem 9.2.
S0 = 32
T = 6/12 = 0.5
K = 30
C = 4.29
P = 2.64
r = 0.04
C + P V (K) = P + S0 P V (Div)
4.29 + 30e0.04(0.5) = 2.64 + 32 P V (Div)
P V (Div) = 0.944
Problem 9.3.
S0 = 800
r = 0.05
=0
T =1
K = 815
C = 75
P = 45
a. Buy stock+ sell call+buy put=buy P V (K)
C + P V (K) = P + S0
P V (K = 815) = S0 + C + |{z}
P = 800 + (75) + 45 = 770
|{z}
|{z}
buy stock

sell call

buy put

So the position is equivalent to depositing 770 in a savings account (or buying a bond with present value equal to 770) and receiving 815 one year later.
770eR = 815
R = 0.056 8
1

CHAPTER 9. PARITY AND OTHER OPTION RELATIONSHIPS


So we earn 5.68%.
b. Buying a stock, selling a call, and buying a put is the same as depositing
P V (K) in the savings account. As a result, we should just earn the risk free
interest rate r = 0.05. However, we actually earn R = 0.056 8 > r. To arbitrage,
we "borrow low and earn high." We borrow 770 from a bank at 0.05%. We use
the borrowed 770 to finance buying a stock, selling a call, and buying a put.
Notice that the net cost of buying a stock, selling a call, and buying a put is
770.
One year later, we receive 770eR = 815. We pay the bank 770e0.05 = 809.
48. Our profit is 815 809. 48 = 5. 52 per transaction.
If we do n such transactions, well earn 5. 52n profit.
Alternative answer: we can burrow at 5% (continuously compounding) and
lend at 5.6 8% (continuously compounding), earning a risk free 0.68%. So if
we borrow $1 at time zero, our risk free profit at time one is e0.0568 e0.05 =
0.00717 3; if we borrow $770 at time zero, our risk free profit at time one is
0.00717 3770 = 5. 52. If we borrow n dollars at time zero, well earn 0.00717 3n
dollars at time one.
c. To avoid arbitrage, we need to have:
P V (K = 815) = S0 + |{z}
C + |{z}
P = 815e0.05 = 775. 25
|{z}
buy stock

sell call

buy put

C P = S0 P V (K) = 800 775. 25 = 24. 75

d. C P = S0 P V (K) = 800 KerT = 800 Ke0.05


If K = 780
C P = 800 780e0.05 = 58. 041
If K = 800
C P = 800 800e0.05 = 39. 016
If K = 820
C P = 800 820e0.05 = 19. 992
If K = 840
C P = 800 840e0.05 = 0.967
Problem 9.4.
To solve this type of problems, just use the standard put-call parity.
To avoid calculation errors, clearly identify the underlying asset.
The underlying asset is 1. We want to find the dollar cost of a put option
on this underlying.
The typical put-call parity:
C + P V (K) = P + S0 eT
C, K, P , and S0 should all be expressed in dollars. S0 is the current (dollar
price) of the underlying. So S0 = $0.95.
C = $0.0571
K = $0.93
is the internal growth rate of the underlying asset (i.e. 1). Hence = 0.04
Since K is expressed in dollars, P V (K) needs to be calculated using the
dollar risk free interest r = 0.06.
0.0571 + 0.93e0.06(1) = P + 0.95e0.04(1)
P = $0.02 02
www.actuary88.com

c
Yufeng
Guo

CHAPTER 9. PARITY AND OTHER OPTION RELATIONSHIPS


Problem 9.5.
As I explained in my study guide, dont bother memorizing the following
complex formula:

1 1
C$ (x0 , K, T ) = x0 KPf
, ,T
x0 K
Just use my approach to solve this type of problems.
Convert information to symbols:
1
The exchange rate is 95 yen per euro. Y 95 =1 or Y 1 =
95
Yen-denominated put on 1 euro with strike price Y100 has a premium Y8.763
(1 Y 100)0 =Y8.763
Whats the strike price of a euro-denominated call on 1 yen? K 1Y
Calculate the price of a euro-denominated call on 1 yen with strike price K
(K 1Y )0 = ?
1
Y1
100
1
The strike price of the corresponding euro-denominated yen call is K =
=0.01
100

1
1
1
Y1 =
(1 Y 100)0 =
(Y 8.763)

100
100
100
0

1 Y 100

1
, we have:
95

1
1
1
(Y 8.763) =
(8.763)
=9. 224 2 104
100
100
95

1

Y 1 =9. 224 2 104
100
0

Since Y 1 =

So the price of a euro-denominated call on 1 yen with strike price K =


is 9. 224 2 104

www.actuary88.com

c
Yufeng
Guo

1
100

CHAPTER 9. PARITY AND OTHER OPTION RELATIONSHIPS


Problem 9.6.
The underlying asset is 1. The standard put-call parity is:
C + P V (K) = P + S0 eT
C, K, P , and S0 should all be expressed in dollars. S0 is the current (dollar
price) of the underlying.
is the internal growth rate of the underlying asset (i.e. 1).
Well solve Part b first.
b. 0.0404 + 0.9e0.05(0.5) = 0.0141 + S0 e0.035(0.5)
S0 = $0.920 04
So the current price of the underlying (i.e. 1) is S0 = $0.920 04. In other
words, the currency exchange rate is $0.920 04 =1
a. According to the textbook Equation 5.7, the forward price is:
F0,T = S0 eT erT = 0.920 04e0.035(0.5) e0.05(0.5) = $0.926 97
Problem 9.7.
The underlying asset is one yen.
a. C + KerT = P + S0 eT
0.0006 + 0.009e0.05(1) = P + 0.009e0.01(1)
0.0006 + 0.008561 = P + 0.008 91
P = $0.00025

b. There are two puts out there. One is the synthetically created put using
the formula:
P = C + KerT S0 eT
The other is the put in the market selling for the price for $0.0004.
To arbitrage, build a put a low cost and sell it at a high price. At t = 0, we:
Sell the expensive put for $0.0004
Build a cheap put for $0.00025. To build a put, we buy a call, deposit
KerT in a savings account, and sell eT unit of Yen.

Sell expensive put


Buy call
Deposit KerT in savings
Short sell eT unit of Yen
Total

t=0
0.0004
0.0006
0.009e0.05(1)
0.009e0.01(1)
$0.00015

T =1
ST < 0.009
ST 0.009
0
0.009
ST
0

T =1
ST 0.009
0
ST 0.009
0.009
ST
0

0.0004 0.0006 0.009e0.05(1) + 0.009e0.01(1) = $0.00015


www.actuary88.com

c
Yufeng
Guo

CHAPTER 9. PARITY AND OTHER OPTION RELATIONSHIPS


At t = 0, we receive $0.00015 yet we dont incur any liabilities at T = 1 (so
we receive $0.00015 free money at t = 0).
c. At-the-money means K = S0 (i.e. the strike price is equal to the current
exchange rate).
Dollar-denominated at-the-money yen call sells for $0.0006. To translate this
into symbols, notice that under the call option, the call holder can give $0.009
and get Y 1.
"Give $0.009 and get Y 1" is represented by ($0.009 Y 1). This options
premium at time zero is $0.0006. Hence we have:
($0.009 Y 1)0 = $0.0006
We are asked to find the yen denominated at the money call for $1. Here
the call holder can give c yen and get $1. "Give c yen and get $1" is represented
by (Y c $1). This options premium at time zero is (Y c $1)0 .
First, we need to calculate c, the strike price of the yen denominated dollar
1
call. Since at time zero $0.009 = Y 1, we have $1 = Y
. So the at-the0.009
1
money yen denominated call on $1 is c =
. Our task is to find this options
0.009

1
$1 =?
premium: Y
0.009
0
Well find the premium for Y 1 $0.009, the option of "give 1 yen and get
$0.009." Once we find this premium, well scale it and find the premium of "give
1
yen and get $1."
0.009
Well use the general put-call parity:
(AT BT )0 + P V (AT ) = (BT AT )0 + P V (BT )
($0.009 Y 1)0 + P V ($0.009) = (Y 1 $0.009)0 + P V (Y 1)
P V ($0.009) = $0.009e0.05(1)
Since we are discounting $0.009 at T = 1 to time zero, we use the dollar
interest rate 5%.
P V (Y 1) = $0.009e0.01(1)
If we discount Y1 from T = 1 to time zero, we get e0.01(1) yen, which is
equal to $0.009e0.01(1) .
So we have:
$0.0006+$0.009e0.05 = (Y 1 $0.009)0 + $0.009e0.01(1)
(Y
= $2. 506 16 104
1 $0.009)0
1
1
2. 506 16 104
=
Y 1 $1
(Y 1 $0.009)0 = $
= $2.
0.009
0.009
0.009
0
2
2. 784 62 10
= Y 3. 094
784 62 102 = Y
0.009
www.actuary88.com

c
Yufeng
Guo

CHAPTER 9. PARITY AND OTHER OPTION RELATIONSHIPS


So the yen denominated at the money call for $1 is worth $2. 784 62 102
or Y 3. 094.
We are also asked to identify the relationship between the yen denominated
at the money call for $1 and the dollar-denominated yen put. The relationship
is that we use the premium of the latter option to calculate the premium of the
former option.
Next, we calculate the premium for the yen denominated at-the-money put
for $1:

$Y

1
0.009

1
($0.009 Y 1)0
0.009

1
=
$0.0006 = $ 0.0 666 7
0.009
1
= Y 7. 407 8
= Y 0.0 666 7
0.009
So the yen denominated at-the-money put for $1 is worth $ 0.0 666 7 or Y
7. 407 8.
I recommend that you use my solution approach, which is less prone to errors
than using complex notations and formulas in the textbook.
Problem 9.8.
The textbook Equations 9.13 and 9.14 are violated.
This is how to arbitrage on the calls. We have two otherwise identical
calls, one with $50 strike price and the other $55. The $50 strike call is more
valuable than the $55 strike call, but the former is selling less than the latter.
To arbitrage, buy low and sell high.
We use T to represent the common exercise date. This definition works
whether the two options are American or European. If the two options are
American, well find arbitrage opportunities if two American options are exercised simultaneously. If the two options are European, T is the common
expiration date.
The payo is:
Transaction
Buy 50 strike call
Sell 55 strike call
Total

t=0
9
10
1

T
ST < 50
0
0
0

T
50 ST < 55
ST 50
0
ST 50 0

T
ST 55
ST 50
(ST 55)
5

At t = 0, we receive $1 free money.


At T , we get non negative cash flows (so we may get some free money, but
we certainly dont owe anybody anything at T ). This is clearly an arbitrage.
www.actuary88.com

c
Yufeng
Guo

CHAPTER 9. PARITY AND OTHER OPTION RELATIONSHIPS


This is how to arbitrage on the two puts. We have two otherwise identical
puts, one with $50 strike price and the other $55. The $55 strike put is more
valuable than the $50 strike put, but the former is selling less than the latter.
To arbitrage, buy low and sell high.
The payo is:
T
T
T
Transaction
t = 0 ST < 50
50 ST < 55 ST 55
Buy 55 strike put 6
55 ST
55 ST
0
Sell 50 strike put 7
(50 ST ) 0
0
Total
1
5
55 ST > 0
0
At t = 0, we receive $1 free money.
At T , we get non negative cash flows (so we may get some free money, but
we certainly dont owe anybody anything at T ). This is clearly an arbitrage.
Problem 9.9.
The textbook Equation 9.15 and 9.16 are violated.
We use T to represent the common exercise date. This definition works
whether the two options are American or European. If the two options are
American, well find arbitrage opportunities if two American options are exercised simultaneously. If the two options are European, T is the common
expiration date.
This is how to arbitrage on the calls. We have two otherwise identical calls,
one with $50 strike price and the other $55. The premium dierence between
these two options should not exceed the strike dierence 15 10 = 5. In other
words, the 50-strike call should sell no more than 10 + 5. However, the 50-strike
call is currently selling for 16 in the market. To arbitrage, buy low (the 55-strike
call) and sell high (the 50-strike call).
The $50 strike call is more valuable than the $55 strike call,
is selling less than the latter.
The payo is:
T
T
Transaction
t = 0 ST < 50 50 ST < 55
Buy 55 strike call 10
0
0
Sell 50 strike call 16
0
(ST 50)
Total
6
0
(ST 50) 5

but the former

T
ST 55
ST 55
(ST 50)
5

So we receive $6 at t = 0. Then at T , our maximum liability is $5. So make


at least $1 free money.
This is how to arbitrage on the puts. We have two otherwise identical calls,
one with $50 strike price and the other $55. The premium dierence between
these two options should not exceed the strike dierence 15 10 = 5. In other
www.actuary88.com

c
Yufeng
Guo

CHAPTER 9. PARITY AND OTHER OPTION RELATIONSHIPS


words, the 55-strike put should sell no more than 7 + 5 = 12. However, the
55-strike put is currently selling for 14 in the market. To arbitrage, buy low
(the 50-strike put) and sell high (the 55-strike put).
The payo is:
T
T
T
Transaction
t = 0 ST < 50
50 ST < 55
ST 55
Buy 50 strike put 14
50 ST
0
0
Sell 55 strike put 7
(55 ST ) (55 ST )
0
Total
7
5
(55 ST ) < 5 0
So we receive $7 at t = 0. Then at T , our maximum liability is $5. So make
at least $2 free money.
Problem 9.10.
Suppose there are 3 options otherwise identical but with dierent strike price
K1 < K2 < K3 where K2 = K1 + (1 ) K2 and 0 < < 1.
Then the price of the middle strike price K2 must not exceed the price of a
diversified portfolio consisting of units of K1 -strike option and (1 ) units
of K2 -strike option:
C [K1 + (1 ) K3 ] C (K1 ) + (1 ) C (K3 )
P [K1 + (1 ) K3 ] P (K1 ) + (1 ) P (K3 )
The above conditions are called the convexity of the option price with respect
to the strike price. They are equivalent to the textbook Equation 9.17 and 9.18.
If the above conditions are violated, arbitrage opportunities exist.
We are given the following 3 calls:
Strike
K1 = 50 K2 = 55
Call premium 18
14

K3 = 60
9.50

50 + (1 ) 60 = 55
= 0.5
0.5 (50) + 0.5 (60) = 55
Lets check:
C [0.5 (50) + 0.5 (60)] = C (55) = 14
0.5C (50) + 0.5C (60) = 0.5 (18) + 0.5 (9.50) = 13. 75
C [0.5 (50) + 0.5 (60)] > 0.5C (50) + 0.5C (60)
So arbitrage opportunities exist. To arbitrage, we buy low and sell high.
The cheap asset is the diversified portfolio consisting of units of K1 -strike
option and (1 ) units of K3 -strike option. In this problem, the diversified
portfolio consists of half a 50-strike call and half a 60-strike call.
The expensive asset is the 55-strike call.
www.actuary88.com

c
Yufeng
Guo

CHAPTER 9. PARITY AND OTHER OPTION RELATIONSHIPS


Since we cant buy half a call option, well buy 2 units of the portfolio (i.e.
buy one 50-strike call and one 60-strike call). Simultaneously,we sell two 55strike call options.
We use T to represent the common exercise date. This definition works
whether the options are American or European. If the options are American,
well find arbitrage opportunities if the American options are exercised simultaneously. If the options are European, T is the common expiration date.
The payo is:
T
ST < 50

T
50 ST < 55

T
55 ST < 60

T
ST 60

18
9.5
27. 5

0
0
0

ST 50
0
ST 50

ST 50
0
ST 50

ST 50
ST 60
2ST 110

2 (14) = 28
0.5

0
0

0
ST 50 0

2 (ST 55)
60 ST > 0

2 (ST 55)
0

Transaction
buy two portfolios
buy a 50-strike call
buy a 60-strike call
Portfolio total

t=0

Sell two 55-strike calls


Total

27. 5 + 28 = 0.5
ST 50 2 (ST 55) = 60 ST
2ST 110 2 (ST 55) = 0
So we get $0.5 at t = 0, yet we have non negative cash flows at the expiration
date T . This is arbitrage.
The above strategy of buying units of K1 -strike call, buying (1 ) units
of K3 -strike call, and selling one unit of K2 -strike call is called the butterfly
spread.
We are given the following 3 puts:
Strike
K1 = 50 K2 = 55
Put premium 7
10.75

K3 = 60
14.45

50 + (1 ) 60 = 55
= 0.5
0.5 (50) + 0.5 (60) = 55
Lets check:
P [0.5 (50) + 0.5 (60)] = P (55) = 10.75
0.5P (50) + 0.5P (60) = 0.5 (7) + 0.5 (14.45) = 10. 725
P [0.5 (50) + 0.5 (60)] > .5P (50) + 0.5P (60)
So arbitrage opportunities exist. To arbitrage, we buy low and sell high.
The cheap asset is the diversified portfolio consisting of units of K1 -strike
put and (1 ) units of K3 -strike put. In this problem, the diversified portfolio
consists of half a 50-strike put and half a 60-strike put.
The expensive asset is the 55-strike put.
www.actuary88.com

c
Yufeng
Guo

CHAPTER 9. PARITY AND OTHER OPTION RELATIONSHIPS


Since we cant buy half a option, well buy 2 units of the portfolio (i.e. buy
one 50-strike put and one 60-strike put). Simultaneously,we sell two 55-strike
put options.
The payo is:

Transaction
buy two portfolios
buy a 50-strike put
buy a 60-strike put
Portfolio total

t=0

T
ST < 50

T
50 ST < 55

T
55 ST < 60

T
ST 60

7
14.45
21. 45

50 ST
60 ST
110 2ST

0
60 ST
60 ST

0
60 ST
60 ST

0
0
0

2 (55 ST )
0

2 (55 ST )
ST 50 0

0
60 ST > 0

0
0

Sell two 55-strike puts 2 (10.75)


Total
0.05
21. 45 + 2 (10.75) = 0.05
50 ST + 60 ST = 110 2ST

21. 45 + 2 (10.75) = 0.05


110 2ST 2 (55 ST ) = 0
60 ST 2 (55 ST ) = ST 50
So we get $0.05 at t = 0, yet we have non negative cash flows at the expiration
date T . This is arbitrage.
The above strategy of buying units of K1 -strike put, buying (1 ) units
of K3 -strike put, and selling one unit of K2 -strike put is also called the butterfly
spread.

Problem 9.11.
This is similar to Problem 9.10.
We are given the following 3 calls:
Strike
K1 = 80 K2 = 100
Call premium 22
9

K3 = 105
5

80 + 105 (1 ) = 100
= 0.2
0.2 (80) + 0.8 (105) = 100
C [0.2 (80) + 0.8 (105)] = C (100) = 9
0.2C (80) + 0.8C (105) = 0.2 (22) + 0.8 (5) = 8. 4
C [0.2 (80) + 0.8 (105)] > 0.2C (80) + 0.8C (105)
So arbitrage opportunities exist. To arbitrage, we buy low and sell high.
The cheap asset is the diversified portfolio consisting of units of K1 -strike
option and (1 ) units of K3 -strike option. In this problem, the diversified
portfolio consists of 0.2 unit of 80-strike call and 0.8 unit of 105-strike call.
The expensive asset is the 100-strike call.
www.actuary88.com

c
Yufeng
Guo

10

CHAPTER 9. PARITY AND OTHER OPTION RELATIONSHIPS


Since we cant buy a fraction of a call option, well buy 10 units of the portfolio (i.e. buy two 80-strike calls and eight 105-strike calls). Simultaneously,we
sell ten 100-strike call options.
We use T to represent the common exercise date. This definition works
whether the options are American or European. If the options are American,
well find arbitrage opportunities if the American options are exercised simultaneously. If the options are European, T is the common expiration date.
The payo is:
T
ST < 80

T
80 ST < 100

Transaction
buy ten portfolios
buy two 80-strike calls
buy eight 105-strike calls
Portfolio total

t=0
2 (22)
8 (5)
84

0
0
0

2 (ST 80)
0
2 (ST 80)

Sell ten 100-strike calls


Total

10 (9)
6

0
0

0
2 (ST 80) 0

Transaction
buy ten portfolios
buy two 80-strike calls
buy eight 105-strike calls
Portfolio total

t=0

T
100 ST < 105

T
ST 105

2 (22)
8 (5)
84

2 (ST 80)
0
2 (ST 80)

2 (ST 80)
8 (ST 105)
10ST 1000

10 (ST 100)
8 (105 ST ) > 0

10 (ST 100)
0

Sell ten 100-strike calls


10 (9)
Total
6
2 (22) 8 (5) = 44 40 = 84

84 + 10 (9) = 84 + 90 = 6
2 (ST 80) + 8 (ST 105) = 10ST 1000
2 (ST 80) 10 (ST 100) = 840 8ST = 8 (105 ST )
10ST 1000 10 (ST 100) = 0
So we receive $6 at t = 0, yet we dont incur any negative cash flows at
expiration T . So we make at least $6 free money.
We are given the following 3 put:
Strike
K1 = 80 K2 = 100
Put premium 4
21

K3 = 105
24.8

80 + 105 (1 ) = 100
= 0.2
0.2 (80) + 0.8 (105) = 100
P [0.2 (80) + 0.8 (105)] = P (100) = 21
0.2P (80) + 0.8P (105) = 0.2 (4) + 0.8 (24.8) = 20. 64
www.actuary88.com

c
Yufeng
Guo

11

CHAPTER 9. PARITY AND OTHER OPTION RELATIONSHIPS


P [0.2 (80) + 0.8 (105)] > 0.2P (80) + 0.8P (105)
So arbitrage opportunities exist. To arbitrage, we buy low and sell high.
The cheap asset is the diversified portfolio consisting of units of K1 -strike
option and (1 ) units of K3 -strike option. In this problem, the diversified
portfolio consists of 0.2 unit of 80-strike put and 0.8 unit of 105-strike put.
The expensive asset is the 100-strike put.
Since we cant buy half a fraction of an option, well buy 10 units of the portfolio (i.e. buy two 80-strike puts and eight 105-strike puts). Simultaneously,we
sell ten 100-strike put options.
The payo is:

Transaction
buy ten portfolios
buy two 80-strike puts
buy eight 105-strike puts
Portfolio total

t=0

T
ST < 80

T
80 ST < 100

2 (4)
8 (24.8)
84

2 (80 ST )
8 (105 ST )
1000 10ST

0
8 (105 ST )
8 (105 ST )

Sell ten 100-strike puts


Total

10 (21)
3. 6

10 (100 ST )
0

10 (100 ST )
2 (ST 80) 0

Transaction
buy ten portfolios
buy two 80-strike puts
buy eight 105-strike puts
Portfolio total

t=0

T
100 ST < 105

T
ST 105

2 (4)
8 (24.8)
84

0
8 (105 ST )
8 (105 ST )

0
0
0

Sell ten 100-strike puts


Total

10 (21)
3. 6

0
8 (105 ST ) > 0

0
0

2 (4) 8 (24.8) = 206. 4


2 (80 ST ) + 8 (105 ST ) = 1000 10ST
206. 4 + 10 (21) = 3. 6
1000 10ST 10 (100 ST ) = 0
8 (105 ST ) 10 (100 ST ) = 2 (ST 80)
We receive $3. 6 at t = 0, but we dont incur any negative cash flows at T .
So we make at least $3. 6 free money.
Problem 9.12.
For two European options diering only in strike price, the following conditions must be met to avoid arbitrage (see my study guide for explanation):
0 CEur (K1 , T ) CEur (K2 , T ) P V (K2 K1 ) if K1 < K2
www.actuary88.com

c
Yufeng
Guo

12

CHAPTER 9. PARITY AND OTHER OPTION RELATIONSHIPS


0 PEur (K2 , T ) PEur (K1 , T ) P V (K2 K1 ) if K1 < K2
a.
Strike
Call premium

K1 = 90
10

K2 = 95
4

C (K1 ) C (K2 ) = 10 4 = 6
K2 K1 = 95 90 = 5
C (K1 ) C (K2 ) > K2 K1 P V (K2 K1 )
Arbitrage opportunities exist.
To arbitrage, we buy low and sell high. The cheap call is the 95-strike call;
the expensive call is the 90-strike call.
We use T to represent the common exercise date. This definition works
whether the two options are American or European. If the two options are
American, well find arbitrage opportunities if two American options are exercised simultaneously. If the two options are European, T is the common
expiration date.
The payo is:
T
T
T
Transaction
t = 0 ST < 90 90 ST < 95
ST 95
Buy 95 strike call 4
0
0
ST 95
Sell 90 strike call 10
0
(ST 90)
(ST 90)
Total
6
0
(ST 90) 5 5
We receive $6 at t = 0, yet we our max liability at T is 5. So well make
at least $1 free money.
b.
T =2
r = 0.1
Strike
K1 = 90
Call premium 10

K2 = 95
5.25

C (K1 ) C (K2 ) = 10 5.25 = 4. 75


K2 K1 = 95 90 = 5
P V (K2 K1 ) = 5e0.1(2) = 4. 094
C (K1 ) C (K2 ) > P V (K2 K1 )
Arbitrage opportunities exist.
Once again, we buy low and sell high. The cheap call is the 95-strike call;
the expensive call is the 90-strike call.
The payo is:
T
T
T
Transaction
t=0
ST < 90
90 ST < 95
ST 95
Buy 95 strike call
5.25 0
0
ST 95
Sell 90 strike call
10
0
(ST 90)
(ST 90)
0.1(2)
0.1(2)
Deposit 4. 75 in savings 4. 75 4. 75e
4. 75e
4. 75e0.1(2)
Total
0
5. 80
95. 80 ST > 0 0.80
www.actuary88.com

c
Yufeng
Guo

13

CHAPTER 9. PARITY AND OTHER OPTION RELATIONSHIPS


4. 75e0.1(2) = 5. 80
(ST 90) + 4. 75e0.1(2) = 95. 80 ST
ST 95 (ST 90) + 4. 75e0.1(2) = 0.80
Our initial cost is zero. However, our payo is always non-negative. So we
never lose money. This is clearly an arbitrage.
Its important that the two calls are European options. If they are American,
they can be exercised at dierent dates. Hence the following non-arbitrage
conditions work only for European options:
0 CEur (K1 , T ) CEur (K2 , T ) P V (K2 K1 ) if K1 < K2
0 PEur (K2 , T ) PEur (K1 , T ) P V (K2 K1 ) if K1 < K2
c.
We are given the following 3 calls:
Strike
K1 = 90 K2 = 100
Call premium 15
10

K3 = 105
6

1
90 + (1 ) 105 = 100
=
3
1
2
(90) + (105) = 100
3

3
2
1
(90) + (105) = C (100) = 10
C
3
3
2
1
2
1
C (90) + C (105) = (15) + (6) = 9
3
3
3
3
2
1
2
1
(90) + (105) > C (90) + C (105)
C
3
3
3
3
Hence arbitrage opportunities exist. To arbitrage, we buy low and sell high.
1
The cheap asset is the diversified portfolio consisting of unit of 90-strike
3
2
call and unit of 105-strike call.
3
The expensive asset is the 100-strike call.
Since we cant buy a partial option, well buy 3 units of the portfolio (i.e.
buy one 90-strike call and two 105-strike calls). Simultaneously,we sell three
100-strike calls.
The payo at expiration T :
T
ST < 90

T
90 ST < 100

T
100 ST < 105

T
ST 105

15
2 (6)
27

0
0
0

ST 90
0
ST 90

ST 90
0
ST 90

ST 90
2 (ST 105)
3ST 300

3 (10)
3

0
0

0
ST 90 0

3 (ST 100)
2 (105 ST ) > 0

3 (ST 100)
0

Transaction
buy 3 portfolios
buy one 90-strike call
buy two 105-strike calls
Portfolios total

t=0

Sell three 100-strike calls


Total
www.actuary88.com

c
Yufeng
Guo

14

CHAPTER 9. PARITY AND OTHER OPTION RELATIONSHIPS


15 + 2 (6) = 27
ST 90 + 2 (ST 105) = 3ST 300
27 + 3 (10) = 3
ST 90 3 (ST 100) = 210 2ST = 2 (105 ST )
3ST 300 3 (ST 100) = 0
So we receive $3 at t = 0, but we incur no negative payo at T . So well
make at least $3 free money.
Problem 9.13.
a. If the stock pays dividend, then early exercise of an American call option
may be optimal.
Suppose the stock pays dividend at tD .
Time 0 ... ... tD ... ... T
Pro and con for exercising the call early at tD .
+. If you exercise the call immediately before tD , youll receive dividend
and earn interest during [tD , T ]
. Youll pay the strike price K at tD , losing interest you could have
earned during [tD , T ]. If the interest rate, however, is zero, you wont lose
any interest.
. You throw away the remaining call option during [tD , T ]. Had you
waited, you would have the call option during [tD , T ]
If the accumulated value of the dividend exceeds the value of the remaining
call option, then its optimal to exercise the stock at tD .
As explained in my study guide, its never optimal to exercise an American
put early if the interest rate is zero.
Problem 9.14.
a. The only reason that early exercise might be optimal is that the underlying
asset pays a dividend. If the underlying asset doesnt pay dividend, then its
never optimal to exercise an American call early. Since Apple doesnt pay
dividend, its never optimal to exercise early.
b. The only reason to exercise an American put early is to earn interest on
the strike price. The strike price in this example is one share of AOL stock.
Since AOL stocks wont pay any dividends, theres no benefit for owning an
AOL stock early. Thus its never optimal to exercise the put.
If the Apple stock price goes to zero and will always stay zero, then theres
no benefit for delaying exercising the put; theres no benefit for exercising the
www.actuary88.com

c
Yufeng
Guo

15

CHAPTER 9. PARITY AND OTHER OPTION RELATIONSHIPS


put early either (since AOL stocks wont pay dividend). Exercising the put
early and exercising the put at maturity have the same value.
If, however, the Apple stock price goes to zero now but may go up in the
future, then its never optimal to exercise the put early. If you dont exercise
early, you leave the door open that in the future the Apple stock price may
exceed the AOL stock price, in which case you just let your put expire worthless.
If the Apple stock price wont exceed the AOL stock price, you can always
exercise the put and exchange one Apple stock for one AOL stock. Theres no
hurry to exercise the put early.
c. If Apple is expected to pay dividend, then it might be optimal to exercise
the American call early and exchange one AOL stock for one Apple stock.
However, as long as the AOL stock wont pay any dividend, its never optimal
to exercise the American put early to exchange one Apple stock for one AOL
stock.
Problem 9.15.
This is an example where the strike price grows over time.
If the strike price grows over time, the longer-lived
valuable as the shorter lived option. Refer to Derivatives
We have two European calls:
Call #1 K1 = 100e0.05(1.5) = 107. 788 T1 = 1.5
Call #2 K2 = 100e0.05 = 105. 127
T2 = 1

option is at least as
Markets Page 298.
C1 = 11.50
C2 = 11.924

The longer-lived call is cheaper than the shorter-lived call, leading to arbitrage opportunities. To arbitrage, we buy low (Call #1) and sell high (Call
#2).
The payo at expiration T1 = 1.5 if ST2 < 100e0.05 = 105. 127
Transaction
Sell Call #2
buy Call #1
Total

t=0
11.924
11.50
0.424

T2
0

T1
ST1 < 100e0.05(1.5)
0
0
0

T1
ST1 100e0.05(1.5)
0
ST1 100e0.05(1.5)
ST1 100e0.05(1.5) 0

We receive $0.424 at t = 0, yet our payo at T1 is always non-negative. This


is clearly an arbitrage.
The payo at expiration T1 = 1.5 if ST2 100e0.05 = 105. 127
Transaction
Sell Call #2
buy Call #1
Total
www.actuary88.com

t=0
11.924
11.50
0.424

T2
100e0.05 ST2

c
Yufeng
Guo

T1
ST1 < 100e0.05(1.5)
100e0.05(1.5) ST1
0
100e0.05(1.5) ST1 < 0

T1
ST1 100e0.05(1.5)
100e0.05(1.5) ST1
ST1 100e0.05(1.5)
0
16

CHAPTER 9. PARITY AND OTHER OPTION RELATIONSHIPS


If ST2 100e0.05 , then payo of the sold Call #2 at T2 is 100e0.05 ST2 .
From T2 to T1 ,

100e0.05 grows into 100e0.05 e0.05(T1 T2 ) = 100e0.05 e0.05(0.5) = 100e0.05(1.5)


ST2 becomes ST1 (i.e. the stock price changes from ST2 to ST1 )

We receive $0.424 at t = 0, yet our payo at T1 can be negative. This is not


an arbitrage.
So as long as ST2 < 100e0.05 = 105. 127 , therell be arbitrage opportunities.
Problem 9.16.
Suppose we do the following at t = 0:
1. Pay C a to buy a call
2. Lend P V (K) = KerL at rL
3. Sell a put, receiving P b
4. Short sell one stock, receiving S0b
The net cost is P b + S0b (C a + KerL ).
The payo at T is:
Transactions
Buy a call
Lend KerL at rL
Sell a put
Short sell one stock
Total

t=0
C a
KerL
Pb
S0b
P b + S0b (C a + KerL )

If ST < K

If ST K

0
K
ST K
ST
0

ST K
K
0
ST
0

The payo is always zero. To avoid arbitrage, we need to have


P b + S0b (C a + KerL ) 0
Similarly, we can do the following at t = 0:
1. Sell a call, receiving C b
2. Borrow P V (K) = KerB at rB
3. Buy a put, paying P a
4. Buy one stock, paying S0a
www.actuary88.com

c
Yufeng
Guo

17

CHAPTER 9. PARITY AND OTHER OPTION RELATIONSHIPS

The net cost is C b + KerB P b + S0b .


The payo at T is:
Transactions
Sell a call
Borrow KerB at rB
Buy a put
Buy one stock
Total

t=0
Cb
KerB
P a
a
S
0b

C + KerB P b + S0b

If ST < K

If ST K

0
K
K ST
ST
0

K ST
K
0
ST
0

The
b payoris always
zero. To avoid arbitrage, we need to have
C + Ke B P b + S0b 0
Problem 9.17.
a. According to the put-call parity, the payo of the following position is
always zero:
1. Buy the call
2. Sell the put
3. Short the stock
4. Lend the present value of the strike price plus dividend
The existence of the bid-ask spread and the borrowing-lending rate dierence
doesnt change the zero payo of the above position. The above position always
has a zero payo whether theres a bid-ask spread or a dierence between the
borrowing rate and the lending rate.
If there is no transaction cost such as a bid-ask spread, the initial gain of
the above position is zero. However, if there is a bid-ask spread, then to avoid
arbitrage, the initial gain of the above position should be zero or negative.
The
b initial
gain of the position is:
P + S0b [C a + P VrL (K) + P VrL (Div)]
Theres
no
b
arbitrage if
P + S0b [C a + P VrL (K) + P VrL (Div)] 0
In this problem, we are given
rL = 0.019
rB = 0.02
S0b = 84.85. We are told to ignore the transaction cost. In addition, we
are given that the current stock price is 84.85. So S0b = 84.85.
The dividend is 0.18 on November 8, 2004.
www.actuary88.com

c
Yufeng
Guo

18

CHAPTER 9. PARITY AND OTHER OPTION RELATIONSHIPS


To find the expiration date, you need to know this detail. Puts and calls
are called equity options at the Chicago Board of Exchange (CBOE). In CBOE,
the expiration date of an equity option is the Saturday immediately following
the third Friday of the expiration month. (To verify this, go to www.cboe.com.
Click on "Products" and read "Production Specifications.")
If the expiration month is November, 2004, the third Friday is November 19.
Then the expiration date is November 20.
11/20/2004 10/15/2004
36
T =
=
= 0.09863
365
365
If the expiration month is January, 2005, the third Friday is January 21.
Then the expiration date is January 22, 2005.
1/22/2005 10/15/2004
T =
365
Calculate the days between 1/22/2005 and10/15/2004 isnt easy. Fortunately,we can use a calculator. BA II Plus and BA II Plus Professional have
"Date" Worksheet. When using Date Worksheet, use the ACT mode. ACT
mode calculates the actual days between two dates. If you use the 360 day
mode, you are assuming that there are 360 days between two dates.
When using the date worksheet, set DT1 (i.e. Date 1) as October 10, 2004
by entering 10.1504; set DT2 (i.e. Date 2) as January 22, 2004 by entering
1.2204. The calculator should tell you that DBD=99 (i.e. the days between two
days is 99 days).
1/22/2005 10/15/2004
99
So T =
=
= 0.271 23
365
365
If you have trouble using the date worksheet, refer to the guidebook of BA
II Plus or BA II Plus Professional.
11/8/2004 10/15/2004
24
=
= 0.06 575
The dividend day is tD =
365
365
P VrL (Div) = 0.18e0.06575(0.019) = 0.18
0.019T
P
VbrL (K)
= Kea
b
P + S0 [C + P VrL (K) + P VrL (Div)]

= P b + 84.85 C a + Ke0.019T + 0.18


b

a
K T
Ca
Pb
+ P VrL (Div)]
P + S0b [C
+ P VrL (K)
75 0.0986 10.3 0.2
0.2 + 84.85 10.3 + 75e0.0190.0986 + 0.18
= 0.29
0.0190.0986
80 0.0986 5.6
0.6
0.6 + 84.85 5.6 + 80e
+ 0.18 = 0.18
85 0.0986 2.1
2.1
2.1 + 84.85 2.1 + 85e0.0190.0986 + 0.18 = 0.17
90 0.0986 0.35 5.5
5.5 + 84.85 0.35 + 90e0.0190.0986 + 0.18 = 1. 15
0.0190.271 2
75 0.271 2 10.9 0.7
0.7 + 84.85 10.9
+ 75e0.0190.271 2 + 0.18 = 0.14
80 0.271 2 6.7
1.45 1.45 + 84.85 6.7 + 80e
+ 0.18
= 0.17
85 0.271 2 3.4
3.1
3.1 + 84.85 3.4 + 85e0.0190.271 2 + 0.18 = 0.19
90 0.271 2 1.35 6.1
6.1 + 84.85 1.35 + 90e0.0190.271 2 + 0.18 = 0.12

b. According to the put-call parity, the payo of the following position is


always zero:
www.actuary88.com

c
Yufeng
Guo

19

CHAPTER 9. PARITY AND OTHER OPTION RELATIONSHIPS


1. Sell the call
2. Borrow the present value of the strike price plus dividend
3. Buy the put
4. Buy one stock
If there is transaction cost such as the bid-ask spread, then to avoid arbitrage,
the initial gain of the above position is zero. However, if there is a bid-ask spread,
the initial gain of the above position can be zero or negative.
The initial gain of the position is:
C b + P VrB (K) + P VrB (Div) (P a + S0a )
Theres no arbitrage if
C b + P VrB (K) + P VrB (Div) (P a + S0a ) 0
P VrB (Div) = 0.18e0.06575(0.02) = 0.18
P VrL (K) = Ke0.02T
C b +P VrB (K)+P VrB (Div)(P a + S0a ) = C b +Ke0.02T +0.18(P a + 84.85)
K
75
80
85
90
75
80
85
90

T
0.0986
0.0986
0.0986
0.0986
0.271 2
0.271 2
0.271 2
0.271 2

Cb
9.9
5.3
1.9
0.35
10.5
6.5
3.2
1.2

Pa
0.25
0.7
2.3
5.8
0.8
1.6
3.3
6.3

C b + P VrB (K) + P VrB (Div) (P a + S0a )


9.9 + 75e0.020.0986 + 0.18 (0.25 + 84.85) = 0.17
5.3 + 80e0.020.0986 + 0.18 (0.7 + 84.85) = 0.23
1.9 + 85e0.020.0986 + 0.18 (2.3 + 84.85) = 0.24
0.35 + 90e0.020.0986 + 0.18 (5.8 + 84.85) = 0.30
10.5 + 75e0.020.271 2 + 0.18 (0.8 + 84.85) = 0.38
6.5 + 80e0.020.271 2 + 0.18 (1.6 + 84.85) = 0.20
3.2 + 85e0.020.271 2 + 0.18 (3.3 + 84.85) = 0.23
1.2 + 90e0.020.271 2 + 0.18 (6.3 + 84.85) = 0.26

Problem 9.18.
Suppose there are 3 options otherwise identical but with dierent strike price
K1 < K2 < K3 where K2 = K1 + (1 ) K2 and 0 < < 1.
Then the price of the middle strike price K2 must not exceed the price of a
diversified portfolio consisting of units of K1 -strike option and (1 ) units
of K2 -strike option:
C [K1 + (1 ) K3 ] C (K1 ) + (1 ) C (K3 )
P [K1 + (1 ) K3 ] P (K1 ) + (1 ) P (K3 )
The above conditions are called the convexity of the option price with respect
to the strike price. They are equivalent to the textbook Equation 9.17 and 9.18.
If the above conditions are violated, arbitrage opportunities exist.
K
80
85
90

T
0.271 2
0.271 2
0.271 2

Cb
6.5
3.2
1.2

www.actuary88.com

Ca
6.7
3.4
1.35
c
Yufeng
Guo

20

CHAPTER 9. PARITY AND OTHER OPTION RELATIONSHIPS


85 = (80) + (1 ) (90)
= 0.5
a.
If we buy a 80-strike call, buy a 90-strike call, sell two 85-strike calls
A 80-strike call and a 90-strike call form a diversified portfolio of calls,
which is always as good as two 85-strike calls
So the cost of buying a 80-strike call and a 90-strike call can never be less
than the revenue of selling two 85-strike calls
What we pay if we buy a 80-strike call and a 90-strike call: 6.7 + 1.35 = 8.
05
What we get if we sell two 85-strike calls: 3.2 2 = 6. 4
8. 05 > 6. 4
So the convexity condition is met.
I recommend that you dont bother memorizing textbook Equation 9.17 and
9.18.
b. If we sell a 80-strike call, sell a 90-strike call, buy two 85-strike calls
A 80-strike call and a 90-strike call form a diversified portfolio of calls,
which is always as good as two 85-strike calls
So the revenue of selling a 80-strike call and a 90-strike call should never
be less than the cost of buying two 85-strike calls.
What we get if we sell a 80-strike call and a 90-strike call: 6.5 + 1.2 = 7. 7
What we pay if we buy two 85-strike calls: 3.4 2 = 6. 8
7. 7 > 6.8
So the convexity condition is met.
c. To avoid arbitrage, the following two conditions must be met:
C [K1 + (1 ) K3 ] C (K1 ) + (1 ) C (K3 )
P [K1 + (1 ) K3 ] P (K1 ) + (1 ) P (K3 )
These conditions must be met no matter you are a market-maker or anyone
else buying or selling options, no matter you pay a bid-ask spread or not.

www.actuary88.com

c
Yufeng
Guo

21

CHAPTER 9. PARITY AND OTHER OPTION RELATIONSHIPS

www.actuary88.com

c
Yufeng
Guo

22

Chapter 10

Binomial option pricing I


Problem 10.1.
The stock price today is S = 100. The stock at T is either
Su = uS = 1.3 100 = 130
Sd = dS = 0.8 100 = 80
a. For a call, the payo at T is
Vu = max (0, Su K) = max (0, 130 105) = 25
Vd = max (0, Sd K) = max (0, 80 105) = 0
We hold a replicating portfolio (4, B) at t = 0. This portfolio will have
value Vu if the stock goes up to Su or Vd if the stock goes down to Sd . We set
up the following equations:

4130 + Be0.08(0.5) = 25
4Su + BerT = Vu

rT
4Sd + Be = Vd
480 + Be0.08(0.5) = 0
B = 38. 431 6
4 = 0.5
So the option premium is:
V = 4S + B = 0.5 100 + (38. 431 6) = 11. 568 4

b. For a put, the payo at T is either


Vu = max (0, K Su ) = max (0, 105 130) = 0
Vd = max (0, Sd K) = max (0, 105 80) = 25
23

CHAPTER 10. BINOMIAL OPTION PRICING I


We hold a replicating portfolio (4, B) at t = 0. This portfolio will have
value Vu if the stock goes up to Su or Vd if the stock goes down to Sd . We set
up the following equations:

4130 + Be0.08(0.5) = 0
4Su + BerT = Vu
rT
4Sd + Be = Vd
480 + Be0.08(0.5) = 25
B = 62. 451 3
4 = 0.5
So the option premium is:
V = 4S + B = 0.5 100 + 62. 451 3 = 12. 451 3

Problem 10.2.
The stock price today is S = 100.The stock at T
Su = uS = 1.3 100 = 130
Sd = dS = 0.8 100 = 80
a. Payo at T is either
Vu = max (0, Su K) = max (0, 130 95) = 35
Vd = max (0, Sd K) = max (0, 80 95) = 0
We hold a replicating portfolio (4, B) at t = 0. This portfolio will have
value Vu if the stock goes up to Su or Vd if the stock goes down to Sd . We set
up the following equations:

4130 + Be0.08(0.5) = 35
4Su + BerT = Vu
rT
4Sd + Be = Vd
480 + Be0.08(0.5) = 0
B = 53. 804 2
4 = 0.7

So the option premium is:


V = 4S + B = 0.7 100 + (53. 804 2) = 16. 195 8
b. There are two calls out there. One can be synthetically built by buying
0.7 share of a stock and borrowing $53. 804 2 from a bank. The other is in the
market selling for $17. To arbitrage, we buy low and sell high.
Buy low. Buy 0.7 share of a stock and borrow $53. 804 2 from a bank. Our
initial cash outgo is 0.7 100 + (53. 804 2) = 16. 195 8. This grows to
either (0.7) 13053. 804 2e0.08(0.5) = 35 if the stock goes up or (0.7) 8053.
804 2e0.08(0.5) = 0 if the stock goes down at T .
Sell high. We sell a call for 17, receiving $17 at time zero. We pay either
max (0, 130 95) = 35 if the stock goes up or max (0, 80 95) = 0 if the
stock goes down at T .
www.actuary88.com

c
Yufeng
Guo

24

CHAPTER 10. BINOMIAL OPTION PRICING I


The net cash flow at T is zero. The net cash inflow at time zero is 17
16. 195 8 = 0.804 2. So we receive $0.804 2 at time zero without incurring any
liability at T ; we have made $0.804 2 free money. Millions of investors will copy
this arbitraging strategy, which will bid down the call price from 17 to the fair
price of 16. 195 8.
c. There are two calls out there. One can be synthetically built by buying
0.7 share of a stock and borrowing $16. 195 8 from a bank. The other is in the
market selling for $15.5. To arbitrage, we buy low and sell high.
Sell high. Short sell 0.7 share of a stock and deposit $53. 804 2 in a bank.
Our initial cash inflow is 0.7 100 + (53. 804 2) = 16. 195 8. At T , we
need to buy back 0.7 share of a stock from the market to close our short
position. In addition, our bank account grows into 53. 804 2e0.08(0.5) . So
our cash inflow at T is either (0.7) 130 + 53. 804 2e0.08(0.5) = 35 if the
stock goes up or (0.7) 80 + 53. 804 2e0.08(0.5) = 0 if the stock goes down.
Buy low. We buy a call for 15.5 at time zero, paying $15.5. We get either
max (0, 130 95) = 35 if the stock goes up or max (0, 80 95) = 0 if the
stock goes down at T .
The net cash flow at T is zero. The initial cash inflow is 16. 195 8 15.5 =
0.695 8. So we receive $0.695 8 at time zero without incurring any liability
at T ; we have made $0.695 8 free money. Millions of investors will copy this
arbitraging strategy, which will bid up the call price from 15.5 to the fair price
of 16. 195 8.
Problem 10.3.
The stock price today is S = 100. The stock at T is
Su = uS = 1.3 100 = 130
Sd = dS = 0.8 100 = 80
a. Payo at T is
Vu = max (0, K Su ) = max (0, 95 130) = 0
Vd = max (0, K Sd ) = max (0, 95 80) = 15
We hold a replicating portfolio (4, B) at t = 0. This portfolio will have
value Vu if the stock goes up to Su or Vd if the stock goes down to Sd . We set
up the
equations:
following rT

4Su + Be = Vu
4130 + Be0.08(0.5) = 0
4Sd + BerT = Vd
480 + Be0.08(0.5) = 15
B = 37. 470 9
4 = 0.3
www.actuary88.com

c
Yufeng
Guo

25

CHAPTER 10. BINOMIAL OPTION PRICING I


So the option premium is:
V = 4S + B = 0.3 100 + 37. 470 9 = 7. 470 9
b. There are two puts out there. One can be synthetically built by short
selling 0.3 share of a stock and depositing $37. 470 9 in a bank. The other is in
the market selling for $8. To arbitrage, we buy low and sell high.
Buy low. Short sell 0.3 share of a stock and deposit $37. 470 9 in a bank.
Our initial cash inflow is 0.3 100 37. 470 9 = 7. 470 9. At T , we need
to buy back 0.3 share of a stock to close our short position. In addition,
our initial deposit grows to 37. 470 9e0.08(0.5) = 39. If the stock goes up,
our cash outgo at T is (0.3) 130 + 37. 470 9e0.08(0.5) = 0; if the stock goes
down to 80 at T , our cash outgo is (0.3) 80 + 37. 470 9e0.08(0.5) = 15.
Sell high. We sell a put for 8, receiving $8 at time zero. At T , we pay
either max (0, 95 130) = 0 if the stock goes up or max (0, 95 80) = 15
if the stock goes down.
The net cash flow of "buy low, sell high" is zero at T . The net cash inflow at
time zero is 8 7. 470 9 = 0.529 1. So we receive $0.529 1 at time zero without
incurring any liability at T ; we have made $0.529 1 free money. Millions of
investors will copy this arbitraging strategy, which will bid down the put price
from 8 to the fair price of 7. 470 9.
c. There are two puts out there. One can be synthetically built by short
selling 0.3 share of a stock and depositing $37. 470 9 in a bank. The other is in
the market selling for $6. To arbitrage, we buy low and sell high.
Buy low. We buy a put for $6 at time zero. At T we receive either
max (0, 95 130) = 0 if the stock goes up or max (0, 95 80) = 15 if the
stock goes down.
Sell high. At time zero we buy 0.3 share of a stock and borrow $37. 470 9
in a bank. Our initial cash inflow is 0.3 100 + 37. 470 9 = 7. 470 9. At
T , we sell 0.3 share of a stock, receiving 0.3 130 = 39 if the stock goes
up or receiving 0.3 80 = 24 if the stock goes down. In addition, we need
to pay the bank 37. 470 9e0.08(0.5) = 39. So our net cash inflow at T is
39 39 = 0 if the stock goes up or 24 39 = 15 if the stock goes down.
The net cash flow of "buy low, sell high" is zero at T . The net cash inflow at
time zero is 7. 470 9 6 = 1. 470 9. So we receive $1. 470 9 at time zero without
incurring any liability at T ; we have made $1. 470 9 free money. Millions of
investors will copy this arbitraging strategy, which will bid up the put price
from 6 to the fair price of 7. 470 9.
Problem 10.4.
www.actuary88.com

c
Yufeng
Guo

26

CHAPTER 10. BINOMIAL OPTION PRICING I


The problem doesnt say the option is European or American. Lets assume
the option is European.
T =1
n=2
period length h = T /2 = 0.5
Period 0
1
2

Suu = 100 1.32 = 169


Su = 100 (1.3) = 130 Vuu = 169 95 = 74
Vu =?
(u , Bu )
Sud = 100 (1.3) (0.8) = 104
S = 100
Vud = 104 95 = 9
V =?
(, B) =?
Sd = 100 (0.8) = 80

Vd =?
Sdd = 100 0.82 = 64
(d , Bd )
Vdd = 0
We start from right to left.
Step 1
Calculate (u , Bu ), the replicating portfolio at Node u which will
produce
the
payo
Vud and Vdd .

4u Suu + Bu erh = Vuu


4u 169 + Bu e0.08(0.5) = 74

rh
4u Sud + Bu e = Vud
4u 104 + Bu e0.08(0.5) = 9
Bu = 91. 2750
4u = 1
The premium at Node u is:
Vu = 4u Su + Bu = 1 (130) 91. 2750 = 38. 725
Step 2
Calculate(d , Bd ), the replicating portfolio at Node d which will
produce
the
payo
Vud and Vdd .

4d Sud + Bd erh = Vud


4d 104 + Bd e0.08(0.5) = 9

rh
4d Sdd + Bd e = Vdd
4d 64 + Bd e0.08(0.5) = 0
4d = 0.225 , Bd = 13. 835 4
The premium at Node d is:
Vd = 4d Sd + Bd = 0.225 (80) 13. 835 4 = 4. 164 6
Step 3
Calculate(, B), the replicating portfolio at time zero, which will
produce
the
payo
Vu and Vd .

4Su + BerT = Vu
4130 + Be0.08(0.5) = 38. 725

rT
4Sd + Be = Vd
480 + Be0.08(0.5) = 4. 164 6
4 = 0.691 2
B = 49. 127 1
The premium at time zero is:
V = 4S + B = 0.691 2 (100) 49. 127 1 = 19. 992 9
The final diagram is:
www.actuary88.com

c
Yufeng
Guo

27

CHAPTER 10. BINOMIAL OPTION PRICING I


Period 0

S = 100
V = 19. 992 9
= 0.691 2
B = $49. 127 1

Su = 100 (1.3) = 130


Vu = 38. 725
u = 1
Bu = $91. 2750

Sd = 100 (0.8) = 80
Vd = 4. 164 6
d = 0.225
Bd = $13. 835 4

Suu = 100 1.32 = 169


Vuu = 169 95 = 74

Sud = 100 (1.3) (0.8) = 104


Vud = 104 95 = 9

Sdd = 100 0.82 = 64


Vdd = 0

Problem 10.5.
This question asks us to redo the previous problem by setting the initial
stock price to 80, 90, 110, 120, and 130.
S = 80.
Period 0

1
Su = 80 (1.3) = 104
Vu =?
(u , Bu ) =?

Suu = 80 1.32 = 135. 2


Vuu = 135. 2 95 = 40. 2
Sud = 80 (1.3) (0.8) = 83. 2
Vud = 0

S = 80
V =?
(, B) =?
Sd = 80 (0.8) = 64
Vd =?
(d , Bd ) =?

Sdd = 80 0.82 = 51. 2


Vdd = 0

4u 135. 2 + Bu e0.08(0.5) = 40. 2


Bu = 61. 7980
4u = 0.773 08
4u 83. 2 + Bu e0.08(0.5) = 0
Vu = 4u Su + Bu = 0.773 08 (104) 61. 7980 = 18. 602 32

4d 83. 2 + Bd e0.08(0.5) = 0
4d = 0.0
4d 51. 2 + Bd e0.08(0.5) = 0
Vd = 4d Sd + Bd = 0 (64) 0 = 0

4104 + Be0.08(0.5) = 18. 602 32


464 + Be0.08(0.5) = 0

www.actuary88.com

Bd = 0

4 = 0.465 058

c
Yufeng
Guo

B = 28. 596 7
28

CHAPTER 10. BINOMIAL OPTION PRICING I


V = 4S + B = 0.465 058 (80) 28. 596 7 = 8. 607 94
The final diagram is:
Period 0
1
Su = 80 (1.3) = 104
Vu = 18. 602 32
u = 0.773 08
Bu = 61. 798
S = 80
V = 8. 607 94
= 0.465 058
B = 28. 596 7

www.actuary88.com

Sd = 80 (0.8) = 64
Vd = 0
d = 0
Bd = 0

Suu = 80 1.32 = 135. 2


Vuu = 135. 2 95 = 40. 2

Sud = 80 (1.3) (0.8) = 83. 2


Vud = 0

Sdd = 80 0.82 = 51. 2


Vdd = 0

c
Yufeng
Guo

29

CHAPTER 10. BINOMIAL OPTION PRICING I


S = 90.
Period 0

1
Su = 90 (1.3) = 117
Vu =?
4u =?
Bu =?

S = 90
V =?
=?
B =?

Suu = 90 1.32 = 152. 1


Vuu = 152. 1 95 = 57. 1

Sud = 90 (1.3) (0.8) = 93. 6


Vud = 0

Sd = 90 (0.8) = 72
Vd =?
d =?
Bd =?

Sdd = 90 0.82 = 57. 6


Vdd = 0

4u 152. 1 + Bu e0.08(0.5) = 57. 1


Bu = 87. 777 7
4u = 0.976 068
4u 93. 6 + Bu e0.08(0.5) = 0
Vu = 4u Su + Bu = 0.976 068 (117) 87. 777 7 = 26. 422 26

4d 93. 6 + Bd e0.08(0.5) = 0
4d = 0.0
4d 57. 6 + Bd e0.08(0.5) = 0
Vd = 4d Sd + Bd = 0 (72) 0 = 0

Bd = 0

4117 + Be0.08(0.5) = 26. 422 26


4 = 0.587 161
472 + Be0.08(0.5) = 0
V = 4S + B = 0.587 161 (90) 40. 617 97 = 12. 226 52
The final diagram is:
Period 0
1

S = 90
V = 12. 226 52
4 = 0.587 161
B = 40. 617 97

www.actuary88.com

Su = 90 (1.3) = 117
Vu = 26. 422 26
4u = 0.976 068
Bu = 87. 777 7

Sd = 90 (0.8) = 72
Vd = 0
d = 0
Bd = 0

B = 40. 617 97

Suu = 90 1.32 = 152. 1


Vuu = 152. 1 95 = 57. 1

Sud = 90 (1.3) (0.8) = 93. 6


Vud = 0

Sdd = 90 0.82 = 57. 6


Vdd = 0
0

c
Yufeng
Guo

30

CHAPTER 10. BINOMIAL OPTION PRICING I


S = 110.
Period 0

1
Su = 110 (1.3) = 143
Vu =?
4u =?
Bu =?

S = 110
V =?
=?
B =?

2
Suu = 110 1.32 = 185. 9
Vuu = 185. 9 95 = 90. 9

Sud = 110 (1.3) (0.8) = 114. 4


Vud = 114. 4 95 = 19. 4
Sd = 110 (0.8) = 88
Vd =?
d =?
Bd =?

Sdd = 110 0.82 = 70. 4


Vdd = 0

4u 185. 9 + Bu e0.08(0.5) = 90. 9


4u = 1
4u 114. 4 + Bu e0.08(0.5) = 19. 4
Vu = 4u Su + Bu = 1 (143) 91. 2750 = 51. 725

Bu = 91. 2750

4d 114. 4 + Bd e0.08(0.5) = 19. 4


4d = 0.440 909
4d 70. 4 + Bd e0.08(0.5) = 0
Vd = 4d Sd + Bd = 0.440 909 (88) 29. 822 9 = 8. 977 092

4143 + Be0.08(0.5) = 51. 725


4 = 0.777 235
488 + Be0.08(0.5) = 8. 977 092
V = 4S + B = 0.777 235 (110) 57. 089 7 = 28. 406 15

The final diagram is:


Period 0
1

S = 110
28. 406 15
4 = 0.777 235
B = 57. 089 7

www.actuary88.com

Su = 110 (1.3) = 143


Vu = 51. 725
4u = 1
Bu = 91. 2750

Sd = 110 (0.8) = 88
Vd = 8. 977 092
d = 0.440 909
Bd = 29. 822 9

Bd = 29. 822 9

B = 57. 089 7

Suu = 110 1.32 = 185. 9


Vuu = 185. 9 95 = 90. 9

Sud = 110 (1.3) (0.8) = 114. 4


Vud = 114. 4 95 = 19. 4

Sdd = 110 0.82 = 70. 4


Vdd = 0

c
Yufeng
Guo

31

CHAPTER 10. BINOMIAL OPTION PRICING I


S = 120.
Period 0

1
Su = 120 (1.3) = 156
Vu =?
4u =?
Bu =?

S = 120
V =?
=?
B =?

Suu = 120 1.32 = 202. 8


Vuu = 202. 8 95 = 107. 8

Sud = 120 (1.3) (0.8) = 124. 8


Vud = 124. 8 95 = 29. 8
Sd = 120 (0.8) = 96
Vd =?
d =?
Bd =?

Sdd = 120 0.82 = 76. 8


Vdd = 0

4u 202. 8 + Bu e0.08(0.5) = 107. 8


4u = 1
4u 124. 8 + Bu e0.08(0.5) = 29. 8
Vu = 4u Su + Bu = 1 (156) 91. 275 = 64. 725

Bu = 91. 275

4d 124. 8 + Bd e0.08(0.5) = 29. 8


Bd = 45. 810 44
4d = 0.620 833
4d 76. 8 + Bd e0.08(0.5) = 0
Vd = 4d Sd + Bd = 0.620 833 (96) 45. 810 44 = 13. 789 528

4156 + Be0.08(0.5) = 64. 725


4 = 0.848 925
496 + Be0.08(0.5) = 13. 789 528
V = 4S + B = 0.848 925 (120) 65. 052 4 = 36. 818 6

The final diagram is:


Period 0

S = 120
V = 36. 818 6
4 = 0.848 925
B = 65. 052 4

www.actuary88.com

Su = 120 (1.3) = 156


Vu = 64. 725
4u = 1
Bu = 91. 275

B = 65. 052 4

2
Suu = 120 1.32 = 202. 8
Vuu = 202. 8 95 = 107. 8

Sud = 120 (1.3) (0.8) = 124. 8


Vud = 124. 8 95 = 29. 8

Sd = 120 (0.8) = 96
Vd = 13. 789 528
4d = 0.620 833
Bd = 45. 810 44
c
Yufeng
Guo

Sdd = 120 0.82 = 76. 8


Vdd = 0

32

CHAPTER 10. BINOMIAL OPTION PRICING I


S = 130.
Period 0

1
Su = 130 (1.3) = 169
Vu =?
4u =?
Bu =?

S = 130
V =?
=?
B =?

Suu = 130 1.32 = 219. 7


Vuu = 219. 7 95 = 124. 7

Sud = 130 (1.3) (0.8) = 135. 2


Vud = 135. 2 95 = 40. 2
Sd = 130 (0.8) = 104
Vd =?
d =?
Bd =?

Sdd = 130 0.82 = 83. 2


Vdd = 0

4u 219. 7 + Bu e0.08(0.5) = 124. 7


Bu = 91. 275
4u = 1
4u 135. 2 + Bu e0.08(0.5) = 40. 2
Vu = 4u Su + Bu = 1 (169) 91. 275 = 77. 725

4d 135. 2 + Bd e0.08(0.5) = 40. 2


Bd = 61. 7980
4d = 0.773 077
4d 83. 2 + Bd e0.08(0.5) = 0
Vd = 4d Sd + Bd = 0.773 077 (104) 61. 7980 = 18. 602

4169 + Be0.08(0.5) = 77. 725


4 = 0.909 585
B = 73. 015
4104 + Be0.08(0.5) = 18. 602
V = 4S + B = 0.909 585 (130) 73. 015 = 45. 231
The final diagram is:
Period 0
1
2

Suu = 130 1.32 = 219. 7


Su = 130 (1.3) = 169 Vuu = 219. 7 95 = 124. 7
Vu = 77. 725
4u = 1
Bu = 91. 275
S = 130
Sud = 130 (1.3) (0.8) = 135. 2
V = 45. 231
Vud = 135. 2 95 = 40. 2
4 = 0.909 585
B = 73. 015
Sd = 130 (0.8) = 104

Vd = 18. 602
Sdd = 130 0.82 = 83. 2
4d = 0.773 077
Vdd = 0
Bd = 61. 7980
Notice that the delta for a call is always positive. A positive delta means
buying stocks. If you sell a call, the risk you face is that the stock price may go
www.actuary88.com

c
Yufeng
Guo

33

CHAPTER 10. BINOMIAL OPTION PRICING I


up to infinity, at which case the call holder will pay only the strike price to buy
a priceless stock from you. To hedge your risk, you need to already own some
stocks at t = 0. If indeed the future stock price goes up at T , your stock will
also go up in value, osetting your loss in the call.
Intuitively, the higher the initial stock price, everything else equal, the higher
the stock price in the future, the higher the payo of the call. Hence to hedge
the risk, the market maker needs to buy more shares of stocks at t = 0. So the
higher the initial stock price, the higher the initial delta, the more stocks the
market maker needs to buy at t = 0.

www.actuary88.com

c
Yufeng
Guo

34

CHAPTER 10. BINOMIAL OPTION PRICING I


Problem 10.6.
Period 0

1
Su = 100 (1.3) = 130
Vu =?
4u =?
Bu =?

S = 100
V =?
=?
B =?

Suu = 100 1.32 = 169


Vuu = 0

Sud = 100 (1.3) (0.8) = 104


Vud = 0

Sd = 100 (0.8) = 80
Vd =?
d =?
Bd =?

Sdd = 100 0.82 = 64


Vdd = 95 64 = 31

4u 169 + Bu e0.08(0.5) = 0
4u = 0
4u 104 + Bu e0.08(0.5) = 0
Vu = 4u Su + Bu = 0 (130) + 0 = 0

Bu = 0

4d 104 + Bd e0.08(0.5) = 0
Bd = 77. 439 6
4d = 0.775
4d 64 + Bd e0.08(0.5) = 31
Vd = 4d Sd + Bd = 0.775 (80) + 77. 439 6 = 15. 439 6

4130 + Be0.08(0.5) = 0
4 = 0.308 792
B = 38. 568 932
480 + Be0.08(0.5) = 15. 439 6
V = 4S + B = 0.308 792 (100) + 38. 568 932 = 7. 689 732
The final diagram is:
Period 0
1
2

Suu = 100 1.32 = 169


Su = 100 (1.3) = 130 Vuu = 0
Vu = 0
4u = 0
Bu = 0
S = 100
Sud = 100 (1.3) (0.8) = 104
V = 7. 689 732
Vud = 0
4 = 0.308 792
B = 38. 568 932
Sd = 100 (0.8) = 80

Vd = 15. 439 6
Sdd = 100 0.82 = 64
4d = 0.775
Vdd = 95 64 = 31
Bd = 77. 439 6

www.actuary88.com

c
Yufeng
Guo

35

CHAPTER 10. BINOMIAL OPTION PRICING I


Problem 10.7.
S = 80
Period 0

1
Su = 80 (1.3) = 104
Vu =?
4u =?
Bu =?

S = 80
V =?
=?
B =?

Suu = 80 1.32 = 135. 2


Vuu = 0

Sud = 80 (1.3) (0.8) = 83. 2


Vud = 95 83. 2 = 11. 8
Sd = 80 (0.8) = 64
Vd =?
d =?
Bd =?

Sdd = 80 0.82 = 51. 2


Vdd = 95 51. 2 = 43. 8

4u 135. 2 + Bu e0.08(0.5) = 0
4u = 0.226 923
4u 83. 2 + Bu e0.08(0.5) = 11. 8
Vu = 4u Su + Bu = 0.226 923 (104) + 29. 477 02 = 5. 877

4d 83. 2 + Bd e0.08(0.5) = 11. 8


4d = 1
4d 51. 2 + Bd e0.08(0.5) = 43. 8
Vd = 4d Sd + Bd = 1 (64) + 91. 275 = 27. 275

Bu = 29. 477 02

Bd = 91. 275

4104 + Be0.08(0.5) = 5. 877


4 = 0.534 95
B = 59. 1
464 + Be0.08(0.5) = 27. 275
V = 4S + B = 0.534 95 (80) + 59. 1 = 16. 304
The final diagram is:
Period 0
1
2

Suu = 80 1.32 = 135. 2


Su = 80 (1.3) = 104 Vuu = 0
Vu = 5. 877
4u = 0.226 923
Bu = 29. 477 02
S = 80
Sud = 80 (1.3) (0.8) = 83. 2
V = 16. 304
Vud = 95 83. 2 = 11. 8
4 = 0.534 95
B = 59. 1
Sd = 80 (0.8) = 64

Vd = 27. 275
Sdd = 80 0.82 = 51. 2
4d = 1
Vdd = 95 51. 2 = 43. 8
Bd = 91. 275
www.actuary88.com

c
Yufeng
Guo

36

CHAPTER 10. BINOMIAL OPTION PRICING I


S = 90
Period 0

1
Su = 90 (1.3) = 117
Vu =?
4u =?
Bu =?

S = 90
V =?
=?
B =?

Suu = 90 1.32 = 152. 1


Vuu = 0

Sud = 90 (1.3) (0.8) = 93. 6


Vud = 95 93. 6 = 1. 4
Sd = 90 (0.8) = 72
Vd =?
d =?
Bd =?

Sdd = 90 0.82 = 57. 6


Vdd = 95 57. 6 = 37. 4

4u 152. 1 + Bu e0.08(0.5) = 0
Bu = 3. 497 27
4u = 0.02 393
4u 93. 6 + Bu e0.08(0.5) = 1. 4
Vu = 4u Su + Bu = 0.02 393 (117) + 3. 497 27 = 0.697 5

4d 93. 6 + Bd e0.08(0.5) = 1. 4
4d = 1
4d 57. 6 + Bd e0.08(0.5) = 37. 4
Vd = 4d Sd + Bd = 1 (72) + 91. 275 = 19. 275

Bd = 91. 275

4117 + Be0.08(0.5) = 0.697 5


4 = 0.412 833
B = 47. 077 722
472 + Be0.08(0.5) = 19. 275
V = 4S + B = 0.412 833 (90) + 47. 077 722 = 9. 923
The final diagram is:
Period 0
1
2

Suu = 90 1.32 = 152. 1


Su = 90 (1.3) = 117 Vuu = 0
Vu = 0.697 5
4u = 0.02 393
Bu = 3. 497 27
S = 90
Sud = 90 (1.3) (0.8) = 93. 6
V = 9. 923
Vud = 95 93. 6 = 1. 4
4 = 0.412 833
B = 47. 077 722
Sd = 90 (0.8) = 72

Vd = 19. 275
Sdd = 90 0.82 = 57. 6
4d = 1
Vdd = 95 57. 6 = 37. 4
Bd = 91. 275

www.actuary88.com

c
Yufeng
Guo

37

CHAPTER 10. BINOMIAL OPTION PRICING I


S = 110
Period 0

1
Su = 110 (1.3) = 143
Vu =?
4u =?
Bu =?

S = 110
V =?
=?
B =?

Suu = 110 1.32 = 185. 9


Vuu = 0

Sud = 110 (1.3) (0.8) = 114. 4


Vud = 0

Sd = 110 (0.8) = 88
Vd =?
d =?
Bd =?

Sdd = 110 0.82 = 70. 4


Vdd = 95 70. 4 = 24. 6

4u 185. 9 + Bu e0.08(0.5) = 0
4u = 0
4u 114. 4 + Bu e0.08(0.5) = 0
Vu = 4u Su + Bu = 0 (143) + 0 = 0

Bu = 0

4d 114. 4 + Bd e0.08(0.5) = 0
4d = 0.559 091
4d 70. 4 + Bd e0.08(0.5) = 24. 6
Vd = 4d Sd + Bd = 0.559 091 (88) + 61. 452 1 = 12. 252

Bd = 61. 452 1

4143 + Be0.08(0.5) = 0
4 = 0.222 76
B = 30. 606 14
488 + Be0.08(0.5) = 12. 252
V = 4S + B = 0.222 76 (110) + 30. 606 14 = 6. 103
The final diagram is:
Period 0
1
2

Suu = 110 1.32 = 185. 9


Su = 110 (1.3) = 143 Vuu = 0
Vu = 0
4u = 0
Bu = 0
S = 110
Sud = 110 (1.3) (0.8) = 114. 4
V = 6. 103
Vud = 0
4 = 0.222 76
B = 30. 606 14
Sd = 110 (0.8) = 88

Vd = 12. 252
Sdd = 110 0.82 = 70. 4
4d = 0.559 091
Vdd = 95 70. 4 = 24. 6
Bd = 61. 452 1

www.actuary88.com

c
Yufeng
Guo

38

CHAPTER 10. BINOMIAL OPTION PRICING I


S = 120
Period 0

1
Su = 120 (1.3) = 156
Vu =?
4u =?
Bu =?

S = 120
V =?
=?
B =?

Suu = 120 1.32 = 202. 8


Vuu = 0

Sud = 120 (1.3) (0.8) = 124. 8


Vud = 0

Sd = 120 (0.8) = 96
Vd =?
d =?
Bd =?

Sdd = 120 0.82 = 76. 8


Vdd = 95 76. 8 = 18. 2

4u 202. 8 + Bu e0.08(0.5) = 0
4u = 0
4u 124. 8 + Bu e0.08(0.5) = 0
Vu = 4u Su + Bu = 0 (156) + 0 = 0

Bu = 0

4d 124. 8 + Bd e0.08(0.5) = 0
4d = 0.379 167
4d 76. 8 + Bd e0.08(0.5) = 18. 2
Vd = 4d Sd + Bd = 0.379 167 (96) + 45. 464 56 = 9. 065

Bd = 45. 464 56

4156 + Be0.08(0.5) = 0
4 = 0.151 083
B = 22. 644 85
496 + Be0.08(0.5) = 9. 065
V = 4S + B = 0.151 083 (120) + 22. 644 85 = 4. 514 89
The final diagram is:
Period 0
1
Su = 120 (1.3) = 156
Vu = 0
4u = 0
Bu = 0
S = 120
V = 4. 514 89
4 = 0.151 083
B = 22. 644 85

Sud = 120 (1.3) (0.8) = 124. 8


Vud = 0

Sd = 120 (0.8) = 96
Vd = 9. 065
4d = 0.379 167
Bd = 45. 464 56

www.actuary88.com

Suu = 120 1.32 = 202. 8


Vuu = 0

c
Yufeng
Guo

Sdd = 120 0.82 = 76. 8


Vdd = 95 76. 8 = 18. 2

39

CHAPTER 10. BINOMIAL OPTION PRICING I


S = 130
Period 0

1
Su = 130 (1.3) = 169
Vu =?
4u =?
Bu =?

S = 130
V =?
=?
B =?

Suu = 130 1.32 = 219. 7


Vuu = 0

Sud = 130 (1.3) (0.8) = 135. 2


Vud = 0

Sd = 130 (0.8) = 104


Vd =?
d =?
Bd =?

Sdd = 130 0.82 = 83. 2


Vdd = 95 83. 2 = 11. 8

4u 219. 7 + Bu e0.08(0.5) = 0
4u = 0
4u 135. 2 + Bu e0.08(0.5) = 0
Vu = 4u Su + Bu = 0 (169) + 0 = 0

Bu = 0

4d 135. 2 + Bd e0.08(0.5) = 0
4d = 0.226 923
4d 83. 2 + Bd e0.08(0.5) = 11. 8
Vd = 4d Sd + Bd = 0.226 923 (104) + 29. 477 0 = 5. 877

Bd = 29. 477 0

4169 + Be0.08(0.5) = 0
4 = 0.09 041 5
B = 14. 681 05
4104 + Be0.08(0.5) = 5. 877
V = 4S + B = 0.09 041 5 (130) + 14. 681 05 = 2. 927 1
The final diagram is:
Period 0
1
2

Suu = 130 1.32 = 219. 7


Su = 130 (1.3) = 169 Vuu = 0
Vu = 0
4u = 0
Bu = 0
S = 130
Sud = 130 (1.3) (0.8) = 135. 2
V = 2. 927 1
Vud = 0
4 = 0.09 041 5
B = 14. 681 05
Sd = 130 (0.8) = 104

Vd = 5. 877
Sdd = 130 0.82 = 83. 2
4d = 0.226 923
Vdd = 95 83. 2 = 11. 8
Bd = 29. 477 0

www.actuary88.com

c
Yufeng
Guo

40

CHAPTER 10. BINOMIAL OPTION PRICING I


Notice that the delta for a put is always negative. A negative delta means
shorting sell stocks. If you sell a put, the risk you face is that the stock price
may go down to zero, at which case the put holder sells you a worthless stock for
the strike price. To hedge your risk, you need to short sell some stocks at t = 0.
If indeed the future stock price goes down at T , you can buy back stocks in
the market at low price to close your short position on stocks, earning a profit.
Your profit from the short sale can oset your loss in the put.
Intuitively, the higher the initial stock price, everything else equal, the higher
the stock price in the future, the lower the payo of the put. Hence to hedge
the risk, the market maker needs to short sell fewer shares of stocks at t = 0. So
the higher the initial stock price, the lower the absolute value of the delta for a
put, the fewer of the stocks that the market maker needs to short sell initially.
Problem 10.8.
Period 0

1
Su = 100 (1.3) = 130
EVu = 0
Vu =?
4u =?
Bu =?

S = 100
EV = 0
V =?
=?
B =?

Suu = 100 1.32 = 169


Vuu = 0

Sud = 100 (1.3) (0.8) = 104


Vud = 0

Sd = 100 (0.8) = 80
EVd = 95 80 = 15
Vd =?
d =?
Bd =?

Sdd = 100 0.82 = 64


Vdd = 95 64 = 31

4u 169 + Bu e0.08(0.5) = 0
Bu = 0
4u = 0
4u 104 + Bu e0.08(0.5) = 0
Vu = max (4u Su + Bu , EVu ) = max [0 (130) + 0, 0] = 0
This is the logic behind the calculation of Vu . The American put can be
exercised at the end of Period 1 and end of Period 2.
If exercised at the end of Period 1, the put is worth EVu = max (0, 95 130) =
0 at the end of Period 1.
If exercised at Period 2, the put is worth 4u Su + Bu = 0 (130) + 0 = 0 at
the end of Period 1.
www.actuary88.com

c
Yufeng
Guo

41

CHAPTER 10. BINOMIAL OPTION PRICING I


We compare the two values and take the greater.

4d 104 + Bd e0.08(0.5) = 0
Bd = 77. 439 6
4d = 0.775
4d 64 + Bd e0.08(0.5) = 31
Vd = max (4d Sd + Bd , EVd ) = max [0.775 (80) + 77. 439 6, 15] = 15. 439 6

4130 + Be0.08(0.5) = 0
4 = 0.308 792
B = 38. 568 932
480 + Be0.08(0.5) = 15. 439 6
V = max (4S + B, EV ) = max [0.308 792 (100) + 38. 568 932 , 0] = 7. 689 732

The final diagram is:


Period 0
1
Su = 100 (1.3) = 130
EVu = 0
Vu = 0
4u = 0
Bu = 0
S = 100
EV = 0
V = 7. 689 7
= 0.308 792
B = 38. 568 932

Suu = 100 1.32 = 169


Vuu = 0

Sud = 100 (1.3) (0.8) = 104


Vud = 0

Sd = 100 (0.8) = 80
EVd = 95 80 = 15
Vd = 15. 439 6
4d = 0.775
Bd = 77. 439 6

Sdd = 100 0.82 = 64


Vdd = 95 64 = 31

Problem 10.9.
a.
time 0

T
Su = 100 (1.2) = 120
Vu = 120 50 = 70

S = 100
V =?
=?, B =?

Sd = 100 (1.05) = 105


Vd = 105 50 = 55
Here d = 1.05 > 1. Is there anything wrong with this? No necessarily. The
non-arbitrage requirement is textbook Equation 10.4: u > e(r)h > d
Lets check. e(r)h = e(0.076960)1 = 1. 08
u = 1.2 > e(r)h > d = 1.05
www.actuary88.com

c
Yufeng
Guo

42

CHAPTER 10. BINOMIAL OPTION PRICING I


So its OK to have d > 1 as long as u > e(r)h > d is met.

4120 + Be0.07696(1) = 70
4=1
B = 46. 296 3
4105 + Be0.07696(1) = 55
V = 4S + B = 1 (100) 46. 296 3 = 53. 703 7
The final diagram is:
time 0

S = 100
V = 53. 703 7
4 = 1, B = 46. 296 3

T
Su = 100 (1.2) = 120
Vu = 120 50 = 70

Sd = 100 (1.05) = 105


Vd = 105 50 = 55

b. Now the stock price has a bigger increase and a bigger decrease. We
are not clear whether the call premium will increase or decrease. We have to
calculate the premium.
time 0
T
Su = 100 (1.4) = 140
Vu = 140 50 = 90
S = 100
V =?
=?, B =?
Sd = 100 (0.6) = 60
Vd = 60 50 = 10

4140 + Be0.07696(1) = 90
4=1
B = 46. 296 3
460 + Be0.07696(1) = 10
V = 4S + B = 1 (100) 46. 296 3 = 53. 703 7
The call premium is the same. Whats going on?
It turns out that as long Su > Sd > K, the call premium is fixed regardless
of how you choose u and d.
time 0
T
Su = uS > K
Vu = uS K
S
V =?
=?, B =?
Sd = dS > K
Vd = dS K

4uS + Berh = uS K
4dS + Berh = dS K

www.actuary88.com

c
Yufeng
Guo

43

CHAPTER 10. BINOMIAL OPTION PRICING I


4 = 1
B = Kerh
50e
= 53. 703 7
0.07696(1)

C = 4S + B = S Kerh = 100

We can explain this using the put-call parity. If Su > Sd > K, the put is
never exercised.
C + P V (K) = P + S
Since P = 0, we have C = S P V (K) = S Kerh = 10050e0.07696(1) =
53. 703 7
c. I think the question wants us to compare b and c, not a and c.
Compared with b, now u is the same but d gets smaller. In fact, now at
Node d, the call expires worthless.
time 0

S = 100
V =?
=?, B =?

T
Su = 100 (1.4) = 140
Vu = 140 50 = 90

Sd = 100 (0.4) = 40
Vd = 0
We might think the call option should be worth less since it expires worthless
at Node
d.

4140 + Be0.07696(1) = 90
4 = 0.9
B = 33. 333
440 + Be0.07696(1) = 0
V = 4S + B = 0.9 (100) 33. 333 = 56. 667 > 53. 703 7
Why did the call premium goes up to 56. 667? Does this lead to arbitrage?
First, Im going to show you this is not an arbitrage. Now we have two call
options:
Call b
Call c
time 0
T
time 0
T
Su = 100 (1.4) = 140
Su = 100 (1.4) = 140
Vu = 140 50 = 90
Vu = 140 50 = 90
S = 100
S = 100
V = 53. 703 7
V = 56. 667
4 = 1, B = 46. 296 3
4 = 0.9, B = 33. 333
Sd = 100 (0.6) = 60
Sd = 100 (0.4) = 40
Vd = 60 50 = 10
Vd = 0
What if we buy Call b and simultaneously sell Call c? Does this lead to
arbitrage?
The answer is No because these two calls are on two dierent stocks. If
these two options have the same underlying asset, then well make free money
www.actuary88.com

c
Yufeng
Guo

44

CHAPTER 10. BINOMIAL OPTION PRICING I


by "buy low and sell high." However, these two calls are on two dierent stocks;
if the stock is the same, there wont be two dierent values of d0 s.
Call b is on a stock whose up price is 140 and down price is 60. Call c is
on the stock whose up price is 140 and down price is 40. Clearly, the stock
under Call c is more volatile than the stock under Call b. Hence Call c is more
valuable than Call b.
We can also explain why Call b is more valuable using the put-call parity.
C + P V (K) = P + S
For b, the put is never exercised. Hence P = 0 and C = S P V (K).
For c, the put is exercised Node d. Hence P > 0 and C = P + S P V (K).
So Call c exceeds Call b.

Problem 10.10.
h = T /n = 1/3
u = e(r)h+
d = e(r)h
u =

= e(0.080)1/3+0.3 1/3 = 1. 221 246

= e(0.080)1/30.3 1/3 = 0.863 693

e(r)h d
e(0.080)1/3 0.863 693
=
= 0.456 806
ud
1. 221 246 0.863 693

d = 1 u = 1 0.456 806 = 0.543 194


a. Calculate the call premium
Period 2

Suu = 100 1. 221 2462 = 149. 144 2


EVuu = 149. 144 2 95 = 54. 144 2
Vuu =?
Sud = 100 (1. 221 246) (0.863 693) = 105. 478 2
EVud = 105. 478 2 95 = 10. 478 2
Vud =?

Sdd = 100 0.863 6932 = 74. 596 6


EVdd = 0
Vdd =?

Suuu = 100 1. 221 2463 = 182. 141 7


Vuuu = 182. 141 7 95 = 87. 141 7

Suud = 100 1. 221 2462 (0.863 693) = 128. 814 8


Vuud = 128. 814 8 95 = 33. 814 8

Sudd = 100 (1. 221 246) 0.863 6932 = 91. 100 8


Vudd = 0

Sddd = 100 0.863 6933 = 64. 428 5


Vddd = 0

Let R represent the roll-back value.


R
Vuu
= erh ( u Vuuu + d Vuud ) = e(0.08)1/3 (0.456 806 87. 141 7 + 0.543 194 33. 814 8) =
56. 644 02
R

Vuu = max Vuu


, EVuu = max (56. 644 02, 54. 144 2) = 56. 644 02
www.actuary88.com

c
Yufeng
Guo

45

CHAPTER 10. BINOMIAL OPTION PRICING I


R
Vud
= erh (u Vuud + d Vudd ) = e(0.08)1/3 (0.456 806 33. 814 8 + 0.543 194 0) =
15. 040 33

R
, EVud = max (15. 040 33, 10. 478 2) = 15. 040 33
Vud = max Vud

R
= erh ( u Vudd + d Vddd ) = e(0.08)1/3 (0.456 806 0 + 0.543 194 0) =
Vdd

Vdd = max Vdd


, EVdd = max (0, 0) = 0

Now we have:
Period 1

Su = 100 (1. 221 246) = 122. 124 6


EVu = 122. 124 6 95 = 27. 124 6
Vu =?

Sd = 100 (0.863 693) = 86. 369 3


EVd = 0
Vd =?

Suu = 100 1. 221 2462 = 149. 144 2


EVuu = 149. 144 2 95 = 54. 144 2
Vuu = 56. 644 02
Sud = 100 (1. 221 246) (0.863 693) = 105. 478 2
EVud = 105. 478 2 95 = 10. 478 2
Vud = 15. 040 33

Sdd = 100 0.863 6932 = 74. 596 6


EVdd = 0
Vdd = 0

VuR = erh (u Vuu + d Vud ) = e(0.08)1/3 (0.456 806 56. 644 02 + 0.543 194 15. 040 33) =
33. 149 27

Vu = max VuR , EVu = max (33. 149 27, 27. 124 6) = 33. 149 27

VdR = erh (u Vud + d Vdd ) = e(0.08)1/3 (0.456 806 15. 040 33 + 0.543 194 0) =
6. 689 72

Vd = max VdR , EVd = max (6. 689 72, 0) = 6. 689 72


Now we have:
Period 0

S = 100
EV = 100 95 = 5
V =?

1
Su = 100 (1. 221 246) = 122. 124 6
EVu = 122. 124 6 95 = 27. 124 6
Vu = 33. 149 27

Sd = 100 (0.863 693) = 86. 369 3


EVd = 0
Vd = 6. 689 72

V R = erh (u Vu + d Vd ) = e(0.08)1/3 (0.456 806 33. 149 27 + 0.543 194 6. 689 72) =
18. 282 51

V = max V R , EV = max (18. 282 51, 5) = 18. 282 51


www.actuary88.com

c
Yufeng
Guo

46

CHAPTER 10. BINOMIAL OPTION PRICING I


From the above calculation you can see that the exercise value is never
greater than the roll back value. Hence the American call option is never exercised early; the American call and European call have the same value of $18.
282 51.
Generally, if a stock doesnt pay dividend, then an American call and an
otherwise identical European call on this stock are worth the same.

c. Calculate the American put premium.

Period 2

Suu = 100 1. 221 2462 = 149. 144 2


EVuu = 0
Vuu =?
Sud = 100 (1. 221 246) (0.863 693) = 105. 478 2
EVud = 0
Vud =?

Sdd = 100 0.863 6932 = 74. 596 6


EVdd = 95 74. 596 6 = 20. 403 4
Vdd =?

Suuu = 100 1. 221 2463 = 182. 141 7


Vuuu = 0

Suud = 100 1. 221 2462 (0.863 693) = 128. 814 8


Vuud = 0

Sudd = 100 (1. 221 246) 0.863 6932 = 91. 100 8


Vudd = 95 91. 100 8 = 3. 899 2

Sddd = 100 0.863 6933 = 64. 428 5


Vddd = 95 64. 428 5 = 30. 571 5

R
Vuu
= erh ( u Vuuu + d Vuud ) = e(0.08)1/3 (0.456 806 0 + 0.543 194 0) =

, EVuu = max (0, 0) = 0


Vuu = max Vuu

R
= erh ( u Vuud + d Vudd ) = e(0.08)1/3 (0.456 806 0 + 0.543 194 3. 899 2) =
Vud
2. 062 29
R

, EVud = max (2. 062 29, 0) = 2. 062 29


Vud = max Vud
R
= erh (u Vudd + d Vddd ) = e(0.08)1/3 (0.456 806 3. 899 2 + 0.543 194 30. 571 5) =
Vdd
17. 903 58

R
, EVdd = max (17. 903 58, 20. 403 4) = 20. 403 4
Vdd = max Vdd

At the dd node, the exercise value is greater than the roll-back value. The
put is exercised at dd. Now we have:
www.actuary88.com

c
Yufeng
Guo

47

CHAPTER 10. BINOMIAL OPTION PRICING I


Period 1

Su = 100 (1. 221 246) = 122. 124 6


EVu = 0
Vu =?

Sd = 100 (0.863 693) = 86. 369 3


EVd = 95 86. 369 3 = 8. 630 7
Vd =?

Suu = 100 1. 221 2462 = 149. 144 2


EVuu = 0
Vuu = 0
Sud = 100 (1. 221 246) (0.863 693) = 105. 478 2
EVud = 0
Vud = 2. 062 29

Sdd = 100 0.863 6932 = 74. 596 6


EVdd = 95 74. 596 6 = 20. 403 4
Vdd = 20. 403 4

VuR = erh (u Vuu + d Vud ) = e(0.08)1/3 (0.456 806 0 + 0.543 194 2. 062 29) =
1. 090 75

Vu = max VuR , EVu = max (1. 090 75, 0) = 1. 090 75


VdR = erh (u Vud + d Vdd ) = e(0.08)1/3 (0.456 806 2. 062 29 + 0.543 194 20. 403 4) =
11. 708 64

Vd = max VdR , EVd = max (11. 708 64, 8. 630 7) = 11. 708 64
Now we have:
Period 0 1
Su = 100 (1. 221 246) = 122. 124 6
EVu = 0
Vu = 1. 090 75
S = 100
EV = 0
V =?
Sd = 100 (0.863 693) = 86. 369 3
EVd = 95 86. 369 3 = 8. 630 7
Vd = 11. 708 64
V R = erh (u Vu + d Vd ) = e(0.08)1/3 (0.456 806 1. 090 75 + 0.543 194 11. 708 64) =
6. 677 85

V = max V R , EV = max (6. 677 85, 0) = 6. 677 85


b. Calculate the European put premium. Verify that the put-call parity
holds.
Since the option is European, we just calculate the roll-back value.
www.actuary88.com

c
Yufeng
Guo

48

CHAPTER 10. BINOMIAL OPTION PRICING I


Period 2

3
Vuuu = 0

Vuu =?
Vuud = 0
Vud =?
Vudd = 3. 899 2
Vdd =?
rh

Vuu = e
0

Vddd = 30. 571 5


( u Vuuu + d Vuud ) = e(0.08)1/3 (0.456 806 0 + 0.543 194 0) =

Vud = erh ( u Vuud + d Vudd ) = e(0.08)1/3 (0.456 806 0 + 0.543 194 3. 899 2) =
2. 062 29
Vdd = erh (u Vudd + d Vddd ) = e(0.08)1/3 (0.456 806 3. 899 2 + 0.543 194 30. 571 5) =
17. 903 58
Now we have:
Period 1 2
Vuu = 0
Vu =?
Vud = 2. 062 29
Vd =?
Vdd = 17. 903 58
Vu = erh (u Vuu + d Vud ) = e(0.08)1/3 (0.456 806 0 + 0.543 194 2. 062 29) =
1. 090 75
Vd = erh ( u Vud + d Vdd ) = e(0.08)1/3 (0.456 806 2. 062 29 + 0.543 194 17. 903 58) =
10. 386 48
Now we have:
Period 0 1
Vu = 1. 090 75
V =?
Vd = 10. 386 48
rh
V =e
( u Vu + d Vd ) = e(0.08)1/3 (0.456 806 1. 090 75 + 0.543 194 10. 386 48) =
5. 978 6
Check whether the put-call parity-holds:
C + KerT = 18. 282 51 + 95e(0.08)1 = 105. 978 6
P + S = 5. 978 6 + 100 = 105. 978 6
C + KerT = P + S
Problem 10.11.
h = T /n = 1/3
u = e(r)h+

www.actuary88.com

= e(0.080.08)1/3+0.3

1/3

= 1. 189 11

c
Yufeng
Guo

49

CHAPTER 10. BINOMIAL OPTION PRICING I


d = e(r)h

u =

= e(0.080.08)1/30.3 1/3 = 0.840 965

e(r)h d
e(0.080.08)1/3 0.840 965
=
= 0.456 807
ud
1. 189 11 0.840 965

d = 1 u = 1 0.456 807 = 0.543 193


a.
Calculate the price of the American call option
Period 2

Suu = 100 1. 189 112 = 141. 398 3


EVuu = 141. 398 3 95 = 46. 398 3
Vuu =?
Sud = 100 (1. 189 11) (0.840 965) = 100
EVud = 100 95 = 5
Vud =?

Sdd = 100 0.840 9652 = 70. 722 2


EVdd = 0
Vdd =?

Suuu = 100 1. 189 113 = 168. 138 1


Vuuu = 168. 138 1 95 = 73. 138 1

Suud = 100 1. 189 112 (0.840 965) = 118. 9110


Vuud = 118. 9110 95 = 23. 911

Sudd = 100 (1. 189 11) 0.840 9652 = 84. 096 5


Vudd = 0

Sddd = 100 0.840 9653 = 59. 474 9


Vddd = 0

R
Vuu
= erh ( u Vuuu + d Vuud ) = e(0.08)1/3 (0.456 807 73. 138 1 + 0.543 193 23. 911) =
45. 177 3
R

, EVuu = max (45. 177 3, 46. 398 3) = 46. 398 3


Vuu = max Vuu
The call is early exercised at the uu node.

R
= erh (u Vuud + d Vudd ) = e(0.08)1/3 (0.456 807 23. 911 + 0.543 193 0) =
Vud
10. 635 3
R

, EVud = max (10. 635 3, 5) = 10. 635 3


Vud = max Vud

R
Vdd
= erh ( u Vudd + d Vddd ) = e(0.08)1/3 (0.456 807 0 + 0.543 193 0) =

, EVdd = max (0, 0) = 0


Vdd = max Vdd

www.actuary88.com

c
Yufeng
Guo

50

CHAPTER 10. BINOMIAL OPTION PRICING I


Period 1

2
Vuu = 46. 398 3

Su = 100 (1. 189 11) = 118. 911


EVu = 118. 911 95 = 23. 911
Vu =?
Vud = 10. 635 3
Sd = 100 (0.840 965) = 84. 096 5
EVd = 0
Vd =?
Vdd = 0
VuR = erh ( u Vuu + d Vud ) = e(0.08)1/3 (0.456 807 46. 398 3 + 0.543 193 10. 635 3) =
26. 262 3

Vu = max VuR , EVu = max (26. 262 3, 23. 911) = 26. 262 3
VdR = erh ( u Vud + d Vdd ) = e(0.08)1/3 (0.456 807 10. 635 3 + 0.543 193 0) =
4. 730 4

Vd = max VdR , EVd = max (4. 730 4, 0) = 4. 730 4


Period 0

1
Vu = 26. 262 3

S = 100
EV = 100 95 = 5
Vu =?
Vd = 4. 730 4
V R = erh ( u Vu + d Vd ) = e(0.08)1/3 (0.456 807 26. 262 3 + 0.543 193 4. 730 4) =
14. 183 0

V = max V R , EV = max (14. 183 0, 5) = 14. 183 0


Calculate the price of the European call option
Period 0

Vuu
Vu
V

Vud
Vd
Vdd

www.actuary88.com

Suuu = 100 1. 189 113 = 168. 138 1


Vuuu = 168. 138 1 95 = 73. 138 1

Suud = 100 1. 189 112 (0.840 965) = 118. 9110


Vuud = 118. 9110 95 = 23. 911

Sudd = 100 (1. 189 11) 0.840 9652 = 84. 096 5


Vudd = 0

Sddd = 100 0.840 9653 = 59. 474 9


Vddd = 0
c
Yufeng
Guo

51

CHAPTER 10. BINOMIAL OPTION PRICING I


Vuu = erh ( u Vuuu + d Vuud ) = e(0.08)1/3 (0.456 807 73. 138 1 + 0.543 193 23. 911) =
45. 177 3
Vud = erh (u Vuud + d Vudd ) = e(0.08)1/3 (0.456 807 23. 911 + 0.543 193 0) =
10. 635 3
Vdd = erh ( u Vudd + d Vddd ) = e(0.08)1/3 (0.456 807 0 + 0.543 193 0) =
0
Vu = erh (u Vuu + d Vud ) = e(0.08)1/3 (0.456 807 45. 177 3 + 0.543 193 10. 635 3) =
25. 719 3
Vd = erh ( u Vud + d Vdd ) = e(0.08)1/3 (0.456 807 10. 635 3 + 0.543 193 0) =
4. 730 4
V = erh (u Vu + d Vd ) = e(0.08)1/3 (0.456 807 25. 719 3 + 0.543 193 4. 730 4) =
13. 941 5
Since the American call is early exercised at Node uu, the price of the American call 14. 183 0 is greater than the price of the European call 13. 941 5.
c. Calculate the American put premium.
Period 2

Suu = 100 1. 189 112 = 141. 398 3


EVuu = 0
Vuu =?
Sud = 100 (1. 189 11) (0.840 965) = 100
EVud = 0
Vud =?

Sdd = 100 0.840 9652 = 70. 722 2


EVdd = 95 70. 722 2 = 24. 277 8
Vdd =?
R
Vuu

Suuu = 100 1. 189 113 = 168. 138 1


Vuuu = 0

Suud = 100 1. 189 112 (0.840 965) = 118. 9110


Vuud = 0

Sudd = 100 (1. 189 11) 0.840 9652 = 84. 096 5


Vudd = 95 84. 096 5 = 10. 903 5

Sddd = 100 0.840 9653 = 59. 474 9


Vddd = 95 59. 474 9 = 35. 525 1
rh
(0.08)1/3
=e
( u Vuuu + d Vuud ) = e
(0.456 807 0 + 0.543 193 0) =

Vuu = max Vuu


, EVuu = max (0, 0) = 0

R
= erh (u Vuud + d Vudd ) = e(0.08)1/3 (0.456 807 0 + 0.543 193 10. 903 5) =
Vud
5. 766 9
R

Vud = max Vud


, EVud = max (5. 766 9, 0) = 5. 766 9

R
= erh ( u Vudd + d Vddd ) = e(0.08)1/3 (0.456 807 10. 903 5 + 0.543 193 35. 525 1) =
Vdd
23. 638 9
R

Vdd = max Vdd


, EVdd = max (23. 638 9, 24. 277 8) = 24. 277 8
The American put is early exercised at Node dd.

www.actuary88.com

c
Yufeng
Guo

52

CHAPTER 10. BINOMIAL OPTION PRICING I


Period 1

2
Vuu = 0

Su = 100 (1. 189 11) = 118. 911


EVu = 0
Vu =?
Vud = 5. 766 9
Sd = 100 (0.840 965) = 84. 096 5
EVd = 95 84. 096 5 = 10. 903 5
Vd =?
Vdd = 24. 277 8
VuR = erh ( u Vuu + d Vud ) = e(0.08)1/3 (0.456 807 0 + 0.543 193 5. 766 9) =
3. 050 1

Vu = max VuR , EVu = max (3. 050 1, 0) = 3. 050 1

VdR = erh ( u Vud + d Vdd ) = e(0.08)1/3 (0.456 807 5. 766 9 + 0.543 193 24. 277 8) =
15. 405 6

Vd = max VdR , EVd = max (15. 405 6, 10. 903 5) = 15. 405 6
Period 0

1
Vu = 3. 050 1

S = 100
EV = 0
Vu =?
Vd = 15. 405 6
V R = erh ( u Vu + d Vd ) = e(0.08)1/3 (0.456 807 3. 050 1 + 0.543 193 15. 405 6) =
9. 504 7

V = max V R , EV = max (9. 504 7, 0) = 9. 504 7


b. Calculate the European put premium. Verify that the put-call parity
holds.
Period 0 1
2
3

Suuu = 100 1. 189 113 = 168. 138 1


Vuuu = 0
Vuu

Vu
Suud = 100 1. 189 112 (0.840 965) = 118. 9110
Vuud = 0
V
Vud

Sudd = 100 (1. 189 11) 0.840 9652 = 84. 096 5


Vd
Vudd = 95 84. 096 5 = 10. 903 5
Vdd

Sddd = 100 0.840 9653 = 59. 474 9


Vddd = 95 59. 474 9 = 35. 525 1
www.actuary88.com

c
Yufeng
Guo

53

CHAPTER 10. BINOMIAL OPTION PRICING I

Vuu = erh ( u Vuuu + d Vuud ) = e(0.08)1/3 (0.456 807 0 + 0.543 193 0) =

Vud = erh (u Vuud + d Vudd ) = e(0.08)1/3 (0.456 807 0 + 0.543 193 10. 903 5) =
5. 766 9
Vdd = erh ( u Vudd + d Vddd ) = e(0.08)1/3 (0.456 807 10. 903 5 + 0.543 193 35. 525 1) =
23. 638 9
Vu = erh (u Vuu + d Vud ) = e(0.08)1/3 (0.456 807 0 + 0.543 193 5. 766 9) =
3. 050 1
Vd = erh ( u Vud + d Vdd ) = e(0.08)1/3 (0.456 807 5. 766 9 + 0.543 193 23. 638 9) =
15. 067 6
V = erh (u Vu + d Vd ) = e(0.08)1/3 (0.456 807 3. 050 1 + 0.543 193 15. 067 6) =
9. 325 9
So the European put is worth 9. 325 9.
Verify the put-call parity:
C + KerT = 13. 941 5 + 95e(0.08)1 = 101. 637 55
P + SeT = 9. 325 9 + 100e(0.08)1 = 101. 637 53
Ignoring rounding dierence, we get C + KerT = P + SeT
The put-call parity holds.
Problem 10.12.
a. h = T /n =0.5/2 = 0.25

u = e(r)h+ h = e(0.08)0.25+0.3 0.25 = 1. 185 305


d = e(r)h
u =

0.25

= e(0.08)0.250.3

= 0.878 095

e(r)h d
e(0.08)0.25 0.878 095
=
= 0.462 57
ud
1. 185 305 0.878 095

d = 1 u = 1 0.462 57 = 0.537 43
b. Since the stock doesnt pay dividend, the American call and an otherwise
identical European call have the same value.
Period 0 1
2

Suu = 40 1. 185 3052 = 56. 197 92


Vuu = 56. 197 92 40 = 16. 197 92
Vu =?
Sud = 40 (1. 185 305) (0.878 095) = 41. 632 4
V =?
Vud = 41. 632 4 40 = 1. 632 4
Vd =?

www.actuary88.com

Sdd = 40 0.878 0952 = 30. 842 03


Vdd = 0
c
Yufeng
Guo

54

CHAPTER 10. BINOMIAL OPTION PRICING I


Vu = erh (u Vuu + d Vud ) = e(0.08)0.25 (0.462 57 16. 197 92 + 0.537 43 1. 632 4) =
8. 204 2
Vd = erh ( u Vud + d Vdd ) = e(0.08)0.25 (0.462 57 1. 632 4 + 0.537 43 0) =
0.740 1
V = erh ( u Vu + d Vd ) = e(0.08)0.25 (0.462 57 8. 204 2 + 0.537 43 0.740 1) =
4. 109 7
So both the American call and the European call are worth 4. 109 7.
We also
Node
uu
ud
dd

calculate the European call price using the following shortcut:


Payo
Risk Neutral Prob
16. 197 92 2u = 0.462 572
1. 632 4
2 u d = 2 0.462 57 0.537 43
0
2d = 0.537 432

P
V = erT payof f RiskN eutralP rob

= e0.08(0.5) 16. 197 92 0.462 572 + 1. 632 4 2 0.462 57 0.537 43 + 0 0.537 432
= 4. 109 8
Please note that the above shortcut works only for European options.
c. Even if the stock doesnt pay dividend, it may be still optimal to exercise
an American put early.
Calculate the premium of the American put
Period 0

1
Su = 40 (1. 185 305) = 47. 412 2
EVu = 0
Vu =?

S = 40
EV = 0
V =?

Suu = 40 1. 185 3052 = 56. 197 92


Vuu = 0
Sud = 40 (1. 185 305) (0.878 095) = 41. 632 4
Vud = 0

Sd = 40 (0.878 095) = 35. 123 8


EVd = 40 35. 123 8 = 4. 876 2
Vd =?

Sdd = 40 0.878 0952 = 30. 842 03


Vdd = 40 30. 842 03 = 9. 157 97

VuR = erh ( u Vuu + d Vud ) = e(0.08)1/3 (0.456 807 0 + 0.543 193 0) =


0

Vu = max VuR , EVu = max (0, 0) = 0

VdR = erh ( u Vud + d Vdd ) = e(0.08)1/3 (0.456 807 0 + 0.543 193 9. 157 97) =
4. 843 6

Vd = max VdR , EVd = max (4. 843 6, 4. 876 2) = 4. 876 2


www.actuary88.com

c
Yufeng
Guo

55

CHAPTER 10. BINOMIAL OPTION PRICING I


So the American put is early exercised at Node d.
V R = erh (u Vu + d Vd ) = e(0.08)1/3 (0.456 807 0 + 0.543 193 4. 876 2) =
2. 579 0

V = max V R , EV = max (2. 579 0, 0) = 2. 579 0


The American put is worth 2. 579 0.

Calculate the premium of the European put


Period 0

Suu = 40 1. 185 3052 = 56. 197 92


Vuu = 0

Vu

Sud = 40 (1. 185 305) (0.878 095) = 41. 632 4


Vud = 0

Sdd = 40 0.878 0952 = 30. 842 03


Vdd = 40 30. 842 03 = 9. 157 97

Vd

Vu = erh (u Vuu + d Vud ) = e(0.08)1/3 (0.456 807 0 + 0.543 193 0) = 0


Vd = erh ( u Vud + d Vdd ) = e(0.08)1/3 (0.456 807 0 + 0.543 193 9. 157 97) =
4. 843 6
V = erh (u Vu + d Vd ) = e(0.08)1/3 (0.456 807 0 + 0.543 193 4. 843 6) =
2. 561 8
The European put is worth 2. 561 8.
Problem 10.13.
a. From the previous problem, we know:
Period 0
1

S = 40
V = 4. 109 7
(, B)

Su = 40 (1. 185 305) = 47. 412 2


Vu = 8. 204 2
(u , Bu )

Sd = 40 (0.878 095) = 35. 123 8


Vd = 0.740 1
(d , Bd )

Suu = 40 1. 185 3052 = 56. 197 92


Vuu = 56. 197 92 40 = 16. 197 92
Sud = 40 (1. 185 305) (0.878 095) = 41. 632 4
Vud = 41. 632 4 40 = 1. 632 4

Sdd = 40 0.878 0952 = 30. 842 03


Vdd = 0

First, let find the replicating portfolio.


www.actuary88.com

c
Yufeng
Guo

56

CHAPTER 10. BINOMIAL OPTION PRICING I


The replicating portfolio at t = 0:

4Su + Berh = Vu
4Sd + Berh = Vd

447. 412 2 + Be0.08(0.25) = 8. 204 2


435. 123 8 + Be0.08(0.25) = 0.740 1

4 = 0.607 41

B = 20. 186 7

The replicating portfolio Node u:

4u Suu + Bu erh = Vuu


4u Sud + Berh = Vud

207 9

4u 56. 197 92 + Bu e0.08(0.25) = 16. 197 92


4u 41. 632 4 + Bu e0.08(0.25) = 1. 632 4

4u = 1

Bu = 39.

The replicating portfolio Node d:

4d Sud + Bd erh = Vud


4d Sdd + Berh = Vdd

4d 41. 632 4 + Bd e0.08(0.25) = 1. 632 4


4d 30. 842 03 + Bd e0.08(0.25) = 0

4d = 0.151 28

Bd = 4.

573 5
At t = 0, we

sell the over-priced call for $5


build a synthetic call by buying 4 = 0.607 41 share of the stock and
borrowing $20. 186 7 from a bank. Cost: 4S + B = 0.607 41 (40) 20.
186 7 = 4. 109 7.
So we receive 5 4. 109 7 = 0.890 3 at t = 0.
b.
Suppose the call in the market at time h is fairly priced
To liquidate our position at t = h (i.e. the end of Period 1) yet hedge our
sold call, well do the following:
1. Sell our 4 = 0.607 41 share of the stock, receiving 4Sh
2. Pay the bank Berh = 20. 186 7e0.08(0.25) = 20. 594 5
www.actuary88.com

c
Yufeng
Guo

57

CHAPTER 10. BINOMIAL OPTION PRICING I


3. Buy the call from the open market for the fair price of Vh , which is either
Vu = 8. 204 2 if the stock price goes up or Vd = 0.740 1 if the stock price
goes down.

Suppose the stock price goes up to Su and we liquidate our position at Node
u. Well
1. Sell our 4 = 0.607 41 share of the stock, receiving 4Su = 0.607 41 (47. 412 2) =
$28. 798 6
2. Pay the bank Berh = 20. 186 7e0.08(0.25) = $20. 594 5
3. Buy the call from the open market for the fair price of Vu = $8. 204 2
The net receipt is: 28. 798 6 20. 594 5 8. 204 2 = 0.000 1 = $0 (if we
ignore rounding errors)
So we receive $0.890 3 free money at t = 0 without incurring any liability at
time h.
Suppose the stock price goes up to Sd and we liquidate our position at Node
u. Well
1. Sell our 4 = 0.607 41 share of the stock, receiving 4Sd = 0.607 41 (35. 123 8) =
$21. 334 5
2. Pay the bank Berh = 20. 186 7e0.08(0.25) = $20. 594 5
3. Buy the call from the open market for the fair price of Vd = $0.740 1
The net receipt is: 21. 334 5 20. 594 5 0.740 1 = 0.000 1 = 0 (if we ignore
rounding errors)
So we receive $0.890 3 free money at t = 0 without incurring any liability at
time h.

Suppose the call in the market at h is over priced


Well not liquidate our position at h. Instead, we rebalance our replicating
portfolio at t = h and liquidate our position at T = 2h. Well consider two
situation.
Situation #1 the stock price goes up to Su = 40 (1. 185 305) = 47. 412 2
at time h
Well change our replicating portfolio from (, B) = (0.607 41, B = $20. 186 7)
to (u , Bu ) = (1, $39. 207 9) at h. We need to buy 1 0.607 41 = 0.392 59
share of the stock. The cost is 0.392 59 (47. 412 2) = 18. 613 6. Well borrow 18.
613 6 from the bank to pay for this.
www.actuary88.com

c
Yufeng
Guo

58

CHAPTER 10. BINOMIAL OPTION PRICING I


So at time h, our total debt to the bank is: 18. 613 4 + 20. 186 7e0.08(0.25) =
39. 207 9. The number of stocks we have is 1. Now at time h, our replicating
portfolio is exactly (u , Bu ) = (1, $39. 207 9).
Then at T =
2h

If Suu = 40 1. 185 3052 = 56. 197 92

1. Our written call is exercised against us. We need to pay the call holder
Vuu = 56. 197 92 40 = 16. 197 92.
2. We sell u stocks in the market and pay o our loan from the bank. Our
net cash receipt is u Suu + Bu erh = 1 56. 197 92 39. 207 9e0.08(0.25) =
16. 197 97.
3. Our net cash flow is zero (ignoring rounding).
If Sud = 40 (1. 185 305) (0.878 095) = 41. 632 4
1. Our written call is exercised against us. We need to pay the call holder
Vud = 41. 632 4 40 = 1. 632 4

2. We sell u stocks in the market and pay o our loan from the bank. Our
net cash receipt is u Sud + Bu erh = 1 41. 632 4 39. 207 9e0.08(0.25) =
1. 632 4
3. Our net cash flow is zero.
So we receive $0.890 3 free money at t = 0 without incurring any liability at
T.
Situation #2 the stock price goes down to Sd = 40 (0.878 095) = 35.
123 8 at time h
Well change our replicating portfolio from (, B) = (0.607 41, B = $20. 186 7)
to (d , Bd ) = (0.151 28, $4. 573 5) at h. We need to sell 0.607 4 0.151 28 =
0.456 12 share of the stock, receiving 0.456 12 (35. 123 8) = $16. 020 7. We immediately send a check of $16. 020 7 to the bank to partially pay our debt. Now
our remaining debt to the bank is
20. 186 7e0.08(0.25) 16. 020 7 = 4. 573 8 = 4. 573 5 (ignore rounding)
Now at time h, our replicating portfolio is exactly (d , Bd ) = (0.151 28, $4. 573 5)
Then at T = 2h
If Sdu = 40 (1. 185 305) (0.878 095) = 41. 632 4
1. Our written call is exercised against us. We need to pay the call holder
Vdu = 41. 632 4 40 = 1. 632 4
2. We sell d stocks in the market and pay o our loan from the bank. Our
net cash receipt is d Sdu +Bd erh = 0.151 2841. 632 44. 573 5e0.08(0.25) =
1. 632 3
3. Our net cash flow is zero (ignoring rounding).
www.actuary88.com

c
Yufeng
Guo

59

CHAPTER 10. BINOMIAL OPTION PRICING I

If Sdd = 40 0.878 0952 = 30. 842 03


1. Our written call expires worthless.
2. We sell d stocks in the market and pay o our loan from the bank. Our
net cash receipt is d Sdd +Bd erh = 0.151 2830. 842 034. 573 5e0.08(0.25)
0
3. Our net cash flow is zero.
So we receive $0.890 3 free money at t = 0 without incurring any liability at
T.

c. If the call option is under priced at h, we just sell o our replicating


portfolio (i.e. sell share of the stock and pay back our loan to the bank) and
purchase a call from the market. Because the value of our replicating portfolio
at h represents the fair price of a call and the call in the market is lower than
the fair price, the money we get from selling our replicating portfolio exceeds
the call price in the market. So well make some profit.
Now we are standing at time h. The call we buy at time h has life from time
h to T = 2h. The call we initially sold at time zero has life from h to T = 2h.
These two calls exactly oset each other so our liability at T is zero.

Problem 10.14.
We can use the general binomial tree formula by setting S = 0.92 and
= r = 0.03.
h = T /n = 0.75/3 = 0.25

u = 1.2
d = 0.9
e(r)h d
e(0.040.03)0.25 0.9
u =
=
= 0.341 677
ud
1.2 0.9
d = 1 u = 1 0.341 677 = 0.658 323
Well calculate part b first.
b. Calculate the American call premium.
www.actuary88.com

c
Yufeng
Guo

60

CHAPTER 10. BINOMIAL OPTION PRICING I


Period 2

Suu = 0.92 1. 22 = 1. 324 8


EVuu = 1. 324 8 0.85 = 0.474 8
Vuu =?
Sud = 0.92 (1. 2) (0.9) = 0.993 6
EVud = 0.993 6 0.85 = 0.143 6
Vud =?

Sdd = 0.92 0.92 = 0.745 2


EVdd = 0
Vdd =?

Suuu = 0.92 1. 23 = 1. 589 76


Vuuu = 1. 589 76 0.85 = 0.739 76

Suud = 0.92 1. 22 (0.9) = 1. 192 32


Vuud = 1. 192 32 0.85 = 0.342 32

Sudd = 0.92 (1. 2) 0.92 = 0.894 24


Vudd = 0.894 24 0.85 = 0.044 24

Sddd = 0.92 0.93 = 0.670 68


Vddd = 0

R
Vuu
= erh ( u Vuuu + d Vuud ) = e(0.04)0.25 (0.341 677 0.739 76 + 0.658 323 0.342 32) =
0.473 359
R

Vuu = max Vuu


, EVuu = max (0.473 359, 0.474 8) = 0.474 8
So the American call is early exercised at the uu node.
R
Vud
= erh ( u Vuud + d Vudd ) = e(0.04)0.25 (0.341 677 0.342 32 + 0.658 323 0.044 24) =
0.144 633

Vud = max Vud


, EVud = max (0.144 633, 0.143 6) = 0.144 633
R
Vdd
= erh (u Vudd + d Vddd ) = e(0.04)0.25 (0.341 677 0.044 24 + 0.658 323 0) =
0.01 496 5
R

Vdd = max Vdd


, EVdd = max (0.01 496 5, 0) = 0.014 965
Period 1

2
Vuu = 0.474 8

Su = 0.92 (1. 2) = 1. 104


EVu = 1. 104 0.85 = 0.254
Vu =?
Vud = 0.144 633
Sd = 0.92 (0.9) = 0.828
EVd = 0
Vd =?
Vdd = 0.01 496 5
VuR = erh ( u Vuu + d Vud ) = e(0.04)0.25 (0.341 677 0.474 8 + 0.658 323 0.144 633) =
0.254 882

Vu = max VuR , EVu = max (0.254 882 , 0.254) = 0.254 882


VdR = erh ( u Vud + d Vdd ) = e(0.04)0.25 (0.341 677 0.144 633 + 0.658 323 0.01 496 5) =
0.05 8680

Vd = max VdR , EVd = max (0.05 8680 , 0) = 0.058 68


www.actuary88.com

c
Yufeng
Guo

61

CHAPTER 10. BINOMIAL OPTION PRICING I


Period 0

1
Vu = 0.254 882

S = 0.92
EV = 0.92 0.85 = 0.07
V =?
Vd = 0.058 68
V R = erh (u Vu + d Vu ) = e(0.04)0.25 (0.341 677 0.254 882 + 0.658 323 0.058 68) =
0.124 467

V = max V R , EV = max (0.124 467 , 0.07) = 0.124 467


So the American call premium is 0.124 467 .

a. Calculate the European call premium.


Period 0 1
2
3

Suuu = 0.92 1. 23 = 1. 589 76


Vuuu = 1. 589 76 0.85 = 0.739 76
Vuu

Suud = 0.92 1. 22 (0.9) = 1. 192 32


Vu
Vuud = 1. 192 32 0.85 = 0.342 32
V

Vud
Vd
Vdd

Sudd = 0.92 (1. 2) 0.92 = 0.894 24


Vudd = 0.894 24 0.85 = 0.044 24

Sddd = 0.92 0.93 = 0.670 68


Vddd = 0

Vuu = erh ( u Vuuu + d Vuud ) = e(0.04)0.25 (0.341 677 0.739 76 + 0.658 323 0.342 32) =
0.473 359
Vud = erh (u Vuud + d Vudd ) = e(0.04)0.25 (0.341 677 0.342 32 + 0.658 323 0.044 24) =
0.144 633
Vdd = erh ( u Vudd + d Vddd ) = e(0.04)0.25 (0.341 677 0.044 24 + 0.658 323 0) =
0.01 496 5
Vu = erh (u Vuu + d Vud ) = e(0.04)0.25 (0.341 677 0.473 359 + 0.658 323 0.144 633) =
0.254 394
VdR = erh (u Vud + d Vdd ) = e(0.04)0.25 (0.341 677 0.144 633 + 0.658 323 0.01 496 5) =
0.05 8680
V = erh (u Vu + d Vu ) = e(0.04)0.25 (0.341 677 0.254 394 + 0.658 323 0.058 68) =
0.124 302
So the European call premium is 0.124 302 .
www.actuary88.com

c
Yufeng
Guo

62

CHAPTER 10. BINOMIAL OPTION PRICING I


Problem 10.15.
We can use the general binomial tree formula by setting S = 0.92 and
= r = 0.03.
h = T /n = 0.75/3 = 0.25

u = 1.2
d = 0.9
e(r)h d
e(0.040.03)0.25 0.9
u =
=
= 0.341 677
ud
1.2 0.9
d = 1 u = 1 0.341 677 = 0.658 323
b. Calculate the American put premium.
Period 2
3

Suuu = 0.92 1. 23 = 1. 589 76


2
Suu = 0.92 1. 2 = 1. 324 8
Vuuu = 0
EVuu = 0

Vuu =?
Suud = 0.92 1. 22 (0.9) = 1. 192 32
Vuud = 0
Sud = 0.92 (1. 2) (0.9) = 0.993 6
EVud = 1 0.993 6 = 0.006 4
Vud =?
2
S
=
0.92
(1.
2)
0.9 = 0.894 24
udd

Sdd = 0.92 0.92 = 0.745 2


Vudd = 1 0.894 24 = 0.105 76
EVdd = 1 0.745 2 = 0.254 8

Vdd =?
Sddd = 0.92 0.93 = 0.670 68
Vddd = 1 0.670 68 = 0.329 32

R
Vuu
= erh ( u Vuuu + d Vuud ) = e(0.04)0.25 (0.341 677 0 + 0.658 323 0) =

, EVuu = max (0, 0) = 0


Vuu = max Vuu

R
= erh ( u Vuud + d Vudd ) = e(0.04)0.25 (0.341 677 0 + 0.658 323 0.105 76) =
Vud
0.06893 1

, EVud = max (0.06893 1, 0.006 4) = 0.068 931


Vud = max Vud

R
= erh (u Vudd + d Vddd ) = e(0.04)0.25 (0.341 677 0.105 76 + 0.658 323 0.329 32) =
Vdd
0.250 418
R

, EVdd = max (0.250 418, 0.254 8) = 0.254 8


Vdd = max Vdd
The American put is early exercised at the dd node.

www.actuary88.com

c
Yufeng
Guo

63

CHAPTER 10. BINOMIAL OPTION PRICING I


Period 1

2
Vuu = 0

Su = 0.92 (1. 2) = 1. 104


EVu = 0
Vu =?
Vud = 0.068 931
Sd = 0.92 (0.9) = 0.828
EVd = 1 0.828 = 0.172
Vd =?
Vdd = 0.254 8
VuR = erh (u Vuu + d Vud ) = e(0.04)0.25 (0.341 677 0 + 0.658 323 0.068 931 ) =
0.04 492 7

Vu = max VuR , EVu = max (0.04 492 7 , 0) = 0.04 492 7


VdR = erh (u Vud + d Vdd ) = e(0.04)0.25 (0.341 677 0.068 931 + 0.658 323 0.254 8) =
0.189 389

Vd = max VdR , EVd = max (0.189 389 , 0.172) = 0.189 389


Period 0

1
Vu = 0.04 492 7

S = 0.92
EV = 1 0.92 = 0.08
V =?
Vd = 0.189 389
V R = erh (u Vu + d Vu ) = e(0.04)0.25 (0.341 677 0.04 492 7 + 0.658 323 0.189 389) =
0.138 636

V = max V R , EV = max (0.138 636 , 0.08) = 0.138 636


So the American put premium is 0.138 636 .

a. Calculate the European put premium.


Period 0 1
2
3

Suuu = 0.92 1. 23 = 1. 589 76


Vuuu = 0
Vuu

Vu
Suud = 0.92 1. 22 (0.9) = 1. 192 32
Vuud = 0
V
Vud

Sudd = 0.92 (1. 2) 0.92 = 0.894 24


Vd
Vudd = 1 0.894 24 = 0.105 76
Vdd

Sddd = 0.92 0.93 = 0.670 68


Vddd = 1 0.670 68 = 0.329 32
www.actuary88.com

c
Yufeng
Guo

64

CHAPTER 10. BINOMIAL OPTION PRICING I


Vuu = erh ( u Vuuu + d Vuud ) = e(0.04)0.25 (0.341 677 0 + 0.658 323 0) =
0
Vud = erh ( u Vuud + d Vudd ) = e(0.04)0.25 (0.341 677 0 + 0.658 323 0.105 76) =
0.06893 1
Vdd = erh (u Vudd + d Vddd ) = e(0.04)0.25 (0.341 677 0.105 76 + 0.658 323 0.329 32) =
0.250 418
Vu = erh (u Vuu + d Vud ) = e(0.04)0.25 (0.341 677 0 + 0.658 323 0.068 931 ) =
0.04 492 7
Vd = erh ( u Vud + d Vdd ) = e(0.04)0.25 (0.341 677 0.068 931 + 0.658 323 0.250 418) =
0.186 533
V = erh ( u Vu + d Vu ) = e(0.04)0.25 (0.341 677 0.04 492 7 + 0.658 323 0.186 533) =
0.136 775
So the European put premium is 0.136 775 .
Problem 10.16.
This problem looks scary, but its actually simple if you know whats the
underlying asset.
The underlying asset is $1. The call option gives the call holder the right to
purchase $1 with a guaranteed price (i.e. strike price) of 120 Yen. So the strike
price and the option price are expressed in Yen.
To simplify the problem, we can treat the underlying asset $1 as a stock and
translate the original problem into the following:
Now we are living in Japan and are interested in options on a stock.
The current stock price is S = 120 Yen.
The continuously compounded risk free interest rate is rf = 0.01 (remember we are living in Japan).
The stocks volatility is = 0.1.
The stocks continuous dividend yield is = 0.05; the dollar interest rate
is like the dividend yield.
T =1
h = T /3 = 1/3
Now we can use the standard binomial formula.

u = e(r)h+ h = e(0.010.05)1/3+0.1 1/3 = 1. 045 402


www.actuary88.com

c
Yufeng
Guo

65

CHAPTER 10. BINOMIAL OPTION PRICING I


d = e(r)h
u =

= e(0.010.05)1/30.1 1/3 = 0.931 398

e(r)h d
e(0.010.05)1/3 0.931 398
=
= 0.485 57
ud
1. 045 402 0.931 398

d = 1 u = 1 0.485 57 = 0.514 43

a. Calculate the price of the American call option


Period 2
3

Suuu = 120 1. 045 4023 = 137. 098 0

Suu = 120 1. 045 4022 = 131. 143 8


Vuuu = 137. 098 0 120 = 17. 098
EVuu = 131. 143 8 120 = 11. 143 8

Vuu =?
Suud = 120 1. 045 4022 (0.931 398) = 122. 147 1
Vuud = 122. 147 1 120 = 2. 147 1
Sud = 120 (1. 045 402) (0.931 398) = 116. 842 2
EVud = 0
Vud =?

Sudd = 120 (1. 045 402) 0.931 3982 = 108. 826 6

Sdd = 120 0.931 3982 = 104. 100 3


Vudd = 0
EVdd = 0

Vdd =?
Sddd = 120 0.931 3983 = 96. 958 8
Vddd = 0
R
Vuu
= erh ( u Vuuu + d Vuud ) = e(0.01)1/3 (0.485 57 17. 098 + 0.514 43 2. 147 1) =
9. 375 5
R

Vuu = max Vuu


, EVuu = max (9. 375 5, 11. 143 8) = 11. 143 8
So the American call is early exercised at the uu node.
R
= erh (u Vuud + d Vudd ) = e(0.01)1/3 (0.485 57 2. 147 1 + 0.514 43 0) =
Vud
1. 039 1
R

, EVud = max (1. 039 1, 0) = 1. 039 1


Vud = max Vud

R
= erh (u Vudd + d Vddd ) = e(0.01)1/3 (0.485 57 0 + 0.514 43 0) =
Vdd

Vdd = max Vdd


, EVdd = max (0, 0) = 0
Period 1

2
Vuu = 11. 143 8

Su = 120 (1. 045 402) = 125. 448 2


EVu = 125. 448 2 120 = 5. 448 2
Vu =?
Vud = 1. 039 1
Sd = 120 (0.931 398) = 111. 767 8
EVd = 0
Vd =?
Vdd = 0
www.actuary88.com

c
Yufeng
Guo

66

CHAPTER 10. BINOMIAL OPTION PRICING I


VuR = erh ( u Vuu + d Vud ) = e(0.01)1/3 (0.485 57 11. 143 8 + 0.514 43 1. 039 1) =
5. 925 9

Vu = max VuR , EVu = max (5. 925 9 , 5. 448 2) = 5. 925 9


VdR = erh ( u Vud + d Vdd ) = e(0.01)1/3 (0.485 57 1. 039 1 + 0.514 43 0) =
0.502 9

Vd = max VdR , EVd = max (0.502 9 , 0) = 0.502 9


Period 0

1
Vu = 5. 925 9

S = 120
EV = 0
V =?
Vd = 0.502 9
V R = erh ( u Vu + d Vu ) = e(0.01)1/3 (0.485 57 5. 925 9 + 0.514 43 0.502 9) =
3. 125 7

V = max V R , EV = max (3. 125 7 , 0) = 3. 125 7


So the American call premium is 3. 125 7 Yen.

Calculate the price of the European call


Period 0 1
2
3

Suuu = 120 1. 045 4023 = 137. 098 0


Vuuu = 137. 098 0 120 = 17. 098
Vuu

Vu
Suud = 120 1. 045 4022 (0.931 398) = 122. 147 1
Vuud = 122. 147 1 120 = 2. 147 1
V
Vud

Sudd = 120 (1. 045 402) 0.931 3982 = 108. 826 6


Vd
Vudd = 0
Vdd

Sddd = 120 0.931 3983 = 96. 958 8


Vddd = 0
Vuu = erh ( u Vuuu + d Vuud ) = e(0.01)1/3 (0.485 57 17. 098 + 0.514 43 2. 147 1) =
9. 375 5
Vud = erh ( u Vuud + d Vudd ) = e(0.01)1/3 (0.485 57 2. 147 1 + 0.514 43 0) =
1. 039 1
Vdd = erh ( u Vudd + d Vddd ) = e(0.01)1/3 (0.485 57 0 + 0.514 43 0) =
0
Vu = erh (u Vuu + d Vud ) = e(0.01)1/3 (0.485 57 9. 375 5 + 0.514 43 1. 039 1) =
5. 070 1
www.actuary88.com

c
Yufeng
Guo

67

CHAPTER 10. BINOMIAL OPTION PRICING I


Vd = erh ( u Vud + d Vdd ) = e(0.01)1/3 (0.485 57 1. 039 1 + 0.514 43 0) =
0.502 9
V = erh (u Vu + d Vu ) = e(0.01)1/3 (0.485 57 5. 070 1 + 0.514 43 0.502 9) =
2. 711 5
So the European call premium is 2. 711 5 Yen.
b. Calculate the price of the American put
Period 2

Suu = 120 1. 045 4022 = 131. 143 8


EVuu = 0
Vuu =?
Sud = 120 (1. 045 402) (0.931 398) = 116. 842 2
EVud = 120 116. 842 2 = 3. 157 8
Vud =?

Sdd = 120 0.931 3982 = 104. 100 3


EVdd = 120 104. 100 3 = 15. 899 7
Vdd =?

Suuu = 120 1. 045 4023 = 137. 098 0


Vuuu = 0

Suud = 120 1. 045 4022 (0.931 398) = 122. 147 1


Vuud = 0

Sudd = 120 (1. 045 402) 0.931 3982 = 108. 826 6


Vudd = 120 108. 826 6 = 11. 173 4

Sddd = 120 0.931 3983 = 96. 958 8


Vddd = 120 96. 958 8 = 23. 041 2

R
Vuu
= erh ( u Vuuu + d Vuud ) = e(0.01)1/3 (0.485 57 0 + 0.514 43 0) =

Vuu = max Vuu


, EVuu = max (0, 0) = 0

R
= erh (u Vuud + d Vudd ) = e(0.01)1/3 (0.485 57 0 + 0.514 43 11. 173 4) =
Vud
5. 728 8
R

, EVud = max (3. 157 8, 5. 728 8) = 5. 728 8


Vud = max Vud

R
= erh ( u Vudd + d Vddd ) = e(0.01)1/3 (0.485 57 11. 173 4 + 0.514 43 23. 041 2) =
Vdd
17. 221 1
R

Vdd = max Vdd


, EVdd = max (17. 221 1, 15. 899 7) = 17. 221 1

Period 1

2
Vuu = 0

Su = 120 (1. 045 402) = 125. 448 2


EVu = 0
Vu =?
Vud = 5. 728 8
Sd = 120 (0.931 398) = 111. 767 8
EVd = 120 111. 767 8 = 8. 232 2
Vd =?
Vdd = 17. 221 1
www.actuary88.com

c
Yufeng
Guo

68

CHAPTER 10. BINOMIAL OPTION PRICING I


VuR = erh ( u Vuu + d Vud ) = e(0.01)1/3 (0.485 57 0 + 0.514 43 5. 728 8) =
2. 937 3

Vu = max VuR , EVu = max (2. 937 3 , 0) = 2. 937 3

VdR = erh ( u Vud + d Vdd ) = e(0.01)1/3 (0.485 57 5. 728 8 + 0.514 43 17. 221 1) =
11. 602 0

Vd = max VdR , EVd = max (11. 602 0 , 8. 232 2) = 11. 602


Period 0

1
Vu = 2. 937 3

S = 120
EV = 0
V =?
Vd = 11. 602
V R = erh ( u Vu + d Vu ) = e(0.01)1/3 (0.485 57 2. 937 3 + 0.514 43 11. 602) =
7. 370 1

V = max V R , EV = max (7. 370 1 , 0) = 7. 370 1

Since the exercise value is never greater than the roll-back value, the American put is not early exercised. Hence the American put and the European put
have the same value.
Both the American put and European put are worth 7. 370 1 Yen.

c. Since the underlying asset generates dividend (the Yen interest rate is
like the dividend rate), it may be optimal to early exercise an American call.
However, even if the asset generates dividend, it may not be optimal to early
exercise an American put.

Problem 10.17.
Method 1
Apply the stock binomial formula to futures
As explained in my study guide, we can apply the stock binomial formula
to options on futures if we do the following three things:
Set the dividend yield equal to the risk free rate (i.e. = r)
Cu Cd
Cu Cd
instead of 4 = erh
Su Sd
Su Sd

1d
u1
Su Cd Sd Cu
B = V = erh Cu
.
+ Cd
instead of B = erh
ud
ud
Su Sd

4=

www.actuary88.com

c
Yufeng
Guo

69

CHAPTER 10. BINOMIAL OPTION PRICING I


h = 1 year long.

u = e(r)h+ h = e h= e0.1 1 = 1. 105 170 9


d = e(r)h h = e0.1 1 = 0.904 837 4
e(r)h d
1d
1 0.904 837 4
u =
=
=
= 0.475 021
ud
ud
1. 105 170 9 0.904 837 4
d = 1 u = 1 0.475 021 = 0.524 979
Period 0

S = 300
EV = 300 290 = 10
V =?
(, B) =?

1
Su = 300 (1. 105 170 9) = 331. 551 27
Vu = 331. 551 27 290 = 41. 551 27

Sd = 300 (0.904 837 4) = 271. 451 22


Vd = 0
V R = erh (u Vu + d Vu ) = e(0.06)1 (0.475 021 41. 551 27 + 0.524 979 0) =
18. 588 29

V = max V R , EV = max (18. 588 29 , 10) = 18. 588 29


So the call premium is $18. 588 29 at t = 0.
41. 551 27 0
Cu Cd
=
= 0.691 368
Su Sd
331. 551 27 271. 451 22
B = V = 18. 588 29

4=

So to form a replicating portfolio at time zero, we buy 0.691 368 unit of


futures contract (i.e. enter 0.691 368 unit of futures contract as a buyer) and
invest $18. 588 29 in a savings account.
Method 2
Just asin Method
1, we calculate

h
= e0.1 1 = 1. 105 170 9
u = e
d = e h = e0.1 1 = 0.904 837 4
Next, we build a futures price tree.
Period 0
1
u
F1,1
= 300 (1. 105 170 9) = 331. 551 27
Vu = 331. 551 27 290 = 41. 551 27
F0,1 = 300
EV = 300 290 = 10
V =?
(, B) =?
d
F1,1
= 300 (0.904 837 4) = 271. 451 22
Vd = 0
www.actuary88.com

c
Yufeng
Guo

70

CHAPTER 10. BINOMIAL OPTION PRICING I


If we buy futures contract at t = 0, our gain in these futures contracts at
T is:

u
F0,1 = F0,1 (u 1) in the u node
F1,1
d

F1,1
F0,1 = F0,1 (d 1) in the d node

If we put $B in a savings account, well get BerT at T .


We want our replicating portfolio and the call option have the same payo
at T:

u
F0,1 + BerT = Vu
F1,1
F0,1 (u 1) + BerT = Vu

d
rT
F0,1 (d 1) + BerT = Vd
F1,1 F0,1 + Be = Vd

300 (1. 105 170 9 1) + Be0.06(1) = 41. 551 27


300 (0.904 837 4 1) + Be0.06(1) = 0
= 0.691 368
B = 18. 588 29

Since it costs nothing to enter a future contract, the call premium is equal
to B:
V = B = 18. 588 29

The statement "Replicating a call option always entails borrowing to buy


the underlying asset" is true for calls on stocks. However, its not true for calls
on futures; it costs nothing to enter a futures contract (so you dont need to
borrow money to finance your transaction on futures). If you write a call on
futures, you need to enter futures contracts and simultaneously deposit the
call premium into a savings account. This way, at call expiration date T , the
combined payo of your futures contracts and your deposit in the savings
account will replicate the payo of a call on futures.

Problem 10.18.
Well apply the stock binomial formula to futures. We need to do the following three things:
Set the dividend yield equal to the risk free rate (i.e. = r)
Cu Cd
Cu Cd
instead of 4 = erh
Su Sd
Su Sd

1d
u1
Su Cd Sd Cu
B = V = erh Cu
.
+ Cd
instead of B = erh
ud
ud
Su Sd

4=

www.actuary88.com

c
Yufeng
Guo

71

CHAPTER 10. BINOMIAL OPTION PRICING I


h = 1/3

= e0.3 1/3 = 1. 189 110


u = e(r)h+ h = e h
d = e(r)h h = e0.3 1/3 = 0.840 965
Notice that ud = 1
e(r)h d
1d
1 0.840 965
=
=
= 0.456 807
u =
ud
ud
1. 189 110 0.840 965
d = 1 u = 1 0.456 807 = 0.543 193

calculate the American call premium


Period 2
Suu = 1000u2 = 1413. 982 6
EVuu = 1413. 982 6 1000 = 413. 982 6
Vuu =?
Sud = 1000ud = 1000
EVud = 0
Vud =?

3
Suuu = 1000u3 = 1681. 380 8
Vuuu = 1681. 380 8 1000 = 681. 380 8
Suud = 1000u2 d = 1000u = 1189. 1100
Vuud = 1189. 11 1000 = 189. 11

Sudd = 1000ud2 = 1000d = 840. 965


Vudd = 0

Sdd = 1000d = 707. 222 1


EVdd = 0
Vdd =?

Sddd = 1000d3 = 1000 0.840 965 3 = 594. 749


Vddd = 0

R
Vuu
= erh ( u Vuuu + d Vuud ) = e(0.05)1/3 (0.456 807 681. 380 8 + 0.543 193 189. 11) =
407. 140 2
R

Vuu = max Vuu


, EVuu = max (407. 140 2, 413. 982 6) = 413. 982 6

The American call is early exercised at Node uu.


R
Vud
= erh (u Vuud + d Vudd ) = e(0.05)1/3 (0.456 807 189. 11 + 0.543 193 0) =
84. 958 9
R

, EVud = max (84. 958 9, 0) = 84. 958 9


Vud = max Vud

R
= erh ( u Vudd + d Vddd ) = e(0.05)1/3 (0.456 807 0 + 0.543 193 0) =
Vdd

Vdd = max Vdd


, EVdd = max (0, 0) = 0

www.actuary88.com

c
Yufeng
Guo

72

CHAPTER 10. BINOMIAL OPTION PRICING I


Period 1

2
Vuu = 413. 982 6

Su = 1000 (1. 189 11) = 1189. 11


EVu = 1189. 11 1000 = 189. 11
Vu =?
Vud = 84. 958 9
Sd = 1000 (0.840 965 ) = 840. 965
EVd = 0
Vd =?
Vdd = 0
VuR = erh ( u Vuu + d Vud ) = e(0.05)1/3 (0.456 807 413. 982 6 + 0.543 193 84. 958 9) =
231. 370 7

Vu = max VuR , EVu = max (231. 370 7 , 189. 11) = 231. 370 7
VdR = erh ( u Vud + d Vdd ) = e(0.05)1/3 (0.456 807 84. 958 9 + 0.543 193 0) =
38. 168 4

Vd = max VdR , EVd = max (38. 168 4 , 0) = 38. 168 4


Period 0

1
Vu = 231. 370 7

S = 1000
EV = 0
V =?
Vd = 38. 168 4
V R = erh ( u Vu + d Vu ) = e(0.05)1/3 (0.456 807 231. 370 7 + 0.543 193 38. 168 4) =
124. 334 9

V = max V R , EV = max (124. 334 9 , 0) = 124. 334 9


So the American call premium is 124. 334 9.
Calculate the European call premium
Period 0

Vuu
Vu
V

Vud
Vd
Vdd

www.actuary88.com

3
Suuu = 1000u3 = 1681. 380 8
Vuuu = 1681. 380 8 1000 = 681. 380 8
Suud = 1000u2 d = 1000u = 1189. 1100
Vuud = 1189. 11 1000 = 189. 11
Sudd = 1000ud2 = 1000d = 840. 965
Vudd = 0

Sddd = 1000d3 = 1000 0.840 965 3 = 594. 749


Vddd = 0
c
Yufeng
Guo

73

CHAPTER 10. BINOMIAL OPTION PRICING I


Vuu = erh ( u Vuuu + d Vuud ) = e(0.05)1/3 (0.456 807 681. 380 8 + 0.543 193 189. 11) =
407. 140 2
Vud = erh (u Vuud + d Vudd ) = e(0.05)1/3 (0.456 807 189. 11 + 0.543 193 0) =
84. 958 9
Vdd = erh ( u Vudd + d Vddd ) = 0
Vu = erh (u Vuu + d Vud ) = e(0.05)1/3 (0.456 807 407. 140 2 + 0.543 193 84. 958 9) =
228. 296 7
Vd = erh ( u Vud + d Vdd ) = e(0.05)1/3 (0.456 807 84. 958 9 + 0.543 193 0) =
38. 168 4
V = erh (u Vu + d Vu ) = e(0.05)1/3 (0.456 807 228. 296 7 + 0.543 193 38. 168 4) =
122. 953 9
So the European call premium is $122. 953 9.
Time zero replicating portfolio for the European call

B = V = $122. 953 9
Vu Vd
228. 296 7 38. 168 4
4=
=
= 0.546 118
Su Sd
1000 (1. 189 11) 1000 (0.840 965 )
calculate the American put premium
Period 2

3
Suuu = 1000u3 = 1681. 380 8
Vuuu = 0

Suu = 1000u2 = 1413. 982 6


EVuu = 0
Vuu =?

Suud = 1000u2 d = 1000u = 1189. 1100


Vuud = 0

Sud = 1000ud = 1000


EVud = 0
Vud =?
2

Sdd = 1000d = 707. 222 1


EVdd = 1000 707. 222 1 = 292. 777 9
Vdd =?

Sudd = 1000ud2 = 1000d = 840. 965


Vudd = 1000 840. 965 = 159. 035

Sddd = 1000d3 = 1000 0.840 965 3 = 594. 749


Vddd = 1000 594. 749 = 405. 251

R
Vuu
= erh ( u Vuuu + d Vuud ) = e(0.05)1/3 (0.456 807 0 + 0.543 193 0) =

Vuu = max Vuu


, EVuu = max (0, 0) = 0
R
Vud
= erh (u Vuud + d Vudd ) = e(0.05)1/3 (0.456 807 0 + 0.543 193 159. 035) =
84. 958 9
www.actuary88.com

c
Yufeng
Guo

74

CHAPTER 10. BINOMIAL OPTION PRICING I


R

Vud = max Vud


, EVud = max (84. 958 9, 0) = 84. 958 9
R
= erh (u Vudd + d Vddd ) = e(0.05)1/3 (0.456 807 159. 035 + 0.543 193 405. 251) =
Vdd
287. 938 62
R

Vdd = max Vdd


, EVdd = max (287. 938 62, 292. 777 9) = 292. 777 9
The American put is early exercised at the node dd.

Period 1

2
Vuu = 0

Su = 1000 (1. 189 11) = 1189. 11


EVu = 0
Vu =?
Vud = 84. 958 9
Sd = 1000 (0.840 965 ) = 840. 965
EVd = 1000 840. 965 = 159. 035
Vd =?
Vdd = 292. 777 9
VuR = erh ( u Vuu + d Vud ) = e(0.05)1/3 (0.456 807 0 + 0.543 193 84. 958 9) =
45. 386 3

Vu = max VuR , EVu = max (45. 386 3 , 0) = 45. 386 3


VdR = erh ( u Vud + d Vdd ) = e(0.05)1/3 (0.456 807 84. 958 9 + 0.543 193 292. 777 9) =
194. 574 6

Vd = max VdR , EVd = max (194. 574 6 , 159. 035) = 194. 574 6
Period 0

1
Vu = 45. 386 3

S = 1000
EV = 0
V =?
Vd = 194. 574 6
V R = erh ( u Vu + d Vu ) = e(0.05)1/3 (0.456 807 45. 386 3 + 0.543 193 194. 574 6) =
124. 334 7

V = max V R , EV = max (124. 334 7 , 0) = 124. 334 7


So the American put premium is 124. 334 7.

Calculate the European put premium.


www.actuary88.com

c
Yufeng
Guo

75

CHAPTER 10. BINOMIAL OPTION PRICING I


Period 0

3
Suuu = 1000u3 = 1681. 380 8
Vuuu = 0

Vuu
Suud = 1000u2 d = 1000u = 1189. 1100
Vuud = 0

Vu
V

Vud
Vd
Vdd

Sudd = 1000ud2 = 1000d = 840. 965


Vudd = 1000 840. 965 = 159. 035

Sddd = 1000d3 = 1000 0.840 965 3 = 594. 749


Vddd = 1000 594. 749 = 405. 251

Vuu = erh ( u Vuuu + d Vuud ) = e(0.05)1/3 (0.456 807 0 + 0.543 193 0) =

Vud = erh (u Vuud + d Vudd ) = e(0.05)1/3 (0.456 807 0 + 0.543 193 159. 035) =
84. 958 9
Vdd = erh ( u Vudd + d Vddd ) = e(0.05)1/3 (0.456 807 159. 035 + 0.543 193 405. 251) =
287. 938 62
Vu = erh (u Vuu + d Vud ) = e(0.05)1/3 (0.456 807 0 + 0.543 193 84. 958 9) =
45. 386 3
Vd = erh ( u Vud + d Vdd ) = e(0.05)1/3 (0.456 807 84. 958 9 + 0.543 193 287. 938 62) =
191. 989 4
V = erh (u Vu + d Vu ) = e(0.05)1/3 (0.456 807 45. 386 3 + 0.543 193 191. 989 4) =
122. 9537
So the European call premium is $122. 9537.
Time zero replicating portfolio for the European call
B = V = $122. 9537
45. 386 3 191. 989 4
Vu Vd
=
= 0.421 1
4=
Su Sd
1000 (1. 189 11) 1000 (0.840 965 )
So at t = 0 we need to enter 0.421 1 futures contract as a seller and deposit
122. 9537 in a savings account.
Why the European call and a European put have the same premium
The standard put-call parity is:
C + P V (K) = P + S
S is the price of the underlying asset at time zero. For the futures contract,
S is the present value of the forward price:
S = P V (F0,T )
In this problem, the forward price and the strike price are equal (i.e. K =
F0,T ). Hence P V (K) = P V (F0,T ) = S. This gives us C = P .
www.actuary88.com

c
Yufeng
Guo

76

CHAPTER 10. BINOMIAL OPTION PRICING I


Problem 10.19.
For options on a stock index, you can
use the standard binomial formula.

(r)h+ h
(0.050.03)1+0.3 1
u=e
= 1. 377 128
=e

d = e(r)h h = e(0.050.03)10.3 1 = 0.755 784


u =

e(r)h d
e(0.050.03)1 0.755 784
=
= 0.425 557
ud
1. 377 128 0.755 784

d = 1 u = 1 0.425 557 = 0.574 443


a. Calculate the price of the European call option
Period 0 1
2
3

Suuu = 100 1. 377 1283 = 261. 169 8


Vuuu = 261. 169 8 95 = 166. 169 8
Vuu

Vu
Suud = 100 1. 377 1282 (0.755 784) = 143. 333 0
Vuud = 143. 333 0 95 = 48. 3330
V
Vud

Sudd = 100 (1. 377 128) 0.755 7842 = 78. 662 9


Vd
Vudd = 0
Vdd

Sddd = 100 0.755 7843 = 43. 171 1


Vddd = 0
Vuu = erh ( u Vuuu + d Vuud ) = e(0.05)1 (0.425 557 166. 169 8 + 0.574 443 48. 333) =
93. 676 4
Vud = erh ( u Vuud + d Vudd ) = e(0.05)1 (0.425 557 48. 3330 + 0.574 443 0) =
19. 565 3
Vdd = erh (u Vudd + d Vddd ) = 0
Vu = erh (u Vuu + d Vud ) = e(0.05)1 (0.425 557 93. 676 4 + 0.574 443 19. 565 3) =
48. 611 4
Vd = erh ( u Vud + d Vdd ) = e(0.05)1 (0.425 557 19. 565 3 + 0.574 443 0) =
7. 920 1
V = erh ( u Vu + d Vu ) = e(0.05)1 (0.425 557 48. 611 4 + 0.574 443 7. 920 1) =
24. 005 8
So the European call premium is $24. 005 8.

b. Calculate the European put premium


www.actuary88.com

c
Yufeng
Guo

77

CHAPTER 10. BINOMIAL OPTION PRICING I


Period 0

Vuu
Vu
V

Vud
Vd
Vdd

Suuu = 100 1. 377 1283 = 261. 169 8


Vuuu = 0

Suud = 100 1. 377 1282 (0.755 784) = 143. 333 0


Vuud = 0

Sudd = 100 (1. 377 128) 0.755 7842 = 78. 662 9


Vudd = 95 78. 662 9 = 16. 337 1

Sddd = 100 0.755 7843 = 43. 171 1


Vddd = 95 43. 171 1 = 51. 828 9

Vuu = erh ( u Vuuu + d Vuud ) = e(0.05)1 (0.425 557 166. 169 8 + 0.574 443 48. 333) =

Vud = erh (u Vuud + d Vudd ) = e(0.05)1 (0.425 557 0 + 0.574 443 16. 337 1) =
8. 927 0
Vdd = erh ( u Vudd + d Vddd ) = e(0.05)1 (0.425 557 16. 337 1 + 0.574 443 51. 828 9) =
34. 934 0
Vu = erh (u Vuu + d Vud ) = e(0.05)1 (0.425 557 0 + 0.574 443 8. 927 0) =
4. 8780
Vd = erh ( u Vud + d Vdd ) = e(0.05)1 (0.425 557 8. 927 0 + 0.574 443 34. 934 0) =
22. 702 6
V = erh (u Vu + d Vu ) = e(0.05)1 (0.425 557 4. 8780 + 0.574 443 22. 702 6) =
14. 379 9
So the European put premium is $14. 379 9.
c. If we switch S and K and switch r and and recalculate the option price,
what happens?
After the switch, we have:
S = 95
K = 100
r = 3%
= 5%

u = e(r)h+ h = e(0.030.05)1+0.31 = 1. 323 13


d = e(r)h h = e(0.030.05)10.3 1 = 0.726 149
u =

e(r)h d
e(0.030.05)1 0.726 149
=
= 0.425 557
ud
1. 323 13 0.726 149

d = 1 u = 1 0.425 557 = 0.574 443


www.actuary88.com

c
Yufeng
Guo

78

CHAPTER 10. BINOMIAL OPTION PRICING I


Calculate the European call premium
Period 0

Vuu
Vu
V

Vud
Vd
Vdd

Suuu = 95 1. 323 133 = 220. 0550


Vuuu = 220. 0550 100 = 120. 055

Suud = 95 1. 323 132 (0.726 149) = 120. 768 7


Vuud = 120. 768 7 100 = 20. 768 7

Sudd = 95 (1. 323 13) 0.726 1492 = 66. 279 3


Vudd = 0

Sddd = 95 0.726 1493 = 36. 374 8


Vddd = 0

Vuu = erh ( u Vuuu + d Vuud ) = e(0.03)1 (0.425 557 120. 055 + 0.574 443 20. 768 7) =
61. 158 1
Vud = erh ( u Vuud + d Vudd ) = e(0.03)1 (0.425 557 20. 768 7 + 0.574 443 0) =
8. 577 1
Vdd = erh ( u Vudd + d Vddd ) = e(0.03)1 (0.425 557 0 + 0.574 443 0) =
0
Vu = erh (u Vuu + d Vud ) = e(0.03)1 (0.425 557 61. 158 1 + 0.574 443 8. 577 1) =
30. 038 5
Vd = erh ( u Vud + d Vdd ) = e(0.03)1 (0.425 557 8. 577 1 + 0.574 443 0) =
3. 542 2
V = erh ( u Vu + d Vd ) = e(0.03)1 (0.425 557 30. 038 5 + 0.574 443 3. 542 2) =
14. 3799
The call premium after the switch is equal to the put premium before the
switch.
Calculate the European put premium
Period 0

Vuu
Vu
V

Vud
Vd
Vdd

www.actuary88.com

Suuu = 95 1. 323 133 = 220. 0550


Vuuu = 0

Suud = 95 1. 323 132 (0.726 149) = 120. 768 7


Vuud = 0

Sudd = 95 (1. 323 13) 0.726 1492 = 66. 279 3


Vudd = 100 66. 279 3 = 33. 720 7

Sddd = 95 0.726 1493 = 36. 374 8


Vddd = 100 36. 374 8 = 63. 625 2
c
Yufeng
Guo

79

CHAPTER 10. BINOMIAL OPTION PRICING I

Vuu = erh ( u Vuuu + d Vuud ) = e(0.03)1 (0.425 557 0 + 0.574 443 0) =

Vud = erh (u Vuud + d Vudd ) = e(0.03)1 (0.425 557 0 + 0.574 443 33. 720 7) =
18. 798 1
Vdd = erh ( u Vudd + d Vddd ) = e(0.03)1 (0.425 557 33. 720 7 + 0.574 443 63. 625 2) =
49. 394 8
Vu = erh (u Vuu + d Vud ) = e(0.03)1 (0.425 557 0 + 0.574 443 18. 798 1) =
10. 479 3
Vd = erh ( u Vud + d Vdd ) = e(0.03)1 (0.425 557 18. 798 1 + 0.574 443 49. 394 8) =
35. 299 1
V = erh (u Vu + d Vd ) = e(0.03)1 (0.425 557 10. 479 3 + 0.574 443 35. 299 1) =
24. 005 8
The put premium after the switch is equal to the call premium before the
switch.
By the way, you can also use the textbooks spreadsheet "optbasics2" to
calculate the European option premium and verify
the European call price is $14. 379 9 (which is the European put price
before the switch)
the European put price is $24. 005 8 (which is the European call price
before the switch)
What a coincidence, you might wonder. Why? This can be explained using
the Black-Scholes option formulas:
The price of a European call option is:
C = SeT N (d1 ) KerT N (d2 )

(Textbook 12.1)

The price of a European put option is:


P = KerT N (d2 ) SeT N (d1 )

(Textbook 12.3)

S
1 2
SeT
SeT
1
ln
+ r+ T
ln
ln
+ 2T
rT
rT
1
K
2
Ke 2
Ke

=
=
+ T
d1 =
2
T
T
T
(Textbook 12.2a)

d2 = d1 T

(Textbook 12.2b)

Before the switch:


www.actuary88.com

c
Yufeng
Guo

80

CHAPTER 10. BINOMIAL OPTION PRICING I


C = SeT N (d1 ) KerT N (d2 )
P = KerT N (d2 ) SeT N (d1 )
SeT
rT
1
Ke

d1 =
+ T
2
T
SeT
SeT
ln
ln

rT
rT
1
1
Ke
Ke

+ T T =
T
d2 = d1 T =
2
2
T
T

SeT
SeT
ln KerT
ln KerT
1
1

+ T
T
KerT N
C = SeT N
T

2
2
T

SeT
SeT
ln KerT
ln
1
1
rT
SeT N Ke

P = KerT N
+
T

2
2
T
T
ln

After we switch S and K and switch


and :
rr
C = SeT N (d1 ) KerT N (d2 ) KS = KerT N (d1 ) SeT N (d2 )

r
P = KerT N (d2 ) SeT N (d1 ) KS = SeT N (d2 )KerT N (d1 )

r
SeT
KerT
ln
ln KerT

T
1
1
Se

+ T
+ T
=
d1 =
T

2
2
T
KS

KerT
SeT
ln
ln

T
rT
1
1
Ke

T =
T
d1 = Se
2
2
T
T
KerT
KerT
ln
ln

T
T
1
1
Se
Se

+ T T =
T
d2 = d1 T =
2
2
T
T
KerT
KerT
SeT
ln

ln
ln

T
rT
1
SeT + 1 T =
Ke

+ T =
+
d2 = Se
2
2
T
T
T
1
T
2

KerT
KerT
ln
ln
SeT
SeT
1
1

+ T
T
SeT N
C = KerT N
T

2
2
T

SeT
SeT
ln KerT
ln KerT
1
1

P = SeT N
+ T
T
KerT N
T

2
2
T

www.actuary88.com

c
Yufeng
Guo

81

CHAPTER 10. BINOMIAL OPTION PRICING I


You can see that the call premium after the switch is equal the put premium before the switch; the put premium after the switch is equal the call
premium before the switch. This conclusion applies to both European options
and American options. Its more complex to prove this is true for American
option. However, we are not going to worry about the proof.
Intuitively, you can think that the by switching S and K and switching r and
, we are switching the strike asset and the underlying asset. So after-the-switch
put is like a before-the-switch call; after-the-switch call is like a before-the-switch
put.
By the way, the payo of the before-switch call is not equal to the payo of
the after-switch put (however the premiums are the same):
Period 3 European
call payo before switch

Suuu = 100 1. 377 1283 = 261. 169 8


Vuuu = 261. 169 8 95 = 166. 169 8

Period 3 European
call

payo after switch


Suuu = 95 1. 323 133 = 220. 0550
Vuuu = 0

Sudd = 100 (1. 377 128) 0.755 7842 = 78. 662 9


Vudd = 0

Sudd = 95 (1. 323 13) 0.726 1492 = 66. 279 3


Vudd = 100 66. 279 3 = 33. 720 7

Suud = 100 1. 377 1282 (0.755 784) = 143. 333 0


Vuud = 143. 333 0 95 = 48. 3330

Sddd = 100 0.755 7843 = 43. 171 1


Vddd = 0

the payo of the before-switch put is not equal


switch call (however the premiums are the same):
Period 3 European put payo before switch
Suuu = 1000u3 = 1681. 380 8
Vuuu = 0
Suud = 1000u2 d = 1000u = 1189. 1100
Vuud = 0
Sudd = 1000ud2 = 1000d = 840. 965
Vudd = 1000 840. 965 = 159. 035

Sddd = 1000d3 = 1000 0.840 965 3 = 594. 749


Vddd = 1000 594. 749 = 405. 251

Suud = 95 1. 323 132 (0.726 149) = 120. 768 7


Vuud = 0

Sddd = 95 0.726 1493 = 36. 374 8


Vddd = 100 36. 374 8 = 63. 625 2
to the payo of the after-

Period 3 European
call

payo after switch


Suuu = 95 1. 323 133 = 220. 0550
Vuuu = 220. 0550 100 = 120. 055

Suud = 95 1. 323 132 (0.726 149) = 120. 768 7


Vuud = 120. 768 7 100 = 20. 768 7

Sudd = 95 (1. 323 13) 0.726 1492 = 66. 279 3


Vudd = 0

Sddd = 95 0.726 1493 = 36. 374 8


Vddd = 0

Problem 10.20.
www.actuary88.com

c
Yufeng
Guo

82

CHAPTER 10. BINOMIAL OPTION PRICING I


For options on a stock index, you can
use the standard binomial formula.

u = e(r)h+ h = e(0.050.03)1+0.3 1 = 1. 377 128


d = e(r)h h = e(0.050.03)10.3 1 = 0.755 784
u =

e(r)h d
e(0.050.03)1 0.755 784
=
= 0.425 557
ud
1. 377 128 0.755 784

d = 1 u = 1 0.425 557 = 0.574 443


a. Calculate the price of the American call option
Period 2
3

Suuu = 100 1. 377 1283 = 261. 169 8

Suu = 100 1. 377 1282 = 189. 648 2


Vuuu = 261. 169 8 95 = 166. 169 8
EVuu = 189. 648 2 95 = 94. 648 2
Vuu =?

Suud = 100 1. 377 1282 (0.755 784) = 143. 333 0


Sud = 100 (1. 377 128) (0.755 784) = 104. 081 1 Vuud = 143. 333 0 95 = 48. 3330
EVud = 104. 081 1 95 = 9. 081 1
Vud =?

Sudd = 100 (1. 377 128) 0.755 7842 = 78. 662 9

Sudd = 100 0.755 7842 = 57. 120 9


Vudd = 0
EVdd = 0

Vdd =?
Sddd = 100 0.755 7843 = 43. 171 1
Vddd = 0
R
Vuu
= erh ( u Vuuu + d Vuud ) = e(0.05)1 (0.425 557 166. 169 8 + 0.574 443 48. 333) =
93. 676 4
R

Vuu = max Vuu


, EVuu = max (93. 676 4, 94. 648 2) = 94. 648 2
R
= erh ( u Vuud + d Vudd ) = e(0.05)1 (0.425 557 48. 3330 + 0.574 443 0) =
Vud
19. 565 3
R

Vud = max Vud


, EVud = max (19. 565 3, 9. 081 1) = 19. 565 3
R
Vdd
= erh (u Vudd + d Vddd ) = 0
R
Vdd = max Vdd
, EVdd = 0

Period 1

2
Vuu = 94. 648 2

Su = 100 (1. 377 128) = 137. 712 8


EVu = 137. 712 8 95 = 42. 712 8
Vu =?
Vud = 19. 565 3
Sd = 100 (0.755 784) = 75. 578 4
EVd = 0
Vd =?
Vdd = 0
www.actuary88.com

c
Yufeng
Guo

83

CHAPTER 10. BINOMIAL OPTION PRICING I


VuR = erh (u Vuu + d Vud ) = e(0.05)1 (0.425 557 94. 648 2 + 0.574 443 19. 565 3) =
49. 004 8

Vu = max VuR , EVu = 49. 004 8


VdR = erh (u Vud + d Vdd ) = e(0.05)1 (0.425 557 19. 565 3 + 0.574 443 0) =
7. 920 1

Vd = max VdR , EVd = 7. 920 1


Period 0

1
Vu = 49. 004 8

S = 100
EV = 100 95 = 5
V =?
Vd = 7. 920 1
V R = erh (u Vu + d Vu ) = e(0.05)1 (0.425 557 49. 004 8 + 0.574 443 7. 920 1) =
24. 165 0

V = max V R , EV = 24. 165 0


So the American call premium is $24. 165 0.
b. Calculate the American put premium
Period 2

Suu = 100 1. 377 1282 = 189. 648 2


EVuu = 0
Vuu =?
Sud = 100 (1. 377 128) (0.755 784) = 104. 081 1
EVud = 0
Vud =?

Sudd = 100 0.755 7842 = 57. 120 9


EVdd = 95 57. 120 9 = 37. 879 1
Vdd =?

Suuu = 100 1. 377 1283 = 261. 169 8


Vuuu = 0

Suud = 100 1. 377 1282 (0.755 784) = 143. 333 0


Vuud = 0

Sudd = 100 (1. 377 128) 0.755 7842 = 78. 662 9


Vudd = 95 78. 662 9 = 16. 337 1

Sddd = 100 0.755 7843 = 43. 171 1


Vddd = 95 43. 171 1 = 51. 828 9

R
Vuu
= erh ( u Vuuu + d Vuud ) = e(0.05)1 (0.425 557 0 + 0.574 443 0) =

Vuu = max Vuu


, EVuu = 0
R
Vud
= erh (u Vuud + d Vudd ) = e(0.05)1 (0.425 557 0 + 0.574 443 16. 337 1) =
8. 927 0

R
Vud = max Vud
, EVud = 8. 927 0
R
Vdd
= erh ( u Vudd + d Vddd ) = e(0.05)1 (0.425 557 16. 337 1 + 0.574 443 51. 828 9) =
34. 934 0
R

Vud = max Vdd


, EVdd = 37. 879 1

www.actuary88.com

c
Yufeng
Guo

84

CHAPTER 10. BINOMIAL OPTION PRICING I


Period 1

2
Vuu = 0

Su = 100 (1. 377 128) = 137. 712 8


EVu = 0
Vu =?
Vud = 8. 927 0
Sd = 100 (0.755 784) = 75. 578 4
EVd = 85 75. 578 4 = 9. 421 6
Vd =?
Vdd = 37. 879 1
VuR = erh ( u Vuu + d Vud ) = e(0.05)1 (0.425 557 0 + 0.574 443 8. 927 0) =
4. 8780

Vu = max VuR , EVu = 4. 8780


VdR = erh ( u Vud + d Vdd ) = e(0.05)1 (0.425 557 8. 927 0 + 0.574 443 37. 879 1) =
24. 311 8

Vd = max VdR , EVd = 24. 311 8


Period 0

1
Vu = 4. 8780

S = 100
EV = 0
V =?
Vd = 24. 311 8
V R = erh ( u Vu + d Vu ) = e(0.05)1 (0.425 557 4. 8780 + 0.574 443 24. 311 8) =
15. 259 3

V = max V R , EV = 15. 259 3


So the American put premium is $15. 259 3

c. If we switch S and K and switch r and and recalculate the option price,
what happens?
After the switch, we have:
S = 95
K = 100
r = 3%
= 5%

u = e(r)h+ h = e(0.030.05)1+0.3 1 = 1. 323 13


d = e(r)h h = e(0.030.05)10.3 1 = 0.726 149
www.actuary88.com

c
Yufeng
Guo

85

CHAPTER 10. BINOMIAL OPTION PRICING I

u =

e(r)h d
e(0.030.05)1 0.726 149
=
= 0.425 557
ud
1. 323 13 0.726 149

d = 1 u = 1 0.425 557 = 0.574 443


Calculate the price of the American call option
Period 2
3

Suuu = 95 1. 323 133 = 220. 0550

Suu = 95 1. 323 132 = 166. 313 9


Vuuu = 220. 0550 100 = 120. 055
EVuu = 166. 313 9 100 = 66. 313 9
Vuu =?

Suud = 95 1. 323 132 (0.726 149) = 120. 768 7


Sud = 95 (1. 323 13) (0.726 149) = 91. 275 0 Vuud = 120. 768 7 100 = 20. 768 7
EVud = 0
Vud =?

Sudd = 95 (1. 323 13) 0.726 1492 = 66. 279 3

Sudd = 95 0.726 1492 = 50. 092 8


Vudd = 0
EVdd = 0

Vdd =?
Sddd = 95 0.726 1493 = 36. 374 8
Vddd = 0
R
Vuu
= erh ( u Vuuu + d Vuud ) = e(0.03)1 (0.425 557 120. 055 + 0.574 443 20. 768 7) =
61. 158 1
R

Vuu = max Vuu


, EVuu = 66. 313 9
R
= erh (u Vuud + d Vudd ) = e(0.03)1 (0.425 557 20. 768 7 + 0.574 443 0) =
Vud
8. 577 1
R

Vud = max Vud


, EVud = 8. 577 1
R
Vdd
= erh ( u Vudd + d Vddd ) = 0
R
Vdd = max Vdd
, EVdd = 0

Period 1

2
Vuu = 66. 313 9

Su = 95 (1. 323 13) = 125. 697 4


EVu = 125. 697 4 100 = 25. 697 4
Vu =?
Vud = 8. 577 1
Sd = 95 (0.726 149) = 68. 984 2
EVd = 0
Vd =?
Vdd = 0
VuR = erh (u Vuu + d Vud ) = e(0.03)1 (0.425 557 66. 313 9 + 0.574 443 8. 577 1) =
32. 167 7

Vu = max VuR , EVu = 32. 167 7


www.actuary88.com

c
Yufeng
Guo

86

CHAPTER 10. BINOMIAL OPTION PRICING I


VdR = erh ( u Vud + d Vdd ) = e(0.03)1 (0.425 557 8. 577 1 + 0.574 443 0) =
3. 542 2

Vd = max VdR , EVd = 3. 542 2


Period 0

1
Vu = 32. 167 7

S = 95
EV = 0
V =?
Vd = 3. 542 2
V R = erh ( u Vu + d Vu ) = e(0.03)1 (0.425 557 32. 167 7 + 0.574 443 3. 542 2) =
15. 2593

V = max V R , EV = 15. 2593


So the American call premium is $15. 2593, which is equal to the American
put premium before the switch.
Calculate the American put premium
Period 2

Suu = 95 1. 323 132 = 166. 313 9


EVuu = 0
Vuu =?
Sud = 95 (1. 323 13) (0.726 149) = 91. 275 0
EVud = 100 91. 275 0 = 8. 725
Vud =?

Sudd = 95 0.726 1492 = 50. 092 8


EVdd = 100 50. 092 8 = 49. 907 2
Vdd =?

Suuu = 95 1. 323 133 = 220. 0550


Vuuu = 0

Suud = 95 1. 323 132 (0.726 149) = 120. 768 7


Vuud = 0

Sudd = 95 (1. 323 13) 0.726 1492 = 66. 279 3


Vudd = 100 66. 279 3 = 33. 720 7

Sddd = 95 0.726 1493 = 36. 374 8


Vddd = 100 36. 374 8 = 63. 625 2

R
Vuu
= erh (u Vuuu + d Vuud ) = e(0.03)1 (0.425 557 0 + 0.574 443 0) =

R
Vuu = max Vuu
, EVuu = 0
R
Vud
= erh ( u Vuud + d Vudd ) = e(0.03)1 (0.425 557 0 + 0.574 443 33. 720 7) =
18. 798 1
R

Vud = max Vud


, EVud = 18. 798 1
R
Vdd
= erh (u Vudd + d Vddd ) = e(0.03)1 (0.425 557 33. 720 7 + 0.574 443 63. 625 2) =
49. 394 8
R

, EVdd = 49. 907 2


Vud = max Vdd
www.actuary88.com

c
Yufeng
Guo

87

CHAPTER 10. BINOMIAL OPTION PRICING I


Period 1

2
Vuu = 0

Su = 95 (1. 323 13) = 125. 697 4


EVu = 0
Vu =?
Vud = 18. 798 1
Sd = 95 (0.726 149) = 68. 984 2
EVd = 100 68. 984 2 = 31. 015 8
Vd =?
Vdd = 49. 907 2
VuR = erh (u Vuu + d Vud ) = e(0.03)1 (0.425 557 0 + 0.574 443 18. 798 1) =
10. 479 3

Vu = max VuR , EVu = 10. 479 3


VdR = erh (u Vud + d Vdd ) = e(0.03)1 (0.425 557 18. 798 1 + 0.574 443 49. 907 2) =
35. 584 8

Vd = max VdR , EVd = 35. 584 8


Period 0

1
Vu = 10. 479 3

S = 95
EV = 100 95 = 5
V =?
Vd = 35. 584 8
V R = erh (u Vu + d Vu ) = e(0.03)1 (0.425 557 10. 479 3 + 0.574 443 35. 584 8) =
24. 165 04

V = max V R , EV = 24. 165 0

So the American put premium is $24. 165 0, which is equal to the American
call premium before the switch.
Problem 10.21.

Suppose u < e(r)h . Since d < u, we have d < u < e(r)h . This means
that the savings account is always better than the stock. So at t = 0, we short
sell eh share of stock and investment the short sale proceeds Seh into the
savings account. Then at time h, we close our short position by buying one
stock from the market. The stock price at time h is either uS or dS.
t=0
t = h, u mode
t = h, d mode
short eh stock
Seh
uS
dS
deposit Seh in savings Seh Se(r)h
Seh

h
Total
0
S e
u > 0 S eh d > 0

So initial cost is zero yet we have positive payo at time h. This is an


arbitrage opportunity.
www.actuary88.com

c
Yufeng
Guo

88

CHAPTER 10. BINOMIAL OPTION PRICING I


Suppose d > e(r)h . Since d < u, we have u > d > e(r)h . This means that
the investing in stocks is always better o than investing in a savings account.
So at t = 0, we buy eh share of stock and borrow money from a bank to
finance the purchase.
t=0
t = h, u mode
t = h, d mode
buy eh stock
Seh uS
dS
(r)h
h
borrow Seh in savings Seh
Se
Se

h
Total
0
S ue
> 0 S d eh > 0
So initial cost is zero yet we have positive payo at time h. This is an
arbitrage opportunity.

www.actuary88.com

c
Yufeng
Guo

89

CHAPTER 10. BINOMIAL OPTION PRICING I

www.actuary88.com

c
Yufeng
Guo

90

Chapter 11

Binomial option pricing II


Problem 11.1.

u = e(r)h+ h = e(00.08)1+0.31 = 1. 246 077


d = e(r)h h = e(00.08)10.3 1 = 0.683 861
e(r)h d
e(00.08)1 0.683 861
u =
=
= 0.425 56
ud
1. 246 077 0.683 861
d = 1 u = 1 0.425 56 = 0.574 44
K = 70
t=0

S = 100
EV = 100 70 = 30
V

t=1
Su = 100 (1. 246 077) = 124. 607 7
Vu = 124. 607 7 70 = 54. 607 7
Sd = 100 (0.683 861 ) = 68. 386 1
Vd = 0

V R = erh ( u Vu + d Vu ) = e0(1) (0.425 56 54. 607 7 + 68. 386 1 0) =


23. 24

V = max V R , EV = max (23. 24, 30) = 30


K = 80
t=0

S = 100
EV = 100 80 = 20
V

t=1
Su = 100 (1. 246 077) = 124. 607 7
Vu = 124. 607 7 80 = 44. 607 7
Sd = 100 (0.683 861 ) = 68. 386 1
Vd = 0
91

CHAPTER 11. BINOMIAL OPTION PRICING II


V R = erh ( u Vu + d Vu ) = e0(1) (0.425 56 44. 607 7 + 68. 386 1 0) =
18. 98

V = max V R , EV = max (18. 98, 20) = 20


K = 90
t=0

S = 100
EV = 100 90 = 10
V

t=1
Su = 100 (1. 246 077) = 124. 607 7
Vu = 124. 607 7 90 = 34. 607 7
Sd = 100 (0.683 861 ) = 68. 386 1
Vd = 0

V R = erh ( u Vu + d Vu ) = e0(1) (0.425 56 34. 607 7 + 68. 386 1 0) =


14. 73

V = max V R , EV = max (14. 73, 10) = 14. 73


K = 100
t=0

S = 100
EV = 100 100 = 0
V

t=1
Su = 100 (1. 246 077) = 124. 607 7
Vu = 124. 607 7 100 = 24. 607 7
Sd = 100 (0.683 861 ) = 68. 386 1
Vd = 0

V R = erh ( u Vu + d Vu ) = e0(1) (0.425 56 24. 607 7 + 68. 386 1 0) =


10. 47

V = max V R , EV = max (10. 47, 0) = 10. 47


a. Early exercise occurs at t = 0 with K = 70, 80

b. The European options satisfy the put-call parity:


CEur + KerT = PEur + SeT
CEur = PEur + SeT KerT = PEur + 100e0.08 K = PEur K + 92.
311 6
Since K = 70, 80, 90, 100 100, we have
EV = max (0, S K) = max (0, 100 K) = 100 K
To have EV > CEur , we need to have:
100 K > PEur K + 92. 311 6
PEur < 100 100e0.08 = 7. 69
To early exercise the American call at t = 0, the European put premium
needs to be less than 7. 69.
www.actuary88.com

c
Yufeng
Guo

92

CHAPTER 11. BINOMIAL OPTION PRICING II


Clearly, the smaller the European put premium is, the more likely the early
exercise of the American call is optimal at t = 0.
Everything else equal, the higher the strike price, the higher the price of a
European put. As a result, it can be optimal to early exercise the American
call with K = 70, 80, while its not optimal to early exercise the American with
K = 90, 100.
Use the European put premium using the Black-Scholes option formula (the
textbook Equation 12.3):
P = KerT N (d2 ) SeT N (d1 )
We find the following put price (S = 100, T = 1, r = 0, = 0.08, = 0.3):
K
PEur
70
$2.3087
80
$5.2537
90
$9.7517
100 $15.7113
Clearly, the condition PEur < 7. 69 is met when K = 70, 80 and violated
when K = 90, 100.
By the way, the price calculated using the Black-Scholes option formula wont
match the price calculated under the binomial option formula under h = 1.
This is because the Black-Scholes option formula is the binomial option pricing
method where n and h = T /n 0 .
c.To early exercise the American call, we need to have PEur < 7. 69. This
condition is met if K = 70, 80 and violated when K = 90, 100.
Problem 11.2.
Now r = 0.08 instead of r = 0. This increases the cost of early exercising
an American call. By early exercising an American call, you lost interest on
the strike asset K. We expect that its still not optimal to early exercise the
American call at K = 90, 100 (its not optimal to early exercise the American
call at these strike prices even when r = 0, let alone when r = 0.08). However,
we are not clear whether its optimal to early exercise the American call when
K = 70, 80. We have to check.

u = e(r)h+ h = e(0.080.08)1+0.3 1 = 1. 349 859


d = e(r)h h = e(0.080.08)10.3 1 = 0.740 818
e(r)h d
e(0.080.08)1 0.740 818
u =
=
= 0.425 558
ud
1. 349 859 0.740 818
d = 1 u = 1 0.425 558 = 0.574 442
www.actuary88.com

c
Yufeng
Guo

93

CHAPTER 11. BINOMIAL OPTION PRICING II


K = 70
t=0

S = 100
EV = 100 70 = 30
V

t=1
Su = 100 (1. 349 859) = 134. 985 9
Vu = 134. 985 9 70 = 64. 985 9
Sd = 100 (0.740 818 ) = 74. 081 8
Vd = 74. 081 8 70 = 4. 081 8

V R = erh (u Vu + d Vu ) = e0.08(1) (0.425 558 64. 985 9 + 0.574 442 4. 081 8) =


27. 69

V = max V R , EV = max (27. 69, 25) = 27. 69


K = 80
t=0

S = 100
EV = 100 80 = 20
V

t=1
Su = 100 (1. 349 859) = 134. 985 9
Vu = 134. 985 9 80 = 54. 985 9
Sd = 100 (0.740 818 ) = 74. 081 8
Vd = 0

V R = erh (u Vu + d Vu ) = e0.08(1) (0.425 558 54. 985 9 + 0.574 442 0) =


21. 60

V = max V R , EV = max (21. 60, 20) = 21. 60


K = 90
t=0

S = 100
EV = 100 90 = 10
V

t=1
Su = 100 (1. 349 859) = 134. 985 9
Vu = 134. 985 9 90 = 44. 985 9
Sd = 100 (0.740 818 ) = 74. 081 8
Vd = 0

V R = erh (u Vu + d Vu ) = e0.08(1) (0.425 558 44. 985 9 + 0.574 442 0) =


17. 67

V = max V R , EV = max (17. 67, 10) = 17. 67


K = 100
www.actuary88.com

c
Yufeng
Guo

94

CHAPTER 11. BINOMIAL OPTION PRICING II


t=0

S = 100
EV = 0
V

t=1
Su = 100 (1. 349 859) = 134. 985 9
Vu = 134. 985 9 100 = 34. 985 9
Sd = 100 (0.740 818 ) = 74. 081 8
Vd = 0

V R = erh ( u Vu + d Vu ) = e0.08(1) (0.425 558 34. 985 9 + 0.574 442 0) =


13. 74

V = max V R , EV = max (13. 74, 0) = 13. 74


a. Early exercise occurs at t = 0 with K = 70. Now the risk free rate is no
longer zero and well lose interest on the strike price by exercising the American
call early. The higher the strike price, the more interest we lose. In contrast,
the previous problem has r = 0 and we dont lose any interest by early exercise.
It makes sense that if r is not zero (i.e. positive) then fewer strike prices will
lead to optimal exercise than if r = 0.
b, c. The European options satisfy the put-call parity:
CEur + KerT = PEur + SeT
CEur = PEur + SeT KerT = PEur + 100e0.08 Ke0.08
Since K = 70, 80, 90, 100 100, we have
EV = max (0, S K) = max (0, 100 K) = 100 K
To have EV > CEur , we need to have:
0.08
100 K > PEur + 100e0.08 Ke

0.08
PEur < 100 100e
K 1 e0.08 = 7. 69 0.0769K

Using the Black-Scholes option pricing formula, we find the following put
price (S = 100, T = 1, r = 0.08, = 0, = 0.3):
K
70
80
90
100

PEur
0.7752
2.0904
4.4524
8.0229

7. 69 0.0769K
7. 69 0.0769 (70) = 2. 307
7. 69 0.0769 (80) = 1. 538
7. 69 0.0769 (90) = 0.769
7. 69 0.0769 (100) = 0

Among the 4 strike prices, only K = 70 satisfies the condition PEur < 7.
69 0.0769K. Hence of the 4 strike prices given, only K = 70 leads to optimal
early exercise.
Problem 11.3.
If = 0, then the stock doesnt pay any dividend. Its never optimal to early
exercise an American call on a non-dividend paying stock. So early exercise will
never occur.
www.actuary88.com

c
Yufeng
Guo

95

CHAPTER 11. BINOMIAL OPTION PRICING II


Problem 11.4.

u = e(r)h+ h = e(0.080)1+0.31 = 1. 462 285


d = e(r)h h = e(0.080)10.3 1 = 0.802 519
e(r)h d
e(0.080)1 0.802 519
u =
=
= 0.425 557
ud
1. 462 285 0.802 519
d = 1 u = 1 0.425 557 = 0.574 443
K = 100
t=0

S = 100
EV = 0
V

t=1
Su = 100 (1. 462 285) = 146. 228 5
Vu = 0

Sd = 100 (0.802 519 ) = 80. 251 9


Vd = 100 80. 251 9 = 19. 748 1

V R = erh (u Vu + d Vu ) = e0.08(1) (0.425 557 0 + 0.574 443 19. 748 1) =


10. 4720

V = max V R , EV = max (10. 4720, 0) = 10. 4720


K = 110
t=0

S = 100
EV = 110 100 = 10
V

t=1
Su = 100 (1. 462 285) = 146. 228 5
Vu = 0

Sd = 100 (0.802 519 ) = 80. 251 9


Vd = 110 80. 251 9 = 29. 748 1

V R = erh (u Vu + d Vu ) = e0.08(1) (0.425 557 0 + 0.574 443 29. 748 1) =


15. 774 8

V = max V R , EV = max (15. 774 8, 10) = 15. 774 8


K = 120
t=0

S = 100
EV = 120 100 = 20
V

www.actuary88.com

t=1
Su = 100 (1. 462 285) = 146. 228 5
Vu = 0

Sd = 100 (0.802 519 ) = 80. 251 9


Vd = 120 80. 251 9 = 39. 748 1
c
Yufeng
Guo

96

CHAPTER 11. BINOMIAL OPTION PRICING II


V R = erh ( u Vu + d Vu ) = e0.08(1) (0.425 557 0 + 0.574 443 39. 748 1) =
21. 077 5

V = max V R , EV = max (21. 077 5, 20) = 21. 077 5


K = 130
t=0

t=1
Su = 100 (1. 462 285) = 146. 228 5
Vu = 0

S = 100
EV = 130 100 = 30
V

Sd = 100 (0.802 519 ) = 80. 251 9


Vd = 130 80. 251 9 = 49. 748 1

V R = erh ( u Vu + d Vu ) = e0.08(1) (0.425 557 0 + 0.574 443 49. 748 1) =


26. 380 3

V = max V R , EV = max (26. 380 3, 30) = 30


a. Early exercise occurs at t = 0 with K = 130.

b, c. The European options satisfy the put-call parity:


CEur + KerT = PEur + SeT
PEur = CEur + KerT SeT = CEur + Ke0.08(1) 100e0(1) = CEur +
0.08
Ke
100
The early exercise value is
EV = max (0, K S)
Since K = 100, 100, 120, 130 100, we have
EV = max (0, K S) = K S = K 100
To have EV > CEur , we need to have:
0.08
K 100 > CEur
+ Ke
100
0.08
CEur < K 1 e
= 0.07 69K

Using the Black-Scholes option pricing formula, we find the following put
price (S = 100, T = 1, r = 0.08, = 0, = 0.3):
K
100
110
120
130

CEur
15.7113
11.2596
7.8966
5.4394

0.07 69K
0.07 69 (100) = 7. 69
0.07 69 (110) = 8. 459
0.07 69 (120) = 9. 228
0.07 69 (130) = 9. 997

Among the 4 strike prices, only K = 130 satisfies the condition CEur < 0.07
69K. Hence of the 4 strike prices given, only K = 130 leads to optimal early
exercise.
www.actuary88.com

c
Yufeng
Guo

97

CHAPTER 11. BINOMIAL OPTION PRICING II


The main reason for early exercising an American put is to earn interest on
the strike price. Hence higher the strike price, everything else equal, the more
likely an American put may be early exercised.
Problem 11.5.
Now = 0.08 as opposed to = 0 in the previous problem. This increases
the cost of early exercising an American put. If you early exercise an American
put, you can earn interest on the strike asset, but you also lose the opportunity
of earning dividend on the stock.
With = 0.08, we expect that its still not optimal to early exercise the
American put when K = 100, 110, 120. Because its not optimal to early exercise
the American put with these strike prices even when = 0, let alone = 0.08.
However, we are not clear whether its optimal to early exercise the American
put when K = 130. We have to check.
The put-call parity is:
CEur + KerT = PEur + SeT
CEur + Ke0.08 = PEur + 100e0.08
PEur = CEur + Ke0.08 100e0.08
EV = max (0, K S) = K S = K 100
Early exercise if:
K 100 > CEur + Ke0.08 100e0.08
CEur < K Ke0.08 100 + 100e0.08 = 0.07 69K 7. 69
Using the Black-Scholes option pricing formula, we find the following put
price (S = 100, T = 1, r = 0.08, = 0.08, = 0.3):
K
CEur
0.07 69K 0.07 69K 7. 69
100 15.6584 0.07 69 (100) 7. 69 = 0
110 11.2165 0.07 69 (110) 7. 69 = 0.769
120 7.8627
0.07 69 (120) 7. 69 = 1. 538
130 5.4136
0.07 69 (130) 7. 69 = 2. 307
The condition CEur < 0.07 69K 7. 69 is met only when K = 130. So its
optimal to early exercise the American put only when K = 130.
Problem 11.6.
The only reason to early exercise an American put is to earn interest on the
strike asset. If r = 0 well never earn any interest on the strike price. So its
never optimal to early exercise an American put if r = 0.
We can also use the put-call parity to verify that its never optimal to early
exercise the American put when r = 0.
CEur + KerT = PEur + SeT
CEur + K = PEur + 100e0.08
www.actuary88.com

c
Yufeng
Guo

98

CHAPTER 11. BINOMIAL OPTION PRICING II


PEur = CEur + K 100e0.08
EV = max (0, K S) = K S = K 100
Early exercise if:
K 100 > CEur + K 100e0.08
CEur < 100 + 100e0.08 = 7. 688
Since CEur 0, its never optimal to early exercise the American put.
Problem 11.7.
Skip. This is a spreadsheet problem. Most likely SOA and CAS wont ask
you to build a 10 period binomial model in the exam.
Problem 11.8.
Skip. This is a spreadsheet problem. Most likely SOA and CAS wont ask
you to build a 10 period binomial model in the exam.
Problem 11.9.
Skip. This is a spreadsheet problem. Most likely SOA and CAS wont ask
you to build a 10 period binomial model in the exam.
Problem 11.10.
Skip. This is a spreadsheet problem. Most likely SOA and CAS wont ask
you to build a 10 period binomial model in the exam.
Problem 11.11.
Skip. This is a spreadsheet problem. Most likely SOA and CAS wont ask
you to build a 10 period binomial model in the exam.
Problem 11.12.
Skip. This is a spreadsheet problem. Most likely SOA and CAS wont ask
you to build a 10 period binomial model in the exam.
Problem 11.13.
Skip. This is a spreadsheet problem. Most likely SOA and CAS wont ask
you to build a 50 period binomial model in the exam.
Problem 11.14.
www.actuary88.com

c
Yufeng
Guo

99

CHAPTER 11. BINOMIAL OPTION PRICING II


The textbook Section 10 Equation 10.7 says that the undiscounted riskneutral stock price is the forward price:
time t time t + h
Su
S
Sd
u Su + d Sd = Se(r)h = Ft,t+h

(Textbook Equation 10.7)

We are reusing the data in the textbook Figure 11.4 except that we set
S = 100. We have:
S = 100
r = 0.08

= 0.3
=0

T =1

h = T /n = 1/3

Then:

u = e(r)h+ h = e(0.080)1/3+0.31/3 = 1. 221 246


d = e(r)h h = e(0.080)1/30.3 1/3 = 0.863 693
e(r)h d
e(0.080)1/3 0.863 693
=
= 0.456 806
u =
ud
1. 221 246 0.863 693
d = 1 u = 1 0.456 806 = 0.543 194
The stock price tree is:
Period 0 1
S u = 100 (1. 221 246) = 122. 124 6
S = 100
S d = 100 (0.863 693) = 86. 369 3
Period 2

S uu = 100 1. 221 2462 = 149. 144 2

S ud = 100 (1. 221 246) (0.863 693) = 105. 478 2


S dd

= 100 0.863 6932 = 74. 596 6

S uuu = 100 1. 221 2462 = 182. 141 7

S uud = 100 1. 221 2462 (0.863 693) = 128. 814 8

S udd = 100 (1. 221 246) 0.863 6932 = 91. 100 8

S ddd = 100 0.863 6933 = 64. 428 5

a.The forward price is calculated using the textbook Equation 5.7:


F0,T = S0 e(r)T
The 4-month forward price is:
F0,1/3 = 100e(0.080)1/3 = 102. 702 5
The 8-month forward price is:
F0,2/3 = 100e(0.080)2/3 = 105. 478 1
www.actuary88.com

c
Yufeng
Guo

100

CHAPTER 11. BINOMIAL OPTION PRICING II


The 1-year forward price is:
F0,1 = 100e(0.080)1 = 108. 328 7
b.

u = e(r)h+ h = e(0.080)1/3+0.31/3 = 1. 221 246


d = e(r)h h = e(0.080)1/30.3 1/3 = 0.863 693
e(r)h d
e(0.080)1/3 0.863 693
u =
=
= 0.456 806
ud
1. 221 246 0.863 693
d = 1 u = 1 0.456 806 = 0.543 194
t = 1/3 Stock Price
Risk neutral prob
S u = 100 (1. 221 246) = 122. 124 6 u
S d = 100 (0.863 693) = 86. 369 3
d
Total
1
The undiscounted risk-neutral expected stock price at t = 1/3 is:
u Su + d Sd = 0.456 806 (122. 124 6) + 0.543 194 (86. 369 3) = 102. 702 53 =
F0,1/3
t = 2/3 Stock
Risk neutral prob
Price
S uu = 100 1. 221 2462 = 149. 144 2
2u
ud
S = 100 (1. 221 246)(0.863 693) = 105. 478 2 2 u d
S dd = 100 0.863 6932 = 74. 596 6
2d
Total
1
The undiscounted risk-neutral expected stock price at t = 2/3 is:
2u Suu + 2d Sdd +2 u d S ud = 0.456 8062 (149. 144 2)+2 (0.456 806) (0.543 194) (105. 478 2)+
0.543 1942 (74. 596 6) = 105. 478 1 = F0,2/3
t = 1 Stock Price

S uuu = 100 1. 221 2462 = 182. 141 7


S uud = 100 1. 221 2462 (0.863 693)
= 128. 814 8
2
S udd = 100 (1.
221
246)
0.863
693
= 91. 100 8

S ddd = 100 0.863 6933 = 64. 428 5


Total

Risk neutral prob


3u
3 2u d
3 u 2d
3d
1

The undiscounted risk-neutral expected stock price at t = 2/3 is:


3u Suuu + 3 2u d S uud + 3 u2d S udd + 3d S ddd

3
= 0.456
141 7)+3 0.456 8062 (0.543 194) 128. 814 8+3 (0.456 806) 0.543 1942 91.
806 (182.

100 8 + 0.543 1943 64. 428 5 = 108. 328 7 = F0,1


Problem 11.15.
Skip. This is a spreadsheet problem. Most likely SOA and CAS wont ask
you to build a 50 period binomial model in the exam.
Problem 11.16.
www.actuary88.com

c
Yufeng
Guo

101

CHAPTER 11. BINOMIAL OPTION PRICING II


Skip. This is a spreadsheet problem. Most likely SOA and CAS wont ask
you to build a 8 period binomial model in the exam.
Problem 11.17.
Skip. This is a spreadsheet problem. Most likely SOA and CAS wont ask
you to build a 8 period binomial model in the exam.
Problem 11.18.
Skip. If you understand the volatility calculation examples in my study
guide, you are fine.
Problem 11.19.
Skip. If you understand the volatility calculation examples in my study
guide, you are fine.
Problem 11.20.
This is a labor intensive problem. However, its a good practice problem
for using the Schroder method. The solution is similar to the textbook Figure
11.11.
First, well build a prepaid forward price tree.
St
P
Ft,T
P
Ft,T
= St P Vt (D) = 50 4e0.08(0.25) = 46. 079 21
St
50
F = S P = 0.3
= 0.325 5265
46. 079 2
Ft,T
F = S

u = erh+F h = e0.08(0.25)+0.325 5265 0.25 = 1. 200 53


d = erhF h = e0.08(0.25)0.325 5265 0.25 = 0.866 959
Prepaid forward price tree:
Time 0

0.25

0.5

0.75

1
95.71879

79.73044
66.41269
55.31947
46.07921

69.12305
57.57710

47.95973
39.94880

49.91701
41.57914

34.63398

36.04742
30.02625
26.03154

www.actuary88.com

c
Yufeng
Guo

102

CHAPTER 11. BINOMIAL OPTION PRICING II


For
example,
u the prepaid forward price at t = 0.25 is calculated as follows:
P
P
= Ft,T
u = 46. 079 21 1. 200 53 = 55. 319 47
Ft+h,T

d
P
P
Ft+h,T
= Ft,T
d = 46. 079 21 0.866 959 = 39. 948 8
The prepaid

uu forward price at t = 0.5 is calculated as follows:


P
P
Ft+2h,T
= Ft,T
u2 = 46. 079 21 1. 200 532 = 66. 412 7

ud
P
P
Ft+2h,T
= Ft,T
ud = 46. 079 21 1. 200 53 0.866 959 = 47. 959 73

dd
P
P 2
Ft+2h,T
= Ft,T
d = 46. 079 21 0.866 9592 = 34. 633 98

(My numbers are calculated using Excel so you might not be able to fully
match mine.)
Next, convert the prepaid forward price tree into a stock price tree. The
one-to-one mapping between the prepaid forward
price and the stock price is

Der(TD tt) if TD t + t
P
P
St+t = Ft+t,T +P V (Div) = Ft+t,T +
0
if TD < t + t
The PV of the dividend at each interval is:
Time
0
0.25 0.5 0.75
Dividend time
0.25
Dividend amount
$4
P V (Div)
3. 920 79 4
0
0

P Vt=0 (Div) = 4e0.08(0.250) = 3. 920 79


We discount $4 from t = 0.25 to time zero
P Vt=0.25 (Div) = 4e0.08(0.250.25) = 4
We discount $4 from t = 0.25 to t = 0.25
For t > 0.25, we have P Vt (Div) = 0
Next, we add the P V (Div) to the prepaid forward price:
Time 0
0.25
0.5
0.75

1
95.71879

79.73044
66.41269
55.31947 + 4
46.07921 + 3. 920 79

69.12305
57.57710

47.95973
39.94880 + 4

49.91701
41.57914

34.63398

36.04742
30.02625
26.03154

Now the stock price tree is:


www.actuary88.com

c
Yufeng
Guo

103

CHAPTER 11. BINOMIAL OPTION PRICING II


Time 0

0.25

0.5

0.75

1
95.718794

79.730440
66.412694
59.319474
50

69.123048
57.577104

47.959733
43.948797

49.917007
41.579138

34.633981

36.047421
30.026253
26.031540

The risk neutral probabilities are:


erh d
e0.08(0.25) 0.866 959
u =
=
= 0.459 399
ud
1. 200 53 0.866 959
d = 1 0.459 399 = 0.540 601
From this point on, we can just use the standard binomial tree formula.
First, we calculate the European call premium. We start from right to left,
calculating the roll-back value.
Time 0 0.25
0.5
0.75
1
50.71879
35.62150
23.17717
24.12305
14.27212
13.46816
8.43381
7.23801
4.91701
3.78761
2.21414
0.99703
0
0
0
In the above table, the final column is the call payo. For example,
50.71879 = max (0, 95.718794 45) = 50. 718 794
The first 4 columns are the roll-back values. For example,
35.62150 = e0.08(0.25) (50.71879 0.459 399 + 24.12305 0.540 601) = 35.
621 504
8.43381 = e0.08(0.25) (14.27212 0.459 399 + 3.78761 0.540 601) = 8. 433 809
The European call premium is 8.43381.
To calculate the American call premium, we still work from right to left.
However, well need to compare the roll back value and the exercise value and
take the greater of the two.
The American call premium is calculated as follows:
www.actuary88.com

c
Yufeng
Guo

104

CHAPTER 11. BINOMIAL OPTION PRICING II


Time 0

0.25

0.5

0.75

1
50.71879

35.62150
23.17717
max (14.27212, 14.31947)
8.45513

24.12305
13.46816

7.23801
3.78761

4.91701
2.21414

0.99703

0
0
0

Its optimal to exercise the American call at the upper node at t = 0.25. The
roll back value is 14.27212. The exercise value is 59.319474 45 = 14. 319 474,
which is greater than the roll back value. The premium of 8.45513 is calculated
as follows:
The roll back value is: e0.08(0.25) (14. 319 474 0.459 399 + 3.78761 0.540 601) =
8. 455 132 8
The early exercise value at t = 0 is 50 45 = 5.
We take the greater of the two. So the American call premium is 8.45513.
By the way, if you bother to calculate the European put and the American
put premium with strike price K = 45, here are the results:
The prepaid forward price tree and the stock price tree wont change whether
the option is a call or put.
The European put premium:
Time 0 0.25
0.5

0.75

1
0

0
0

1.33205
3.89484

0
2.51380

6.21822

0
4.74394

9.59857

8.95258
14.08269
18.96846

So the European put premium is 3.89484.


Check the put-call parity: C + P V (K) = P + S P V (Div)
C + P V (K) = 8.43381 + 45e0.08(1) = 49. 974 05
P + S P V (Div) = 3.89484 + 50 3. 920 79 = 49. 974 05
So C + P V (K) = P + S P V (Div) holds.
The American put premium:
www.actuary88.com

c
Yufeng
Guo

105

CHAPTER 11. BINOMIAL OPTION PRICING II


Time 0

0.25

0.5

0.75

1
0

0
0

1.33205
4.11033

0
2.51380

6.62489

0
4.74394
8.95258

10.36602
14.97375

18.96846
The two bold numbers indicate that early exercise is optimal.
The American put premium is 4.11033.

www.actuary88.com

c
Yufeng
Guo

106

Chapter 12

Black-Scholes formula
Problem 12.1.
Skip this spreadsheet problem.
Problem 12.2.
Skip this problem but remember the following key point. As n gets bigger,
the price calculated using the discrete binomial tree method approach the price
calculated using the Black-Scholes formula.
Problem 12.3.
a. r = 8%
=0
T
European call price
1
7.8966
10
56.2377
100
99.9631
1, 000
100
1, 0000 100
1, 0000 100
As T , the European call premium approaches the current stock price.
b. r = 8%
T
1
10
100
1, 000
1, 0000
10, 0000

= 0.1%
European call price
7.8542
55.3733
90.4471
36.7879
0.0045
0
107

CHAPTER 12. BLACK-SCHOLES FORMULA


Dividend reduces the value of the stock. Over a long period of time, the
value of the underlying stock is reduced to zero; the value of the call is reduced
to zero.
Problem 12.4.
a. r = 0%
= 8%
T
European call price
1
18.6705
10
10.1571
100
0.0034
1, 000 0.0000
Dividend reduces the value of the stock. Over a long period of time, the
value of the underlying stock is reduced to zero; the value of the call is reduced
to zero.
b. r = 0.1%
= 8%
T
European call price
1
18.7281
10
10.2878
100
0.0036
1, 000 0.0000
Dividend reduces the value of the stock, but you can earn interest on the
strike asset with r = 0.1% (vs. r = 0 in the previous problem).
Consequently, the call is more valuable when r = 0.1% than when r = 0.
However, even with r = 0.1%, over a long period of time, the value of the
underlying stock is reduced to zero due to the dividend paid out; the call value
approaches zero.
Problem 12.5.
a. We need to find the 90-strike yen-denominated euro put.
The underlying asset is 1 euro.
The strike asset is expressed in yen.
The current price of the underlying is 95 yen. S = 95
The strike asset is 90 yen. K = 90
The strike asset 90 yen earns 1.5%. So the risk free rate is r = 1.5%
(always remember that r is the earning rate of the strike asset)
www.actuary88.com

c
Yufeng
Guo

108

CHAPTER 12. BLACK-SCHOLES FORMULA


The underlying (i.e. 1 euro) earns 3.5%. So the dividend rate is = 3.5%
(always remember that is the earning rate of the underlying asset)
T = 0.5
ln
d1 =
0.658 560

95
1
1
S
ln
+ r + 2 T
+ 0.015 0.035 + 0.12 0.5
K
2
90
2

=
=
0.1
0.5
T

N (d1 ) = NormalDist (0.658 560) = 0.744 910 8


N (d1 ) = 1 N (d1 ) = 1 0.744910 8 = 0.255 089
d2 = d1 T = 0.658 560 0.1 0.5 = 0.587 849
N (d2 ) = 0.721 683
N (d2 ) = 1 N (d2 ) = 1 0.721 683 = 0.278 317
P = SeT N (d1 ) + KerT N (d2 )
= 95e0.035(0.5) 0.255 089 + 90e0.015(0.5) 0.278 317 = 1. 048 3 (yen)
1
b. Now we need to find the price of a
strike euro-denominated yen call
90
with 6 months to expiration.
The underlying asset is 1 yen. The earning rate of the underlying asset is
1.5%. So = 1.5%
1
1
euro. So S =
The current price of the underlying asset is
95
95
1
1
The strike asset is
euro. So K =
. The earning rate of the strike
90
90
asset is 3.5%. So r = 3.5%
Volatility is still 10%
C = SeT N (d1 ) KerT N (d2 )

1/95
S
1 2
1
2
ln
ln
+ r+ T
+ 0.035 0.015 + 0.1 0.5
K
2
1/90
2

=
d1 =
=
0.1 0.5
T
0.587 849
N (d1 ) = 0.278 316 8

d2 = d1 T = 0.587 849 0.1 0.5 = 0.658 559 7


N (d2 ) = 0.255 089
C = SeT N (d1 )KerT N (d2 ) =
0.255 089 = 0.0001 226 1 (euro)

1 0.015(0.5)
1
0.278 316 8 e0.035(0.5)
e
95
90

c. A 90-strike yen-denominated euro put is worth 1. 048 3 (yen). This put is


"give 1 euro and get 90 yen."
www.actuary88.com

c
Yufeng
Guo

109

CHAPTER 12. BLACK-SCHOLES FORMULA


The privilege of "give 1 euro and get 90 yen" at T = 0.5 is worth 1. 048 3
(yen) at t = 0 (statement a)
This can be expressed at (1 90Y ) = 1. 048 3Y
1
strike euro-denominated yen call is worth 0.0001 226 1 (euro). This
90
1
call is "give
euro and get 1 yen."
90
A

1
euro and get 1 yen" at T = 0.5 is worth 0.0001
90
226 1 (euro) at t = 0 (statement b)

1
This can be expressed at
1Y = 0.0001 226 1
90
The privilege of "give

Statement a and b are essentially the same statement. Using Statement a,


we can derive Statement b; using Statement b, we can derive Statement a.
First, we derive Statement b from Statement a.
(1 90Y
(statement a)
) = 1. 0483Y
1
1. 048 3
1
1Y =
(1 90Y ) =
Y

90
90
90
The above equation means this: If you can sell 1 for a guaranteed price
1
for a guaranteed price of 1Y . This
90Y , then you must be able to sell
90
should make intuitive sense.
1
1. 048 3
Y into euros. Since at t = 0, 1=95Y or 1Y =
,
We convert
90
95
then
1. 048 3
1
1. 048 3
Y =
= 0.0001226 1
90
90 95
1
1. 048 3
1
1Y =
(1 90Y ) =
Y = 0.0001226 1

90
90
90
This is exactly Statement b
Next,
we derive
Statement b from Statement a.

1
1Y = 0.0001 226 1 (statement b)
90

1
1Y = 90 (0.0001 226 1) = 90 (0.0001 226 1) 95Y =

(1 90Y ) = 90
90
1. 048 3Y
This is exactly Statement a.

Problem 12.6.
www.actuary88.com

c
Yufeng
Guo

110

CHAPTER 12. BLACK-SCHOLES FORMULA


a.
C = SeT N (d1 ) KerT N (d2 )

100
1
1
S
ln
+ r + 2 T
+ 0.06 0 + 0.42 1
ln
K
2
105
2

=
= 0.228 024 589
d1 =
0.4 1
T
0.228 025
N (d1 ) = 0.590 186 6

d2 = d1 T = 0.228 025 0.4 1 = 0.171 975


N (d2 ) = 0.431 729
C = SeT N (d1 ) KerT N (d2 ) = 100e0(1) 0.590 186 6 105e0.06(1)
0.431 729 = 16. 327 019
b.
F0,T = Se(r)T = 100e(0.060)1 = 106. 183 65
c
According to the textbook Equation 12.7, if the underlying asset is futures
instead of stocks, we can use the general Black-Scholes formula except
set F = S (replace the stock price with futures price)
set = r

1
S
F
100e(0.060)1 1
1 2
+ r + 2 T
ln
ln
T
+

+ 0.42 (1)
K
2
K 2
105

2
d1 =
=
=
=
0.4 1
T
T

1
100
+ 0.06 0 + 0.42 1
ln
105
2

= 0.228 024 589 0.228 025


0.4 1
N (d1 ) = 0.590 186 6

d2 = d1 T = 0.228 025 0.4 1 = 0.171 975


N (d2 ) = 0.431 729
C = SerT N (d1 ) KerT N (d2 ) = 100e0(1) 0.590 186 6 105e0.06(1)
0.431 729 = 16. 327 019
ln

The call premium in c is equal to the call premium in a. Why? We can


prove this mathematically. Replace S with F = Se(r)T and set = r , we
get:

S
1
1
Se(r)T
ln
+ r r + 2 T
+ r + 2 T
ln
K
2
K
2

d1 =
=
T
T

d2 = d1 T

C = Se(r)T erT N (d1 ) KerT N (d2 ) = SeT N (d1 ) KerT N (d2 )

www.actuary88.com

c
Yufeng
Guo

111

CHAPTER 12. BLACK-SCHOLES FORMULA


This is the
asset is a stock.
formula when the underlying

same call
P = Se(r)T erT N (d1 ) + KerT N (d2 ) = SeT N (d1 ) +
KerT N (d2 )
This is the same put formula when the underlying asset is a stock.
We can also prove that the futures option premium is equal to the underlying
stock option premium intuitively. On the maturity date T , the futures price is
equal to the stock price. So if we stand at T , the payo of a futures option and
the payo of an otherwise identical stock option are identical. Consequently, the
premium of a futures option is equal to the premium of an otherwise identical
stock option.
Problem 12.7.
a.

100
1
1
S
ln
+ r + 2 T
+ 0.08 0.03 + 0.32 0.75
K
2
95
2

d1 =
=
=
0.3 0.75
T
0.471 669

d2 = d1 T = 0.471 669 0.3 0.75 = 0.211 861


N (d1 ) = 0.681 418
N (d2 ) = 0.583 892
C = SeT N (d1 ) KerT N (d2 ) = 100e0.03(0.75) 0.681 418 95e0.08(0.75)
0.583 892 = 14. 386 295
ln

b.

1
100e0.03(0.75)
1
S
2
+
0

0
+
0.75
ln
+ r + 2 T

0.3
K
2
2
95e0.08(0.75)

=
d1 =
=
0.3 0.75
T
0.471 669100

d2 = d1 T = 0.471 669 0.3 0.75 = 0.211 861


ln

N (d1 ) = 0.681 418


N (d2 ) = 0.583 892
C = SeT N (d1 ) KerT N (d2 ) = 100e0.03(0.75) 0.681 418 95e0.08(0.75)
0.583 892 = 14. 386 295
The call premium is b is the same as the call premium in a. This is because
the call and
putpremium formula can be rewritten as:
C = SeT N (d1 ) KerT N (d2 )

P = SeT N (d1 ) + KerT N (d2 )

1 2
SeT
S
1
+
0

0
+
ln
T
ln
+ r + 2 T

K
2
KerT
2

=
d1 =
T
T
d2 = d1 T
www.actuary88.com

c
Yufeng
Guo

112

CHAPTER 12. BLACK-SCHOLES FORMULA


From the above equations, you see that instead of using r and , we can
replace S with SeT , replace K with KerT , and set r = = 0. This will also
give us the correct option premium.
Problem 12.8.
a. F0,T =0.75 = Se(r)T = 100e(0.080.03)0.75 = 103. 821 20
b. To find the futures option premium, we replace S with F0,T and replace
with r in the standard
Black-Scholes
formula:

103. 821 20
S
1
1
ln
ln
+ r + 2 T
+ 0.08 0.08 + 0.32 0.75
K
2
95
2

=
=
d1 =
0.3 0.75
T
0.471 669

d2 = d1 T = 0.471 669 0.3 0.75 = 0.211 861


N (d1 ) = 0.681 418
N (d2 ) = 0.583 892
C = SeT N (d1 )KerT N (d2 ) = 103. 821 20e0.08(0.75) 0.681 41895e0.08(0.75)
0.583 892 = 14. 386 295
Here is another method. Since future option premium is equal to the stock
option premium, we can just calculate the stock option premium. Actually, the
stock call option premium is calculated in 12.7 a.
c. 12.7 a and 12.8 b have the same premium.
Problem 12.9.
a.When a stock pays discrete dividend, well use the textbook Equation 12.5.
P
N (d1 ) P V (K) N (d2 )
C = F0,T
P
F0,T = S0 P V (Div) = 50 P V (Div)
P V (Div) = 2e0.08(1/360) = 1. 999 555 6 = 2
P
F0,T
(S) = 50 2 = 48
P
F0,T
1
1
48
+ 2 T
+ 0.32 0.5
ln
0.08(0.5)
P V (K) 2
40e
2

d1 =
=
= 1. 154 1
0.3
0.5

d2 = d1 T = 1. 154 1 0.3 0.5 = 0.9420

ln

N (d1 ) = 0.875 77
N (d2 ) = 0.826 90
C = 48 (0.875 77) 40e0.08(0.5) (0.826 90) = 10. 258
So the European call premium is 10. 258
www.actuary88.com

c
Yufeng
Guo

113

CHAPTER 12. BLACK-SCHOLES FORMULA


If its ever optimal to early exercise an American call, the best time to early
exercise is immediately before the dividend payment. Since the dividend is
paid tomorrow, the best time to early exercise is today. The exercise value is
EV = 50 40 = 10, which is less than the European call premium 10. 258. So
its not optimal to early exercise the American call.
Some of you might think that its optimal to early exercise the American
call. If we early exercise today, we get a stock, which will pay us a dividend 2.
So the total exercise value is 12.
This reasoning is flawed. If you exercise the call today and take ownership
of a stock, youll get $2 dividend tomorrow. However, after the dividend is paid,
the price of your stock is reduced by the amount of the dividend $2 (so you also
lose $2). So your exercise value is 10, not 12. And its not optimal to early
exercise the American call.
P
b. C = F0,T
N (d1 ) P V (K) N (d2 )
P
F0,T = S0 P V (Div) = 50 P V (Div)
P V (Div) = 2e0.08(1/360) = 1. 999 555 6 = 2
P
F0,T
(S) = 50 2 = 48
P
F0,T
1
1
48
+ 2T
+ 0.32 0.5
ln
0.08(0.5)
P V (K) 2
2
40e

d1 =
=
= 1. 154 1
0.3 0.5
T

d2 = d1 T = 1. 154 1 0.3 0.5 = 0.9420

ln

N (d1 ) = 0.875 77
N (d2 ) = 0.826 90
C = 48 (0.875 77) 40e0.08(0.5) (0.826 90) = 10. 258
The exercise value is EV = 60 40 = 20 > C
Hence its optimal to early exercise the American put at t = 0.
c. Its optimal to early exercise the American call at t = 0 if the exercise
value is greater than the European call premium. Its not optimal to exercise
the American call at t = 0 if the exercise value is equal to or less than the he
European call premium. However, keep in mind that if the stock doesnt pay
dividend, then its never optimal to early exercise an American call.
Problem 12.10.
The statement means that the absolute value | (t) | = | V
t | reaches its maximum value when t T . In other words, the closer to the expiration date, the
higher the | (t) |. This statement is not correct. I dont know of any intuitive
way to explain why this statement is not correct. Thats why the textbook asks
you to test this statement using a spreadsheet.
www.actuary88.com

c
Yufeng
Guo

114

CHAPTER 12. BLACK-SCHOLES FORMULA


If you want to test the statement using a spreadsheet, you can use the
spreadsheet titled "optbasic2." Then you can make up a case and validate the
statement.
However, for the purpose of passing the exam, you can ignore this problem.
Problem 12.11.
a. If you look at the Appendix 12.A (which is excluded from both CAS and
SOA exam), you can see that Vega is the derivative of an option price regarding
volatility:
V ega = V

( )
The formula V ega = V (+ )V
is an approximation. For this approx2
imation to work, needs to be small. Since Appendix 12.A is excluded from
both CAS and SOA exam, you can ignore this part.
b. If you want to solve this problem, you can set up some test cases and
compare the approximated Vega with the actual Vega (using the spreadsheet).
For the purpose of passing the exam, you can ignore this part.
Problem 12.12.
Since Appendix 12.A is excluded from both CAS and SOA exam, you can
ignore this problem.
Problem 12.13.
Lets not worry about drawing a diagram and focus on how to calculate the
profit. Ill do some sample calculations.
Lets calculate the profit after 6 months (i.e. at expiration) assuming the
stock price after 6 months is $60.
Using the Black-Scholes formula, we can find:
The 40-strike call premium is 4.1553. This call premium is calculated
using the Black-Scholes formula by setting S = 40, K = 40, T = 0.5, r =
0.08, = 0.3, = 0
The 45-strike call premium is 2.1304. This call premium is calculated
using the Black-Scholes formula by setting S = 40, K = 45, T = 0.5, r =
0.08, = 0.3, = 0
The net cost of the bull spread at t = 0 is 4.1553 2.1304 = 2. 024 9
Its future value at expiration is 2. 024 9e0.08(0.5) = 2. 107 5
The stock price at expiration is 60.
www.actuary88.com

c
Yufeng
Guo

115

CHAPTER 12. BLACK-SCHOLES FORMULA


The 40-strike call payo is 60 40 = 20
The 45-strike call payo is (60 45) = 15
The total payo is 20 15 = 5
So our profit is: 2. 107 5 + 5 = 2. 892 5
Lets calculate the profit after 3 months assuming the stock price after 3
months is $60.
Using the Black-Scholes formula, we can find:
The 40-strike call premium is 4.1553
The 45-strike call premium is 2.1304
The net cost of the bull spread at t = 0 is 4.1553 2.1304 = 2. 024 9
Its future value after 3 months is 2. 024 9e0.08(0.25) = 2. 065 8
3 months later (i.e. at t = 0.25), we close our position. Right now, our
purchased 40-strike call and sold 45-strike call both have 3 months to expiration.
To close our position (i.e. to cancel out our position), at t = 0.25, we sell a 40strike call and buy a 45-strike call. After this, our net position is zero. At
t = 0.25, the stock price is 60.
At t = 0.25, the 40-strike call with 3 months to expiration is worth 20.7969
(so we receive 20.7969). This call premium is calculated using the BlackScholes formula by setting S = 60, K = 40 T = 0.25, r = 0.08, = 0.3, =
0.
At t = 0.25, the 45-strike call with 3 months to expiration is worth 15.9480
(so we pay 15.9480). This call premium is calculated using the BlackScholes formula by setting S = 60, K = 45 T = 0.25, r = 0.08, =
0.3, = 0
The net receipt is 20.7969 15.9480 = 4. 848 9
So our profit is: 2. 065 8 + 4. 848 9 = 2. 783 1
Lets calculate the profit after 1 day assuming the stock price after 1day is
$60.
Using the Black-Scholes formula, we can find:
The 40-strike call premium is 4.1553
The 45-strike call premium is 2.1304
The net cost of the bull spread at t = 0 is 4.1553 2.1304 = 2. 024 9
Its future value after 1 day is 2. 024 9e0.08(1/365) = 2. 025 34
www.actuary88.com

c
Yufeng
Guo

116

CHAPTER 12. BLACK-SCHOLES FORMULA


1 day later (i.e. at t = 364/365), we close our position. Right now, our
purchased 40-strike call and sold 45-strike call both have 364 days to expiration.
To close our position (i.e. to cancel out our position), at t = 364/365 we sell a
40-strike call and buy a 45-strike call. After this, our net position is zero. At
t = 364/365, the stock price is 60.
At t = 364/365, the 40-strike call with 364 days to expiration is worth
23.3775 (so we receive 23.3775). This call premium is calculated using
the Black-Scholes formula by setting S = 60, K = 40 T = 364/365, r =
0.08, = 0.3, = 0.
At t = 364/365, the 45-strike call with 364 days to expiration is worth
19.2391 (so we pay 19.2391). This call premium is calculated using the
Black-Scholes formula by setting S = 60, K = 45 T = 364/365, r =
0.08, = 0.3, = 0
The net receipt is 23.3775 19.2391 = 4. 138 4
So our profit is: 2. 038 44 + 4. 138 4 = 2. 099 96
Problem 12.14.
Appendix 12.A is excluded from both CAS and SOA exam. So SOA and
CAS cant ask you to calculate option Greeks using formulas in Appendix 12.A.
(SOA and CAS can ask you to calculate delta using the textbook formula
10.1).
However, SOA and CAS can ask you to calculate the Greeks for a portfolio
using the formula presented in Page 389. Page 389 is on the syllabus.
So you need to learn how to calculate portfolios Greeks using the following
formula (in the textbook Page 389):
Greekoption =
a.

S = 40
1

Price
Delta
Gamma
Vega
Theta
Rho

Pn

1
1
1
1
1
1

i=1

i Greeki

Greek1
bought 40K call
4.1553
0.6159
0.0450
0.1080
0.0134
0.1024

2
1
1
1
1
1
1

Greek2
sold 45K call
2.1304
0.3972
0.0454
0.1091
0.0120
0.0688

Pn
Greekoption = i=1 i Greeki
Portfolio Price and Greeks
1 (4.1553) + 1 (2.1304) = 2. 024 9
1 (0.6159) + 1 (0.3972) = 0.218 7
1 (0.0450) + 1 (0.0454) = 0.000 4
1 (0.1080) + 1 (0.1091) = 0.001 1
1 (0.0134) + 1 (0.0120) = 0.001 4
1 (0.1024) + 1 (0.0688) = 0.033 6

Column 3 and 5 are calculated using the "optbasic2" spreadsheet. You


dont need to worry about these two columns. YouP
just need to focus on the
n
last column and practice the formula Greekoption = i=1 i Greeki .
www.actuary88.com

c
Yufeng
Guo

117

CHAPTER 12. BLACK-SCHOLES FORMULA


b.

S = 45
1

Price
Delta
Gamma
Vega
Theta
Rho

1
1
1
1
1
1

Greek1
bought 40K call
7.7342
0.8023
0.0291
0.0885
0.0135
0.1418

2
1
1
1
1
1
1

Greek2
sold 45K call
4.6747
0.6159
0.0400
0.1216
0.0150
0.1152

P
Greekoption = ni=1 i Greeki
Portfolio Price and Greeks
1 (7.7342) + 1 (4.6747) = 3. 059 5
1 (0.8023) + 1 (0.6159) = 0.186 4
1 (0.0291) + 1 (0.0400) = 0.010 9
1 (0.0885) + 1 (0.1216) = 0.033 1
1 (0.0135) + 1 (0.0150) = 0.001 5
1 (0.1418) + 1 (0.1152) = 0.026 6

c. Ignore this part. Not sure what this problem wants to accomplish.
Problem 12.15.
a.

S = 40
1

Price
Delta
Gamma
Vega
Theta
Rho
b.

1
1
1
1
1
1

S = 45
1

Price
Delta
Gamma
Vega
Theta
Rho

1
1
1
1
1
1

Greek1
bought 40K call
2.5868
0.3841
0.0450
0.1080
0.0049
0.0898

Greek1
bought 40K call
1.1658
0.1977
0.0291
0.0885
0.0051
0.0503

1
1
1
1
1
1

1
1
1
1
1
1

Greek2
sold 45K call
5.3659
0.6028
0.0454
0.1091
0.0025
0.1474

P
Greekoption = ni=1 i Greeki
Portfolio Price and Greeks
1 (2.5868) + 1 (5.3659) = 2. 779 1
1 (0.3841) + 1 (0.6028) = 0.218 7
1 (0.0450) + 1 (0.0454) = 0.000 4
1 (0.1080) + 1 (0.1091) = 0.001 1
1 (0.0049) + 1 (0.0025) = 0.002 4
1 (0.0898) + 1 (0.1474) = 0.057 6

Greek2
sold 45K call
2.9102
0.3841
0.0400
0.1216
0.0056
0.1010

Pn
Greekoption = i=1 i Greeki
Portfolio Price and Greeks
1 (1.1658) + 1 (2.9102) = 1. 744 4
1 (0.1977) + 1 (0.3841) = 0.186 4
1 (0.0291) + 1 (0.0400) = 0.010 9
1 (0.0885) + 1 (0.1216) = 0.033 1
1 (0.0051) + 1 (0.0056) = 0.000 5
1 (0.0503) + 1 (0.1010) = 0.050 7

c. Ignore
Problem 12.16.
Theres no easy way to solve this problem manually. This type of problems
shouldnt show up in the exam.
Problem 12.17.
Theres no easy way to solve this problem manually. This type of problems
shouldnt show up in the exam.
www.actuary88.com

c
Yufeng
Guo

118

CHAPTER 12. BLACK-SCHOLES FORMULA


Problem 12.18.
S = 50

K = 60

r = 0.06

= 0.4

= 0.03

a.
Solve for h.
1 2
h (h 1) + (r ) h r = 0
2
1
0.42 h (h 1) + (0.06 0.03) h 0.06 = 0
2
h = 0.608 182 5
h = 1. 233 182 5
Use the bigger h for call and the smaller h for put.
hcall
1. 233 182 5

HCall
=
K=
60 = 317. 309 19
hcall 1
1. 233 182 5 1

K)
Cperpetual = (HCall

26. 351 83

HCall

hcall

= (317. 309 19 60)

50
317. 309 19

1. 233 182 5

The call should be exercised when the stock price reaches 317. 309 19. This
price is called the barrier.
The call premium is 26. 351 83.
b. Now = 0.04 (instead of = 0.03). Everything else is the same as in a.
Higher dividend yield means that the stock price will decrease more quickly.
Recall that the stock price drops by the dividend amount immediately after
the dividend payment time. We expect that under = 0.04 the optimal stock
price (i.e. barrier) is lower than the barrier when = 0.03. In addition, we
expect that the option price under = 0.04 is lower than the option price under
= 0.03.
Solve for h.
1 2
h (h 1) + (r ) h r = 0
2
1
0.42 h (h 1) + (0.06 0.04) h 0.06 = 0
2
h = 0.568 729 3
h = 1. 318 729 3
Use the bigger h for call and the smaller h for put.
hcall
1. 318 729 3

HCall
=
K=
60 = 248. 247 52
hcall 1
1. 318 729 3 1
Cperpetual =

(HCall

22. 751 3

K)

HCall

hcall

= (248. 247 52 60)

50
248. 247 52

1. 318 729 3

The call should be exercised when the stock price reaches 248. 247 52.
The call premium is 22. 751 28
c.Now r = 0.07 (instead of r = 0.06). Everything else is the same as in a.
www.actuary88.com

c
Yufeng
Guo

119

CHAPTER 12. BLACK-SCHOLES FORMULA


When the risk free interest rate goes up, stocks are expected to generate high
returns. Consequently, the value of a stock goes up; the call option premium
goes up (while the put option premium goes down). We expect that both the
barrier and the call premium go up when r goes up from 0.06 to 0.07.
Solve for h.
1 2
h (h 1) + (r ) h r = 0
2
1
0.42 h (h 1) + (0.07 0.03) h 0.07 = 0
2
h = 1. 218 245 8
h = 0.718 245 8
Use the bigger h for call and the smaller h for put.
hcall
1. 218 245 8

=
HCall
K=
60 = 334. 919 4
hcall 1
1. 218 245 8 1
Cperpetual =

(HCall

K)

27. 100 1

HCall

hcall

= (334. 919 4 60)

50
334. 919 4

1. 218 245 8

The call should be exercised when the stock price reaches 334. 919 4
The call premium is 27. 1001
d.
The higher the volatility, the more valuable an option is. As goes
up from 0.4 to 0.5, we expect both the barrier and the call option premium will
go up.
1 2
h (h 1) + (r ) h r = 0
2
1
0.52 h (h 1) + (0.06 0.03) h 0.06 = 0
2
h = 1. 170 189 9
h = 0.410 189 9
Use the bigger h for call and the smaller h for put.
hcall
1. 170 189 9

=
K=
60 = 412. 547 36
HCall
hcall 1
1. 170 189 9 1

K)
Cperpetual = (HCall

29. 835 5

HCall

hcall

= (412. 547 36 60)

50
412. 547 36

1. 170 189 9

The call should be exercised when the stock price reaches 412. 547 36
The call premium is 29. 835 5
Problem 12.19.
S = 50

K = 60

a.Solve for h.
www.actuary88.com

r = 0.06

= 0.4

= 0.03

1 2
h (h 1) + (r ) h r = 0
2
c
Yufeng
Guo

120

CHAPTER 12. BLACK-SCHOLES FORMULA


1
0.42 h (h 1) + (0.06 0.03) h 0.06 = 0
2
h = 0.608 182 5
h = 1. 233 182 5
Use the bigger h for call and the smaller h for put.
hput
0.608 182 5
K=
60 = 22. 690 80
HP ut =
hcall 1
0.608 182 5 1
Pperpetual = (K HP ut )
23. 074 7

S
HP ut

hput

= (60 22. 690 80)

50
22. 690 80

0.608 182 5

The put should be exercised when the stock price reaches 22. 690 80.
The put premium is 23. 074 7
b. Now = 0.04 (instead of = 0.03). Everything else is the same as in a.
Higher dividend yield means that the stock price will decrease more quickly.
Since the value of a put goes up if the stock price goes down, we expect that the
barrier price will go down (i.e. exercise occurs later because we want to exercise
when the stock price is low) and the put premium will go up.
1 2
Solve for h.
h (h 1) + (r ) h r = 0
2
1
0.42 h (h 1) + (0.06 0.04) h 0.06 = 0
2
h = 0.568 729 3
h = 1. 318 729 3
Use the bigger h for call and the smaller h for put.
hput
0.568 729 3
K=
60 = 21. 752 5
HP ut =
hcall 1
0.568 729 3 1
Pperpetual = (K HP ut )
23. 824 8

S
HP ut

hput

= (60 21. 752 5)

50
21. 752 5

0.568 729 3

The put should be exercised when the stock price reaches 21. 752 5
The put premium is 23. 824 8
c. As r goes up, people expect to get increased return from the stock. The
stock price goes up and the put option premium goes down.
We expect the barrier will go up (compared with a, meaning that exercise
occurs sooner).
1 2
h (h 1) + (r ) h r = 0
2
1
0.42 h (h 1) + (0.07 0.03) h 0.07 = 0
2
h = 1. 218 245 8
h = 0.718 245 8
Use the bigger h for call and the smaller h for put.
hput
0.718 245 8
K=
60 = 25. 080 67
HP ut =
hcall 1
0.718 245 8 1
www.actuary88.com

c
Yufeng
Guo

121

CHAPTER 12. BLACK-SCHOLES FORMULA

Pperpetual = (K

HP ut )

23. 824 8

S
HP ut

hput

= (60 25. 080 67)

50
25. 080 67

0.568 729 3

The put should be exercised when the stock price reaches 25. 080 67
The put premium is 23. 824 8
d.
The higher the volatility, the more valuable an option is. As goes
up from 0.4 to 0.5, we expect both the barrier will go down (i.e. exercise occurs
later) and the put option premium will go up.
1 2
h (h 1) + (r ) h r = 0
2
1
0.52 h (h 1) + (0.06 0.03) h 0.06 = 0
2
h = 1. 170 189 9
h = 0.410 189 9
Use the bigger h for call and the smaller h for put.
hput
0.410 189 9
HP ut =
K=
60 = 17. 452 5
hcall 1
0.410 189 9 1
Pperpetual = (K HP ut )
27. 629 4

S
HP ut

hput

= (60 17. 452 5)

50
17. 452 5

0.410 189 9

The put should be exercised when the stock price reaches 17. 452 5
The put premium is 27. 629 4

Problem 12.20.
For a and b, if you use the Black-Scholes formula, youll find that call and
put are both worth 17.6988.
For part c. After we switch S and K and switch r and , the put after
the switch and the call before the switch have the same value. This is not a
coincidence. Its explained in my solution to Problem 10.19.

Problem 12.21.
1 2
a.
h (h 1) + (r ) h r = 0
2
1
0.32 h (h 1) + (0.08 0.05) h 0.08 = 0
2
h = 1. 177 043
h = 1. 510 376
Use the bigger h for call and the smaller h for put.
hcall
1. 510 376

=
HCall
K=
90 = 266. 340 6
hcall 1
1. 510 376 1
www.actuary88.com

c
Yufeng
Guo

122

CHAPTER 12. BLACK-SCHOLES FORMULA

Cperpetual =

(HCall

K)

HCall

40. 158 9
The call premium is 40. 158 9

hcall

= (266. 340 6 90)

100
266. 340 6

1. 510 376

1 2
b.
h (h 1) + (r ) h r = 0
2
1
0.32 h (h 1) + (0.05 0.08) h 0.05 = 0
2
h = 0.510 38
h = 2. 177 04
Use the bigger h for call and the smaller h for put.
hput
0.510 38
HP ut =
K=
100 = 33. 791 5
hcall 1
0.510 38 1
Pperpetual = (K

HP ut )

St
HP ut

40. 158 9
The put premium is 40. 158 9

hput

= (100 33. 791 5)

90
33. 791 5

0.510 38

c. After we switch S and K and switch r and , the perpetual American


put after the switch and the perpetual American call before the switch have the
same value. This is a minor fact probably not worth memorizing. We are not
going to worry about the proof.

www.actuary88.com

c
Yufeng
Guo

123

CHAPTER 12. BLACK-SCHOLES FORMULA

www.actuary88.com

c
Yufeng
Guo

124

Chapter 13

Market making and delta


hedging
Problem 13.1.

91
40
S
1
1
ln
+ r + 2 T
+ 0.08 0 + 0.32
K
2
45
2
365
r

=
=
d1 =
T
91
0.3
365
0.578 25
r

91
= 0.728 04
d2 = d1 T = 0.578 25 0.3
365
N (d1 ) = 0.281 55
N (d2 ) = 0.233 29
C = SeT N (d1 ) KerT N (d2 ) = 40e0(91/365) 0.281 55 45e0.08(91/365)
0.233 29 = 0.971 3
ln

A call on 100 stocks is worth 100 (0.971 3) = 97. 13


Use the formula in Appendix 12.8 to find
= eT N (d1 ) = e091/365 N (d1 ) = 0.281 55 (positive delta means buying stocks)
Suppose a trader sells a call option on 100 stocks. To hedge his risk, the
trader should at t = 0
sell the call and receive 100 (0.971 3) = 97. 13
buy 0.281 55 (100) = 28. 155 stocks costing 40 (28. 155) = 1126. 2
borrow 1126. 2 97. 13 = 1029. 07 from the bank
125

CHAPTER 13. MARKET MAKING AND DELTA HEDGING


The traders net position is zero at t = 0
If the stock price is 39 on Day 1.
To close his short call position, the trader can buy, from the open market,
a 45-strike call expiring in 90 days. This new (purchased) call and the original
(sold) call have the same underlying, same expiration date, same strike price.
They will cancel out each other and the trader doesnt have any liabilities associated either call.
The cost of thenew call is 70.55.
Its calculated

as follows:
90
39
1 2
1
S
2
ln
+ r+ T
+ 0.08 0 + 0.3
ln
K
2
45
2
365
r

d1 =
=
=
T
90
0.3
365
0.753 706
r

90
d2 = d1 T = 0.753 706 346 0.3
= 0.902 675
365
N (d1 ) = 0.774 49
N (d2 ) = 0.183 349
C = SeT N (d1 ) KerT N (d2 ) = 39e0(90/365) 0.225 51345e0.08(90/365)
0.183 349 = 0.705 5
A call on 100 stocks is worth 100 (0.705 5) = 70. 55
So the trader needs to pay 70. 55 to buy a new call to cancel out the original
call he sold.
At t = 0, the trader owns 28. 155 stocks. Now one day later, the stocks
are worth 28. 155 (39) = 1098. 045. So the trader sells out his stocks, receiving
1098. 05.
In addition, the trader needs to pay back the loan borrowed from the bank.
The future value of the loan one day later is:
1029. 07e0.08(1/365) = 1029. 30
So the traders net wealth at t = 1/365 is:
1098. 05 (1029. 30 + 70. 55) = 1. 8
So the trader lost $1.8.
If you want to following the textbook calculation on Page 417 "Day 1:
Marking-to-market," here it is:
Day 1
Gain on 28. 155 shares
28. 155 (39 40) = 28. 155
Gain on the written call 97. 13 70. 55 = 26. 58
Interest
1029. 07 e0.08(1/365) 1 = 0.226
Overnight profit
28. 155 + 26. 58 0.226 = 1. 8
If the stock price is 40.5 on Day 1.
To close his short call position, the trader can buy, from the open market,
a 45-strike call expiring in 90 days. This new (purchased) call and the original
(sold) call have the same underlying, same expiration date, same strike price.
www.actuary88.com

c
Yufeng
Guo

126

CHAPTER 13. MARKET MAKING AND DELTA HEDGING


They will cancel out each other and the trader doesnt have any liabilities associated either call.
The cost of the new call is 110.46.
Its calculated

as follows:
40.5
90
S
1 2
1
2
ln
ln
+ r+ T
+ 0.08 0 + 0.3
K
2
45
2
365
r

=
=
d1 =
90
T
0.3
365
0.500 363
r

90
d2 = d1 T = 0.500 363 0.3
= 0.649 332
365
N (d1 ) = 0.308 41
N (d2 ) = 0.258 062
C = SeT N (d1 )KerT N (d2 ) = 40.5e0(90/365) 0.308 4145e0.08(90/365)
0.258 062 = 1. 104 65
A call on 100 stocks is worth 100 (1. 104 65) = 110. 465
One Day 1, the trader
pays 110. 46 and buy a new call to cancel out the original call he sold
sells out his 28. 155 stocks, receiving 28. 155 (40.5) = 1140. 28
pays 1029. 07e0.08(1/365) = 1029. 30 to the bank to payo the loan
So the traders net wealth on Day 1 is:
1140. 28 (1029. 30 + 110. 46) = 0.52
If you want to use the textbook notation, here you go:
Day 1
Gain on 28. 155 shares
28. 155 (40.5 40) = 14. 077 5
Gain on the written call 97. 13 110. 465 = 13.335
Interest
1029. 07 e0.08(1/365) 1 = 0.226
Overnight profit
14. 077 5 13. 335 0.226 = 0.52
So the trader gained $0.52.
Problem 13.2.

91
40
1
1
S
ln
+ r + 2 T
+ 0.08 0 + 0.32
K
2
40
2
365
r

=
=
d1 =
T
91
0.3
365
0.208 05
r

91
d2 = d1 T = 0.208 05 0.3
= 0.05 825
365
N (d2 ) = 0.476 77
N (d1 ) = 0.417 59
P = 40e0.08(91/365) 0.476 77 40e0(91/365) 0.417 59 = 1. 990 6
ln

www.actuary88.com

c
Yufeng
Guo

127

CHAPTER 13. MARKET MAKING AND DELTA HEDGING


A put on 100 stocks is worth 100 (1. 990 6) = 199. 06
Use the formula in Appendix 12.8 to find
= eT N (d1 ) = e091/365 0.417 59 = 0.417 6 (negative delta
means short selling stocks)
Suppose a trader sells a put option on 100 stocks. To hedge his risk, the
trader should at t = 0
sell the put and receive 100 (1. 990 6) = 199. 06
buy 0.417 6 (100) = 41. 76 stocks (i.e. short sell 41. 76 stocks) receiving
41. 76 (40) = 1670. 4
lend 199. 06 + 1670. 4 = 1869. 46 to a bank
The traders net position is zero at t = 0
If the stock price is 39.5 on Day 1.
To close his short put position, the trader can buy, from the open market,
a 40-strike put expiring in 90 days. This new (purchased) put and the original
(sold) put have the same underlying, same expiration date, same strike price.
They will cancel out each other and the trader doesnt have any liabilities associated either put.
The cost of the
as follows:

new put is

90
39
S
1 2
1
ln
ln
+ r+ T
+ 0.08 0 + 0.32
K
2
40
2
365
r

d1 =
= 0.0
=
T
90
0.3
365
3695
r

90
d2 = d1 T = 0.03695 0.3
= 0.112 02
365
N (d2 ) = 0.544 60
N (d1 ) = 0.485 26
P = KerT N (d2 )SeT N (d1 ) = 40e0.08(90/365) 0.544 6039e0(90/365)
0.485 26 = 2. 433 4
On Day 1 a put on 100 stocks is worth 100 (2. 433 4) = 243. 34
One Day 1, the trader
pays 243. 34, buying a new put to cancel out the original put he sold
buys back 41. 76 stocks to close the short sale position, paying 41. 76 (39) =
1628. 64
receives 1869. 46e0.08(1/365) = 1869. 87 from the bank
www.actuary88.com

c
Yufeng
Guo

128

CHAPTER 13. MARKET MAKING AND DELTA HEDGING


So the traders net wealth on Day 1 is:
243. 34 1628. 64 + 1869. 87 = 2. 11
So the trader lost 2. 11.
If you want to use the textbook notation, here you go:
Day 1
Gain on 41. 76 shares
41. 76 (40 39) = 41. 76
Gain on the written put 199. 06 243. 34 = 44.28
Interest income
1869. 46 e0.08(1/365) 1 = 0.41
Overnight profit
41. 76 44. 28 + 0.41 = 2. 11
If the stock price is 40.5 on Day 1.
To close his short call position, the trader can buy, from the open market,
a 40-strike call expiring in 90 days. This new (purchased) call and the original
(sold) call have the same underlying, same expiration date, same strike price.
They will cancel out each other and the trader doesnt have any liabilities associated either call.
The cost of the new call is calculated
as follows:

90
40.5
S
1 2
1
2
ln
ln
+ r+ T
+ 0.08 0 + 0.3
K
2
40
2
365
r

d1 =
=
=
T
90
0.3
365
0.290 29
r

90
d2 = d1 T = 0.290 29 0.3
= 0.141 32
365
N (d1 ) = 0.385 80
N (d2 ) = 0.443 81
P = KerT N (d2 )SeT N (d1 ) = 40e0.08(90/365) 0.443 8140.5e0(90/365)
0.385 80 = 1. 780 8
On Day 1 a put on 100 stocks is worth 100 (1. 780 8) = 178. 08
One Day 1, the trader
pays 178. 08, buying a new put to cancel out the original put he sold
buys back 41. 76 stocks to close the short sale position, paying 41. 76 (40.5) =
1691. 28
receives 1869. 46e0.08(1/365) = 1869. 87 from the bank
So the traders net wealth on Day 1 is:
178. 08 1691. 28 + 1869. 87 = 0.51
So the trader gains 0.51.
If you want to use the textbook notation, here you go:
Day 1
Gain on 41. 76 shares
41. 76 (40 40.5) = 20. 88
Gain on the written put 199. 06 178. 08 = 20. 98

Interest income
1869. 46 e0.08(1/365) 1 = 0.41
Overnight profit
20. 88 + 20. 98 + 0.41 = 0.51
www.actuary88.com

c
Yufeng
Guo

129

CHAPTER 13. MARKET MAKING AND DELTA HEDGING


Next, Im going to redo Problem 13.2 using the strike price K = 45. This
way, Problem 13.2 is similar to Problem 13.1 except that in Problem 13.1 theres
a call and in Problem
a put.

13.2 theres

91
40
S
1 2
1
ln
ln
+ r+ T
+ 0.08 0 + 0.32
K
2
45
2
365
r

=
=
d1 =
T
91
0.3
365
0.578 25
r

91
d2 = d1 T = 0.578 25 0.3
= 0.728 04
365
N (d1 ) = 0.718 45
N (d2 ) = 0.766 71
P = KerT N (d2 )SeT N (d1 ) = 45e0.08(91/365) 0.766 7140e0(91/365)
0.718 45 = 5. 082 6
A put on 100 stocks is worth 100 (5. 082 6) = 508. 26
Use the formula in Appendix 12.8 to find
= SeT N (d1 ) = e091/365 0.718 45 = 0.718 45 (negative delta
means short selling stocks)
Suppose a trader sells a put option on 100 stocks. To hedge his risk, the
trader should at t = 0
sell the call and receive 100 (5. 082 6) = 508. 26
buy 0.718 45 (100) = 71. 845 stocks (i.e. short sell 71. 845 stocks) receiving 71. 845 (40) = 2873. 8
lend 2873. 8 + 508. 26 = 3382. 06 to a bank
The traders net position is zero at t = 0
If the stock price is 39.5 on Day 1.
To close his short put position, the trader can buy, from the open market,
a 45-strike put expiring in 90 days. This new (purchased) put and the original
(sold) put have the same underlying, same expiration date, same strike price.
They will cancel out each other and the trader doesnt have any liabilities associated either put.
The cost of thenew call is 110.46.
Its calculated

as follows:
90
39
S
1 2
1
2
ln
ln
+ r+ T
+ 0.08 0 + 0.3
K
2
45
2
365
r

d1 =
=
=
T
90
0.3
365
0.753 706
r

90
d2 = d1 T = 0.753 706 346 0.3
= 0.902 675
365
N (d1 ) = 0.774 49
www.actuary88.com

c
Yufeng
Guo

130

CHAPTER 13. MARKET MAKING AND DELTA HEDGING


N (d2 ) = 0.816 65
P = KerT N (d2 )SeT N (d1 ) = 45e0.08(90/365) 0.816 6539e0(90/365)
0.774 49 = 5. 826 3
On Day 1 a put on 100 stocks is worth 100 (5. 826 3) = 582. 63
One Day 1, the trader
pays 582. 63, buying a new put to cancel out the original put he sold
buys back 71. 845 stocks to close the short sale position, paying 71. 845 (39) =
2801. 955
receives 3382. 06e0.08(1/365) = 3382. 801 from the bank
So the traders net wealth on Day 1 is:
582. 63 2801. 955 + 3382. 801 = 1. 784
So the trader lost 1.78.
If you want to use the textbook notation, here you go:
Day 1
Gain on 71. 845 shares
71. 845 (40 39) = 71. 845
Gain on the written put 508. 26 582. 63 = 74.37
Interest income
3382. 06 e0.08(1/365) 1 = 0.741
Overnight profit
71. 845 74. 37 + 0.741 = 1. 784
If the stock price is 40.5 on Day 1.
To close his short call position, the trader can buy, from the open market,
a 45-strike call expiring in 90 days. This new (purchased) call and the original
(sold) call have the same underlying, same expiration date, same strike price.
They will cancel out each other and the trader doesnt have any liabilities associated either call.
The cost of the new call is 110.46.
Its calculated

as follows:
90
40.5
1 2
1
S
2
ln
+ r+ T
+ 0.08 0 + 0.3
ln
K
2
45
2
365
r

d1 =
=
=
T
90
0.3
365
0.500 363
r

90
= 0.649 332
d2 = d1 T = 0.500 363 0.3
365
N (d1 ) = 0.691 59
N (d2 ) = 0.741 94
P = KerT N (d2 )SeT N (d1 ) = 45e0.08(90/365) 0.741 9440.5e0(90/365)
0.691 59 = 4. 725 76
On Day 1 a put on 100 stocks is worth 100 (4. 725 76) = 472. 576
One Day 1, the trader
www.actuary88.com

c
Yufeng
Guo

131

CHAPTER 13. MARKET MAKING AND DELTA HEDGING


pays 472. 576, buying a new put to cancel out the original put he sold
buys back 71. 845 stocks to close the short sale position, paying 71. 845 (40.5) =
2909. 72
receives 3382. 06e0.08(1/365) = 3382. 801 from the bank
So the traders net wealth on Day 1 is:
472. 576 2909. 72 + 3382. 801 = 0.505
So the trader gains 0.505.
If you want to use the textbook notation, here you go:
Day 1
Gain on 71. 845 shares
71. 845 (40 40.5) = 35. 92
Gain on the written put 508. 26 472. 576 = 35. 684
Interest income
3382. 06 e0.08(1/365) 1 = 0.741
Overnight profit
35. 92 + 35. 684 + 0.741 = 0.505
Problem 13.3.
buy a 40-45 bull spread=buy a 40-strike call and sell a 45-strike put
I used a spreadsheet to calculate the following so you might not be able to
fully match my result
Day 0:
Expiration T = 91/365
Stock price 40
Position
Strike Call premium
buy
40
2.7804
sell
45
0.9710
Net (one stock)
2.7804 + 0.9710 = 1. 809 4
Net (100 stocks)
100 (1. 809 4) = 180. 94

Delta
0.58240
0.28155
0.28155 + 0.58240 = 0.300 85
100 (0.300 85) = 30. 085

In the above table


positive premium means cash inflow (i.e. receiving premium)
negative premium means cash outgo (i.e. paying premium)
positive delta means buying stocks
negative delta means short selling stocks
On Day 0, the trader:
Buy a 40-strike call and sell a 45-strike call, paying 180. 94
Short sell 30. 085 stocks, paying 30. 085 (40) = 1203. 4. The negative delta
means short selling.
www.actuary88.com

c
Yufeng
Guo

132

CHAPTER 13. MARKET MAKING AND DELTA HEDGING


Deposit 1203. 4 180. 94 = 1022. 46 to a savings bank to earn a risk free
rate
The traders net position is zero.
Day 1:
Expiration T = 90/365
Stock price 39
Position
Strike price Call premium
Sell
40
2.2144
Buy
45
0.7054
Net (one stock)
2.2144 0.7054 = 1. 509
Net (100 stocks)
100 (1. 509) = 150. 9
Day 1, the trader closes out his position
Sell a 40-strike call to cancel out the original 40-strike call he bought; buy
a 45-strike call to cancel out the original 45-strike call he sold. The trader
receives 150. 9
Buy 30. 085 stocks from the market to close the short sale, paying 30.
085 (39) = 1173. 315
Receive 1022. 46e0.08(1/365) = 1022. 684 from the savings account
The traders net profit: 150. 9 1173. 315 + 1022. 684 = 0.269
If the stock price is 39 on Day 1, the trader gets 0.269 profit.
Day 1:
Expiration T = 90/365
Stock price 40.5
Position
Strike price Call premium
Sell
40
3.0621
Buy
45
1.1046
Net (one stock)
3.0621 1.1046 = 1. 957 5
Net (100 stocks)
100 (1. 957 5) = 195. 75
Day 1, the trader closes out his position
Sell a 40-strike call to cancel out the original 40-strike call he bought; buy
a 45-strike call to cancel out the original 45-strike call he sold. The trader
receives 195. 75
Buy 30. 085 stocks from the market to close the short sale, paying 30.
085 (40.5) = 1218. 442 5
Receive 1022. 46e0.08(1/365) = 1022. 684 from the savings account
The traders net profit: 195. 75 1218. 442 5 + 1022. 684 = 0.008 5 = 0.01
If the stock price is 39 on Day 1, the trader losses 0.01.
Problem 13.4.
www.actuary88.com

c
Yufeng
Guo

133

CHAPTER 13. MARKET MAKING AND DELTA HEDGING


Day 0:
Expiration T = 91/365
Stock price 40
Position Strike Put premium
buy
45
5.0824
sell
40
1.9905
Net
5.0824 + 1.9905 2 = 1. 101 4
Net
100 (1. 101 4) = 110. 14

Delta
(0.71845) = 0.718 451
0.41760
0.71845 0.41760 2 = 0.116 75
100 (0.116 75) = 11. 675

On Day 0, the trader:


Buy a 45-strike put and sell two 45-strike put, paying 110. 14
Short sell 11. 675 stocks, receiving 11. 675 (40) = 467
Deposit 467 110. 14 = 356. 86 to a savings bank to earn a risk free rate
The traders net position is zero.
Day 1:
Expiration T = 90/365
Stock price 39
Position Strike price Call premium
Sell
45
5.8265
Buy
40
2.4331
Net
5.8265 2.4331 2 = 0.960 3
Net
100 (0.960 3) = 96. 03
Day 1, the trader closes out his position
Sell a 45-strike put to cancel out the original 40-strike put he bought; buy
two 40-strike puts to cancel out the original two 40-strike puts he sold.
The trader receives 96. 03
Buy 11. 675 stocks from the market to close the short sale, paying 11.
675 (39) = 455. 325
Receive 356. 86e0.08(1/365) = 356. 938 2 from the savings account
The traders net profit: 96. 03 455. 325 + 356. 938 2 = 2. 36
If the stock price is 39 on Day 1, the trader loses 2.36.
Day 1:
Expiration T = 90/365
Stock price 40.5
Position Strike price Call premium
Sell
45
4.7257
Buy
40
1.7808
Net
4.7257 1.7808 2 = 1. 164 1
Net
100 (1. 164 1) = 116. 41
Day 1, the trader closes out his position
1 The delta formula in Appendix 12.B is the delta for the market maker if he sells an option.
If the market maker buys an option, add a negative sign to the formula. In this case, the delta
is -0.71845 if the trader sells the 45-strike put. However, since the trader buys a 45-strike put,
the delta is (0.91845) = 0.918 45

www.actuary88.com

c
Yufeng
Guo

134

CHAPTER 13. MARKET MAKING AND DELTA HEDGING


Sell a 45-strike put to cancel out the original 40-strike put he bought; buy
two 40-strike puts to cancel out the original two 40-strike puts he sold.
The trader receives 116. 41
Buy 11. 675 stocks from the market to close the short sale, paying 11.
675 (40.5) = 472. 84
Receive 356. 86e0.08(1/365) = 356. 938 2 from the savings account
The traders net profit: 116. 41 472. 84 + 356. 938 2 = 0.51
If the stock price is 39 on Day 1, the trader gains 0.51.
Problem 13.5.
T = 91/365
r = 8%
= 30%
K = 40

=0

Lets walk through Day 0 and Day 1 calculations.


Day
0
1
stock
$40.00
$40.50
put
$199.05
$178.08
delta
0.417596
0.385797
Investment
1, 869.43 $1, 740.56
Interest credited (end of the day)
$0.41
$0.381 5
Capital gain (end of the day)
$0.09
$4. 245 4
Daily profit (end of the day)
$0.50
$3. 864
Day 0:
Using the Black-Scholes formula, you can verify the put premium is C0 =
$199.05 and delta per stock is 0.417596. Negative delta means buying negative
number of stocks (i.e. the trader needs to short sell stocks). The trader buys
0 = 100 (0.417596) = 41. 759 6 stocks (i.e. short-sell 41. 759 6 stocks),
receiving 41. 759 640 = 1670. 384. In addition, the trader receives put premium
C0 =$199.05. So at t = 0 the traders asset (i.e. investment)
M V (0) = 0 S0 C0 = (41. 759 6) 40 199.05 = 1869. 434
The negative amount means that the trader receives 1869. 434
The trader lends out $1869. 434 (i.e. depositing $1869. 434) in a savings
account. Now his net position is zero.
At the end of Day 0, the stock price goes up from S0 = 40 to S1 = 40.5.
The put liability goes down from C0 = $199.05 to C1 = $178.08. The traders
asset before he rebalances his portfolio is (BR stands for before rebalance):
M V BR (1) = 0 S1 C1 = (41. 759 6) 40.5 178.08 = 1869. 343 8
The traders profit at the end of Day 0 is:
M V BR (1)M V (0) erh = 1869. 343 8(1869. 434) e0.08(1/365) = 0.499 98 =
0.5
www.actuary88.com

c
Yufeng
Guo

135

CHAPTER 13. MARKET MAKING AND DELTA HEDGING


To find the capital gain and the interest earned at the end of Day 0, we just
need to break down the profit at the end of Day 0 into two parts:

M V BR (1) M V (0) erh = M V BR (1) M V (0) + M V (0) erh 1


{z
} |
|
{z
}
capital gain

interest earned

Capital gain at the end of Day 0: M V BR (1) M V (0) = 1869. 343 8


(1869. 434) = 0.090 2

Interest earned at the end of Day 0: M V (0) erh 1 = (1869. 434) e0.08/365 1 =
0.409 78 = 0.41
The investment at the beginning of Day 0 is: M V (0) = 1869. 434
Day 1:
In the beginning of Day 1, the trader starts from a clean slate. He buys sells
1 = 100 (0.385797) = 38. 579 7 stocks (i.e. short sell stocks) and receives
the put premium C1 = $178.08.
His asset (or investment) is M V (1) = 1 S1 C1 = (38. 579 7) 40.5
178.08 = 1740. 557 85
The traders asset at the end of Day 1 before he rebalances the portfolio (i.e.
before he starts over from a clean slate the next day) is:
M V BR (2) = 1 S2 C2 = (38. 579 7) 39.25 230.55 = 1744. 803 225
The traders profit at the end of Day 1 is:
M V BR (2) M V (1) erh = 1744. 803 225 (1740. 557 85) e0.08(1/365) =
3. 864
To find the capital gain and the interest earned at the end of Day 1, we just
need to break down the profit at the end of Day 1 into two parts:

M V BR (2) M V (1) erh = M V BR (2) M V (1) + M V (1) erh 1


{z
} |
|
{z
}
capital gain

interest earned

The capital gain credited at the end of Day 1 is:

M V BR (2) M V (1) = 1744. 803 225 (1740. 557 85) = 4. 245 4


The interest
earned
at the end of Day 1is:

M V (1) erh 1 = (1740. 557 85) e0.08(1/365) 1 = 0.381 5

Ill omit the calculations for the other days. Here is the result for all days:
Day
0
1
2
stock
$40.00
$40.50
$39.25
put
$199.05
$178.08
$230.55
delta
0.417596
0.385797
0.46892
Investment
1, 869.43 $1, 740.56 $2, 071.07
Interest credited (end of the day)
$0.41
$0.381 5
$0.45
Capital gain (end of the day)
$0.09
$4. 245 4
$0.05
Daily profit (end of the day)
$0.50
$3. 864
$0.40
www.actuary88.com

c
Yufeng
Guo

136

CHAPTER 13. MARKET MAKING AND DELTA HEDGING


Day
stock
put
delta
Investment
Interest credited (end of the day)
Capital gain (end of the day)
Daily profit (end of the day)

3
$38.75
$254.05
0.50436
$2, 208.46
$0.48
$4.48
$4.00

4
$40.00
$195.49
0.41940
$1, 873.10
$0.41
$0.91
$1.32

5
$40.00
$194.58
0.41986
$1, 874.02

Day
stock
put
delta
Investment
Interest credited (end of the day)
Capital gain (end of the day)
Daily profit (end of the day)

0
$40.00
$199.05
0.41760
$1, 869.43
$0.41
$0.42
$0.01

1
$40.642
172.6644
0.37684
$1, 704.22
$0.37
$0.35
$0.02

2
40.018
196.5319
0.41731
$1, 866.51
$0.41
$0.40
$0.01

Day
stock
put
delta
Investment
Interest credited (end of the day)
Capital gain (end of the day)
Daily profit (end of the day)

3
39.403
222.5962
0.45918
$2, 031.89
$0.45
$0.45
$0.00

4
$38.80
250.8701
0.50202
$2, 198.56
$0.48
$0.48
$0.00

5
39.420
220.0727
0.45940
$2, 031.02

Problem 13.6.

Problem 13.7.
If SOA tests this type of problems in the exam, theyll need to give you at
least and . You can calculate the option premium V and delta . Once you
have Greeks, just use the formula:
1
V (St+h , T t h) V (St , T t) + t + h + t
2

(Textbook 13.6)

Im not going to do all the parts in the problem. Im just going to show you
some examples.
First, youll need to get Greeks. You can use the Black-Scholes formula and
find the option premium V and delta . To find and , youll need to use the
spreadsheet attached to the textbook.
= 0.3
r = 8%
=0
K = 40
www.actuary88.com

c
Yufeng
Guo

137

CHAPTER 13. MARKET MAKING AND DELTA HEDGING


Day
0
1
time t
0
h = 1/365
Expiration T t 180/365 = 0.493 150 684 9 179/365 = 0.490 410 958 9
stock price St
40
44
Call price Vt
4.1217
6.8991
Delta t
0.6151
0.7720
Gamma t
0.0454
0.0327
Theta t
0.0134
0.0137
V (St , T t) = V (40, 180/365

0)
=
4.1217

V (St+h , T t h) = V S0+1/365 , 180/365 0 1/365 = 6.8991


= St+h St = 44 40 = 4
t = 0.6151
t = 0.0134 365 (The spreadsheet gives the per-day theta; we need to
annualize it.)
t = 0.0454
1
V (St , T t) + t + t h + t 2
2
1
1
= 4.1217 + 0.6151 4 0.0134 365
+ 0.0454 42
365 2
= 6. 931 9
The true value is V (St+h , T t h) = 6.8991
The error percentage is:
6. 931 9 6.8991
= 0.4754 %
6.8991
= 0.3

r = 8%

Day
time t
Expiration T t
stock price St
Call price Vt
Delta t
Gamma t
Theta t

=0

K = 40

0
0
180/365 = 0.493 150 684 9
40
4.1217
0.6151
0.0454
0.0134

5
h = 5/365
175/365 = 0.479 452 054 8
44
6.8440
0.7726
0.0330
0.0138

V (St , T t) = V (40, 180/365


0) = 4.1217

V (St+h , T t h) = V S0+1/365 , 180/365 0 5/365 = 6.8440


= St+h St = 44 40 = 4
t = 0.0134 365
t = 0.0454
t = 0.6151
1
V (St , T t) + t + t h + t 2
2
5
1
= 4.1217 + 0.6151 4 0.0134 365
+ 0.0454 42
365 2
= 6. 878 3
The true value is V (St+h , T t h) = 6.8440
The error percentage is:
www.actuary88.com

c
Yufeng
Guo

138

CHAPTER 13. MARKET MAKING AND DELTA HEDGING


6. 878 3 6.8440
= 0.5 01%
6.8440
Problem 13.8.
Im not going to do
some examples.
= 0.3
r = 8%
Day
time t
Expiration T t
Stock price St
Put price Vt
Delta t
Gamma t
Theta t

all the parts in the problem. Im just going to show you


=0
K = 40
0
0
180/365 = 0.493 150 684 9
40
2.5744
0.3849
0.0454
0.0050

1
h = 1/365
179/365 = 0.490 410 958 9
44
1.3602
0.2280
0.0327
0.0053

V (St , T t) = V (40, 180/365


0) = 2.5744

V (St+h , T t h) = V S0+1/365 , 180/365 0 1/365 = 1.3602


= St+h St = 44 40 = 4
t = 0.0050 365
t = 0.0454
t = 0.3849
1 2
V (St , T t) + t + t h + t
2
1
1
= 2.5744 0.3849 4 0.0050 365
+ 0.0454 42
365 2
= 1. 393
The true value is V (St+h , T t h) = 1.3602
The error percentage is:
1. 393 1.3602
= 0.2 41%
1.3602
= 0.3

r = 8%

=0

K = 40

Day
0
5
time t
0
h = 5/365
Expiration T t 180/365 = 0.493 150 684 9 175/365 = 0.479 452 054 8
Stock price St
40
44
Put price Vt
2.5744
1.3388
Delta t
0.3849
0.2274
Gamma t
0.0454
0.0330
Theta t
0.0050
0.0054
V (St , T t) = V (40, 180/365

0)
=
2.5744

V (St+h , T t h) = V S0+1/365 , 180/365 0 1/365 = 1.3602


= St+h St = 44 40 = 4
t = 0.0050 365
t = 0.0454
t = 0.3849
1 2
V (St , T t) + t + t h + t
2
www.actuary88.com

c
Yufeng
Guo

139

CHAPTER 13. MARKET MAKING AND DELTA HEDGING


5
1
= 2.5744 0.3849 4 0.0050 365
+ 0.0454 42
365 2
= 1. 373
The true value is V (St+h , T t h) = 1.3602
The error percentage is: 1.3388
1. 373 1.3388
= 2. 55%
1.3388
Problem 13.9.
Im going to do one set of calculation assuming the stock price one day later
is $30. However, Im not going to produce a graph.
= 0.3

S = 40

r = 8%

=0

K = 40

Day
0
1
time t
0
h = 1/365
Expiration T t 91/365
90/365
Stock price St
40
30
Call price Vt
2.7804
0.0730
Delta t
0.5824
0.0423
Gamma t
0.0652
0.0202
Theta t
0.0173 0.9134
a. The price of a 40-strike, 90 day to expiration call is worth 0.0730
b. Use delta approximation:
= 30 40 = 10
0
V = V0 + t = 2.7804 + 0.5824 (10) = 3. 043 6
We get a nonsense value of 3. 043 6. This is because the delta approximation is good when is small. Here we have a large change of = 10.
c. Use delta-gamma approximation:
1
0
V = V0 + t + t 2
2
1
= 2.7804 + 0.5824 (10) + 0.0652 (10)2 = 0.216 4
2
d. Use delta-gamma-theta approximation:
1
0
V = V0 + t + t 2 + t h
2
1
1
2
= 2.7804 + 0.5824 (10) + 0.0652 (10) 0.0173 365
2
365
= 0.199 1
We see in a, b, c, and d, the approximation is not good. This is because the
approximation is good when is small. Here we have a large change of = 10.
Problem 13.10.
www.actuary88.com

c
Yufeng
Guo

140

CHAPTER 13. MARKET MAKING AND DELTA HEDGING


Im not going to produce a graph. Ill do one set of calculation assuming the
stock price one day later is $41.
= 0.3

S = 40

r = 8%

=0

K = 40

Day
0
1
time t
0
h = 1/365
Expiration T t 1
364/365
Stock price St
40
30
Call price Vt
6.2845
6.9504
Delta t
0.6615
0.6909
Gamma t
0.0305
0.0287
Theta t
0.0104 0.0106
a. The price of a 40-strike, 364 day to expiration call is worth 6.9504
b. Use delta approximation:
= 41 40 = 1
0
V = V0 + t = 6.2845 + 0.6615 (1) = 6. 946
c. Use delta-gamma approximation:
1
0
V = V0 + t + t 2
2
1
= 6.2845 + 0.6615 (1) + 0.0305 (1)2 = 6. 961 3
2
d. Use delta-gamma-theta approximation:
1
0
V = V0 + t + t 2 + t h
2
1
1
= 6.2845 + 0.6615 (1) + 0.0305 (1)2 0.0104 365
= 6. 950 9
2
365
We see in a, b, c, and d, the approximation is good. This is because is
small.
Problem 13.11.
Im going to do one set of calculation assuming the stock price one day later
is $30.
= 0.3

S = 40

r = 8%

Day
0
time t
0
Expiration T t 91/365
Stock price St
40
Put price Vt
1.9905
Delta t
0.4176
Gamma t
0.0652
Theta t
0.0088
a. The price of a 40-strike, 90
www.actuary88.com

=0

K = 40

1
h = 1/365
90/365
30
9.2917
0.9577
0.0202
0.0061
day to expiration put is worth 9.2917

c
Yufeng
Guo

141

CHAPTER 13. MARKET MAKING AND DELTA HEDGING


b. Use delta approximation:
= 30 40 = 10
0
V = V0 + t = 1.9905 0.4176 (10) = 6. 166 5
c. Use delta-gamma approximation:
1
0
V = V0 + t + t 2
2
1
= 1.9905 0.4176 (10) + 0.0652 (10)2 = 9. 426 5
2
d. Use delta-gamma-theta approximation:
1
0
V = V0 + t + t 2 + t h
2
1
1
= 1.9905 0.4176 (10) + 0.0652 (10)2 0.0088 365
2
365
= 9. 417 7
Problem 13.12.
Im going to do one set of calculation assuming the stock price one day later
is $41.
= 0.3

S = 40

r = 8%

=0

K = 40

Day
0
1
time t
0
h = 1/365
Expiration T t 1
364/365
Stock price St
40
41
Put price Vt
3.2092
2.8831
Delta t
0.3385 0.3091
Gamma t
0.0305
0.0287
Theta t
0.0023 0.0025
a. The price of a 40-strike, 364 day to expiration put is worth 2.8831
b. Use delta approximation:
= 41 40 = 1
0
V = V0 + t = 3.2092 0.3385 (1) = 2. 870 7
c. Use delta-gamma approximation:
1
0
V = V0 + t + t 2
2
1
2
= 3.2092 0.3385 (1) + 0.0305 (1) = 2. 885 95
2
d. Use delta-gamma-theta approximation:
1
0
V = V0 + t + t 2 + t h
2
1
1
= 3.2092 0.3385 (1) + 0.0305 (1)2 0.0023 365
2
365
= 2. 883 65
Problem 13.13.
www.actuary88.com

c
Yufeng
Guo

142

CHAPTER 13. MARKET MAKING AND DELTA HEDGING


Please note that in DM 13.9, is annualized. However, in Table 13.1, is
a per-day theta. (DM page 411 says that if the expiry is measured in years,
then theta is the annualized change in option value; to get the per-day theta,
divide by 365. The in Table 13.1 and in the authors Excel spreadsheet is
daily
theta.) So we need to annualize before using DM 13.9.

1
2
2

0.3 40 0.0652 0.0173 365 + 0.08 (0.5824 40 2.7804)


2
1
= 5. 79 4 105 0
365
Problem 13.14.
Well use the Excel spreadsheet attached in the DM textbook and calculate
the following:
Inputs:
Stock Price
40
Exercise Price
45
Volatility
30%
Risk-free interest rate
8%
Time to Expiration (years) 0.5
Dividend Yield
0%
Outputs:
Black-Scholes (European)
Call
Put
Price
2.1304
5.3659
Delta
0.3972
0.6028
Gamma
0.0454
0.0454
Vega
0.1091
0.1091
Theta
0.0120 0.0025
Rho
0.0688
0.1474
Psi
0.0794 0.1206
Elasticity 7.4578
4.4936
DM

13.9 is:
1

0.32 402 0.0454 0.0025 365 + 0.08 (0.6028 40 5.3659)


2

1
= 5. 29 106 0
365
Problem 13.15.

Use DMs spreadsheet. The inputs are:

www.actuary88.com

c
Yufeng
Guo

143

CHAPTER 13. MARKET MAKING AND DELTA HEDGING


Stock Price
Exercise Price
Volatility
Risk-free interest rate
Time to Expiration (years)
Dividend Yield

40
45
30%
8%
= 180/365
0%

Well get the following outputs:


Black-Scholes (European)
40-strike call
Price
4.1217
Delta
0.6151
Gamma 0.0454

45-strike call
2.1004
0.3949
0.0457

We bought one 45-strike call. Suppose we need to sell (i.e. write) X unit
of 40-strike call. The Gamma of the bought 45-strike call is 0.0457 (negative
because we bought a call). The Gamma of the sold X unit of 40-strike call is
0.0454X. The total Gamma of our portfolio is 0.0454X 0.0457. To Gamma
hedge, set 0.0454X 0.0457 = 0. This gives us X = 1. 006 6. The total Delta
of our portfolio is 0.6151 1. 006 6 0.3949 = 0.224 3. To delta hedge, we need
to buy 0.224 3 share of the underlying stock.
This is our final portfolio at time zero:
Transactions at time zero
Cost
Buy a 45-strike call
2.1004
Sell 1. 006 6 unit of 40-strike call (to Gamma hedge) 1. 006 6 4.1217 = 4. 148 9
Buy 0.224 3 share of the stock (to Delta hedge)
0.224 3 40 = 8. 972
Borrow 2.1004 4. 148 9 + 8. 972 = 6. 923 5
6. 923 5
Total
2.1004 4. 148 9 + 8. 972 6. 923 5 = 0
One day later, you close out your position:
Transactions (one day later)
Revenue
0
Sell a 45-strike call
C1
0
Buy 1. 006 6 unit of 40-strike call 1. 006 6C2
0
Sell 0.224 3 share of the stock
0.224 3S
Repay the borrowed amount
6. 923 5e0.08/365
0
0
0
Total
C1 1. 006 6C2 + 0.224 3S 6. 923 5e0.08/365
0

C1 is the price of a 45-strike call with 179 day to expiration.


0
C2 is the price of a 40-strike call with 179 day to expiration.
0
S is the price of the stock one day later.
0
0
If you want to draw the diagram of the overnight profit C1 1. 006 6C2 +
0
0
0.224 3S 6. 923 5e0.08/365 , you can get dierent profit by changing S . For
example,
www.actuary88.com

c
Yufeng
Guo

144

CHAPTER 13. MARKET MAKING AND DELTA HEDGING


0

assume S = 50. Also assume that the stock volatility and risk free interest
rate are not changed. Using the Black-Scholes formula (use T = 179/365), we
get:
0
0
C1 = 8.1511
C2 = 12.0043
The overnight profit is:
8.1511 1. 006 6 12.0043 + 0.224 3 50 6. 923 5e0.08/365 = 0.36
0

If S = 50, then the overnight profit is 0 1. 006 6 0 + 0.224 3 0 6.


923 5e0.08/365 = 6. 93.
0

By changing S , youll get dierent profits. Then you draw a graph on how
0
the overnight profit varies by S . Since its time-consuming to calculate the
0
overnight profit by changing S , Im not going to do it.
Problem 13.16.
Use DMs spreadsheet. The inputs are:
Stock Price
Exercise Price
Volatility
Risk-free interest rate
Time to Expiration (years)
Dividend Yield

40
45
30%
8%
= 180/365
0%

Well get the following outputs:


Black-Scholes (European)
40-strike call 45-strike put
Price
4.1217
5.3596
Delta
0.6151
0.6051
Gamma 0.0454
0.0457
We sell one 45-strike put and buy X unit of 40-strike call. The total Gamma
is 0.0457 0.0454X. To Gamma hedge, set 0.0457 0.0454X = 0. This gives
us X = 1. 006 6.
The delta of 1. 006 6 unit of 40-strike call is 0.6151 1. 006 6.
The total Delta is 0.6151 1. 006 6 0.6051 = 1. 224 3, meaning that we
need to short sell 1. 224 3 unit of the underlying stock.
Final portfolio at time zero:
Transactions at time zero
Sell a 45-strike put
Buy 1. 006 6 unit of 40-strike call (to Gamma hedge)
Short sell 1. 224 3 share of the stock (to Delta hedge)
Lend 5.3596 4. 148 9 + 48. 972 = 50. 182 7
Total
www.actuary88.com

c
Yufeng
Guo

Revenue
5.3596
1. 006 6 4.1217 = 4. 148 9
1. 224 3 40 = 48. 972
50. 182 7
5.3596 4. 148 9 + 48. 972 50. 182 7 = 0
145

CHAPTER 13. MARKET MAKING AND DELTA HEDGING


One day later, you close out your position:
Transactions (one day later)
Revenue
0
Buy a 45-strike put
P1
0
Sell 1. 006 6 unit of 40-strike call 1. 006 6C2
0
Buy 1. 224 3 share of the stock
1. 224 3S
Collect loan
50. 182 7e0.08/365
0
0
0
Total
P1 + 1. 006 6C2 1. 224 3S + 50. 182 7e0.08/365
0

P1 is the price of a 45-strike put with 179 day to expiration.


0
C2 is the price of a 40-strike call with 179 day to expiration.
0
S is the price of the stock one day later.
Problem 13.17.
Assume the butterfly spread is asymmetric.
K3 K2
45 40
=
=
= 0.5
K3 K1
45 35
Written butterfly spread consists of selling one 35-strike call, buying two
40-strike calls, and selling one 45-strike call.
Inputs: = 0.3
S = 40
r = 8%
=0
T = 91/365
strike K
35
40
45
butterfly spread
call price 6.1315
2.7804
0.9710
6.1315 2 (2.7804) + 0.9710 = 1. 541 7
Delta
0.8642
0.5824
0.2815
0.8642 2 (0.5824) + 0.2815 = 0.019 1
Gamma
0.0364
0.0652
0.0563
0.0364 2 (0.0652) + 0.0563 = 0.037 7
Theta
0.0134 0.0173 0.0134 0.0134 2 (0.0173) 0.0134 = 0.007 8
Inputs: = 0.3
S = 40
strike K
49
call price 4.1217
Delta
0.6151
Gamma
0.0454
Theta
0.0134

r = 8%

=0

T = 180/365

Suppose we sell X units of 180 day to expiration call. The total Gamma of
the written butterfly spread and written X units of 180 day to expiration call
is 0.037 7 + 0.0454X.
To Gamma hedge, set 0.037 7 + 0.0454X = 0, X = 0.830 4. The total
Delta of the written butterfly and the written X units of 180 day to expiration
call is 0.019 1 + 0.830 4 0.6151 = 0.491 7. So we need to buy 0.491 7 share of
the underlying stock.
Transactions at time zero
Sell the spread and buy 0.491 7 stock
Borrow 18. 126 3 from a bank
Total
www.actuary88.com

Cost (negative cost means receiving money)


1. 541 7 + 0.491 7 40 = 18. 126 3
18. 126 3
0

c
Yufeng
Guo

146

CHAPTER 13. MARKET MAKING AND DELTA HEDGING


One day later, we close out our position:
Transactions (one day later)
Buy a 35-strike 90 day to expiry call
Sell two 40-strike 90 day to expiry calls 1. 006 6
Buy a 45-strike 90 day to expiry call
Sell 0.491 7 stock
Repay the borrowed amount
Total
0

Revenue
0
C1
0
2C2
0
C3
0
0.491 7S
18. 126 3e0.08/365
0
0
0
0
C1 + 2C2 C3 + 0.491 7S 18. 126 3e0.08/365
0

So the total overnight profit is C1 +2C2 C3 +0.491 7S 18. 126 3e0.08/365 .


Problem 13.18.
inputs: = 0.3
S = 40
r = 8%
=0
T = 91/365
strike K
40
45
ratio spread
put price 1.9905
5.0824
2 1.9905 5.0824 = 1. 101 4
Delta
0.4176 0.7185 2 (0.4176) (0.7185) = 0.116 7
Gamma
0.0652
0.0563
2 0.0652 0.0563 = 0.074 1
Theta
0.0088 0.0037 2 (0.0088) (0.0037) = 0.013 9
in puts: = 0.3
S = 40
strike K
40
call price 4.1217
Delta
0.6151
Gamma
0.0454
Theta
0.0134

r = 8%

=0

T = 180/365

We buy X unit of 40-strike 180-day to expiration call. The total Gamma


of the ratio spread and bought X unit of 40-strike 180-day to expiration call
is 0.074 1 + 0.0454X. To Gamma hedge, set 0.074 1 + 0.0454X = 0, X = 1.
632 2. So we need to short sell 1. 632 2 shares of the underlying stock.
Transactions at time zero
Enter the ratio spread and short 1. 632 2 stock
Deposit 66. 389 4 in a savings account
Total
One day later, we close out our position:
Transactions (one day later)
Buy two 40-strike 90 day to expiry puts
Sell a 45-strike 90 day to expiry put
Buy 1. 632 2 stock
Receive money from the bank
Total
www.actuary88.com

c
Yufeng
Guo

Revenue
1. 101 4 + 1. 632 2 40 = 66. 389 4
66. 389 4
0

Revenue
0
2P1
0
P2
0
1. 632 2S
66. 389 4e0.08/365
0
0
0
2P1 + P2 1. 632 2S + 66. 389 4e0.08/365
147

CHAPTER 13. MARKET MAKING AND DELTA HEDGING


0

So the total overnight profit is 2P1 + P2 1. 632 2S + 66. 389 4e0.08/365 .


Skip Problem 13.19 and 13.20. Vega hedge and rho hedge are far outside
the scope of the Exam MFE syllabus.

www.actuary88.com

c
Yufeng
Guo

148

Chapter 14

Exotic options: I
Problem 14.1.
For n non-negative numbers (such as stock prices) S1 , S2 , ..., Sn , the arithmetic mean can never be less than the geometric mean:
S1 + S2 + ... + Sn
1/n
(S1 S2 ...Sn )
n
If and only if S1 = S2 = ... = Sn , we have:
S1 + S2 + ... + Sn
= (S1 S2 ...Sn )1/n
n
The proof can be found at Wikipedia:
http://en.wikipedia.org/wiki/Inequality_of_arithmetic_and_geometric_
means
For example, this is the proof for n = 2.
(S1 S2 )2 0
S12 + S22 2S1 S2
2
(S1 + S2 ) = S12 + S22 + 2S1 S2 4S1 S2

2
S1 + S2
S1 + S2
S1 S2
S1 S2
2
2
Problem 14.2.
Arithmetic average:
5+4+5+6+5
= 5.0
5
Geometric average:
(5 4 5 6 5)1/5 = 4. 959
Ignore the question "What happens to the dierence between the two measures of the averages as the standard deviation of the observations increase?"
This question is vague. Im not sure what the author is after.
149

CHAPTER 14. EXOTIC OPTIONS: I


Problem 14.3.

u = e(r)h+ h = e(0.080)0.5+0.30.5 = 1. 286 765 9


u = e(r)h h = e(0.080)0.50.3 0.5 = 0.841 868 0
e(r)h d
e(0.080)0.5 0.841 868
u =
=
= 0.447 165
ud
1. 286 766 0.841 868
d = 1 u = 1 0.447 165 = 0.552 835
t=0

t = 0.5

t=1
Suu = 165.57665

Su = 128.67659
S = 100

Sud = 108.32871
Sd = 84.18680
Sdd = 70.87417

The four arithmetic averages are:


128.67659 + 165.57665
= 147. 126 62
2

(The path is uu or up up)

128.67659 + 108.32871
= 118. 502 65
2

(ud)

84.18680 + 108.32871
= 96. 257 8
2
84.18680 + 70.87417
= 77. 530 485
2
Path
uu
ud
du
dd

Average asset
147. 126 62
118. 502 65
96. 257 8
77. 530 485

Call Payo
47. 126 62
18. 502 65
0
0

(du)

(dd)
Risk Neutral Prob
2u
u d
u d
2d

The price of the Asian arithmetic average price call:


e0.08 (47. 126 62 0.447 1652 + 18. 502 65 0.447 165 0.552 835
+0 0.447 165 0.552 835 + 0 0.552 8352 ) = 12. 921
If we were to calculate the price of the Asian arithmetic average asset put,
then
Path Average asset Put Payo Risk Neutral Prob
uu
147. 126 62
0
2u
ud
118. 502 65
0
u d
du
96. 257 8
3. 742 2
u d
dd
77. 530 5
22. 469 5
2d
The price of the Asian arithmetic average price put:
www.actuary88.com

c
Yufeng
Guo

150

CHAPTER 14. EXOTIC OPTIONS: I


e0.08 (0 0.447 1652 + 0 0.447 165 0.552 835
+3. 742 2 0.447 165 0.552 835 + 22. 469 5 0.552 8352 = 7. 193
The
four geometric averages are:
128.67659 165.57665 = 145. 965 2

(uu)

128.67659 108.32871 = 118. 065 1

(ud)

84.18680 108.32871 = 95. 497 9

(du)

84.18680 70.87417 = 77. 244 2


Path
uu
ud
du
dd

Average asset
145. 965 2
118. 065 1
95. 497 9
77. 244 2

(dd)

Call Payo
45. 965 2
18. 065 1
0
0

Risk Neutral Prob


2u
u d
u d
2d

The price of the Asian geometric average price call:


e0.08 (45. 965 2 0.447 1652 + 18. 065 1 0.447 165 0.552 835
+0 0.447 165 0.552 835 + 0 0.552 8352 ) = 12. 607
If we were asked to calculate the price
Path Average asset Call Payo
uu
145. 965 2
0
ud
118. 065 1
0
du
95. 497 9
4. 502 1
dd
77. 244 2
22. 755 8

of the Asian average price put, then:


Risk Neutral Prob
2u
u d
u d
2d

The price of the Asian geometric average price put:


e0.08 (0 0.447 1652 + 0 0.447 165 0.552 835
+4. 502 1 0.447 165 0.552 835 + 22. 755 8 0.552 8352 ) = 7. 447
Problem 14.4.
a. Asian arithmetic
Path ST
uu
165.57665
ud
108.32871
du
108.32871
dd
70.87417

average strike
K
147. 126 62
118. 502 65
96. 257 8
77. 530 485

call:
Call Payo
18. 450 03
0
12. 070 91
0

Risk Neutral Prob


2u
ud
ud
2d

The price of the Asian arithmetic average strike call:


e0.08 (18. 450 03 0.447 1652 + 0 0.447 165 0.552 835
+12. 070 91 0.447 165 0.552 835 + 0 0.552 8352 ) = 6. 160
www.actuary88.com

c
Yufeng
Guo

151

CHAPTER 14. EXOTIC OPTIONS: I


If we were to calculate the price of the Asian arithmetic average strike put,
then
Path ST
K
Put Payo Risk Neutral Prob
uu
165.57665 147. 126 62 0
2u
ud
108.32871 118. 502 65 10. 173 94
u d
du
108.32871 96. 257 8
0
u d
dd
70.87417
77. 530 485 76. 656 315 2d
The price of the Asian arithmetic average strike put:
e0.08 (0 0.447 1652 + 10. 173 94 0.447 165 0.552 835
+0 0.447 165 0.552 835 + 6. 656 315 0.552 8352 ) = 4. 20
b. The price of the Asian geometric average strike call:
Path ST
K
Call Payo Risk Neutral Prob
uu
165.57665 145. 965 2 19. 611 45
2u
ud
108.32871 118. 065 1 0
u d
du
108.32871 95. 497 9
12. 830 81
u d
dd
70.87417
77. 244 2
0
2d
The price of the Asian geometric average strike call:
e0.08 (19. 611 45 0.447 1652 + 0 0.447 165 0.552 835
+12. 830 81 0.447 165 0.552 835 + 0 0.552 8352 ) = 6. 548
If we were to calculate the price
then
Path ST
K
uu
165.57665 145. 965 2
ud
108.32871 118. 065 1
du
108.32871 95. 497 9
dd
70.87417
77. 244 2

of the Asian geometric average strike put,


Put Payo
0
9. 736 39
0
6. 370 03

Risk Neutral Prob


2u
u d
u d
2d

The price of the Asian geometric average strike put:


e0.08 (0 0.447 1652 + 9. 736 39 0.447 165 0.552 835
+0 0.447 165 0.552 835 + 6. 370 03 0.552 8352 ) = 4. 019
Problem 14.5.
Stock price tree:
182.14179
165.57665
128.67659
100

128.814749
108.32871

84.18680

91.10067
70.87417
64.42843

www.actuary88.com

c
Yufeng
Guo

152

CHAPTER 14. EXOTIC OPTIONS: I


Calculate the price of the arithmetic average asset call.
Path Arithmetic Average Payo
Risk Neutral Prob
uuu
151.1369
51.1369 3u = 0.0953
uud
133.3612
33.3612 2u d = 0.1133
udu
118.8058
18.8058 2u d = 0.1133
udd
106.2345
6.2345
u 2d = 0.1348
duu
106.8874
6.8874
2u d = 0.1133
dud
94.3160
0
u 2d = 0.1348
ddu
84.0221
0
u 2d = 0.1348
ddd
75.1314
0
3d = 0.1603
Total
1
The price of the arithmetic average asset call is:
e0.08 (51.1369 0.0953 + 33.3612 0.1133
+18.8058 0.1133 + 6.2345 0.1348 + 6.8874 0.1133) = 11. 45
Calculate the price of the geometric average asset call.
Path Arithmetic Average Payo
Risk Neutral Prob
uuu
149.1442
49.1442 3u = 0.0953
uud
132.8796
32.8796 2u d = 0.1133
udu
118.3887
18.3887 2u d = 0.1133
udd
105.4781
5.4781
u 2d = 0.1348
duu
105.4781
5.4781
2u d = 0.1133
dud
93.9754
0
u 2d = 0.1348
ddu
83.7272
0
u 2d = 0.1348
ddd
74.5965
0
3d = 0.1603
Total
1
The price of the geometric average asset call is:
e0.08 (49.1442 0.0953 + 32.8796 0.1133
+18.3887 0.1133 + 5.4781 0.1348 + 5.4781 0.1133) = 10. 94
Problem 14.6.
a. Using the Black-Scholes formula, we find that the call price is $4.1293.
b and c.
knock-in call + knock-out call = ordinary call
However, the knock-out call is worthless because the barrier 44 is lower than
the strike price 45. For the call to be worth something, the stock price must
exceed the strike price 45. However, as soon as the stock reaches the barrier 44,
the call is knocked out dead (i.e. the call doesnt exist any more). Hence this
knock-out call will never have a positive payo.
Hence the knock-in call becomes the ordinary call. The premium of the
knock-in call is also $4.1293.
Key point to remember:
www.actuary88.com

c
Yufeng
Guo

153

CHAPTER 14. EXOTIC OPTIONS: I


1. A knock-out call is worthless if the barrier is less than or equal to the
strike price.
2. A knock-in call is just an ordinary call if the barrier is less than or equal
to the strike price.

Problem 14.7.
DM Chapter 14 doesnt have any formula on the price of a barrier option. To
calculate the barrier option price, you have to use the textbooks spreadsheet.
So this problem is out of the scope of Exam MFE. I used the textbooks
Excel spreadsheet and calculated the following:
T
0.25
0.5
1
2
3
4
5
100

BS
0.9744
2.1304
4.1293
7.4398
10.2365
12.6969
14.9010
39.9861

DO
0.7323
1.2482
1.8217
2.4505
2.8529
3.1559
3.4003
5.3112

BS/DO
1.3306
1.7067
2.2667
3.0360
3.5881
4.0232
4.3823
7.5286

BS: the price of a standard option using the Black-Scholes formula.


DO: Down-and-out barrier option. Since the current stock price 40 exceeds
the barrier 38, the knock-out call with 38 barrier is really a down-and-out option
(if the stock price drops to 38, the call is dead).
The greater the expiration, the greater the ratio of BS/DO. As the expiration increases, both the standard option and the down-and-out option become
more valuable. However, as T increases, the value of a standard option goes up
much faster than a down-and-out option.
The value of a down-and-out option has a moderate increase. As T increase,
the stock price may be more volatile, increasing the value of the barrier option.
However, theres also more chance that the stock price may hit the barrier.

Problem 14.8.
Out of the scope of MFE. However, I used the textbook spreadsheet and
found the following:
www.actuary88.com

c
Yufeng
Guo

154

CHAPTER 14. EXOTIC OPTIONS: I


T
BS
UO
BS/U O
0.25 5.0833 3.8661 1.3149
0.5
5.3659 3.4062 1.5753
1
5.6696 2.8626 1.9806
2
5.7862 2.2233 2.6026
3
5.6347 1.8109 3.1115
4
5.3736 1.5094 3.5601
5
5.0654 1.2761 3.9695
100 0.0012 0.0001 24.4960
As T increases, the BS/U O increases too (just like in Problem 14.7). However, if T is too big, the value of a standard put and the value of an up-and-out
put both approach zero. If T is too big, we lose the time value of money (the
money we spent in buying a put could have been invested elsewhere and earned
lot of interest).

Problem 14.9.
I used the spreadsheet and calculated the following (T is expressed in months):
T
BS
UO
BS/UO
1
0.1727
0.1727
1.0003
2
0.5641
0.5479
1.0296
3
0.9744
0.8546
1.1401
4
1.3741
1.0384
1.3234
5
1.7593
1.1243
1.5649
6
2.1304
1.1468
1.8577
7
2.4886
1.1316
2.1991
8
2.8353
1.0954
2.5885
9
3.1718
1.0482
3.0260
10
3.4991
0.9962
3.5124
11
3.8180
0.9430
4.0488
12
4.1293
0.8906
4.6365
As T goes up, the BS/UO ratio goes up too.

Problem 14.10.
With K = 0.9, the standard put is worth 0.0188. With barrier 1 or 1.05, the
up-and-out barrier is also worth 0.0188. Why? DM page 452 and 453 have an
explanation. Here is the main point. If the exchange rate never hits 1, then the
standard put and the up-and-out put with 1 or 1.5 barrier have the same value.
The only way that the standard put is more valuable than the up-and-out put
is when the exchange rate goes up from 0.9 to 1 or to 1.05 and then falls below
0.9, in which case the up-and-out is dead yet the standard put has a positive
payo of K ST = 0.9 ST . However, such a scenario is rare and 0.9 ST is
www.actuary88.com

c
Yufeng
Guo

155

CHAPTER 14. EXOTIC OPTIONS: I


small. Hence the standard put and the up-and-out put have roughly the same
value.
In contrast, when the strike price K = 1, the payo of the standard put at
the above mentioned scenario is 1 ST , which is greater than 0.9 ST . Hence
the standard put is slightly more valuable than the up-and-out put.
Problem 14.11.
a.Using the Black-Scholes formula, we find the call premium is $9.6099.
b. If we buy the compound option, then we can, at t1 = 1, buy, for a
guaranteed price of x = 2, a call that expires at T = 2. Well exercise the
compound option at t1 = 1 only if the stock price at t = 1 is such that the call
we are entitled to buy is equal to or greater than the premium 2.
Well use the textbook Equation 14.11 to find S , the minimum price such
that exercising the compound option is worthwhile.
C (S , K, T t1 ) = x
C (S , 40, 2 1) = 2
C (S , 40, 1) = 2
To solve for S , we have use the trial-and-error approach. I found that

S = 31.723.
In other words, if the stock price at t1 = 1 is 31.723, then a call option
written at t1 = 1 and expires at T = 2 is exactly worth $2.In this case, well not
exercise the compound option. Well let the compound option expire worthless.
If the stock price at t1 = 1 is less than 31.723, then the call written at t1 = 1
and expires at T = 2 is worth less than $2 and well not exercise the compound
option. Well let the compound option expire worthless.
Only if the stock price at t1 = 1 is greater than 31.723 should we exercise
the compound option.
c. To find the price of the compound call (i.e. call on call), well use worksheet
called "Compound" provided by the DM textbook.
The inputs are:
Stock Price
40
Exercise Price to buy asset
40
Exercise Price to buy option
2
Volatility
30%
Risk-free interest rate
8%
Expiration for Option on Option (years)
1
Expiration for Underlying Option (years)
2
Dividend Yield
0%

Compound Option Prices


Call on Call
7.9482
Put on Call
0.1845
Call on Put
2.2978
www.actuary88.com

c
Yufeng
Guo

156

CHAPTER 14. EXOTIC OPTIONS: I


Put on Put
0.4484
Critical S for compound call
31.723
Critical S for compound put
44.3494
So the compound call premium is 7.9482. (From the output, we see that the
critical stock price for the compound call is 31.723, which matches our answer
in part b).
d. From Part c, we see that the price of the put on call is 0.1845.
Alternatively, we can use DM 14.12 to calculate the put on the call, given
we know the price of the call on call.
CallOnCall P utOnCall + xert1 = BSCall
7.9482 P utOnCall + 2e0.081 = 9.6099
P utOnCall = 0.184 5
Problem 14.12.
a. Use the Black-Scholes formula, we find the put price is 3.6956.
b. Use the "Compound" worksheet.
Inputs
Stock Price
40
Exercise Price to buy asset
40
Exercise Price to buy option
2
Volatility
30%
Risk-free interest rate
8%
Expiration for Option on Option (years)
1
Expiration for Underlying Option (years)
2
Dividend Yield
0%
Outputs:
Call on Call
7.9482
Put on Call
0.1845
Call on Put
2.2978
Put on Put
0.4484
Critical S for compound call
Critical S for compound put

31.723
44.3494

So if S1 < 44.3494, well exercise the put at t = 1.


c. The put on put price is 0.4484.
Problem 14.13.
a. DM 14.15 gives you the price formula for a gap call. A gap put formula
is
P (K1 , K2 ) = K1 erT N (d2 ) SeT N (d1 )
www.actuary88.com

c
Yufeng
Guo

157

CHAPTER 14. EXOTIC OPTIONS: I

1
S
+ r + 2 T
K2
2

d1 =

d2 = d1 T
For foreign currency, = r , r = r$ , and S = x0 .
ln

b. The gap put payo is 0.8 x if x < 1. If = 0, then x = x0 = 0.9. The


gap out payo is 0.1. The gap put premium is 0.8e0.060.5 0.9e0.030.5 =
0.110 2.
As the volatility increases, the value of the gap put increases.
Problem 14.14.
Skip.
Problem 14.15.
Skip.
Problem 14.16.
Price of a standard 40-strike call on S. The inputs to the Black Scholes
formulas are:
S = K = 40
= 0.3
r = 0.08
T =1
=0
The call price is 6.2845.
The price of an exchange option with S as underlying and 0.667Q as the
strike asset is 7. 577.
S =
40
K = 0.667 60 = 40
= 0.32 + 0.52 2 0.5 0.3 0.5 = 0.435 89
40e01
1
ln
+ 0.435 892 1
0.041
40e
2

d1 =
= 0.309 7
0.435 89 1

d2 = 0.309 7 0.435 89 1 = 0.126 2


N (d1 ) = 0.621 6
N (d2 ) = 0.449 8
C = 40e01 0.621 6 40e0.041 0.449 8 = 7. 577
Problem 14.17.
a. Inputs to the exchange call formula:
S = 0.3
S = 40
S = 0
K = 0.5
K = 60
K = 0
= 0.5
T =1
www.actuary88.com

c
Yufeng
Guo

158

CHAPTER 14. EXOTIC OPTIONS: I


You should get: C = 2. If you change S = 0.1, then the call price is
C = 1.2178
In the replicating portfolio, the number of underlying stocks to hold at time
zero is eS T . If we increase S , then we hold fewer shares of the underlying
asset and the call price drops.
b. inputs to the exchange call formula:
S = 40
S = 0
S = 0.3
K = 60
K = 0
K = 0.5
= 0.5
T =1
Once again, C = 2.
If K = 0.1, then C = 2.86.
If K goes up, then the present value of the strike price goes down, making
the call more valuable.
c. inputs to the exchange call formula:
S = 40
S = 0
S = 0.3
K = 60
K = 0
K = 0.5
= 0.5
T =1
Once again, C = 2.
If = 0.5, then C = 5.79.
p
If is negative, then = 2S + 2K 2 S K is higher then if is positive.
So a negative increases , making an option more valuable.
Problem 14.18.
= 1p

= 2S + 2K 2 S K = 0.32 + 0.32 2 1 0.3 0.3 = 0


a. The call price is zero. Because V ar [ln (S/K)] = 2S + 2K 2 S K = 0,
S/K is a constant during [0, T ]. Since at time zero S = K, then S = K during
[0, T ]. So the payo of the call is zero and the call is worthless.
p

b. Now = 2S + 2K 2S K = 0.42 + 0.32 2 1 0.4 0.3 =


0.1. Using DM 14.16, we find that the exchange call premium is 1.60.
Skip the remaining problems (Problem 14.19 through 14.22).

www.actuary88.com

c
Yufeng
Guo

159

CHAPTER 14. EXOTIC OPTIONS: I

www.actuary88.com

c
Yufeng
Guo

160

Chapter 18

Lognormal distribution
Problem 18.1.
x
Using DM 18.4 z =
, we find the equivalent draws from the a standard

normal distribution are:


11 (8)
3 (8)
7 (8)

= 0.258 2
= 0.774 6
= 1. 2910
15
15
15
2 (8)

= 2. 5820
15

15 (8)

= 1. 807 4
15

Problem 18.2.
x
, we get x = + z. The equivalent draws are:
Using DM 18.4 z =

0.8 + (1.7) 25 = 7. 7
0.8 + (0.55)
25 = 3. 55
0.8 + (0.3) 25 = 0.7
0.8 + (0.02) 25 = 0.7
0.8 + (0.85) 25 = 5. 05
Problem 18.3.
Linear combination of normal random variables is also normal.
x1 + x2 is normal. Its mean is E (x1 + x2 ) = E (x1 ) + E (x2 ) = 1 2 = 1.
Its variance is
V ar (x1 + x2 ) = V ar (x1 ) + V ar (x2 ) + 2Cov (x1 , x2 ) = 5 + 2 + 2 (1.3) = 9. 6
x1 x2 is normal. Its mean is E (x1 x2 ) = E (x1 ) E (x2 ) = 1 (2) = 3.
Its variance is
V ar (x1 x2 ) = V ar (x1 ) + V ar (x2 ) 2Cov (x1 , x2 ) = 5 + 2 2 (1.3) = 4. 4
Problem 18.4.
161

CHAPTER 18. LOGNORMAL DISTRIBUTION


x1 + x2 is normal with mean E (x1 + x2 ) = E (x1 ) + E (x2 ) = 2 + 8 = 10
and variance
V ar (x1 + x2 ) = V ar (x1 ) + V ar (x2 ) + 2Cov (x1 , x2 )
Cov (x1 , x2 ) = x1 x2

V ar (x1 + x2 ) = 0.5 + 14 + 2 (0.3) 0.5 14 = 12. 91
x1 x2 is normal with mean E (x1 x2 ) = 2 8 = 6 and variance
V ar (x1 x2 ) = V ar (x1 ) + V ar (x2
) 2Cov
(x1 ,x2 )
V ar (x1 x2 ) = 0.5 + 14 2 (0.3) 0.5
14 = 16. 09
Problem 18.5.
Use DM 18.10.
x1 + x2 + x3 is normal with mean
E (x1 + x2 + x3 ) = E (x1 ) + E (x2 ) + E (x3 ) = 1 + 2 + 2.5 = 5. 5
variance
V ar (x1 + x2 + x3 ) = V ar (x1 )+V ar (x2 )+V ar (x3 )+2Cov (x1 , x2 )+2Cov (x1 , x3 )+
2Cov (x2 , x3 )



= 5 + 3 + 7 + 2 (0.3) 5
3 + 2 (0.1) 5
7 + 2 (0.4) 3
7 =
22. 17
x1 + 3x2 + x3 is normal with mean
E (x1 + 3x2 + x3 ) = E (x1 ) + 3E (x2 ) + E (x3 ) = 1 + 3 (2) + 2.5 = 9. 5
variance
V ar (x1 + 3x2 + x3 ) = V ar (x1 )+32 V ar (x2 )+V ar (x3 )+23Cov (x1 , x2 )+
2Cov (x1 , x3 ) + 2 3Cov (x2
, x3 )


2
3 +2 (0.1) 5
7 +23 (0.4) 3
7 =
= 5+3 3+7+23 (0.3) 5
58. 15
x1 + x2 + 0.5x3 is normal with mean
E (x1 + x2 + 0.5x3 ) = E (x1 ) + E (x2 ) + 0.5E (x3 ) = 1 + 2 + 0.5 (2.5) = 4. 25
variance
V ar (x1 + x2 + 0.5x3 ) = V ar (x1 )+V ar (x2 )+0.52 V ar (x3 )+2Cov (x1 , x2 )+
2 0.5Cov (x1 , x3 ) + 2 0.5Cov (x
2 , x3 )

= 5 + 3 + 0.52 7 + 2 (0.3) 5
3 + 2 0.5 (0.1) 5
7 +2

0.5 (0.4) 3
7 = 14. 50
Problem 18.6.
www.actuary88.com

c
Yufeng
Guo

162

CHAPTER 18. LOGNORMAL DISTRIBUTION


Use DM 18.13.
E (ex ) = e2+0.5(5) = 90. 017 1
Let a represent the median of y = ex . Then P (y a) = 0.5.
y = ex is an increasing function. Hence of the median of ex corresponds to
the median of x.
Let b represent the median of x.
P (x b) = 0.5. Hence b = 2 (the median of a normal random variable is
its mean).
= a = eb = e2 = 7. 389 1

Problem 18.7.
Theres a typo in the problem. The table should say "Month" instead of
"Day" (i.e. should be Month 0, Month 1, ...,Day 4, not Day 0, Day 1, ...,Day
4).
This is similar to DM Table 11.1.
a. Stock A:

St
2
Month Stock price
rt =ln
rt r
St1
0
100
105
1
105
ln
= 0.04879 016 0.00238048
100
102
2
102
ln
= 0.02898 75 0.00084028
105
97
3
97 ln
= 0.05026 183 0.00252625
102
100
4
100
ln
= 0.03 045 921 0.00092776
97
Total
0.00000000 0.00667477
4
P

rt

0
mean monthly continuously compounded return: r =
= =0
4
4
v
uP

2
u 4
r
u
rt r
t t=1
0.00667477
=
= 4. 716 909 3
monthly standard deviation:
41
3

t=1

102

annual standard deviation: 12 4. 716 909 3 102 = 16.34%


b. Stock B
www.actuary88.com

c
Yufeng
Guo

163

CHAPTER 18. LOGNORMAL DISTRIBUTION

Month

Stock price

0
1
2
3
4
Total

100
105
102
97
100

rt =ln

St
St1

0.04879016
0.35667494
0.43592432
0.00000000
0

rt r

0.00238048
0.12721702
0.19003001
0.00092776
0.32055527
4
P

rt
0

mean monthly continuously compounded return: r = t=1 = = 0


4
4
v
uP

2
u 4
r
u
rt r
t t=1
0.32055527
monthly standard deviation:
=
= 0.326 881 87
41
3

annual standard deviation: 12 (0.326 881 87) = 1. 132 4 = 113.24%


c. The statement is correct.
For example, the mean monthly continuously compounded return for Stock
A is

4
P

rt

r = t=1
4
100
ln
100 = 0
4

4
P

t=1

4
P
St
S4
(ln St ln St1 )
ln
ln S4 ln S0
St1
S0
t=1
=
=
=
=
4
4
4
4

ln

So the mean monthly continuously compounded return for Stock A depends


only on S4 and S0 ; S1 , S2 , and S3 are irrelevant.
However, S1 , S2 , and S3 matters whenvwe are calculating the monthly stanuP

2
u 4
u
rt r
t t=1
dard deviation in the following formula:
.
41
Problem 18.8.

ln St is normally distributed with mean ln S0 + 0.5 2 t and variance


2 t.

ln St N ln S0 + 0.5 2 t, 2 t
Then for a given stock price c

P (St c) = P (ln St ln c) =
www.actuary88.com

!
ln c ln S0 0.5 2 t

c
Yufeng
Guo

164

CHAPTER 18. LOGNORMAL DISTRIBUTION


One subtle point. Since St is continuous, the probability is zero that St
takes on a single value. Hence P (St = c) = 0, P (St c) = P (St < c), and
P (St > c) = P (St > c).
In this problem, S0 = 100, = 0.08, = 0, = 0.3

ln St N ln 100 + 0.08 0 0.5 0.32 t, 0.32 t


ln St N 4. 605 17 + 0.035 t, 0.32 t
P (St c) =

ln c 4. 605 17 0.035 t

0.3 t

For t = 1, we have:

ln S1 N 4. 640 17, 0.32


P (S1 c) = P (ln S1 ln c) =
P (S1 105) =
0.518 3

ln c 4. 640 17
0.3

ln 105 4. 640 17
0.3

= (0.0460) = NormalDist (0.0460) =

P (S1 > 105) = 1 0.518 3 = 0.481 7


Next, we consider how P (St >c) changes if we change t or . !

ln c ln S0 0.5 2 t

We know that P (St c) =


t

ln c ln S0 0.5 2 t

, where z is a normal random variSet z =


t
able.

0.5 2
0.5 2 d
d ln c ln S0 0.5 2 t
d

t=
=
z=
dt
dt

dt
t
2 t
For this problem, = 0.08, = 0, = 0.3
d
0.005 833
0.08 0 0.5 0.32

=
z=
dt
2 0.3 t
t
So z is a decreasing function of t. Remember that the accumulative normal
distribution (z) is an increasing function of z, we conclude:
The higher the t, the lower the z, the lower the P (St c) = (z), and the
higher the P (St > c) = 1 (z).
Similarly,
www.actuary88.com

c
Yufeng
Guo

165

CHAPTER 18. LOGNORMAL DISTRIBUTION

d 0.5 2
d 0.5 2 t
d
d ln c ln S0 0.5 2 t

=
= t
z=
d
d
d
d

t
t

= t
0.5 = t 0.5 +
d

2
For this problem, = 0.08, = 0 so

0.08
d
> 0.
z = t 0.5 + 2
d

So z is an increasing function of .
The higher the , the higher the z, the higher the P (St c) = (z), and
the lower the P (St > c) = 1 (z).
Problem 18.9.
Use DM 18.30.

N d1
E (St |St > K) = Se()t
N d2

d1

ln
=

S0
+ + 0.5 2 t
K

d2

= d1 T

In this problem,

100
+ 0.08 0 + 0.5 0.32 1
105

= 0.254 0
=
0.3 1

N d1 = NormalDist (0.2540) = 0.600 3

d1

ln

d2
= 0.2540
0.3 1 = 0.046

N d2 = NormalDist (0.046 )

= 0.481 7

E (S1 |S1 > 105) = 100e(0.080)1

0.600 3
= 135
0.481 7

Next, we wan to analyze how E (St |St > 105) changes if we change t (while
keeping other parameters unchanged), (while keeping other parameters unchanged),
and (while keeping other parameters unchanged). We see that E (St |St > 105)
increases if we increase t, , or . We consider t = 0.25, 0.5, 0.75, 1, ..., 6.
www.actuary88.com

c
Yufeng
Guo

166

CHAPTER 18. LOGNORMAL DISTRIBUTION


t
0.25
0.5
0.75
1
1.25
1.5
1.75
2
2.25
2.5
2.75
3
3.25
3.5
3.75
4
4.25
4.5
4.75
5
5.25
5.5
5.75
6

E (S1 |S1 > 105)


117.19
123.98
129.74
135.00
139.97
144.76
149.42
154.01
158.54
163.05
167.54
172.04
176.54
181.06
185.61
190.19
194.81
199.48
204.19
208.95
213.76
218.64
223.58
228.58

0.1
0.2
0.3
0.4
0.5
0.6
0.7
0.8
0.9
1
1.1
1.2
1.3
1.4
1.5
1.6
1.7
1.8
1.9
2
2.1
2.2
2.3
2.4

E (S1 |S1 > 105)


115.15
124.65
135.00
146.25
158.48
171.78
186.26
202.04
219.26
238.06
258.61
281.11
305.77
332.82
362.55
395.25
431.27
471.02
514.93
563.51
617.34
677.07
743.46
817.37

0.01
0.02
0.03
0.04
0.05
0.06
0.07
0.08
0.09
0.1
0.11
0.12
0.13
0.14
0.15
0.16
0.17
0.18
0.19
0.2
0.21
0.22
0.23
0.24

E (S1 |S1 > 105)


131.63
132.08
132.54
133.01
133.49
133.98
134.49
135.00
135.53
136.07
136.63
137.19
137.78
138.37
138.98
139.60
140.24
140.90
141.57
142.25
142.95
143.67
144.41
145.16

Problem 18.10.
Use DM 18.23. P (St < K) = N (d2 )

d1

0.484 0

ln
=

S0
100
ln
+ + 0.5 2 t
+ 0.08 0 + 0.5 0.32 1
K
98

=
=
t
0.3 1

d2
= d1
t = 0.484 0 0.3 1

N d2 = NormalDist (0.184

= 0.184
) = 0.427 0

P (St < 98) = 0.427 0


Next, we analyze how P (St < 98) changes if we change t. We find that
P (St < 98) increases with t initially and then decreases with t.
www.actuary88.com

c
Yufeng
Guo

167

CHAPTER 18. LOGNORMAL DISTRIBUTION


t
0.25
0.5
0.75
1
1.25
1.5
1.75
2
2.25
2.5
2.75
3
3.25
3.5
3.75
4
4.25
4.5
4.75
5
5.25
5.5
5.75
6

d2
0.1930
0.1777
0.1788
0.1840
0.1907
0.1979
0.2052
0.2126
0.2199
0.2271
0.2341
0.2410
0.2477
0.2543
0.2607
0.2670
0.2732
0.2792
0.2852
0.2910
0.2967
0.3023
0.3078
0.3133

P (St < 98) = N (d2 )


0.4235
0.4295
0.4290
0.4270
0.4244
0.4216
0.4187
0.4158
0.4130
0.4102
0.4075
0.4048
0.4022
0.3996
0.3972
0.3947
0.3924
0.3900
0.3878
0.3855
0.3833
0.3812
0.3791
0.3770

Problem 18.11.
Use DM 18.28.

N d1
E (St |St < K) = Se()t
N d2
If K = 98:

S0
100
ln
ln
+ + 0.5 2 t
+ 0.08 0 + 0.5 0.32 1

K
98

d1 =
=
=
t
0.3 1
0.484 0

= d1 t = 0.484 0 0.3 1 = 0.184

N d1 = NormalDist (0.484 0 ) = 0.3142

N d2 = NormalDist (0.184 ) = 0.427 0

d2

www.actuary88.com

c
Yufeng
Guo

168

CHAPTER 18. LOGNORMAL DISTRIBUTION


E (S1 |S1 < 98) = 100e(0.080)1

0.3142
= 79. 71
0.427 0

If K = 120:

100
ln
+ 0.08 0 + 0.5 0.32 1

120

d1 =
= 0.191 1
0.3 1

d2 = d1 t = 0.191 1 0.3 1 = 0.491 1


N
N

d 1

d 2

= NormalDist (0.191 1 ) = 0.575 8


= NormalDist (0.491 1 ) = 0.688 3

E (S1 |S1 < 120) = 100e(0.080)1

0.575 8
= 90. 62
0.688 3

If K = 98. We find that E (S1 |S1 < K) decreases with t.


t
d1
d2 E (S1 |S1 < K)
0.25 0.4235 0.1930
88.12
0.5 0.4295 0.1777
84.41
0.75 0.4290 0.1788
81.79
1 0.4270 0.1840
79.71
1.25 0.4244 0.1907
77.97
1.5 0.4216 0.1979
76.47
1.75 0.4187 0.2052
75.15
2 0.4158 0.2126
73.97
2.25 0.4130 0.2199
72.90
2.5 0.4102 0.2271
71.92
2.75 0.4075 0.2341
71.01
3 0.4048 0.2410
70.18
3.25 0.4022 0.2477
69.39
3.5 0.3996 0.2543
68.66
3.75 0.3972 0.2607
67.97
4 0.3947 0.2670
67.32
4.25 0.3924 0.2732
66.71
4.5 0.3900 0.2792
66.12
4.75 0.3878 0.2852
65.57
5 0.3855 0.2910
65.04
5.25 0.3833 0.2967
64.53
5.5 0.3812 0.3023
64.04
5.75 0.3791 0.3078
63.58
6 0.3770 0.3133
63.13
If K = 120. We find that E (S1 |S1 < K) decreases with t.
www.actuary88.com

c
Yufeng
Guo

169

CHAPTER 18. LOGNORMAL DISTRIBUTION


t
0.25
0.5
0.75
1
1.25
1.5
1.75
2
2.25
2.5
2.75
3
3.25
3.5
3.75
4
4.25
4.5
4.75
5
5.25
5.5
5.75
6

d1
0.8764
0.7814
0.7260
0.6883
0.6602
0.6381
0.6198
0.6044
0.5910
0.5792
0.5687
0.5591
0.5504
0.5424
0.5350
0.5281
0.5216
0.5156
0.5098
0.5044
0.4992
0.4942
0.4895
0.4850

d2
1.1571
0.7770
0.6007
0.4911
0.4131
0.3533
0.3051
0.2647
0.2302
0.1999
0.1730
0.1488
0.1268
0.1066
0.0879
0.0705
0.0543
0.0390
0.0246
0.0109
0.0021
0.0145
0.0263
0.0377

E (S1 |S1 < K)


98.14
95.09
92.64
90.62
88.88
87.36
86.00
84.78
83.66
82.63
81.68
80.80
79.97
79.20
78.46
77.77
77.11
76.49
75.89
75.32
74.78
74.26
73.76
73.28

The following table lists the value of E (St |St < K)


www.actuary88.com

c
Yufeng
Guo

170

CHAPTER 18. LOGNORMAL DISTRIBUTION

t
0.25
0.5
0.75
1
1.25
1.5
1.75
2
2.25
2.5
2.75
3
3.25
3.5
3.75
4
4.25
4.5
4.75
5
5.25
5.5
5.75
6

K = 98
= 0.3
88.12
84.41
81.79
79.71
77.97
76.47
75.15
73.97
72.90
71.92
71.01
70.18
69.39
68.66
67.97
67.32
66.71
66.12
65.57
65.04
64.53
64.04
63.58
63.13

K = 120
= 0.3
98.14
95.09
92.64
90.62
88.88
87.36
86.00
84.78
83.66
82.63
81.68
80.80
79.97
79.20
78.46
77.77
77.11
76.49
75.89
75.32
74.78
74.26
73.76
73.28

K = 98
= 0.1
95.26
94.22
93.52
92.99
92.56
92.21
91.90
91.64
91.40
91.19
91.00
90.83
90.68
90.53
90.40
90.28
90.16
90.06
89.96
89.86
89.78
89.69
89.61
89.54

K = 120
= 0.1
102.01
103.69
104.72
105.35
105.74
105.99
106.16
106.28
106.37
106.43
106.48
106.51
106.53
106.55
106.56
106.57
106.57
106.57
106.57
106.57
106.57
106.57
106.56
106.56

The diagram:

www.actuary88.com

c
Yufeng
Guo

171

CHAPTER 18. LOGNORMAL DISTRIBUTION

Problem 18.12.
This problem states that KT = S0 erT , but it doesnt specify r .To solve the
problem, we set r = .

As T increases, P (ST < KT ) increases, P (ST > KT ) decreases, and the


call/put price increases.
www.actuary88.com

c
Yufeng
Guo

172

CHAPTER 18. LOGNORMAL DISTRIBUTION


t
0.25
0.5
0.75
1
1.25
1.5
1.75
2
2.25
2.5
2.75
3
3.25
3.5
3.75
4
4.25
4.5
4.75
5
5.25
5.5
5.75
6

d2
0.0750
0.1061
0.1299
0.1500
0.1677
0.1837
0.1984
0.2121
0.2250
0.2372
0.2487
0.2598
0.2704
0.2806
0.2905
0.3000
0.3092
0.3182
0.3269
0.3354
0.3437
0.3518
0.3597
0.3674

P (ST < KT )
0.5299
0.5422
0.5517
0.5596
0.5666
0.5729
0.5786
0.5840
0.5890
0.5937
0.5982
0.6025
0.6066
0.6105
0.6143
0.6179
0.6214
0.6248
0.6281
0.6313
0.6345
0.6375
0.6405
0.6433

P (ST > KT )
0.4701
0.4578
0.4483
0.4404
0.4334
0.4271
0.4214
0.4160
0.4110
0.4063
0.4018
0.3975
0.3934
0.3895
0.3857
0.3821
0.3786
0.3752
0.3719
0.3687
0.3655
0.3625
0.3595
0.3567

Call price
$5.98
$8.45
$10.34
$11.92
$13.32
$14.58
$15.73
$16.80
$17.80
$18.75
$19.64
$20.50
$21.32
$22.10
$22.85
$23.58
$24.29
$24.97
$25.63
$26.27
$26.89
$27.50
$28.09
$28.67

Put price
$5.98
$8.45
$10.34
$11.92
$13.32
$14.58
$15.73
$16.80
$17.80
$18.75
$19.64
$20.50
$21.32
$22.10
$22.85
$23.58
$24.29
$24.97
$25.63
$26.27
$26.89
$27.50
$28.09
$28.67

Why do the European call and the European put have the same price? Use
the call-put parity:
KerT
C = S0 eT N (d1 ) KerT N (d2 )
Since K = S0 erT and = 0, we have:
C = S0 N (d1 ) S0 N (d2 )

1 2
1 2
S0
+
r

+
T
T
rT
+
r
+

ln

S0 erT
2
2

d1 =
=
= 0.5 T
T
T

d2 = d2 T = 0.5 T





=
=
C = S0 N 0.5 T S0 N 0.5 T = S0 N 0.5 T S0 1 N 0.5 T
h

i

S0 2N 0.5 T 1



P = S0 eT N (d1 )+KerT N (d2 ) = S0 N 0.5 T +S0 N 0.5 T =




S0 1 N 0.5 T
+ S0 N 0.5 T = S0 2N 0.5 T 1

www.actuary88.com

c
Yufeng
Guo

173

CHAPTER 18. LOGNORMAL DISTRIBUTION


For example, for T = 0.25

C = P = 100 (2N (0.075) 1) = 100 (2 0.529 9 1) = 5. 98


By the way, we can use the put-call parity to see why C = P .
C + KerT = P + S0 eT
Since K = S0 erT and = 0, we have: C + S0 = P + S0 and C = P .

Typically, the strike price K is a fixed amount (as opposed to the increasing
amount K = S0 erT ); the call price and put price move in opposite directions.
In this problem, K = S0 erT increases with T ; the call and the put options both
become more valuable as T increases. This is a pure mathematical coincidence.
Intuitively, how to reconcile the fact that as T increases, P (ST < KT ) increases, P (ST > KT ) decreases, and the call and the put prices both increase?
Please note that the call/put price not only depends on the probability of the
option being in the money, not also depends on the payo. For example, even
though P (ST > KT ) decreases with T , if ST KT increases at a faster speed,
the call price will go up.

Problem 18.13.

Parameters:
S = 100

K = 90

= 0.3

= 4.5%

=0

As T increases, P (St < K) increases and then decreases. So the impact of


T on P (St < K) is ambiguous. However, E(St |St < K) decreases over time.
www.actuary88.com

c
Yufeng
Guo

174

CHAPTER 18. LOGNORMAL DISTRIBUTION


T
0.5
1
1.5
2
2.5
3
3.5
4
4.5
5
5.5
6
6.5
7
7.5
8
8.5
9
9.5
10
10.5
11
11.5
12

d2
0.5438
0.4179
0.3684
0.3426
0.3275
0.3182
0.3124
0.3089
0.3070
0.3061
0.3061
0.3067
0.3077
0.3091
0.3108
0.3127
0.3148
0.3171
0.3194
0.3219
0.3244
0.3270
0.3296
0.3323

www.actuary88.com

P (St < K) = N (d2)


0.2933
0.3380
0.3563
0.3659
0.3716
0.3752
0.3774
0.3787
0.3794
0.3798
0.3798
0.3795
0.3791
0.3786
0.3780
0.3772
0.3764
0.3756
0.3747
0.3738
0.3728
0.3718
0.3708
0.3698

E(St |St < K)


$79.20
$74.64
$71.45
$68.96
$66.89
$65.13
$63.59
$62.23
$61.00
$59.88
$58.86
$57.91
$57.04
$56.22
$55.46
$54.74
$54.06
$53.42
$52.82
$52.24
$51.70
$51.17
$50.68
$50.20

c
Yufeng
Guo

175

CHAPTER 18. LOGNORMAL DISTRIBUTION

www.actuary88.com

c
Yufeng
Guo

176

Chapter 19

Monte Carlo simulation


Please note that due to rounding, you may not be able to fully reproduce my
result. For example, when you see my z = 0.1234, the actual z used in my
calculation could be 0.123436. If you plug in z = 0.1234, you may not be able
to reproduce my result.

Problem 19.1.
Here is the snapshot of the simulation done in Excel:

1
2
3
4
5
6
...
1000
1001
1002

A
i
1
2
3
4
5
...
999
1000
Total

B
u
0.104689
0.491579
0.085629
0.878402
0.199163
...
0.132422
0.869963
498.294032

C
u2
0.010960
0.241650
0.007332
0.771590
0.039666
...
0.017536
0.756835
334.277115

Sample formulas
Cell B2 = rand()
B3 = rand()
C2 = B22
C3 = B32
B1002 = sum(B2 : B1001)
C1002 = sum(C2 : C1001)
E (u) =

498.294032
ui
=
= 0.498 294
n
1000
177

CHAPTER 19. MONTE CARLO SIMULATION


n
V ar (u) =
n1
0.08 606 6

1P 2
ui
n

ui
n

2 !

1000
=
999

334.277115
0.498 2942
1000

The correct mean of u (0, 1) is: E (u) = 0.5


1
The correction variance: V ar (u) =
= 0.08333 3
12
Next, we graph the histogram.
D
E
F
1
bin
range frequency
2
0.1
0 < u < 0.1
104
3
0.2 0.1 u < 0.2
106
4
0.3 0.2 u < 0.3
94
5
0.4 0.3 u < 0.4
100
6
0.5 0.4 u < 0.5
90
7
0.6 0.5 u < 0.6
108
8
0.7 0.6 u < 0.7
91
9
0.8 0.7 u < 0.8
93
10
0.9 0.8 u < 0.9
114
11
1
0.9 u < 1
100
12 Total
1000
Sample formulas for Column F.
F 2 = countif (B2 : B1001, < &D2)
F 3 = countif (B2 : B1001, < &D3) F 2
F 4 = countif (B2 : B1001, < &D4) (F 2 + F 3)
F 5 = countif (B2 : B1001, < &D5) (F 2 + F 3 + F 4)
...
F 11 = countif (B2 : B1001, < &D11) (F 2 + F 3 + F 4 + ... + F 10)
Problem 19.2.
1
2
3
4
5
6
7
...
1000
Total

u1
0.6922
0.0553
0.6137
0.2610
0.7009
0.1991
0.8552
...
0.4822

www.actuary88.com

u2
0.1621
0.1343
0.5859
0.5720
0.5851
0.8738
0.3089
...
0.9639

u3
0.0013
0.5132
0.1285
0.5668
0.4098
0.1330
0.2594
...
0.9245

...
...
...
...
...
...
...
...
...
...

u12
0.1888
0.4550
0.6732
0.3890
0.1757
0.0924
0.6550
...
0.7599

c
Yufeng
Guo

x=

ui 6
0.8461
0.2583
0.5420
1.1239
0.6006
0.2029
0.4612
...
0.9899
45.381885

x2
0.715842
0.066742
0.293817
1.263203
0.360681
0.041173
0.212701
...
0.979991
949.311003
178

CHAPTER 19. MONTE CARLO SIMULATION

Figure 19.1:

www.actuary88.com

c
Yufeng
Guo

179

CHAPTER 19. MONTE CARLO SIMULATION

Figure 19.2:
45.381885
= 0.045 382 (close to zero)
1000

1
1000
2
949.311003 0.045 382 = 0.948 200 (close to 1)
V ar (x) =
999 1000
The histogram looks like a bell curve.
E (x) =

Problem 19.3.
Snapshot of Excel for the simulation:
A
B
C
1
i
x1
x2
2
1 0.88028 0.535024
3
2 0.71431 0.209528
4
3 0.114864
0.1811
5
4
0.8346 0.156575
6
5 0.646959 1.65712
7
6 0.628469
0.48616
...
...
...
2001 2000 0.779131 0.509077
2002 sum

D
ex1
0.414668
0.489528
1.121721
0.434047
1.909724
1.874738
...
2.179578
3, 210.5069

E
ex2
1.70749
1.233096
0.834356
1.169498
0.190687
1.626059
...
1.663754
17, 452.8493

G
(ex1 )2
0.171949
0.239637
1.258259
0.188397
3.647045
3.514643
...
4.750559
13, 501.1984

Sample formulas.
www.actuary88.com

c
Yufeng
Guo

180

H
(ex2 )2
2.915522
1.520526
0.696150
1.367726
0.036361
2.644069
...
2.768078
1, 753, 120.4685

CHAPTER 19. MONTE CARLO SIMULATION


Cell B2 = N ormInv (Rand(), 0, 1)
D2 = exp (B2)
E2 = exp (C2)

C2 = N ormInv (Rand(), 0.7, 0.30.5)

3210.5069
= 1. 605 3
2000
The correct mean (using DM 18.13): E (ex1 ) = e0.5 = 1. 648 7

E (ex1 ) =

The estimated variance:

2000
1
x1
2
V ar (e ) =
= 4. 1757
13501.1984 1. 605 3
1999 2000
The correct variance (DM18.14) : V ar (ex1 ) = e (e 1) = 4. 670 8
17452.8493
= 8. 726 4
2000
The correct mean: E (ex2 ) = e0.7+0.53 = 9. 025 0

E (ex2 ) =

The estimated variance:

2000
1
V ar (ex2 ) =
1753120.4685 9. 025 0 2 = 795. 507 4
1999 2000
The correct variance is:

V ar (ex2 ) = e20.7+3 e3 1 = 1554. 533 6

Our estimate of E (ex2 ) and V ar (ex2 ) are way o. To improve our estimate,
we need to increase the number of simulations.

Problem 19.4.
I performed 5,000
i
z
1 0.2273
2 0.0408
3
1.7371
4 0.8544
5 0.5836
...
...
5000
0.4792
sum

simulations.
ST
put payo
38.9990
1.0010
40.1055
0.0000
52.3623
0.0000
35.4978
4.5022
36.9693
3.0307
...
...
43.3588
0.0000
10, 197.3969

P
0.9812
0.0000
0.0000
4.4131
2.9707
...
0.0000
9, 995.4749

P2
0.9627
0.0000
0.0000
19.4754
8.8248
...
0.0000
61, 151.4337

The z column in the above table is generated using Excels formula N ormInv (Rand () , 0, 1)
Sample calculations for Row 1.
Excels formula N ormInv(Rand () , 0, 1) happens to return z = 0.2273.
2
2
S = S e(r0.5 )T + T z = 40e(0.0800.50.3 )0.25+0.3 0.25(0.2273) =
T

38. 9990
www.actuary88.com

c
Yufeng
Guo

181

CHAPTER 19. MONTE CARLO SIMULATION

Figure 19.3:

www.actuary88.com

c
Yufeng
Guo

182

CHAPTER 19. MONTE CARLO SIMULATION

Figure 19.4:
The put payo: 40 38. 9990 = 1. 001
The put price: P = e0.080.25 1. 001 = 0.9812
P 2 = 0.98122 = 0.962 7
The estimate put price is:

9995.4749
= 1. 999 1
5000

You can verify that the put price based on the Black-Scholes formula is
1.9927.
The estimated
variance of the put price
per simulation is:

1
5000
61151.4337 1. 999 12 = 8. 235 5
4999 5000
Suppose we want to perform n simulations and take the average put price
of these n simulations as an estimate of the put price.
P1 + P2 + ... + Pm
Then P =
n
here Pi is the put price calculated from the i-th simulation
=
0.012 =

nV ar (P per simulation)
V ar (P per simulation)
=
V ar P =
n2
n

8. 235 5
n

n=

8. 235 5
= 82355
0.012

So we need to perform roughly 82, 400 simulations.


www.actuary88.com

c
Yufeng
Guo

183

CHAPTER 19. MONTE CARLO SIMULATION


Problem 19.5.
i
1
2
3
4
5
6
7
8
...
5000
sum

z
0.03768
0.531987
2.062351
2.46722
2.579892
1.23246
0.477051
1.37135
...
1.421245

S1
40.9592
48.5928
76.9061
19.7610
89.8235
28.6210
47.7985
27.4530
...
63.4500

1/S1
0.0244
0.0206
0.0130
0.0506
0.0111
0.0349
0.0209
0.0364
...
0.0158

F = erT (1/S1 )
0.0225
0.0190
0.0120
0.0467
0.0103
0.0323
0.0193
0.0336
...
0.0145
116.9386

F2
0.000508
0.000361
0.000144
0.002182
0.000106
0.001040
0.000373
0.001131
...
0.000212
2.9935

For example, if z = 0.03768, then

2
2
S1 = S0 e(r0.5 )T + T z = 40e(0.0800.50.3 )1+0.3 1(0.03768) = 40.
959 2
The forward price is:
1
F = erT (1/S1 ) = e0.081
= 0.0225
40.9592
116.9386
= 0.023 39
5000
The estimated variance of the forward price per simulation is:

5000
1
2.9935 0.023 392 = 0.00005162
4999 5000

The estimated forward price is:

We can calculate the true forward price using DM 20.30:

2
P
F0,T
[S a (T )] = erT S a (0) e[a(r)+0.5a(a1) ]T
Set a = 1. The true forward price is:

2
P
S (T ) = e0.08 401 e(1(0.080)+0.5(1)(11)0.3 )1 = 0.02331
F0,T

Problem 19.6.
a.
www.actuary88.com

c
Yufeng
Guo

184

CHAPTER 19. MONTE CARLO SIMULATION


i
1
2
3
4
5
6
7
8
...
5000
sum

z
0.5992
1.3882
1.2717
0.3870
0.7793
0.4702
1.1824
0.4278
...
0.1042

S1
49.5825
27.3147
28.2864
46.5239
32.7893
47.7004
59.0623
47.0974
...
40.1502

S12
2, 458.4195
746.0939
800.1177
2, 164.4688
1, 075.1384
2, 275.3243
3, 488.3595
2, 218.1649
...
1, 612.0360


F = erT S12
2, 269.4072
688.7315
738.6017
1, 998.0565
992.4778
2, 100.3891
3, 220.1616
2, 047.6243
...
1, 488.0968
9, 517, 170.0248

F2
5, 150, 209.1192
474, 351.0158
545, 532.5311
3, 992, 229.8443
985, 012.2592
4, 411, 634.2656
10, 369, 440.9962
4, 192, 765.3642
...
2, 214, 432.0185
26, 459, 271, 681.8572

The estimate forward price at time zero for a claim paying S12 at T = 1 is:
9517170.0248
= 1903. 43
5000
You can verify that the true forward price (using DM 20.30) is 1, 896.49
The estimated
variance of the forward price per
simulation is:

1
5000
26, 459, 271, 681.8572 1903. 432 = 166, 9142. 40
4999 5000
b.
i
1
2
3
4
5
6
7
...
5000
sum

z
0.112
0.8019
0.0557
0.9828
2.0755
1.0444
1.0135
...
1.2727

S1
42.8403
32.5673
42.1228
30.8470
77.2100
30.2822
56.1445
...
28.2776

S10.5
6.5453
5.7068
6.4902
5.5540
8.7869
5.5029
7.4930
...
5.3177

F = erT S10.5
6.0420
5.2680
5.9912
5.1270
8.1114
5.0798
6.9169
...
4.9088
30, 059.267012

F2
36.506096
27.752022
35.894682
26.286079
65.794022
25.804789
47.843187
...
24.096581
184, 880.552079

The estimate forward price at time zero for a claim paying S10.5 at T = 1 is:
30059.267012
= 6. 01185
5000
You can verify that true forward price (using DM 20.30) is 6.0086
The estimated variance of the forward price per simulation is:

1
5000
184880.552079 6.011852 = 0.833 9
4999 5000
www.actuary88.com

c
Yufeng
Guo

185

CHAPTER 19. MONTE CARLO SIMULATION


c.
i
1
2
3
4
5
6
7

z
0.0111
0.1893
0.6352
0.4972
1.2449
0.1526
0.5234

S1
41.5630
39.1378
34.2374
48.0883
28.5144
43.3653
35.4052

S12
0.0006
0.0007
0.0009
0.0004
0.0012
0.0005
0.0008

...
8
sum

...
0.2497

...
44.6471

...
0.0005

The estimated forward price is:

F = erT S12
0.0005
0.0006
0.0008
0.0004
0.0011
0.0005
0.0007

F2
0.00000029
0.00000036
0.00000062
0.00000016
0.00000129
0.00000024
0.00000054

...
0.0005
3.216862

...
0.00000021
0.002918

3.216862
= 0.000 643
5000

You can verify that the true forward price is 0.000644


The estimated
variance of the forward
price per simulation is:

1
5000
2
0.002918 0.000644 = 1. 69 107
4999 5000
Skip the remaining problems.

www.actuary88.com

c
Yufeng
Guo

186

Chapter 20

Brownian motion and Itos


lemma
Problem 20.1.
I dont want you to memorize Itos lemma. For problems related to Its
lemma, all you need to know is two things:
2

1. Unlike a deterministic random variable X where (dX) = 0, for a stochastic random variable Z, the term (dZ)2 is not zero and hence cant be
ignored. In fact, the textbook and my study guide have explained that
2
3
(dZ) = dt. However, you can ignore higher order such as (dZ) .
2. Itos lemma is just the stochastic counterpart of the Taylor series. To
derive Itos lemma, first write the Taylor series. For a stochastic random
variable X and a function y = f (t, X), first write the Taylor expansion:
f
1 2f
f
1 2f
dy = d f (t, X) =
(dt)2 +
(dX)2 +....Here we
dt+
dX +
2
t
X
2 t
2 X 2
3
3
ignored the higher order terms (dt) , (dX) , and above. Next, throw away
the term (dt)2 since t is a deterministic random variable and (dt)2 0.
f
f
1 2f
Now we have dy = d f (t, X) =
(dX)2 . This is
dt +
dX +
t
X
2 X 2
Itos lemma.
With this point in mind, lets solve the problem.
a. If the stock price S follows the textbook Equation 20.8, then:
dS (t) = dt + dZ (t)

(DM 20.8)

dS (t) is a linear function of dZ (t). Since [dZ (t)]2 = dt, we should keep
2
(dS) in the Taylor expansion but throw away all other higher order terms:
187

CHAPTER 20. BROWNIAN MOTION AND ITOS LEMMA


ln S
ln S
1 2 ln S
(dS)2
dt +
dS +
t
S
2 S 2
ln S
ln S
1
Since ln S doesnt contain t,
= 0. In addition,
=
and
t
S
S
2
1
1
ln S
=
= 2
S 2
S S
S
ln S
1
1 2 ln S
1 1
2
2
d ln S =
(dS) = dS
(dS)
dS +
2
S
2 S
S
2 S2
d ln S =

(dS)2 = (dt + dZ)2


= 2 (dt)2 + 2 (dZ) (dt) + 2 (dZ)2
Next, use the multiplication rule:
dZ dt = 0

(DM 20.17a)

(dt) = 0

(DM 20.17b)

(DM 20.17c)

(dZ) = dt
2

(dS) = (dt + dZ) = 2 (dZ) = 2 dt


1
1 1 2
d ln S = (dt + dZ)
dt
S
2
S2

1
1 1 2
dt + dZ
=

S
2 S2
S
b.If the stock price S follows the textbook Equation 20.9, then:
dS (t) = ( S) dt + dZ
d ln S =

(DM 20.9)

1
1 2 ln S
1 1
ln S
2
2
(dS) = dS
(dS)
dS +
S
2 S 2
S
2 S2

(dS)2 = [ ( S) dt + dZ]2
= 2 ( S)2 (dt)2 + 2 ( S) dt dZ + 2 (dZ)2 = 2 dt
1
1 1
2
d ln S = dS
(dS)
S
2 S2
( S) dt + dZ
1 2 dt
=

2 S2
( S) 1 2

=
dt + dZ

S
2 S2
S
c.If the stock price S follows the textbook Equation 20.27, then:

(DM 20.8)
dS (t) = (S, t) (S, t) dt + (S, t) dZ (t)
When S (t) follows a geometric Brownian motion,
www.actuary88.com

c
Yufeng
Guo

188

CHAPTER 20. BROWNIAN MOTION AND ITOS LEMMA

(S, t) = S (t)
(S, t) = S (t)
Hence DM Equation 20.8 becomes:

(S, t) = S (t)

dS (t) = S ( ) dt + SdZ (t)


d ln S =

(20.1)

ln S
1
1 2 ln S
1 1
2
2
(dS) = dS
(dS)
dS +
S
2 S 2
S
2 S2

(dS)2 = [S ( ) dt + SdZ (t)]2 = 2 S 2 (dZ)2 = 2 S 2 dt


1
1 1
d ln S = dS
(dS)2
S
2 S2
S ( ) dt + SdZ (t) 1 2 S 2 dt
=

S
2 S2

1 2
= dt + dZ
2
Problem 20.2.
S 2
S 2
1 2S2
2
(dS)
dt +
dS +
t
S
2 S 2
S 2
1 2S2
=
(dS)2 = 2SdS + (dS)2
dS +
S
2 S 2
a.
Under DM Equation 20.8:
dS (t) = dt + dZ (t)
(dS)2 = 2 dt
dS 2 = 2SdS + (dS)2

= 2S (dt + dZ) + 2 dt = 2S + 2 dt + 2SdZ


dS 2 =

b.
Under DM Equation 20.9:
dS (t) = ( S) dt + dZ
(dS)2 = 2 dt
2
dS 2 = 2SdS + (dS)
= 2S [ ( S) dt + dZ] + 2 dt
= 2S ( S) + 2 dt + 2SdZ

c.
Under Geometric Brownian motion
dS (t) = S ( ) dt + SdZ (t)
(dS)2 = 2 S 2 dt
dS 2 = 2SdS + (dS)2
= 2S [S ( ) dt + SdZ] + 2 S 2 dt
+ 2 S 2 dt
= 2S 2 [( ) dt + dZ]

2
2
= S 2 ( ) + dt + 2S 2 dZ
Problem 20.3.
www.actuary88.com

c
Yufeng
Guo

189

CHAPTER 20. BROWNIAN MOTION AND ITOS LEMMA


S 1
S 1
1 2 S 1
(dS)2
dt +
dS +
t
S
2 S 2
S 1
2 S 1
S 1
= 2S 3
=0
= S 2
t
S
S 2
dS 1 = S 2 dS + S 3 (dS)2
a.
Under DM Equation 20.8:
dS 1 =

dS (t) = dt + dZ (t)
(dS)2 = 2 dt
2
dS 1 = S 2 dS + S 3 (dS)
2
3 2
= S
(dt + dZ) + S dt

= S 2 + S 3 2 dt S 2 dZ

b.
Under DM Equation 20.9:
dS (t) = ( S) dt + dZ
(dS)2 = 2 dt
2
dS 1 = S 2 dS + S 3 (dS)
3 2
= S 2 ( ( S) dt + dZ)
dt
+ S 2
2
3 2
= S ( S) + S dt S dZ

c.
Under Geometric Brownian motion
dS (t) = S ( ) dt + SdZ (t)
(dS)2 = 2 S 2 dt
2
dS 1 = S 2 dS + S 3 (dS)
2
= S [S ( ) dt + SdZ (t)] + S 3 2 S 2 dt
S 1 2 dt
= S 1
[( ) dt + 2dZ
(t)] +
1
1
=S
( ) + dt S dZ (t)
Problem 20.4.
S 0.5
S 0.5
1 2 S 0.5
2
(dS)
dt +
dS +
t
S
2 S 2
S 0.5
2 S 0.5
S 0.5
= 0.25S 1.5
=0
= 0.5S 0.5
t
S
S 2
dS 0.5 = 0.5S 0.5 dS 0.125S 1.5 (dS)2
a.
Under DM Equation 20.8:

dS 0.5 =

dS (t) = dt + dZ (t)
(dS)2 = 2 dt
dS 0.5 = 0.5S 0.5 [dt + dZ (t)] 0.125S 1.5 2 dt
b.
Under DM Equation 20.9:
dS (t) = ( S) dt + dZ
2
(dS) = 2 dt
dS 0.5 = 0.5S 0.5 [ ( S) dt + dZ] 0.125S 1.5 2 dt
www.actuary88.com

c
Yufeng
Guo

190

CHAPTER 20. BROWNIAN MOTION AND ITOS LEMMA


c.
Under Geometric Brownian motion
dS (t) = S ( ) dt + SdZ (t)
(dS)2 = 2 S 2 dt
dS 0.5 = 0.5S 0.5 [S ( ) dt + SdZ (t)] 0.125S 1.5 2 S 2 dt
= 0.5S 0.5 [( ) dt + dZ (t)] 0.125S 0.5 2 dt
Problem 20.5.
dS (t) = S (s s ) dt + s SdZS

(DM 20.37)

dQ (t) = Q (Q Q ) dt + Q QdZQ

(DM 20.38)

dSdQ = dt
(DM 20.17d)
2 0.5

2 0.5
2 0.5
2
2 0.5 S Q
S Q
S Q
1 2 S 2 Q0.5
d S Q
(dS)2 +
=
dS+
dQ+
dSdQ+
S
Q
SQ
2
S 2

1 2 S 2 Q0.5
(Q)2
2
Q2

S 2 Q0.5
S 2 Q0.5
2 S 2 Q0.5
= 2SQ0.5
= 0.5Q0.5 S 2
=
S
Q
SQ
SQ0.5

2 S 2 Q0.5
2 S 2 Q0.5
0.5
= 2Q
= 0.25S 2 Q1. 5
S 2
Q2

d S 2 Q0.5 =
2SQ0.5 [S (s s ) dt + s SdZS ]
+0.5Q0.5 S 2 [Q (Q Q ) dt + Q QdZQ ]
+SQ0.5 dt
+Q0.5 ( Q Q)2 dt
2
0.125S 2 Q1. 5 [Q (Q Q ) dt + Q QdZQ ]
Problem 20.6.
From Problem
20.1.c, we
have:

1 2
d ln S = S S S dt + S dZS
2

1
d ln Q = Q Q 2Q dt + Q dZQ
2

1 2
1 2
d ln (SQ) = d ln S+d ln Q+ S S S dt+ S dZS + Q Q Q dt+
2
2
Q dZQ

www.actuary88.com

c
Yufeng
Guo

191

CHAPTER 20. BROWNIAN MOTION AND ITOS LEMMA


Problem 20.7.
dS (t)
= ( ) dt + dZ
S (t)

2
P
F0,T
S A (T ) = erT S A (0) e[A(r)+0.5A(A1) ]T
Assume T = 1
If A = 2
2
P
F0,1
S 2 (1) = e0.061 1002 e(2(0.060)+0.52(21)0.4 )1 = 12460. 77

If A = 0.5
2
P
F0,1
S 0.5 (1) = e0.061 1000.5 e(0.5(0.060)+0.50.5(0.51)0.4 )1 = 9. 51

If A = 2
2
P
F0,1
S 2 (1) = e0.061 1002 e(2(0.060)+0.5(2)(21)0.4 )1 = 1. 35 104

The textbook asks you to compare your solution to the solution to Problem
19.7. However, Problem 19.7 is out of the scope of Exam MFE. So you dont to
do such a comparison.
Skip the remaining problems (Problem 20.8 and beyond); they are out of
the scope of Exam MFE.

www.actuary88.com

c
Yufeng
Guo

192

Chapter 21

The Black-Scholes equation


Problem 21.1.
Equation 21.12 is V (t, T ) = er(T t)
1
BS PDE is Equation 21.11: Vt + 2 S 2 VSS + (r ) SVS rV = 0
2
We want to prove that V (t, T ) = er(T t) satisfies Equation 21.11.
V (t, T ) = er(T t)
V
2V
V
=
VS =
=0
Vt =
=0
VSS =
= rer(T t) = rV
2
S
S
t
1
=
Vt + 2 S 2 VSS + (r ) SVS rV = rV rV = 0
2
The boundary condition is that V (T, T ) = 1. Clearly, V (t, T ) = er(T t)
satisfies this boundary condition.
Problem 21.2.
V (t, T ) = AS a et

aV
VS = a AS a1 et =
S
Vt = (AS a et ) = V

a (a 1) V
VSS = a (a 1) AS a2 et =
S2

1
1
Vt + 2 S 2 VSS + (r ) SVS rV = V + 2 a (a 1) V + (r ) aV rV
2
2
To satisfy BS PDE, we just need to choose the a such that
1
1
V + 2 a (a 1) V +(r ) aV rV = 0 or + 2 a (a 1)+(r ) ar =
2
2

1 2 2
1
a + r 2 a + ( r) = 0
2
2
Solving this equation, we get:
193

CHAPTER 21. THE BLACK-SCHOLES EQUATION


s
2

1
1 2
r 2 2 2 ( r)
r

2
2
1 r
a=
=
2
2
2

s
2
2 ( r)
r 1

2
2
If we set a above, then V (t, T ) = AS a et satisfies the BS PDE.
Problem 21.3.
According to Proposition 20.30, the prepaid forward price at time zero value
of a claim paying S a (T ) is
2
F0,T [S a (T )] = erT S a (0) e[a(r)+0.5a(a1) ]T
In the above formula, if we replace T with (T t) AS a et and S (0) with
S (t), well get the prepaid forward price at time t value of a claim paying
S a (T ):
2
V (t, T ) = Ft,T [S a (T )] = er(T t) S a (t) e[a(r)+0.5a(a1) ](T t)
We need to prove that Ft,T [S a (T )] satisfies the BS PDE.
Notice that V (t, T ) is in the form of AS a et where = r a (r )
0.5a (a 1)
According to Problem
21.2, V (t, T ) satisfies the BS PDE if we set

s
2
1 r
2 ( r)
r 1
a=

2
2
Problem 21.4.
First, lets prove that if V 1 (S, t, T ) and V 2 (S, t, T ) each satisfy the BS PDE
1
Vt + 2 S 2 VSS + (r ) SVS rV = 0, then for any constants k1 and k2 the
2
linear combination V (S, t, T ) = k1 V 1 (S, t, T ) + k2 V 2 (S, t, T ) also satisfies the
BS PDE.
Proof.
1
2
Vt = k1 Vt1 + k2 Vt2
VS = k1 VS1 + k2 VS2
VSS = k1 VSS
+ k2 VSS
1
Vt + 2 S 2 VSS + (r ) SVS rV
2

1 2 2 1
1 2 2 2
1
1
1
2
2
2
= k1 Vt + S VSS + (r ) SVS rV +k2 Vt + S VSS + (r ) SVS rV
2
2
= k1 0 + k2 0 = 0
In this problem, Ker(T t) and S (t) e(T t) are each in the form of AS a et .
According to Problem 21.2, Ker(T t) and S (t) e(T t) each satisfy the BS
PDE. Hence the linear combination Ker(T t) + S (t) e(T t) satisfy the BS
PDE.
The boundary condition is that V (S, T, T ) = K + S.
www.actuary88.com

c
Yufeng
Guo

194

CHAPTER 21. THE BLACK-SCHOLES EQUATION


Problem 21.5.
Some basics.
1 R x s2 /2
e
ds
N (x) =
2
1
d Rx
0
2
N (x) = ex /2 Generally
f (s) ds = f (x)
dx a
2
1
00
x2
1 d x2 /2
1
0
N (x) =
= xe 2 = xN (x)
e
2 dx
2
Now lets prove.
V = S (t) e(T t) N (d1 )

1 2
S (t)
+ r + (T t)
ln
K
2

d1 =
T t

1
d1
1
S (t)
1

=
ln
=
S (t)
K
T t S (t)
T t S (t)
1

d1
1
0
0

N (d1 ) = N (d1 )
= N (d1 )
S
S
S
(t)
T t
00
00
1
d1
1
0

N (d1 ) = N (d1 )
= N (d1 )
S
S
S
(t)
T t
#
"

0
N
(d
N
)
(d
)
1
1
VS = e(T t) N (d1 ) + S (t)
= e(T t) N (d1 ) +
S
T t
#
"
0
N (d1 )
=
(r ) SVS = (r ) Se(T t) N (d1 ) +
T t
(r )
0
Se(T t) N (d1 )
= (r ) V +
T t

0
1
VSS = e(T t)
N (d1 ) +
N (d1 )
S
T t S

00
1
1
1
1
1
0
= e(T t) N (d1 )

N (d1 )

+
T t S (t) T t
T t S (t)

1
1
1
1
1
0
0

d1 N (d1 )

= e(T t) N (d1 )

S
(t)
S
(t)
T t
T t
T t
(T t)
d1
e
0

N (d1 ) 1
=
S (t) T t
T t

1 2 2
1 2 2 e(T t) 0
d1

=
N (d1 ) 1
S VSS = S
2
2
S T
t
T t

e(T t) 0
d1
= S
N (d1 ) 1
2 T t
T t
www.actuary88.com

c
Yufeng
Guo

195

CHAPTER 21. THE BLACK-SCHOLES EQUATION

Vt = S (t) e(T t) N (d1 ) = S (t) N (d1 ) e(T t) + e(T t) N (d1 )


t
t
t

(T t)
(T t) 0
= S (t) N (d1 ) e
+e
N (d1 ) d1
t
Please note that S (t) is a fixed constant for a given time t.

S (t)
1
ln
+ r + 2 (T t)

K
2

d1 =
t
t
T t
1
S (t)
r + 2 (T t)
ln

2
K +

=
t T t t

T t
T t
1
S (t)
1
=
ln
+ r + 2
3
K
2
t
2 (T t) 2

1 2
S (t)
1
=
ln

+r
3
K
2 T t 2
2 (T t) 2

S (t)

ln K
T t 1 2
1

=
+r

1
2 (T t)

2
(T t) 2

S (t)
1 2
ln
(T

t)
+
r

1
K
2

1
2 (T t)
2
(T t)

1 2

(T

t)

T t 1 2
1
2

+
r

1
2 (T t)
2
2
(T t)

1
2 r + 2

1
2
d1
T t
=

2 (T t)

(T t)
(T t) 0
= Vt = S (t) N (d1 ) e
+e
N (d1 ) d1

1 2
2
r

1
0
2
d1
T t
= V + S (t) e(T t) N (d1 )

2 (T t)

www.actuary88.com

c
Yufeng
Guo

196

CHAPTER 21. THE BLACK-SCHOLES EQUATION


1
Vt + 2 S 2 VSS + (r ) SVS
2

1 2
2
r

1
0
2
d1
= V + S (t) e(T t) N (d1 )
T t

2 (T t)

d1
e(T t) 0
N (d1 ) 1
+S
2 T t
T t

(r )
0
+ (r ) V +
Se(T t) N (d1 )
T t
= rV
End of the proof.

Problem 21.6.
V = er(T t) N (d2 )

d2 = d1 T t

1
d2 = d1
T t = d1 +
t
t
t
t
2 T t
1

1
1

d2 =
d1 =
=
S
S
T t S (t)
S T t
From the previous problem, we know that

1 2

2
r

2
d1
T t
d1 =

t
2 (T t)

1 2
2
r

1
2
d2 + T t
=
T t

2 (T t)

1
2 (r )
T t
d2
2 (T t)

1
2 (r )
1

T t +
d2 =
d2
t
2 (T t)

2 T t

www.actuary88.com

c
Yufeng
Guo

197

CHAPTER 21. THE BLACK-SCHOLES EQUATION

1 2
2
r

1
2
d1
=
T t

2 (T t)

1 2
2
r

1
2
d2 + T t
=
T t

2 (T t)

1 2
2 r+

1
2
d2 + T t
=
T t

2 (T t)

1 2
2
r

1
2
d2

=
T

2 (T t)

0
rN (d2 ) + N (d2 ) = rV + er(T t) N (d2 ) d2
Vt = e
t
t

1 2

2
r

1
0
2
d2
= rV + er(T t) N (d2 )
T t

2 (T t)

r(T t)

VS = er(T t) N (d2 )

N (d2 )

d2 = er(T t)
S
S T t

1
1
0

= (r ) SVS = (r ) Ser(T t) N (d2 )


T t S
1
(r ) er(T t)
0
0

= N (d2 )
= (r ) er(T t) N (d2 )
T t
T t
#
" 0
#
0
er(T t) N (d2 )
N (d2 )

=
VSS = e
S
S T t
T t S
" 00
#
0
N (d2 )
er(T t) N (d2 )
d2
=
S
S
S2
T t
"
#
0
0
N (d2 )
er(T t) d2 N (d2 )
1

=
S
S2
T t
S T t
r(T t)

= N (d2 )

"

er(T t)

S 2 T t

www.actuary88.com

d
er(T t)
0
2
+ 1 = N (d2 ) 2 2
d2 + T t
S (T t)
T t
c
Yufeng
Guo

198

CHAPTER 21. THE BLACK-SCHOLES EQUATION


1
Vt + 2 S 2 VSS + (r ) SVS
2

1 2

2
r

1
0
2
d2
= rV + er(T t) N (d2 )
T t

2 (T t)

1
er(T t)
0
2 S 2 N (d2 ) 2 2
d2 + T t
2
S (T t)
0
N (d2 )
= rV
+ (r ) Ser(T t)
S T t
Problem 21.7.

S (t) e(T t) N (d1 ) and e(T t) N (d2 ) each satisfy BS PDE. Hence their
linear combination S (t) e(T t) N (d1 )Ke(T t) N (d2 ) also satisfies BS PDE.
You can verify on your own that S (t) e(T t) N (d1 )Ke(T t) N (d2 ) satisfies
the boundary condition when t = T .
Problem 21.8.
(r)T
The forward price
is Se(r)T
,
then
. If K = Se

1 2
S
S
1 2
ln (r)T + r + T
ln
+ r+ T
K
2
2
Se

=
d1 =
T

1 2
(r ) T + r + T
1
2

=
T
=
2
T

1
1
T T =
T
d2 = d1 T =
2
2
(r)T
a. Bet #1: Getting $1 if ST > K = Se
and zero otherwise.
Bet #2: Getting $1 if ST < K = Se(r)T and zero otherwise.

Why Bet #1 is always worth less than Bet #2?


Bet #1 is worth er(T t) N (d2 ). Bet #2 is worth er(T t) N (d2 ).
Since d2 < 0, we have d2 > 0 > d2 . Since the normal cdf is increasing
function, N (d2 ) > N (d2 ). Hence Bet #2 is greater than Bet #1.
b. To make the bet fair, we need to have P (ST > K) = N (d2 ) = 0.5
so therell be equal risk-neutral chance of ST > K and ST < K. To have
N (d2 ) = 0.5, we need to have
d1 = T .
d2 = 0 and
1 2
S

+ r+ T
ln

S
1 2
K
2

= T
ln + r + T =
=
d1 =
K
2
T
2T
www.actuary88.com

c
Yufeng
Guo

199

CHAPTER 21. THE BLACK-SCHOLES EQUATION

S
1
ln
= r 2 T
K
2
#

So x = K = Se

1 2

K
1
ln
= r 2 T
S
2

K = Se

1 2

c. price of an asset call option is Ser(T t) N (d1 ).


N (d1 ) is the expected fractional share of the stock *if* the call is exercised.
To make the bet fair, set N (d1) = 0.5. This gives us d1 = 0
S
1

ln
+ r + 2 T
S
1
K
2

d1 =
=0
ln
+ r + 2 T
K
2
T

S
K
1
1
ln
ln
= r + 2 T
= r + 2 T
K
2
S
2
#
$
1 2
r+ T
2
=
K = Se
#

r+

So we need to set x = K = Se

1 2

Problem 21.9.
ln
Since K = Se(r)T is the forward price of the stock, d1 =

1
1
1
T and d2 = d1 T =
T T =
T .
2
2
2

S
Se(r)T

1
+ r + 2 T
2

=
T

valueis V1 = SerT N (d1 )KerT N (d2 ) =


Bet #1
STK if ST > K.
pays
Its

SeT N 0.5 T KerT N 0.5 T

rT
Bet #2 pays K ST
if K > S
. Its value
N (d2 )
2 = Ke
is V
T
rT
T
Se
N (d1 ) = Ke
N 0.5 T Se
N 0.5 T
h

i
h
i

V1 V2 = SeT N 0.5 T + N 0.5 T KerT N 0.5 T + N 0.5 T
T

Using the formula N (x) = 1 N (x), we get: V1 V2 = SeT KerT =


Se
erT Se(r)T = 0
The two bets have the same value.
T

Problem 21.10.
Please note that the continuous payment rate in this problem is not the
option Gamma. To avoid confusion, well use to represent the options continuous payment rate.
At time t
www.actuary88.com

c
Yufeng
Guo

200

CHAPTER 21. THE BLACK-SCHOLES EQUATION


We buy one option on the stock. We pay V
We buy N shares of the stock (a negative N means short selling stocks).
We pay N S .
We deposit W means into a savings account. We pay W .
To have zero-financing, we set our total initial cost to zero
I = V + NS + W = 0

(DM 21.7)

Next, lets consider the change of I during [t, t + dt]:


dI = dt + dV + N (dS + Sdt) + dW

(DM 21.8)

dI is the interest we earned during [t, t + dt]. Since W is invested in a savings


account, we have dW = rW dt. This says that the interest earned on W during
[t, t + dt] is rW dt.
dt is the payment we receive from the derivative during [t, t + dt].
Notice that the change of S is dS + Sdt (the sum of the change of the stock
price dS and the dividend received Sdt). Apply Itos lemma:
dV = Vt dt + VS dS + 0.5 2 S 2 VSS dt
dI = dt + Vt dt + VS dS + 0.5 2 S 2 VSS dt + N (dS + Sdt) + dW = dt +
Vt dt + (VS + N ) dS + 0.5 2 S 2 VSS dt + N Sdt + rW dt
Set N = VS . Then dS term becomes zero and W = (V VS S).
Because our initial cost is zero, the interest we earned dI should be zero.
dt + Vt dt + 0.5 2 S 2 VSS dt VS Sdt r (V VS S) dt = 0
This gives us:
+ Vt + 0.5 2 S 2 VSS VS S r (V VS S) = 0

Vt + 0.5 2 S 2 VSS + (r ) VS S + rV = 0
Skip the remaining problems.

www.actuary88.com

c
Yufeng
Guo

201

CHAPTER 21. THE BLACK-SCHOLES EQUATION

www.actuary88.com

c
Yufeng
Guo

202

Chapter 22

Exotic options: II
Skip all the problems.

203

CHAPTER 22. EXOTIC OPTIONS: II

www.actuary88.com

c
Yufeng
Guo

204

Chapter 23

Volatility
Problem 23.1.
This problem can be solved using the approach used in DM Table 11.1 (DM
page 361). In other words, if you can reproduce DM Table 11.1, you should be
able to solve this problem.
Problem 23.2.
This problem can be solved using the approach used in DM Table 11.1 (DM
page 361).
Problem 23.3.
This problem is out of the scope of the exam MFE. However, if you want to
solve it, you can use DM 23.6 to find the answer.
Problem 23.4.
Out of the scope of the exam MFE. See DM Example 23.2 if you want to
know how to solve it.
Problem 23.5.
Out of the scope of the exam MFE. See DM Example 23.2 if you want to
know how to solve it.
Problem 23.6.
205

CHAPTER 23. VOLATILITY


a. You can do this using the option price formula. Alternatively, you can
find the call price using the put-call parity C + P V (K) = P + S. Since the
maturity is very short, P is close to zero; theres little chance that the stock price
will drop from $100 to below $50 during T = 0.01. Hence C = S P V (K) =
100 50e0.060.01 = 50. 03.
b. Using the spreadsheet in the CD that comes with the textbook, you should
find vega is almost zero. Vega measures the sensitivity of the option price to
volatility (see DM section 12.3). Vega is almost zero because the option maturity
T = 0.01 is very short. Its hard for the stock price to change significantly during
this short amount of time (unless the volatility is very very big).
c. Under 5% or 100% volatility, the option price is still about 50.03. The
volatility isnt huge enough to move the stock price during T = 0.01.
However, under 500% volatility, the option price is 51.3. The volatility is
huge; the stock price can change significantly during T = 0.01
d. Its dicult to calculate the implied volatility for a call option thats deep
in the money and that has a short time to maturity.
Problem 23.7.
a. Using the put-call parity, we have C = P + S P V (K) > S P V (K) =
100 50e0.060.01 = 50. 03. So the call is worth at east 50.03; it can never be
worth less than 50. 03. Hence the bid price 50 is never possible. We cant find
the implied volatility under this bid price.
b. Once again, we use the put-call parity. The ask price 50.1 is greater than
the minimum value of the call 50.03. In order for the call to be worth more
than 50. 03, the put value must be greater than zero. To make this happen,
there must be some change that the stock price will drop from 100 to below 50
during T = 0.01. The only way this can happen is that the stock volatility is
very high.
c. Skip.
d. Its dicult to calculate the implied volatility for a deep in-the-money
call with a short maturity.
Problem 23.8.
a. For the call to have any value, the stock price must move up from $50
and be greater than K = 100 at maturity. Theres little chance that the stock
price will move up by that much during the short maturity T = 0.01 and under
a small volatility of 30%. Hence the call price is virtually zero.
www.actuary88.com

c
Yufeng
Guo

206

CHAPTER 23. VOLATILITY


b. The maturity is short. The option price is not very sensitive to the
volatility. Hence vega is close to zero.
c. The implied volatility is about 30% if the bid price is zero; the implied
volatility must be huge if the ask price is 0.05 (there must be some chance that
the stock price can move up by more than $50 at maturity in order for the call
to be worth 0.05).
d. The market maker wants to buy the option for $0 price and sell the option
for $0.05. The market-maker thinks that the option isnt worth much. At the
same time, the market maker still wants to make a little profit.
e. Its dicult to calculate and interpret the implied volatility for a deep
out-of-the-money option that has a short maturity.
Skip the remaining problems.

www.actuary88.com

c
Yufeng
Guo

207

CHAPTER 23. VOLATILITY

www.actuary88.com

c
Yufeng
Guo

208

Chapter 24

Interest rate models


Problem 24.1.
a. We need to find the price of a 1-year bond issued at t = 1. The price of
this bond is just the PV of $1 discounted from t = 2 to t = 1.
P (0, 2)
P (1, 2) =
P (0, 1)
To understand this formula, notice that P (0, 2) = P (1, 2) P (0, 1). This
equation means that to calculate the PV of $1 discounted from t = 2 to t = 0,
we first discount $1 from t = 2 to t = 1 and next discount it from t = 1 to t = 0.
P (0, 2)
0.8495
P (1, 2) =
=
= 0.917 485 7
P (0, 1)
0.9259
b. C =Time zero cost of what you get at T N (d1 ) Time zero cost of
what you
give at T N (d2 )

Time zero cost of what you get at T
2
d1 = ln Time
+
0.5
T
/ T
zero cost of what you give at T

d2 = d1 T
Make sure you know the formula for . DM page 790 shows that
p
= V ar (ln (Ft,T (P [T, T + s])))
If you buy this option, then at T = 1, you can pay K = 0.9009 and buy a
1-year bond. This 1-year bond will give you $1 at time T + s = 2.
The value of this bond at T = 0 is P (0, 2) = 0.8495, the PV of $1
discounted from T + s = 2 to T = 1
Time zero cost of the strike price K at T = 1 is just PV of K discounted
from T = 1 to time zero. So P V (K) = 0.9009 0.9259
C =Time zero cost of what you get at T N (d1 ) Time zero cost of what
you give at T N (d2 )
209

CHAPTER 24. INTEREST RATE MODELS


d1 =

ln

2
0.8495
0.90090.9259 +0.50.1 1

= 0.232 43
0.1 1

d2 = d1 T = 0.232 43 0.1 1 = 0.132 43

N (d2 ) = 0.552 68
N (d1 ) = 0.591 90
C = 0.8495 0.591 90 0.9009 0.9259 0.552 68 = 0.0418
c.
P =Time zero cost of what you give at T N (d2 ) Time zero cost of what
you get at T N (d1 )
P = 0.9009 0.9259 (1 0.552 68) 0.8495 (1 0.591 90) = 0.0264
Alternative calculation using the put-call parity.
C + P (0, T ) K = P + P (0, T + s)
0.0418 + 0.9009 0.9259 = P + 0.8495
P = 0.0264
d. Lets walk through the notations and formula.
Notation
RT =1 (T = 1, T + s = 2). The (not annualized) interest rate agreed upon
at time T = 1 that applies to the time interval [T = 1, T + s = 2].
Caplet. A Caplet gives the buyer the right to buy the time-T = 1 market interest rate RT (T, T + s) = RT =1 (T = 1, T + s = 2) by paying a
fixed strike interest rate KR = 11%. If KR RT =1 (T = 1, T + s = 2),
the caplet expires worthless. The payo of the caplet at T + s = 2 is
max [0, RT =1 (T = 1, T + s = 2) 0.11]. The payo of the caplet at T is
max [0, RT =1 (T = 1, T + s = 2) 0.11]
RT =1 (T = 1, T + s = 2)

Please note that the actual interest rate during [T = 1, T + s = 2] is a random variable. One might be tempted to think that we already know the Year 2
interest rate as:
1
1 = 8. 993 5%
P (1, 2)
However, this thinking is flawed. 8. 993 5% is the implied Yr 2 interest rate
based on the information available to us at t = 0. This rate can be dierent
from the Year 2 spot rate.
To calculate the price of the caplet, we first modify the payo:

RT =1 (T = 1, T + s = 2) 1.11
max [0, RT =1 (T = 1, T + s = 2) 0.11]
= 1.11 max 0,
=
RT =1 (T = 1, T + s = 2) 1.11
RT =1 (T = 1, T + s = 2)

1
1
1.11 max 0,

1.11 RT =1 (T = 1, T + s = 2)
www.actuary88.com

c
Yufeng
Guo

210

CHAPTER 24. INTEREST RATE MODELS

1
1
1
max 0,

= max 0, 0.900 9
1.11 RT =1 (T = 1, T + s = 2)
RT =1 (T = 1, T + s = 2)
is the payo of a put on a bond. This put gives the buyer the right, at T = 1,
1
to sell a bond that matures at T + s = 2 for a guaranteed price
= 0.900 9.
1.11
From Part c, we already know that this put price is 0.0264. Hence the caplet
price is:
1.11 0.0264 = 0.02 93
Problem 24.2.
a. P (2, 3) =

P (0, 3)
0.7722
=
= 0.909 01
P (0, 2)
0.8495

b. C = P (0, T + s) N (d1 ) P (0, T ) KN (d2 )


P (0, T + s) = P (0, 2 + 1) = P (0, 3) = 0.7722
P (0, T ) = P (0, 2) = 0.8495
C = 0.7722N (d1 ) 0.8495 0.9N (d2 )
ln
d1 =

P (0, T + s)
0.7722
+ 0.52 T
ln
+ 0.5 0.1052 2
P (0, T ) K
0.8495

0.9

=
= 0.141 29
0.105 2
T

d2 = d1 T = 0.141 29 0.105 2 = 0.007 2


N (d1 ) = 0.556 18
N (d2 ) = 0.497 13

C = 0.7722 0.556 18 0.8495 0.9 0.497 13 = 0.04 94


c.
P = P (0, T ) KN (d2 ) P (0, T + s) N (d1 )
= 0.8495 0.9 (1 0.497 13) 0.7722 (1 0.556 18)
= 0.04175 1
Alternative calculation using the put-call parity.
C + P (0, T ) K = P + P (0, T + s)
0.04 94 + 0.8495 0.9 = P + 0.7722
P = 0.041 75
d.
To calculate the price of the caplet, we first modify
the payo:

1
1
max [0, RT =2 (T = 2, T + s = 3) 0.11]
= 1.11 max 0,

1.11 RT =2 (T = 2, T + s = 3)

RT =2 (T = 2, T + s = 3)
1
1
1

= max 0, 0.900 9
max 0,
1.11 RT =2 (T = 2, T + s = 3)
RT =2 (T = 2, T + s = 3)
is the payo of a put on a bond. This put gives the buyer the right, at T = 2,
1
to sell a bond that matures at T + s = 3 for a guaranteed price
= 0.900 9.
1.11
www.actuary88.com

c
Yufeng
Guo

211

CHAPTER 24. INTEREST RATE MODELS


1
We can estimate the put price. Since
= 0.900 9 is close to 0.9, from Part
1.11
c we know that P = 0.041 75. Hence the price of the caplet is 1.10.041 75 = 0.0
459
Or we can calculate the put price.
P = P (0, T ) KN (d2 ) P (0, T + s) N (d1 ) = 0.8495 0.900 9N (d2 )
0.7722N (d1 )
P (0, T + s)
0.7722
ln
+ 0.5 2 T
ln
+ 0.5 0.1052 2
P (0, T ) K
0.8495

0.900
9

d1 =
=
=
0.105 2
T
0.134 56

d2 = d1 T = 0.134 56 0.105 2 = 1. 393 2 102


N (d1 ) = 0.446 48
N (d2 ) = 0.505 56
P = 0.8495 0.900 9 0.505 56 0.7722 0.446 48 = 0.04214
Hence the price of the caplet is 1.1 0.04214 = 0.0464
Problem 24.3.
Make sure you understand that this problem is dierent from SOA May 2007
#9. In SOA May 2007 #9, the first cash flow occurs at t = 1.The first year
market rate 6% is below the cap rate 7.5%. At the end of Year 1 (at t = 1), we
get nothing from the cap .
In this problem, the author wants us to repeatedly use using DM Equation
24.36 to calculate the cap price. So the author wants the first cash flow to occur
at t = 2.
The cap contract is signed at t = 0. During Yr 2, the cap rate 11.5% is compared with the actual Yr 2 interest. The payo at t = 2 is the max (0, 11.5% Yr 2 rate).
The PV of this payo at t = 1 is:

max (0, 11.5% Yr 2 rate)


1
1
= 1.115 max 0,

1 + Yr 2 rate
1.115 1 + Yr 2 rate

1
1
max 0,

is the payo a put. This put gives the buyer


1.115 1 + Yr 2 rate
the right at T = 1 to buy a one-year bond maturing at T + s = 2. You can
verify that the time zero cost of this put is 0.0248.
P = P (0, 1) KN (d2 ) P (0, 2) N (d1 )
0.8495
ln
+ 0.5 0.12 1
P (0, 2)
1
2
ln
+ 0.5 T
0.9259
P (0, 1) K
1.115

d1 =
=
= 0.277 36
0.1 1
T

d2 = d1 T = 0.277 36 0.1 1 = 0.177 36


www.actuary88.com

c
Yufeng
Guo

212

CHAPTER 24. INTEREST RATE MODELS


1
N (0.177 36) 0.8495N (0.277 36)
1.115
1
= 0.9259
0.429 61 0.8495 0.390 75 = 0.0248
1.115
So the time zero cost of the payo at t = 2 is 1.115 0.0248.
P = 0.9259

Similarly, the time zero cost of the payo at t = 3 is 1.115 0.0404


P = P (0, 2) KN (d2 ) P (0, 3) N (d1 )
The put gives the buyer the right at T = 2 to buy a bond maturing at
T +s=3
0.7722
+ 0.5 0.1052 2
ln
P (0, 3)
1
2
ln
+ 0.5 T
0.8495
P (0, 2) K
1.115

=
d1 =
= 0.164 82
0.105 2
T

d2 = d1 T = 0.164 82 0.105 2 = 0.01633


1
P = 0.8495
N (0.01633) 0.7722N (0.164 82)
1.115
1
= 0.8495
0.493 49 0.7722 0.434 54 = 0.040 4
1.115
The time zero cost of the payo at t = 4 is 1.115 0.0483
P = P (0, 2) KN (d2 ) P (0, 3) N (d1 )
The put gives the buyer the right at T = 3 to buy a bond maturing at
T +s=4
0.7020
ln
+ 0.5 0.112 3
P (0, 4)
1
ln
+ 0.5 2 T
0.7722
P (0, 3) K
1.115

=
d1 =
= 0.166 35
0.11 3
T

d2 = d1 T = 0.166 35 0.11 3 = 0.02417


1
P = 0.7722
N (0.02417) 0.7020N (0.166 35)
1.115
1
= 0.7722
0.509 64 0.7020 0.433 94 = 0.04832 8
1.115
The time zero cost of the cap is:
1.115 (0.0248 + 0.0404 + 0.0483) = 0.126 6
Problem 24.4.
At time zero, we
But a 3-year bond. Cost: P (t, T2 ) = e0.083 = 0.786 63
(T2 t) P (t, T2 )
1 e0.083
= 0.635 62
=
(T1 t) P (t, T1 )
2 e0.086
unit of 6-year bond (i.e. sell 0.635 62 unit of 6-year bond), receiving
0.635 62e0.086 = 0.393 31

To hedge, we buy N =

www.actuary88.com

c
Yufeng
Guo

213

CHAPTER 24. INTEREST RATE MODELS


Borrow 0.786 63 0.393 31 = 0.393 32 from a bank at 8% interest rate.
Our net position is 0.786 63 (0.393 31 + 0.393 32) = 0
If at the end of the day the risk-free rate is 8.25%, well close o our position.
Sell the (3 1/365)-year bond, receiving e0.0825(31/365) = 0.780 93
Buy back the (6 1/365)-year bond, paying 0.635 62e0.0825(61/365) =
0.387 54
Pay o the loan, paying 0.393 32e0.081/365 = 0.393 41 (we borrow the
loan for only one day).
Our net position: 0.780 93 0.387 54 0.393 41 = 0.000 02. So we lose
0.000 02

Why do we lose money? Because of the convexity mismatch.


We buy the 3-year bond. The convexity is (see DM 7.14 and DM Example
7.9):
34
= 10. 288 07
1.082
We sell 0.635 62 unit of the 6-year bond. The convexity is:
67
= 22. 887 55
0.635 62
1.082
Even though we hedged the duration, we didnt hedge convexity. The two
portfolios have dierent convexities. As explained in my study guide, the highconvexity bond has a better value. To arbitrage, we should have bought the
high-convexity bond and sold the low-convexity bond. So to arbitrage, we need
to reverse our position (i.e. at t = 0, sell one unit of 3-year bond and buy
0.635 62 unit of 6-year bond). Then well earn 0.000 02 free money during Day
1.
How to reverse our position. At time zero, we
Sell a 3-year bond, receiving P (t, T2 ) = e0.083 = 0.786 63
To hedge, we buy 0.635 62 unit of 6-year bond, paying 0.635 62e0.086 =
0.393 31
Lend 0.786 63 0.393 31 = 0.393 32 at 8% interest rate.
Our net position is 0.786 63 (0.393 31 + 0.393 32) = 0
At the end of the day, the risk-free rate is 8.25%. We close o our position.
Buy the (3 1/365)-year bond, paying e0.0825(31/365) = 0.780 93
Sell the (6 1/365)-year bond, receiving 0.635 62e0.0825(61/365) = 0.387 54
www.actuary88.com

c
Yufeng
Guo

214

CHAPTER 24. INTEREST RATE MODELS


Close the loan, receiving 0.393 32e0.081/365 = 0.393 41.
Our net position: 0.387 54 + 0.393 41 0.780 93 = 0.000 02. So we gain
0.000 02
At time zero, our cost is zero. At the end of the day, we receive 0.000 02
profit. This is arbitrage.
If at the end of the day the risk-free rate is 7.75%, well close o our position.
Sell the (3 1/365)-year bond, receiving e0.0775(31/365) = 0.792 72
Buy back the (6 1/365)-year bond, paying 0.635 62e0.0775(61/365) =
0.399 34
Pay o the loan, paying 0.393 32e0.081/365 = 0.393 41 (we borrow the
loan for only one day).
Our net position: 0.792 72 0.399 34 0.393 41 = 0.000 03. So we lose
0.000 03

To arbitrage, we reverse out position and gain 0.000 03 free money.


This example shows that the assumption of the parallel shift of a flat yield
curve leads to arbitrage. To build a good model for bond price, we need to
throw away this bad assumption.
Problem 24.5.
a.4-year 5% annual
coupon bond with yield 6% (Bond 1)
Price: P1 = 0.05 e0.061 + e0.062 + e0.063 + e0.064 + e0.064 =
0.959 16
Macaulay duration:

0.05 e0.061 + 2e0.062 + 3e0.063 + 4e0.064 + 4e0.064


D1 =
= 3.
0.959 16
716 7
8-year 7% annual coupon bond with yield 6% (Bond 2)
Price:

P2 = 0.07 e0.061 + e0.062 + e0.063 + e0.064 + ... + e0.068 +e0.068 =


1. 050 3
Macaulay duration:

0.07 e0.061 + 2e0.062 + 3e0.063 + ... + 8e0.068 + 8e0.068


D2 =
=
1. 050 3
6. 433 2
b. Use DM 7.13:
D1 B1 (y1 ) (1 + y1 )
N =
D2 B2 (y2 ) (1 + y2 )
www.actuary88.com

c
Yufeng
Guo

215

CHAPTER 24. INTEREST RATE MODELS


Please also note that DM 24.7 is the special case of DM 7.13. If the two
bonds each have one cash flow and two bonds have the same yield (i.e. y1 = y2 ),
then DM 7.13 becomes DM 24.7.
In this problem, the yield is the same for the two bonds. So y1 = y2 .
D1 P1 (y)
3. 716 7 0.959 16
D1 B1 (y1 ) (1 + y1 )
=
=
= 0.527 6
N =
D2 B2 (y2 ) (1 + y2 )
D2 P2 (y)
6. 433 2 1. 050 3
we

So we need to buy 0.527 6 (i.e. sell 0.527 6) unit of Bond 2. At time zero,
sell 0.527 6 unit of Bond 2, receiving 0.527 6 1. 050 3 = 0.554 14
buy one Bond 1, paying 0.959 16
borrow the dierence 0.959 16 0.554 14 = 0.405 02 from a bank
0.959 16 0.527 6 1. 050 3 = 0.405 021 72
Our net position is zero.

If at the end of the day, the yield is 6.25%:


Back back 0.527 6 unit of Bond 2, which has (8 1/365) year to maturity.
Now we are
at t = 1/365. Bond 2s price now is:
standing

P2 = 0.07 e0.06251 + e0.05752 + e0.06253 + ... + e0.06258 + e0.06258 e0.0625/365


= 1. 034 39
Sell Bond 1, which has (4 1/365) year to maturity
The price
is

P1 = 0.05 e0.0.06251 + e0.06252 + e0.06253 + e0.06254 + e0.06254 e0.0625/365 =


0.953 48
Repay the bank: 0.405 02e0.06/365 = 0.405 09
The net profit at the end of Day 1 is: 0.953 480.527 61. 034 390.405 09 =
0.002 65
If at the end of the day, the yield is 5.75%:
Back back 0.527 6 unit of Bond 2, which has (8 1/365) year to maturity.
Now we are standing
at t = 1/365. Bond 2s price now is:

P2 = 0.07 e0.0575(11/365) + e0.0575(21/365) + ... + e0.0575(81/365) +


e0.0575(81/365)

= 0.07 e0.05751 + e0.05752 + e0.05753 + ... + e0.05758 + e0.05758 e0.0575/365


= 1. 067 54
Sell Bond 1, which has (4 1/365) year to maturity
The price
is

P1 = 0.05 e0.05751 + e0.05752 + e0.05753 + e0.05754 + e0.05754 e0.0575/365 =


0.968 27
Repay the bank: 0.405 02e0.06/365 = 0.405 09
www.actuary88.com

c
Yufeng
Guo

216

CHAPTER 24. INTEREST RATE MODELS


The net profit at the end of Day 1 is: 0.968 270.527 61. 067 540.405 09 =
0.000 05
If we reserve our position, we can have 0.000 05 at the end of Day 1. Refer
to Problem 24.4 to see how to reverse our position.
Summary: If the yield moves up to 6.25% in Day 1, we can make 0.002 65
profit; if the yield moves down to 5.75% in Day 1, we can make 0.000 05 profit.
Once again, the assumption of the flat yield curve leads to arbitrage.
Problem 24.6.
Vasicek model:
a.
Zero risk premium means that (r, t, T ) = r. Hence (r, t) =
(r, t, T ) r
=0
q (r, t, T )
2
0.12
r = b 0.5 2 = 0.1 0.5
= 0.025
DM 24.26:

0.22
2-year bond:
1 e(T t)
1 e0.2(2)
B (T t = 2) = aT t| =
=
= 1. 648 4

0.2
aT t| is a T t year continuous annuity with the force of interest .
2

A (T t = 2) = er[B(2)2]B (2) /4 = e0.025(1. 648 42)1. 648 4 0.1 /(40.2) =


0.975 14
The 2-year bond is worth:
P (0, 2) = A (2) eB(2)r = 0.975 14e1. 648 40.05 = 0.897 99
The delta is:
d
d
P (0, 2) =
A (2) eB(2)r = B (2) A (2) eB(2)r = B (2) P (0, 2) =
dr
dr
1. 648 4 0.897 99 = 1. 480 2
The gamma is:
d2
d
P (0, 2) = B (2) P (0, 2) = B 2 (2) P (0, 2) = 1. 648 42 0.897 99 = 2.
2
dr
dr
44
10-year bond:
1 e0.210
= 4. 323 3
0.2
2
2
r[B(10)10]B 2 (10) 2 /4
= e0.025(4. 323 310)4. 323 3 0.1 /(40.2) =
A (10) = e
0.912 36
The 10-year bond is worth:
P (0, 10) = A (10) eB(10)r = 0.912 36e4. 323 30.05 = 0.735
The delta is:
d
P (0, 10) = B (10) P (0, 10) = 4. 323 3 0.735 = 3. 177 6
dr
The gamma is:
d2
P (0, 10) = B 2 (10) P (0, 10) = 4. 323 32 0.735 = 13. 738
dr2
B (10) = a10|0.2 =

www.actuary88.com

c
Yufeng
Guo

217

CHAPTER 24. INTEREST RATE MODELS


The delta is:
d
d
P (0, 2) =
A (2) eB(2)r = B (2) A (2) eB(2)r = B (2) P (0, 2) =
dr
dr
1. 648 4 0.897 99 = 1. 480 2
The gamma is:
d
d2
P (0, 2) = B (2) P (0, 2) = B 2 (2) P (0, 2) = 1. 648 42 0.897 99 = 2.
dr2
dr
44
b. If we buy a 2-year bond, to duration hedge our risk, we need to buy
2P (0, 2)
2 0.897 99
=
= 0.244 35
10P (0, 10)
10 0.735
So we need to sell 0.244 35 unit of 10-year bond.

N =

At t = 0, we
buy a 2-year bond, paying P (0, 2) = 0.897 99
sell 0.244 35 unit of 10-year bond, receiving 0.244 35 0.735 = 0.179 60
borrow 0.897 99 0.179 60 = 0.718 39 at 5%
Our net position is zero.
p
The one
pstandard deviation of the interest rate under Vasicek is r 1/365 =
0.05 0.1 1/365 p
ru = 0.05 + 0.1p 1/365 = 0.055
rd = 0.05 0.1 1/365 = 0.045
p
Under ru = 0.05 + 0.1 1/365 = 0.055
2 1/365 year bond:
1 e(T t)
1 e0.2(21/365)
B (2 1/365) =
=
= 1. 647

0.2
0.025(1. 646 72+1/365)1. 6472 0.12 /(40.2)
= 0.975 2
A (2 1/365) = e
The 2 1/365-year bond is worth:
P (0, 2 1/365) = 0.975 2e1. 64670.055 = 0.890 8
10 1/365-year bond:
1 e0.2(101/365)
B (10 1/365) =
= 4. 323 0
0.2
2
2
r[B(101/365)]B 2 (101/365) 2 /4
= e0.025(4. 323 10+1/365)4. 323 0.1 /(40.2) =
A (10 1/365) = e
0.912 34
The 10 1/365-year bond is worth:
P (0, 10 1/365) = A (10 1/365) eB(101/365)r = 0.912 34e4. 323 0.055 =
0.719 3
At the end of the day, we close our position:
www.actuary88.com

c
Yufeng
Guo

218

CHAPTER 24. INTEREST RATE MODELS


sell a 2 1/365-year bond, receiving 0.890 8
buy 0.244 35 unit of 101/365-year bond, paying 0.244 35 0.719 3 =
0.175 8
pay back the loan, paying 0.718 39e0.05/365 = 0.718 5
our net position is: 0.890 8 0.175 8 0.718 5 = 0.003 5
So we lose 0.003 5
p
Under rd = 0.05 0.1 1/365 = 0.045
2 1/365 year bond:
1 e(T t)
1 e0.2(21/365)
B (2 1/365) =
=
= 1. 647

0.2
0.025(1. 646 72+1/365)1. 6472 0.12 /(40.2)
= 0.975 2
A (2 1/365) = e
The 2 1/365-year bond is worth:
P (0, 2 1/365) = 0.975 2e1. 64670.045 = 0.905 6
10 1/365-year bond:
1 e0.2(101/365)
B (10 1/365) =
= 4. 323 0
0.2
2
2
r[B(101/365)]B 2 (101/365) 2 /4
= e0.025(4. 323 10+1/365)4. 323 0.1 /(40.2) =
A (10 1/365) = e
0.912 34
The 10 1/365-year bond is worth:
P (0, 10 1/365) = A (10 1/365) eB(101/365)r = 0.912 34e4. 323 0.045 =
0.751 1
At the end of the day, we close our position:
sell a 2 1/365 year bond, receiving 0.905 6
buy 0.244 35 unit of 10 1/365 year bond, paying 0.244 35 0.751 1 =
0.183 5
pay back the loan, paying 0.718 39e0.05/365 = 0.718 5
our net position is: 0.905 6 0.183 5 0.718 5 = 0.003 6
So we gain 0.003 6
Delta hedge. At t = 0, we buy a 2-year bond and buy N units of 10-year
bond. We already calculated the following:
The delta of the 2-year bond is 1. 480 2
The delta of the 10-year bond is 3. 177 6
www.actuary88.com

c
Yufeng
Guo

219

CHAPTER 24. INTEREST RATE MODELS


The total delta of the portfolio is: 1. 480 2 + N (3. 177 6).
To delta hedge, set 1. 480 2 + N (3. 177 6) = 0
N = 0.465 8
So at t = 0, we
buy a 2-year bond, paying P (0, 2) = 0.897 99
sell 0.465 8 unit of 10-year bond, receiving 0.465 8 0.735 = 0.342 36
borrow 0.897 99 0.342 36 = 0.555 63 at 5%
Our net position is zero.

p
Under ru = 0.05 + 0.1 1/365 = 0.055
The 2 1/365-year bond is worth: P (0, 2 1/365) = 0.890 8
The 10 1/365-year bond is worth: P (0, 10 1/365) = 0.719 3
At the end of the day, we close our position:
sell a 2 1/365-year bond, receiving 0.890 8
buy 0.465 8 unit of 101/365-year bond, paying 0.465 80.719 3 = 0.335 05
pay back the loan, paying 0.555 63e0.05/365 = 0.555 71
our net position is: 0.890 8 0.335 05 0.555 71 = 0.000 04
So we gain 0.000 04
p
Under rd = 0.05 0.1 1/365 = 0.045
The 2 1/365-year bond is worth: P (0, 2 1/365) = 0.905 6
The 10 1/365-year bond is worth: P (0, 10 1/365) = 0.751 1
At the end of the day, we close our position:
sell a 2 1/365 year bond, receiving 0.905 6
buy 0.465 8 unit of 101/365 year bond, paying 0.465 80.751 1 = 0.349 86
pay back the loan, paying 0.555 63e0.05/365 = 0.555 71
our net position is: 0.905 6 0.349 86 0.555 71 = 0.000 03
So we gain 0.000 03
www.actuary88.com

c
Yufeng
Guo

220

CHAPTER 24. INTEREST RATE MODELS


CIR:
a. Zero risk premium means that (r, t, T ) = r. Hence (r, t) =
0

(r, t, T ) r
=
q (r, t, T )

DM 24.28: (r, t) = r/. Hence = 0


DM q
24.29:
q
2
2
= (a + ) + 2 2 = (0.2 + 0) + 2 0.447212 = 0.663 32

a + + = 0.2 + 0 + 0.663 32 = 0.863 32


2-year bond:
"

2e(a++ )(T t)/2

A (T t = 2) =
a + + e(T t) 1 + 2

2 0.663 32e0.863 322/2


=
0.863 32 (e0.663 322 1) + 2 0.663 32

# 2ab
2

2 0.2 0.1
0.447212

= 0.967 18

2 e(T t) 1

B (T t = 2) =
a + + e(T t) 1 + 2

2 e0.663 322 1
=
= 1. 489 72
0.863 32 (e0.663 322 1) + 2 0.663 32

P (0, 2) = A (2) eB(2)r = 0.967 18e1. 489 720.05 = 0.897 76


The delta is:
d
d
P (0, 2) =
A (2) eB(2)r = B (2) P (0, 2) = 1. 489 72 0.897 76 =
dr
dr
1. 337 41
The gamma is:
d
d2
P (0, 2) = B (2) P (0, 2) = B 2 (2) P (0, 2) = 1. 489 722 0.897 76 =
dr2
dr
1. 992 37
10-year bond:
2 0.2 0.1

2 0.663 32e0.863 3210/2


0.447212
=
A (T t = 10) =
0.663
3210
0.863 32 (e
1) + 2 0.663 32
0.685 54

2 e0.663 3210 1
B (T t = 10) =
= 2. 311 96
0.863 32 (e0.663 3210 1) + 2 0.663 32

P (0, 10) = A (10) eB(10)r = 0.685 54e2. 311 960.05 = 0.610 70


www.actuary88.com

c
Yufeng
Guo

221

CHAPTER 24. INTEREST RATE MODELS


The delta is:
d
P (0, 10) = B (10) P (0, 10) = 2. 311 96 0.610 70 = 1. 411 91
dr
The gamma is:
d2
P (0, 10) = B 2 (10) P (0, 10) = 2. 311 962 0.610 70 = 3. 264 29
dr2
b. If we buy a 2-year bond, to duration hedge our risk, we need to buy N
units of 10 year bond.
2P (0, 2)
2 0.897 76
=
= 0.294 01
10P (0, 10)
10 0.610 70
So we need to sell 0.294 01 unit of 10-year bond.

N =

At t = 0, we
buy a 2-year bond, paying P (0, 2) = 0.897 76
sell 0.294 01 unit of 10-year bond, receiving 0.294 01 0.610 70 = 0.179 55
borrow 0.897 76 0.179 55 = 0.718 21 at 5%
Our net position is zero.

Notice that under CIR, dr = a (b r) dt+p


rdz. The one standard
deviation
of
the
interest
rate
under
CIR
is
r

r
1/365
=
0.05

0.44721
0.05
p
1/365
p

0.44721 0.05 1/365 = 0.005 234 2


ru = 0.05 + 0.005 234 2 = 0.055 23
rd = 0.05 0.005 234 2 = 0.044 77
Under ru = 0.055 23
The 2 1/365-year bond is worth:
"

0.863 32(21/365)/2

# 2 0.2 0.1
0.447212

2 0.663 32e

0.863 32 e0.663 32(21/365) 1 + 2 0.663 32


0.967 26

2 e0.663 32(21/365) 1

B (T t = 2 1/365) =
=
0.863 32 e0.663 32(21/365) 1 + 2 0.663 32
1. 488 40
A (T t = 2 1/365) =

P (0, 2 1/365) = A (2 1/365) eB(21/365)r = 0.967 26e1. 488 400.055 23 =


0.890 93
The 10 1/365-year bond is worth:
www.actuary88.com

c
Yufeng
Guo

222

CHAPTER 24. INTEREST RATE MODELS

A (T t = 10 1/365) =
0.685 63

"

0.863 32(101/365)/2

2 0.663 32e

0.863 32 e0.663 32(101/365) 1 + 2 0.663 32

# 2 0.2 0.1
0.447212

2 e0.663 32(101/365) 1

B (T t = 10 1/365) =
=
0.863 32 e0.663 32(101/365) 1 + 2 0.663 32
2. 311 95
P (0, 10 1/365) = 0.685 63e2. 311 950.055 23 = 0.603 44
At the end of the day, we close our position:
sell a 2 1/365-year bond, receiving 0.890 93
buy 0.294 01 unit of 10 1/365-year bond, paying 0.294 01 0.603 44 =
0.177 42
pay back the loan, paying 0.718 21e0.05/365 = 0.718 31
our net position is: 0.890 93 0.177 42 0.718 31 = 0.004 8
So we lose 0.004 8
Under rd = 0.044 77
P (0, 2 1/365) = A (2 1/365) eB(21/365)r = 0.967 26e1. 488 400.044 77 =
0.904 91
P (0, 10 1/365) = 0.685 63e2. 311 950.044 77 = 0.618 21
At the end of the day, we close our position:
sell a 2 1/365-year bond, receiving 0.904 91
buy 0.294 01 unit of 10 1/365-year bond, paying 0.294 01 0.618 21 =
0.181 76
pay back the loan, paying 0.718 21e0.05/365 = 0.718 31
our net position is: 0.904 91 0.181 76 0.718 31 = 0.004 84
So we gain 0.004 84
Delta hedge:
1. 337 41 + N (1. 411 91) = 0
N = 0.947 23
So we need to sell 0.947 23 unit of the 10-year bond.
At t = 0, we
www.actuary88.com

c
Yufeng
Guo

223

CHAPTER 24. INTEREST RATE MODELS


buy a 2-year bond, paying P (0, 2) = 0.897 76
sell 0.947 23 unit of 10-year bond, receiving 0.947 23 0.610 70 = 0.578 47
borrow 0.897 76 0.578 47 = 0.319 29 at 5%
Our net position is zero.
One day later, under ru = 0.055 23
P (0, 2 1/365) = A (2 1/365) eB(21/365)r = 0.967 26e1. 488 400.055 23 =
0.890 93
P (0, 10 1/365) = 0.685 63e2. 311 950.055 23 = 0.603 44
At the end of the day, we close our position:
sell a 2 1/365-year bond, receiving 0.890 93
buy 0.947 23 unit of 10 1/365-year bond, paying 0.947 23 0.603 44 =
0.571 60
pay back the loan, paying 0.319 29e0.05/365 = 0.319 33
our net position is: 0.890 93 0.571 60 0.319 33 = 0
So we gain 0
Under rd = 0.044 77
P (0, 2 1/365) = 0.967 26e1. 488 400.044 77 = 0.904 91
P (0, 10 1/365) = 0.685 63e2. 311 950.044 77 = 0.618 21
At the end of the day, we close our position:
sell a 2 1/365-year bond, receiving 0.904 91
buy 0.947 23 unit of 10 1/365-year bond, paying 0.947 23 0.618 21 =
0.585 59
pay back the loan, paying 0.319 29e0.05/365 = 0.319 33
our net position is: 0.904 91 0.585 59 0.319 33 0
So we gain 0
Problem 24.7.
www.actuary88.com

c
Yufeng
Guo

224

CHAPTER 24. INTEREST RATE MODELS


t=0

t=1

t=2

t=3
0.16

0.04
0.12
0.12
0.12
0.10
0.08
0.10
0.12
0.10
0.08
0.08
0.08
0.06
0.04
Price of 1-Yr bond: P (0, 1) = e0.1 = 0.904 84
The yield for 1-Yr bond is 0.1.
Price of
Path
0u
0d

2-Yr bond:
Prob Price
0.5
e0.12 e0.1
0.5
e0.08 e0.1

P (0, 2) = 0.5e0.12 e0.1 + 0.5e0.08 e0.1 = 0.818 89


The yield for 2-Yr bond is y
ln 0.818 89
y=
= 9.990 3%
e2y = 0.818 89
2
Price of 3-Yr bond:
Path
Prob
0 u uu 0.25
0 u ud 0.25
0 d du 0.25
0 d dd 0.25

Price
e0.14 e0.12 e0.1
e0.1 e0.12 e0.1
e0.1 e0.08 e0.1
e0.06 e0.08 e0.1

P (0, 5) = 0.25 e0.14 e0.12 e0.1 + e0.1 e0.12 e0.1 + e0.1 e0.08 e0.1 + e0.06 e0.08 e0.1 =
0.741 56
1
Yield for the 3-Yr bond: ln 0.741 56 = 9. 966 6%
3
Price of 4-Yr bond:
www.actuary88.com

c
Yufeng
Guo

225

CHAPTER 24. INTEREST RATE MODELS


Path
0 u uu uuu
0 u uu uud
0 u ud udu
0 u ud udd
0 d du duu
0 d du dud
0 d dd ddu
0 d dd ddd

Prob
1/8
1/8
1/8
1/8
1/8
1/8
1/8
1/8

Price
e0.16 e0.14 e0.12 e0.1
e0.12 e0.14 e0.12 e0.1
e0.12 e0.10 e0.12 e0.1
e0.08 e0.10 e0.12 e0.1
e0.12 e0.10 e0.08 e0.1
e0.08 e0.10 e0.08 e0.1
e0.08 e0.06 e0.08 e0.1
e0.04 e0.06 e0.08 e0.1

= 0.594 52
= 0.618 78
= 0.644 04
= 0.670 32
= 0.670 32
= 0.697 68
= 0.726 15
= 0.755 78

0.594 52 + 0.618 78 + 0.644 04 + 0.670 32 + 0.670 32 + 0.697 68 + 0.726 15 + 0.755 78


P (0, 4) =
=
8
0.672 20
1
Yield for the 4-Yr bond: ln 0.672 20 = 9. 930 0%
4
Yields decline with maturity. This is explained in DM page 796. The average
of the exponential interest rates is less than the exponentiated average.
Problem 24.8.
Instead of working path by path, we can work backwards.
At t = 3, the bond price is:
Vuuu = 0.8331
Vuud = 0.8644
Vddu = 0.8906
At t = 2, the bond price is:
0.8331 + 0.8644
Vuu = 0.8321
= 0.706 24
2
Vud = Vdu = 0.8798
Vdd = 0.9153

Vddd = 0.9123

0.8644 + 0.8906
= 0.772 02
2

0.8906 + 0.9123
= 0.825 10
2

At t = 1, the bond price is:


0.706 24 + 0.772 02
Vu = 0.8832
= 0.652 80
2
Vd = 0.9023

0.772 02 + 0.825 10
= 0.720 54
2

At t = 0, the bond price is:


0.652 80 + 0.720 54
Vu = 0.9091
= 0.624 25 = 0.6243
2
Problem 24.9.
www.actuary88.com

c
Yufeng
Guo

226

CHAPTER 24. INTEREST RATE MODELS


Next year, the bond price is either Vu = 0.652 80 with a yield of 0.652 801/3
1 = 0.152 76 or Vd = 0.720 54 with a yield of 0.720 541/3 1 = 0.115 44.
0.152 76
Using DM 24.48, the yield volatility is 0.5 ln
= 0.140 06 = 0.14
0.115 44
Problem 24.10.
See DM page 805.
Problem 24.11.
P (0, 3)
0.7118
1 =
1 = 0.140 16.
P (0, 4)
0.6243
Since the interest rate cap applies during the interval [t = 3, t = 4], the fair cap
rate must be the implied forward rate rA = 0.140 16 during [t = 3, t = 4] . We
can verify that rA = 0.140 16 using the binomial tree.
Let r (3, 4) represent the actual interest rate during the interval [t = 3, t = 4].
The reference rate rA is the 1-year forward rate 3 years hence. So rA is also
the rate during [t = 3, t = 4]. This is the dierence between r (3, 4) and rA .
r (3, 4) is the actual interest rate observed in the market during Year 3; rA is
rate agreed upon at t = 0 that applies to Year 3.
Set the notional principal to $100. At t = 4, the payo is 100 [r (3, 4) rA ]
= 100r (3, 4) 100rA . We need to find rA such that the PV of the payo is
zero.
First, we calculate PV of 100r (3, 4). At t = 4, 100r (3, 4) has 4 possible
values: 20.03,15.68,12.28, and 9.62. Discounting these 4 values to t = 3, we get
the 4 values:
100 0.2003
100 0.1568
= 16. 687
= 13. 555
1 + 0.2003
1 + 0.1568
DM Page 806 and 807 explain that rA =

100 0.1228
= 10. 937
1 + 0.1228

100 0.0962
= 8. 775 8
1 + 0.0962

We have 3 values at t = 2 (using 0.5 as the risk-neutral probability of up or


down)

16. 687
13. 555
Vuu = 0.5
+
= 12. 583
1 + 0.2017 1 + 0.2017
Vud = 0.5

Vdd = 0.5

13. 555
10. 937
+
1 + 0.1366 1 + 0.1366
10. 937
8. 775 8
+
1 + 0.0925 1 + 0.0925

= 10. 774

= 9. 021 9

The valueat t = 1:

12. 583
10. 774
+
= 10. 315
Vu = 0.5
1 + 0.1322 1 + 0.1322
www.actuary88.com

c
Yufeng
Guo

227

CHAPTER 24. INTEREST RATE MODELS

Vd = 0.5

10. 774
9. 021 9
+
1 + 0.1082 1 + 0.1082

= 8. 931 6

The value
at t = 0:

10. 315 8. 931 6


V = 0.5
+
= 8. 748 5
1 + 0.1
1 + 0.1
So PV of 100r (3, 4) is 8. 748 5.
PV of 100rA is 100rA P (0, 4) = 100rA 0.6243
P (0, 4) = 0.6243 is from DM Table 24.2.
100rA 0.6243 = 8. 748 5

rA = 0.140 13

Problem 24.12.
I just solve for Tree #1. Once you understand the logic, you can do Tree
#2.
1-Yr bond price:
P (0, 1) =

1
= 0.925 93
1.08

2-Yr bond price:


1
1
Vd =
Vu =
1 + 0.07676
1 + 0.10363

Vu + Vd
1
1
1
V = 0.5
= 0.5
+
= 0.849 45
1.08
1.08 1 + 0.07676 1 + 0.10363
3-Yr bond price:
1
Vdd =
1 + 0.08170

1
1 + 0.13843

1
1
1
Vd = 0.5
+
= 0.849
1 + 0.07676 1 + 0.08170 1 + 0.10635
1
1
1
Vu = 0.5
+
= 0.807 46
1 + 0.10363 1 + 0.10635 1 + 0.13843
1
(0.849 + 0.807 46) = 0.766 88
V = 0.5
1.08
Vud =

1
1 + 0.10635

Vuu =

4-Yr bond price:


Value at t = 3
1
1 + 0.07943

1
1 + 0.09953

1
1 + 0.12473

1
1 + 0.15630

Value at t = 2
Vdd = 0.5

1
1 + 0.08170

www.actuary88.com

1
1
+
1 + 0.07943 1 + 0.09953
c
Yufeng
Guo

= 0.848 62
228

CHAPTER 24. INTEREST RATE MODELS

1
1
1
+
= 0.812 84
1 + 0.10635 1 + 0.09953 1 + 0.12473
1
1
1
= 0.5
+
= 0.770 33
1 + 0.13843 1 + 0.12473 1 + 0.15630

Vud = 0.5
Vuu

Value at t = 1
1
(0.848 62 + 0.812 84) = 0.771 51
Vd = 0.5
1 + 0.07676
1
(0.812 84 + 0.770 33) = 0.717 26
Vu = 0.5
1 + 0.10363
Value at t = 0
V = 0.5

1
(0.771 51 + 0.717 26) = 0.689 25
1.08

5-Yr bond price:


value at t = 4
1
1
1 + 0.07552
1 + 0.09084
1
1
1 + 013143
1 + 0.15809
value at t = 3

1
1
0.5
1 + 0.07943 1 + 0.07552
1
1
0.5
1 + 0.09953 1 + 0.09084
1
1
0.5
1 + 0.12473 1 + 0.10927
1
1
0.5
1 + 0.15630 1 + 0.13143

1
1 + 0.10927

1
= 0.855 32
1 + 0.09084
1
+
= 0.826 82
1 + 0.10927
1
+
= 0.793 67
1 + 0.13143
1
+
= 0.755 57
1 + 0.15809
+

value at t = 2
1
(0.855 32 + 0.826 82) = 0.777 54
Vdd = 0.5
1 + 0.08170
1
Vud = 0.5
(0.826 82 + 0.793 67) = 0.732 36
1 + 0.10635
1
(0.793 67 + 0.755 57) = 0.680 43
Vuu = 0.5
1 + 0.13843
value at t = 1
1
Vd = 0.5
(0.777 54 + 0.732 36) = 0.701 13
1 + 0.07676
1
(0.732 36 + 0.680 43) = 0.640 07
Vu = 0.5
1 + 0.10363
Value at t = 0
1
V = 0.5
(0.701 13 + 0.640 07) = 0.620 93
1.08
www.actuary88.com

c
Yufeng
Guo

229

CHAPTER 24. INTEREST RATE MODELS


Problem 24.13.
Im going to solve for only Tree #1.
DM Page 799 states that the volatility in Year 1 is the standard deviation
of the natural log of the yield for that bond 1 year hence. So the volatility in
Yr 1 for an n-year bond is the standard deviation of the natural log of the yield
of an (n 1)-year bond issued at t = 1.
Yr-1 yield for 1-yr bond is unknown.
Yr-1 yield for 2-yr bond . The up yield of an 2 1 = 1 year bond issued at
t = 1 is ru = 0.10362; the down yield is 0.07676.Using DM 24.48, we get:
0.10362
ru
= 0.5 ln
= 0.150 02
0.5 ln
rd
0.07676
Yr-1 yield for a 3-yr bond. We first calculate the price of a 2-yr bond issued
at t = 1.
From the previous problem
regarding the price of the 3-year bond, we know
1
1
1
+
= 0.849
Vd = 0.5
1 + 0.07676 1 + 0.08170 1 + 0.10635
1
1
1
+
= 0.807 46
Vu = 0.5
1 + 0.10363 1 + 0.10635 1 + 0.13843
ru = 0.807 461/2 1 = 0.112 86
rd = 0.8491/2 1 = 0.08529 1
Volatility: 0.5 ln

ru
0.112 85
= 0.5 ln
= 0.140 00
rd
0.08529 1

Yr-1 yield for a 4-yr bond. We first calculate the price of a 3-yr bond issued
at t = 1.
From the previous problem about the price of a 4-yr bond,
1
(0.848 62 + 0.812 84) = 0.771 51
Vd = 0.5
1 + 0.07676
1/3
1 = 9. 031 7 102
rd = 0.771 51
1
Vu = 0.5
(0.812 84 + 0.770 33) = 0.717 26
1 + 0.10363
rd = 0.717 261/3 1 = 0.117 14
ru
0.117 14
= 0.5 ln
= 0.130 02
rd
9. 031 7 102
Yr-1 yield for a 5-yr bond. We first calculate the price of a 4-yr bond issued
at t = 1.
From the previous problem about the price of a 5-yr bond,
1
Vd = 0.5
(0.777 54 + 0.732 36) = 0.701 13
1 + 0.07676
rd = 0.701 131/4 1 = 9. 282 4 102
Volatility: 0.5 ln

www.actuary88.com

c
Yufeng
Guo

230

CHAPTER 24. INTEREST RATE MODELS


1
(0.732 36 + 0.680 43) = 0.640 07
1 + 0.10363
1/4
1 = 0.118
ru = 0.640 07
Vu = 0.5

ru
0.118
= 0.5 ln
= 0.119 99 = 0.12
rd
9. 282 4 102
Dont worry about the question "Can you unambiguously say that rates in
one tree are more volatile than the other?"
Volatility: 0.5 ln

Problem 24.14.
Skip. This problem is not worth your time.

Problem 24.15.
Ill solve for only Tree #1.
time 0 1
2

4
0.15809

0.15630
0.13843

0.13143

0.10362
0.08

0.12473
0.10635

0.10927

0.07676

0.09953
0.08170

0.09084
0.07943
0.07552

Set the notional amount to $1. The payo at each node is


Payo:
time 0

0.13843 0.105
1 + 0.13843
0
0.10635 0.105
1 + 0.10635

0
0

0.15630 0.105
1 + 0.15630
0.12473 0.105
1 + 0.12473

1
max (0, r 0.105)
1+r

4
0.15809 0.105
1 + 0.15809
0.13143 0.105
1 + 0.13143
0.10927 0.105
1 + 0.10927

0
0

0
0
0

www.actuary88.com

c
Yufeng
Guo

231

CHAPTER 24. INTEREST RATE MODELS


0.15630 0.105
1
+
1 + 0.15630
1 + 0.15630
7. 428 977 9 102
Vuuu =

0.15809 0.105
0.13143 0.105
0.5
+ 0.5
=
1 + 0.15809
1 + 0.13143

0.12473 0.105
1
0.13143 0.105
0.10927 0.105
Vuud =
+
0.5
+ 0.5
=
1 + 0.12473
1 + 0.12473
1 + 0.13143
1 + 0.10927
2. 963 786 7 102

1
0.10927 0.105
Vdud = 0 +
0.5
+ 0.5 0 = 1. 750 465 4
1 + 0.09953
1 + 0.10927
103

0.13843 0.105
1
+
0.5 7. 428 977 9 102 + 0.5 2. 963 786 7 102 =
1 + 0.13843
1 + 0.13843
7. 501 016 6 102
Vuu =

0.10635 0.105
1
+
0.5 2. 963 786 7 102 + 0.5 1. 750 465 4 103 =
1 + 0.10635
1 + 0.10635
1. 540 576 3 102
Vud =

1
0.5 1. 750 465 4 103 + 0.5 0 = 8. 091 270 2104
1 + 0.08170

1
Vu =
0.5 7. 501 016 6 102 + 0.5 1. 540 576 3 102 =
1 + 0.10362
4. 096 334 3 102
Vdd =

1
0.5 1. 540 576 3 102 + 0.5 8. 091 270 2 104 =
1 + 0.07676
7. 529 482 0 103
Vd =

1
0.5 4. 096 334 3 102 + 0.5 7. 529 482 0 103 = 2.
1 + 0.08
245 038 2 102
V =

If the notional amount is $1, the interest cap is worth 2. 245 038 2 102 at
t = 0. Since the notional amount is 250 million, the interest cap is worth at
t=0
250 2. 245 038 2 102 = 5. 612 595 5 (million)

www.actuary88.com

c
Yufeng
Guo

232

Das könnte Ihnen auch gefallen